Download as pdf or txt
Download as pdf or txt
You are on page 1of 535

Index

Indian Legal System & Court Procedures - Page 1


Legal Sections & Medical Jurisprudence - Page 26
Previous Year Questions (Indian Legal System & Medical Jurisprudence) - Page 43
Sexual Offences & Trace Evidences - Page 68
Virginity, Impotency, Pregnancy and Abortion - Page 89
Child Abuse & Infant Deaths - Page 98
Previous Year Questions (Sexual Jurisprudence) - Page 105
Strangulation & Hanging - Page 127
Asphyxial Deaths & Drowning - Page 142
Previous Year Questions (Violent Asphyxia Deaths) - Page 153
Forensic Traumatology Part - 1 (Mechanical Injuries) - Page 165
Forensic Traumatology Part - 2 (Regional Injuries) - Page 183
Thermal Injuries & Torture Methods - Page 204
Forensic Ballistics - Page 216
Previous Year Questions (Forensic Traumatology) - Page 239
Introduction to Toxicology & Diagnosis of Poisoning - Page 275
Metallic & Non-metallic Poisons - Page 287
Animal & Plant Irritants - Page 309
Corrosives - Page 329
Delirients, Cardiac & Spinal Poisons - Page 343
Cerebral Poisons - Page 358
Agricultural & Miscellaneous Poisons - Page 368
Previous Year Questions (Forensic Toxicology) - Page 377
Techniques in Postmortem - Page 431
Thanatology - Page 440
Human Identification - Page 464
Previous Year Questions (Forensic Thanatology ) - Page 487
Forensics and Psychiatry - Page 526
Indian Legal System & Court Procedures
1. Oral evidence is more important than written testimony. This is due to which of the following?
A. Oral evidence cannot be cross-examined
B. Oral evidence can be cross-examined
C. Documentary evidence requires no proof
D. Oral evidence is seen, heard or perceived.
----------------------------------------
2. The lawyer who is defending the suspect is making inquiries with the suspect to gather information to
support his bail plea. Among the details he is looking for, is the method by which the police inquest was
carried out. All the statements regarding a police inquest are true except?
A. Senior head constable can investigate
B. Most common inquest
C. Panchnama has to be signed by investigation officer
D. Police inquest includes dowry death
----------------------------------------
3. The trial of Mr. Gupta is in court now. After having thoroughly studied the case, the judge declared
that a magistrate inquest was not needed in this case. What among the following could be the offence
Mr. Gupta is being charged for?
A. Homicide
B. Exhumation
C. Police custody death
D. Dowry death
----------------------------------------
4. Which of the following is the lowest court that can give a life imprisonment verdict?
A. Chief metropolitan magistrate
B. Metropolitan magistrate
C. High court
D. First class judicial magistrate
----------------------------------------
5. Who will pay the conduct money while serving the summons in criminal cases?
A. Court
B. Opposite party
C. Judge
D. No conduct money is given
----------------------------------------

1
6. Under which section of the Indian Penal Code (IPC) can a husband or his relatives be punished with
3 years of imprisonment and a fine for subjecting a woman to cruelty?
A. Sec. 113 A IEA
B. Sec. 498 A IPC
C. Sec. 304 B IPC
D. Sec.113 B IEA
----------------------------------------
7. Assertion: A mere stay in the hospital for 20 days does not make the injury grievous Reason: The
injured person must be in severe bodily pain or unable to follow his ordinary pursuits for a period of 20
days
A. A is True, R is the correct reason of A
B. A is false, R is true
C. A is true, R is not the correct reason of A
D. Both A and R are false
----------------------------------------
8. A person is found causing hurt by poisoning with the intent of committing an offence. Identify the IPC
section which describes this.
A. Sec. 326 IPC
B. Sec.327 A IPC
C. Sec.328 IPC
D. Sec. 329 IPC
----------------------------------------
9. Under which section, if a witness is forced to give an answer admitting his guilt, he cannot be
arrested or prosecuted for it and it is also not taken as proof against him in any criminal proceeding?
A. S. 132 IEA
B. S.151 IEA
C. S.152 IEA
D. S.148 IEA
----------------------------------------
10. Under which IPC section is a person punishable for maliciously performing an act likely to spread a
life-threatening disease?
A. Sec. 269 IPC
B. Sec. 270 IPC
C. Sec. 299 IPC
D. Sec. 370 IPC
----------------------------------------
11. Section IPC 44 describes an injury as any harm, whatever illegally, caused to any person. What
aspect of a person, among the following, is not included as a component that can be injured under this

Page 2

2
section?
A. Mind
B. Body
C. Soul
D. Property
----------------------------------------
12. In the ongoing trial of a 28-year-old man who had allegedly thrown acid on his 25-year-old female
friend, the Judge allowed the recording of a voluntary statement made by a particular individual,
according to Section 45. Who among the following is eligible to volunteer a statement?
A. Eyewitness
B. Chemical examiner
C. Hostile witness
D. Magistrate
----------------------------------------
13. In case of custodial deaths, the following are the autopsy guidelines issued by National Human
Rights Commission ,except?
A. A magisterial inquiry has to be conducted, and to report within 24 hours of occurrence
B. Clothing should be carefully removed, preserved and sealed by the police
C. Video recording of postmortem examination is to be done
D. All reports, viz. postmortem, videograph and magisterial inquiry reports must be sent to the
Commission within 2 months of the incident
----------------------------------------
14. A 45-year-old chronic alcoholic who was under regular treatment for diabetes died from acute
exacerbation of chronic pancreatitis. As per Medical certification of cause of death, column part I(a) of
the death certificate is filled with which of the following?
(or)
What should be filled in part I(a) of the death certificate for a 45-year-old chronic alcoholic on Diabetic
medication ,who died from an acute exacerbation of chronic pancreatitis?
A. Acute exacerbation of Chronic Pancreatitis
B. Chronic Pancreatitis
C. Chronic alcoholism
D. Diabetes Mellitus
----------------------------------------
15. A person with an unsound mind can be released with pending investigation or trial under which
section?
A. Section 416 Cr P C
B. Section 510 IPC
C. Section 330 Cr P C

Page 3

3
D. Section 84 IPC
----------------------------------------
16. What is the punishment that can be given under PCPNDT Act to the doctor for their 1st offense?
A. 3 yrs and Rs 10000/-
B. 5 yrs and Rs 10000/
C. 3 yrs and Rs 50000/-
D. 5 yrs and Rs 50000/-
----------------------------------------
17. As per the mental health care act, which of the following is not true regarding advanced directives?
A. Every person except minor has right to take advance directive by writing
B. It empowers the patient to choose his/ her treatment and appoint a representative to take decision
on behalf of patient.
C. If patient is minor, his/ her parent or care giver will act as representative.
D. It is the duty of every psychiatrist to plan treatment at the time of emergency, keeping advance
directive in mind.
----------------------------------------
18. In a case of murder, the defendant's witness states that he saw the defendant commit the
murder.What is the term used for a witness
(or)
What is the term used for a witness in a trial who testifies for the opposing party ?
A. Medical witness
B. Expert witness
C. Hostile witness
D. Atheist witness
----------------------------------------
19. Which of the following section deals with fabricating false evidence?
A. IPC 191
B. IPC 192
C. IPC 196
D. IPC 197
----------------------------------------
20. A married woman committed suicide after she found out that her husband is having a consensual
sexual affair. She had stated all the facts in her suicide note. Under which of the following section can
the husband be taken?
A. IPC 306
B. IPC 354A
C. IPC 420

Page 4

4
D. IPC 497
----------------------------------------
21. The defending team had appealed to the court to pronounce the evidence invalid as the
investigating team had not followed protocols as defined under CrPC 174. Which type of inquest does
fall under CrPC 174?
A. Magistrate inquest
B. Police inquest
C. Medical examiner inquest
D. None of the above
----------------------------------------
22. As per drugs and cosmetic rules, the following drug belongs to which of the following schedule of
drugs?

A. Schedule B
B. Schedule C
C. Schedule J
D. Schedule G
----------------------------------------
23. A 27-year-old woman has consulted the doctor to abort her 10-week-old fetus. The woman
allegedly has been harassed by her in-laws and is not willing to continue with the pregnancy. The
husband and his parents are threatening the doctor to not proceed with the abortion. According to the
MTP Act of 1971, consent from whom is needed by the doctor to legally proceed with the abortion?
(or)
According to the MTP Act of 1971, consent from whom is needed by the doctor to legally proceed with
the abortion?
A. Guardian
B. Pregnant women
C. Pregnant women and her husband
D. Husband of the lady
----------------------------------------

Page 5

5
24. According to Section 141 of the Indian Evidence Act of 1872, which of the following is permitted
during cross-examination but is not permitted during the documentation of a dying declaration?
A. The rules for taking oath
B. A leading question
C. An insane person giving evidence
D. Gives the correct sequence in which medical evidence should be recorded
----------------------------------------
25. After questioning the husband, his parents, family and friends of the victim and after analyzing the
social media posts of the victim herself, the police have decided to define the death of the 23-year-old
woman as Dowry death. Which IPC section provides them with the description to press these charges?
(or)
Under which section of the Indian Penal Code (IPC) is the death of the 23-year-old woman classified as
a dowry death ?
A. 302
B. 305
C. 304 A
D. 304 B
----------------------------------------
26. What is the IPC section for causing abortion without a woman’s consent?
A. 312
B. 313
C. 314
D. 315
----------------------------------------
27. One of the victims of the fire which had destroyed 6 huts in a nearby slum has decided to give a
dying declaration. The doctor was called in as part of the team to record the statement. Which among
the following is the role of the doctor?
A. To assess compos mentis
B. To record statement even in presence of magistrate
C. Cross-examine the person
D. Put person under oath before declaration
----------------------------------------
28. An 11-year-old female in the school was brought to the principal by a teacher who stated that she is
always crying, unattentive, and not taking interest in any activity. On further investigation, the girl told
that she was inappropriately touched by her uncle in private parts at her home. To whom should the
Principal report next?
A. Child welfare
B. Parents

Page 6

6
C. Police
D. Magistrate
----------------------------------------
29. The doctor who had treated the victim for the gun shot injury was summoned with a Subpoena to
testify in court with the relevant medical documentation. What category of the document does a
subpoena fall under?
A. Legal document
B. Medicolegal document
C. Medical document
D. First Information Report
----------------------------------------
30. Cognizable offenses are those where the police can proceed with an arrest without acquiring a
warrant for the arrest. All of the following acts are cognizable ,except?
A. Forgery
B. Voluntary causing grievous hurt
C. Murder
D. Dowry death
----------------------------------------
31. Commutation of death sentence cannot be done by 1. Additional sessions court 2. Sessions court
3. High court 4. Supreme court 5. President
A. 1 only
B. 1, 2
C. 1, 2, 3
D. 1, 2, 3, 4
----------------------------------------
32. Consider the following statements: 1. The Consumer Protection Act (CPA) applies to all goods, but
not any services. 2. The CPA provides for establishing 4-tier consumer dispute redressal machinery at
National, State, District and block levels. Which of the statements given above is/are correct?
A. 1 only
B. Both 1 and 2
C. 2 only
D. Neither 1 nor 2
----------------------------------------
33. After confirming the age of the victim, the police have decided to press charges under Section 376
AB I.P.C. This section specifically deals with which of the following offense?
A. Rape on a woman below 16 years
B. Rape on a woman below 12 years
C. Gang rape on a woman below 12 years

Page 7

7
D. Gang rape on a woman below 16 years
----------------------------------------
34. During the proceedings of the trial, the plaintiff was also summoned to the court to record her
statement. Who can be referred to as a plaintiff?
A. Files case in civil court
B. Acts as defender
C. Gives judgment
D. Same as public prosecutor
----------------------------------------
35. This criminal act is punishable under which section?

A. 320 IPC
B. 326 (B) IPC
C. 326 IPC
D. 326 (A) IPC
----------------------------------------
36. Choose the correct declarations with their respective intended purposes.
A. Declaration of Oslo - Abortion
B. Declaration of Helsinki - Hippocratic oath
C. Declaration of Tokyo - Patient rights
D. Declaration of Sydney - Torture
----------------------------------------

Correct Answers
Question Correct Answer

Question 1 2
Question 2 4
Question 3 1

Page 8

8
Question 4 3
Question 5 4
Question 6 2
Question 7 1
Question 8 3
Question 9 1
Question 10 2
Question 11 3
Question 12 2
Question 13 2
Question 14 1
Question 15 3
Question 16 1
Question 17 4
Question 18 3
Question 19 2
Question 20 1
Question 21 2
Question 22 2
Question 23 2
Question 24 2
Question 25 4
Question 26 2
Question 27 1
Question 28 3
Question 29 1
Question 30 1
Question 31 2
Question 32 4
Question 33 2
Question 34 1
Question 35 4
Question 36 1

Solution for Question 1:

Page 9

9
Correct Option B - Oral evidence can be cross-examined:
• Oral evidence is superior to documentary evidence, as the witness can be cross-examined.
• A doctor’s report is accepted only when he comes and deposes oral evidence in court (S. 45 IEA).
Incorrect Options:
Option A, C & D:
These statements are incorrect due to the above given reason

Solution for Question 2:


Correct Option D - Police inquest includes dowry death:
• It is an incorrect statement
• Magistrate inquest includes dowry death not police inquest
• Police Inquest: Conducted in cases of unnatural death not due to disease, such as accidents,
suicides, and homicides. Examples include: Accidental cases Suicide cases Homicide cases
• Conducted in cases of unnatural death not due to disease, such as accidents, suicides, and
homicides.
• Examples include: Accidental cases Suicide cases Homicide cases
• Accidental cases
• Suicide cases
• Homicide cases
• Magistrate Inquest: Conducted in specific cases of death that require a more formal investigation.
Examples include: Deaths occurring in mental asylums Deaths related to police beating and borstals
Deaths in police custody Dowry deaths (alleged deaths of married women within seven years of
marriage) Exhumation cases (digging up a dead body) Deaths due to police firing
• Conducted in specific cases of death that require a more formal investigation.
• Examples include: Deaths occurring in mental asylums Deaths related to police beating and borstals
Deaths in police custody Dowry deaths (alleged deaths of married women within seven years of
marriage) Exhumation cases (digging up a dead body) Deaths due to police firing
• Deaths occurring in mental asylums
• Deaths related to police beating and borstals
• Deaths in police custody
• Dowry deaths (alleged deaths of married women within seven years of marriage)
• Exhumation cases (digging up a dead body)
• Deaths due to police firing
• Conducted in cases of unnatural death not due to disease, such as accidents, suicides, and
homicides.
• Examples include: Accidental cases Suicide cases Homicide cases

Page 10

10
• Accidental cases
• Suicide cases
• Homicide cases
• Accidental cases
• Suicide cases
• Homicide cases
• Conducted in specific cases of death that require a more formal investigation.
• Examples include: Deaths occurring in mental asylums Deaths related to police beating and borstals
Deaths in police custody Dowry deaths (alleged deaths of married women within seven years of
marriage) Exhumation cases (digging up a dead body) Deaths due to police firing
• Deaths occurring in mental asylums
• Deaths related to police beating and borstals
• Deaths in police custody
• Dowry deaths (alleged deaths of married women within seven years of marriage)
• Exhumation cases (digging up a dead body)
• Deaths due to police firing
• Deaths occurring in mental asylums
• Deaths related to police beating and borstals
• Deaths in police custody
• Dowry deaths (alleged deaths of married women within seven years of marriage)
• Exhumation cases (digging up a dead body)
• Deaths due to police firing
Incorrect Options:
Option A - Senior head constable can investigate:
• The minimum cadre of police for conducting an inquest is a Station Officer (Senior Head Constable).
Option B - Most common inquest:
• Police inquest is the most common type of inquest.
Option C - CrPC for police inquest is 174:
• CrPC for police inquest is 174
• CrPC for magistrate inquest is 176

Solution for Question 3:


Correct Option A - Homicide:
• Magistrate inquest is not done in Homicide.
Incorrect Options:

Page 11

11
Option B - Exhumation: In Exhumation cases (where the body is dug out of a
grave), the inquest is done by the Executive Magistrate.
Option C - Police custody death: The disappearance or death of a person in police custody or when a
woman is raped in police custody or any custody permitted by the court or during police interrogation is
done by Magistrate inquest.
Option D - Dowry death: Dowry deaths (suicide/death of a woman within 7
years of marriage) include in the indication for Magistrate inquest.

Solution for Question 4:


Correct Option C - High court:
• The lowest court that can give a life imprisonment verdict is the High court.
• Punishment: Imprisonment for any period including the death sentence.
Incorrect Options:
Option A - Chief metropolitan magistrate:
• A chief metropolitan magistrate can sentence a guilty for imprisonment up to 7 years.
Option B - Metropolitan magistrate:
• In metropolitan cities with more than 1 million population, the Chief Judicial Magistrate and first-class
Judicial Magistrate are designated as Chief Metropolitan Magistrate and Metropolitan Magistrate
respectively.
Option D - First class judicial magistrate:
• A first-class judicial magistrate can sentence a guilty for imprisonment up to 3 years.

Solution for Question 5:


Correct Option D - No conduct money is given:
• In criminal cases, no fee is paid to the witness at the time of serving the summons & the witness has
to attend the court in the interest of the state.
Incorrect Options:
Option A - Court:
• In criminal cases, no fee is paid to the witness at the time of serving the summons.
• He must attend the court and give evidence because of the interest of the State in securing justice;
otherwise, he will be charged with contempt of court.
• However, conveyance charges and daily allowance are paid according to the Government rules after
the testimony.
Option B - Opposite party:
• In criminal cases, no fee is paid to the witness at the time of serving the summons & the witness has
to attend the court in the interest of the state.

Page 12

12
Option C - Judge:
• If he feels the amount is less or not paid, he can bring it to the notice of the judge.
• In responding to a summon in civil cases, witnesses can request for adequate conduct money from
the Judge..

Solution for Question 6:


Correct Option B - Sec. 498 A IPC:
• Sec. 498 A IPC describes cruelty to a woman by husband/relatives shall be punished with 3 years
imprisonment and also liable to a fine.
Incorrect Options:
Option A - Sec. 113 A IEA: Sec. 113 A IEA: Presumption as to abetment of suicide by a
married woman.
Option C - Sec. 304 B IPC: Sec. 304 B IPC describes dowry's death- the death of a
married woman within 7 yrs of marriage under suspicious conditions.
Option D - Sec.113 B IEA: Sec.113 B IEA: Presumption as to dowry death.

Solution for Question 7:


Correct Option A - A is True, R is the correct reason of A:
Assertion: A mere stay in the hospital for 20 days does not make the injury grievous
• Sec 320 IPC defines grievous hurt and lists only eight kinds of hurt that it labels as “grievous”.
Reason: The injured person must be in severe bodily pain or unable to follow his ordinary pursuits for a
period of 20 days
• A and R both are true and R is the correct explanation for A
• The injured person must be in severe bodily pain or unable to follow his ordinary pursuits for a period
of 20 days
Incorrect Options:
Options B, C & D

Solution for Question 8:


Correct Option C - Sec. 328 IPC:
• Section 328: Causing hurt by means of poison, etc., with intent to commit an offence.
Incorrect Options:
Option A
- Sec. 326 IPC: Section 326: Voluntarily causing grievous hurt by dangerous weapons or means.

Page 13

13
Option B - Sec. 327 A
IPC: Section327: Voluntarily causing hurt to extort property, or to constrain to an illegal act.
Option D - Sec. 329 IPC: Section 329: Voluntarily causing grievous hurt to extort property, or to constra
in an illegal act.

Solution for Question 9:


Correct Option A - S. 132 IEA:
• Witness not excused from answering on ground that answer will criminate.
• If a witness is forced to give an answer admitting his guilt, he cannot be arrested or prosecuted for it
and it is also not taken as proof against him in any criminal proceeding.
Incorrect Options:
Option B - S.151 IEA:
• Indecent or Scandalous questions.
• Unless they relate to the facts at issue or to information that is required to be known in order to
determine whether the facts at issue existed, the Court may prohibit any questions or inquiries that it
deems to be indecent or scandalous, even though they may have some bearing on the questions
before the Court.
Option C - S.152 IEA:
• Questions intended to insult or annoy.
• Any inquiry that the Court determines to be needlessly offensive in form or that the Court believes is
meant to offend or annoy someone will be prohibited.
Option D - S.148 IEA:
• Court to decide when question shall be asked and when witness compelled to answer.
• The Court will decide whether or not the witness will be required to answer any such question if it
relates to a subject not pertinent to the lawsuit or proceeding, with the exception of how it might
jeopardise the witness's reputation.
• The Court may also, if it sees fit, inform the witness that he is not required to answer the question.

Solution for Question 10:


Correct Option B - Sec. 270 IPC:
• Sec.270 IPC: The conduct in order to be punishable must be malicious or negligent, so as to cause
the spread of an infectious disease dangerous to life. (imprisonment up to 2 years or with a fine or both.
Incorrect Options:
Option A - Sec. 269 IPC:
• Sec.269 IPC: Whoever negligently does an act which is likely to spread infection of any disease
dangerous to life is punishable up to 6 months or with a fine or both.

Page 14

14
Option C - Sec. 299 IPC:
• Sec. 299 IPC: Culpable homicide.
Option D - Sec. 370 IPC:
• Sec. 370 IPC: Whoever commits the offence of trafficking shall be punished with rigorous
imprisonment for a term which shall not be less than seven years, but which may extend to ten years,
and shall also be liable to fine.

Solution for Question 11:


Correct Option C - Soul:
• Injury: Any harm, whatever illegal, caused to any person in body, mind, reputation or property (Sec. 44
IPC).
• The soul is not included as a component that can be injured under Section IPC 44.
Incorrect Options:
Option A - Mind: Section IPC 44 describes any harm, whatever illegally, caused to any person in body,
mind, reputation, or property.
Option B - Body: Section IPC 44 describes any harm, whatever illegally, caused to any person in body,
mind, reputation, or property.
Option D - Property: Section IPC 44 describes any harm, whatever illegally, caused to any person in bo
dy, mind, reputation, or property.

Solution for Question 12:


Correct Option B - Chemical examiner:
• Expert witness (Sec. 45 IEA ): Person who has been trained or skilled in a technical or scientific
subject.
• Examples of expert witnesses: Handwriting or fingerprint expert, doctor, and chemical examiner.
• An expert witness is a person who has been trained or is skilled or has knowledge, experience, or
education in a technical or scientific subject, and is capable of drawing opinions and conclusions from
the facts observed by himself or noticed by others.
Incorrect Options:
Option A - Eyewitness:
• Oral evidence is more important than documentary evidence, as it permits cross-examination.
• It must be direct-it must be evidence of an eyewitness (Sec. 60 IEA).
Option C - Hostile witness:
• A hostile witness is a witness in a trial who testifies for the opposing party or a witness who offers
adverse testimony to the calling party during direct examination.
Option D - Magistrate:

Page 15

15
• A judge/ magistrate can function as an expert witness when the proceeding requires the services of
an expert lawyer unlike in this case.

Solution for Question 13:


Correct Option B - Clothing should be carefully removed, preserved and sealed by the police:
• Clothing should not be removed by the police or any other person as it must be examined, preserved,
and sealed by the doctor conducting the autopsy.
• It should be sent for further examination at the FSL.
Incorrect Options:
Option A - A magisterial inquiry has to be conducted, and reported within 24 hours of occurrence: In ev
ery case of custodial death/rape, a
magisterial inquiry has to be conducted and reported within 24 hours of occurrence.
Option C - Video recording of postmortem examination is to be done: To prevent tampering and supple
ment the postmortem report, a video recording of the postmortem examination is to be done.
Option D - All reports, viz. postmortem, videography and magisterial inquiry reports must be sent to the
Commission within 2 months of the incident: All reports, viz. postmortem, videography and magisterial
inquiry reports must be sent to the Commission within 2 months of the incident.

Solution for Question 14:


Correct Option A - Acute exacerbation of Chronic Pancreatitis:
• Acute exacerbation of Chronic Pancreatitis can be filled in Column part I(a), as it is the immediate
cause of death.
Incorrect Options:
Option B - Chronic Pancreatitis:
• Chronic Pancreatitis can be filled in Column part I(b), as it is the antecedent cause of death.
Option C - Chronic alcoholism:
• Chronic alcoholism can be filled in Column part I(c), as it is the underlying cause of death.
Option D - Diabetes Mellitus:
• Diabetes Mellitus can be filled in Column part II, as it is the other health condition present in the
person but does not cause the death of the person.

Solution for Question 15:


Correct Option C - Section 330 Cr P C:
• Release of the person of unsound mind pending investigation or trial.

Page 16

16
Incorrect Options:
Option A - Section 416 Cr P C :
• Postponement of death sentence in a pregnant female/ commutation of death sentence to life
imprisonment in a pregnant female.
• It is practised in India.
Option B - Section 510 IPC:
• Punishment for causing public nuisance under the influence of alcohol.
Option D - Section 84 IPC:
• Sec. 84 IPC: Insane - Act of a person of unsound mind. - Nothing is an offence which is done by a
person who, at the time of doing it, by reason of unsoundness of mind, is incapable of knowing the
nature of the act, or that he is doing what is either wrong or contrary to law.”

Solution for Question 16:


Correct Option A - 3 yrs and Rs 10000/-:
• As per PCPNDT Act 1994, the doctor can be punished for 3 yrs and Rs 10,000/- for 1st offense.
Incorrect Options:
Option B - 5 yrs and Rs 10000/-
Option C - 3 yrs and Rs 50000/-
Option D - 5 yrs and Rs 50000/-: As per PCPNDT Act 1994, the doctor can be punished for 5
yrs and Rs 50,000/- for the 2nd offense.

Solution for Question 17:


Correct Option D - It is the duty of every psychiatrist to plan treatment at the time of emergency, keepin
g advance directive in mind:
• The advance directive shall not apply to emergency treatment.
• It is the duty of every psychiatrist to plan treatment, keeping advance directive in mind.
Incorrect Options:
Option A - Every person except minor has right to take advance directive by writing:
• It is true regarding advanced directives.
• Every person except a minor has the right to take advance directive by writing.
Option B - It empowers the patient to choose his/her treatment and appoint a
representative to take decision on behalf of patient:
• It is true regarding advanced directives.
Option C - If patient is minor, his/her parent or care giver will act as representative:
• It is true regarding advanced directives.

Page 17

17
• If the patient is a minor, his/ her parent or caregiver will act as a representative.

Solution for Question 18:


Correct Option C - Hostile witness:
• A hostile witness is a witness in a trial who testifies for the opposing party or a witness who offers
adverse testimony to the calling party during direct examination.
Incorrect Options:
Option A - Medical witness:
• Medical students and newly qualified doctors should attend the Courts, where they can follow the
proceedings, hear the evidence given by medical witnesses, and note the questions put and the replies
given.
• In case of hurt, the duty of the medical witness is only to describe the facts. It is the Court that must
decide ~ whether it is simple or grievous.
• The opinion of the medical witness must be based on his own personal observations. The doctor
should obtain a detailed history from the accused person and from other sources, and then carry out
physical examination and investigations.
• When a medical witness deposes regarding the existence, characters, and extent of the mental
disease, the Judge has to decide whether the disease justifies an acquittal on the ground of
unsoundness of mind.
Option B - Expert witness:
• An expert witness is a person who has been trained or is skilled or has knowledge, experience, or
education in a technical or scientific subject, and is capable of drawing opinions and conclusions from
the facts observed by himself, or noticed by others, e.g., doctor, firearms expert, fingerprints expert,
handwriting expert, etc. (S. 45, I.E.A.).
Option D - Atheist witness:
• If the witness is an atheist, he makes a solemn affirmation in the same terms, instead of swearing by
God. (S.51, IPC).

Solution for Question 19:


Correct Option B - IPC 192:
• Section 192 of IPC deals with fabricating false evidence.
Incorrect Options:
Option A - IPC 191: S. 191, I.P.C : Giving false evidence.
Option C - IPC 196: IPC 196: Using evidence known to be false.
Option D
- IPC 197: S. 197, I.P.C : Issuing or signing false certificate (imprisonment up to seven years).

Page 18

18
Solution for Question 20:
Correct Option A - IPC 306:
• The scenario is suggestive of the abetment of suicide and the man can be taken under section 306 of
IPC.
Incorrect Options:
Option B - IPC 354A:
• Sexual harassment is defined as physical contact and advances involving unwelcome and explicit
sexual overtures or demanding sexual favours, showing pornography against her will or making
sexually coloured remarks.
• It is punishable with (rigorous) imprisonment for 1–3 years with/without a fine (Sec. 354-A IPC).
• The offense is cognizable and bailable.
Option C - IPC 420:
• IPC 420: Cheating and dishonestly inducing delivery of property.
Option D - IPC 497:
• Section 497 of IPC with adultery was invalidated by the hon’ble supreme court of India on 27th
September 2018.
• However, there might be civil liabilities including grounds for divorce and dissolution of marriage.

Solution for Question 21:


Correct Option B - Police inquest:
• 174 CrPC: Police inquest.
Incorrect Options:
Option A - Magistrate inquest: 176 CrPC: Magistrate inquest.
Option C - Medical examiner inquest: 53 CrPC: Examination of the accused by a medical practitioner a
t the request of the police officer (a doctor can examine the arrested person even without his consent).
Option D - None of the above: Incorrect.

Solution for Question 22:


Correct Option B - Schedule C:
• Schedule C and C1: Biological products, e.g. serums & antisera.
Incorrect Options:
Option A - Schedule B: Schedule B: Contains fee structure for government-run labs.

Page 19

19
Option C - Schedule J: Schedule J: Diseases and ailments (by whatever name described) which a drug
may not purport to prevent or cure or make claims to prevent or cure, e.g. AIDS, cancer, cataract, con
genital malformations, deafness, blindness, hydrocoele, hernia, piles, leucoderma, stammering, paraly
sis, etc.
Option D - Schedule G: Schedule G: Chemotherapeutic agents for cancer, antihistaminics, and hypogl
ycaemic agents.

Solution for Question 23:


Correct Option B - pregnant women:
• Only the consent of the pregnant woman is necessary.
Incorrect Options:
Option A - Guardian:
• For a minor or insane person – consent from a guardian is required.
Option C - Pregnant women and husband:
• Only the consent of the pregnant woman is necessary.
• Consent from the husband is not necessary.
Option D - Husband of the lady:
• Consent from husband- not necessary.

Solution for Question 24:


Correct Option B - A leading question:
• A leading question is defined Under The Indian Evidence Act Section 141.
• Any question suggesting the answer that the person asking it wishes or expects to receive is called a
leading question.
• It includes a material fact and admits of a conclusive answer by a simple ‘Yes’ or ‘No’.
Incorrect Options:
Option A - The rules for taking the oath:
• Section 51 IPC - Oath definition.
• Section 178 IPC - Not taking the oath is punishable.
• For a dying declaration, the oath is not required.
Option C - An insane person giving evidence
• Section 84 IPC: offense committed by an insane person isn't liable/responsible for the offense.
• Section 118 IEA: Evidence: Competency for giving evidence depends upon understanding, but not on
age. A child of any age can give evidence, if the court is satisfied that the child is truthful.
Option D - Gives the correct sequence in which medical evidence should be recorded:

Page 20

20
• A leading question is defined Under The Indian Evidence Act Section 141.

Solution for Question 25:


Correct Option D - 304 B:
• Dowry death (Sec. 304-B IPC): Where the death of a woman is caused by any burn or bodily injury or
occurs in a manner other than under normal circumstances within 7 years of her marriage, and it is
shown that she was subjected to cruelty or harassment by her husband or any relative of her husband
for, or in connection with any demand for dowry, such death is called dowry death.
Incorrect Options:
Option A - 302:
• Sec. 302 IPC-Punishment for murder – death or imprisonment for life.
Option B - 305:
• Sec. 305 IPC-Punishment for Abetment [influencing/forcing] to the suicide of children/ insane.
Option C - 304 A:
• Sec. 304-A IPC-Punishment for causing death due to a rash/ negligent act.
• Death due to medical negligence is punishable under this section.
• Gives 2 yrs imprisonment ± fine.

Solution for Question 26:


Correct Option B - 313:
• Sec. 313 IPC: If the miscarriage is caused without the consent of the woman, imprisonment may be
up to ten years.
Incorrect Options:
Option A - 312: Sec. 312 I.P.C: Whoever voluntarily causes criminal abortion is liable for imprisonment
of up to upto 10 years and also be liable to fine .
Option C - 314: Sec. 314 IPC: If a pregnant woman dies from an act intended to cause miscarriage.
Option D - 315: Sec. 315 IPC: A person doing an act intended to prevent the child from being born aliv
e or to cause it to die after its birth, is liable.

Solution for Question 27:


Correct Option A - To assess compos mentis:
• A dying declaration is a written or oral statement of a person, who is dying as a result of some
unlawful act, relating to the material facts of the cause of his death or bearing on the circumstances
(S.32, IEA).

Page 21

21
• If there is time, the Executive magistrate should be called to record the declaration.
• Before recording the statement, the doctor should certify that the person is conscious and his mental
faculties are normal (compos mentis).
Incorrect Options:
Option B - To record statements even in presence of a
magistrate: A magistrate can record the statements during the dying declaration.
Option C - Cross-examine the person: Cross-examination is not done during the dying declaration.
Option D - Put the person under oath before the declaration: An oath is not required before a
dying declaration.

Solution for Question 28:


Correct Option C - Police:
• Reporting the offense to the police is mandatory.
• Failure to report attracts punishment with imprisonment of up to 6 months or a fine or both.
• Section 19 of the POCSO Act, 2012 provides a mandatory report of child sexual offenses to the
Special Juvenile Police Unit or the local police.
Incorrect Options:
Options: A, B & D

Solution for Question 29:


Correct Option A - Legal document:
• Subpoenas fall under the category of Legal documents.
• The legal document compelling the attendance of a witness in a court of law, under penalty, on a
particular day, time, and place to give evidence.
Incorrect Options:
Option B - Medicolegal document:
• Medicolegal Reports: They are reports prepared by a doctor at the request of the investigating officer,
usually in criminal cases, e.g., assault, rape, murder, etc.
• The examination of an injured person or a dead body is made, when there is a requisition from a
police officer or Magistrate.
• Subpoenas do not fall under this category
Option C - Medical document:
• Medical records are legal records.

Page 22

22
• They include information of a medical, preventive, and social nature, as well as the information
required for providing patients with the proper care.
• Subpoenas do not fall under this category
Option D - First Investigation Report(FIR):
• When the police learn of the commission of a crime that is punishable by law, they must create a First
Information Report (FIR).
• The First Information Report is a report of information that is received by the police first in time, for this
reason.
• Subpoenas do not fall under this category

Solution for Question 30:


Correct Option A - Forgery:
• The forgery act is not a cognizable offense.
Incorrect Options:
Option B - Voluntary causing grievous hurt:
• It belongs to a cognizable offense, and police can arrest a person without an arrest warrant.
Option C - Murder:
• Murder belongs to cognizable offenses.
• Cognizable offenses are those where the police can proceed with an arrest without acquiring a
warrant for the arrest.
Option D - Dowry death:
• Dowry death is a cognizable offense, and police can arrest a person without an arrest warrant.

Solution for Question 31:


Correct Option B - 1, 2:
• Commutation of the death sentence cannot be done by the additional sessions court and sessions
court.
• Commutation of the death sentence is possible by the High court, Supreme court & the President.
• The Sessions Court is established by the State Government (S.9, Cr.P.C.) and is usually located at
the district headquarters.
• It can only try cases which have been committed to it by a Magistrate (S.l93,Cr.P.C.).
• It can pass any sentence authorised by law, but a sentence of death passed by it must be confirmed
by the High Court (S.28 (2) and S.366, Cr.P.C.).
• District Court: deals with civil cases.

Page 23

23
• An Assistant Sessions Court can pass sentence of imprisonment up to ten years and unlimited fine
(S.28 (3), Cr.P.C.).
Incorrect Options:
Options B, C & D: Due to the above given reasons

Solution for Question 32:


Correct Option D - Neither 1 nor 2:
1. The Consumer Protection Act (CPA) applies to all goods, but not any services.
• Consumer protection act applies to all goods and services. So this option is incorrect.
2. The CPA provides for establishing 4-tier consumer dispute redressal machinery at National, State, D
istrict, and block levels.
• CPA provides for establishing “three” consumer dispute, redressal machinery at tier the National,
State, and District levels.
• So both options are incorrect.
Incorrect Options:
Options A, B & C:

Solution for Question 33:


Correct Option B - Rape on a woman below 12 years:
• Section 376 AB IPC deals with the rape on girls below 12 years on the basis of a recent amendment
in 2018.
Incorrect Options:
Option A - Rape on a woman below 16 years: Section 376(3) :Rape on a woman below 16 years
Option C - Gang rape on a woman below 12 years: 376 DB: The gang rape of a girl <
12 years (life/ death).
Option D - Gang rape on a woman below 16 years: 376 DA: The gang rape of a girl <
16 years (life/ death).

Solution for Question 34:


Correct Option A - Files case in civil court:
• Plaintiff is the person who files a case in civil court.
Incorrect Options:
Option B - Acts as defender:

Page 24

24
• A defendant is a person against whom a civil action is filed.
Option C - Gives judgment:
• Plaintiff is not the person who gives judgment, plaintiff is the person who files a case in civil court.
Option D - Same as public prosecutor:
• Plaintiff is the person who files a case in civil court.
• In criminal cases, the public prosecutor commences the Examination-in-Chief (Direct Examination).

Solution for Question 35:


Correct Option D - 326 (A) IPC:
• The image shows an acid attack (vitriolage).
• Vitriolage is punishable under 326 (A) I.P.C.
Incorrect Options:
Option A - 320 IPC: 320 IPC: Definition of grievous hurt.
Option B - 326 (B) IPC: 326 (B) IPC: Attempt of acid attack.
Option C - 326 IPC: 326 IPC: Grievous hurt by dangerous weapon.

Solution for Question 36:


Correct Option A - Declaration of Oslo - Abortion:
• It is the correct option.
Incorrect Options:
Option B - Declaration of Helsinki - Hippocratic oath: Declaration of Helsinki - Clinical research.
Option C - Declaration of Tokyo - Patient rights: Declaration of Tokyo - Torture.
Option D - Declaration of Sydney - Torture: Declaration of Sydney - Brain death.

Page 25

25
Legal Sections & Medical Jurisprudence
1. Stripping naked a female patient without her consent "by a male doctor" for medical examination is
punishable under which section?
(or)
Stripping naked a female patient without her consent "by a male doctor" for medical examination is
punishable under which section?
A. S.228 A IPC
B. S.354 IPC
C. S.354 C IPC
D. S.493 IPC
----------------------------------------
2. A 30-year-old man and his parents have been arrested under IPC 314 after the death of the man's
26-year-old wife. What offence are they being charged for?
(or)
For what offense are the 30-year-old man and his parents being charged under IPC 314 following the
death of the man's 26-year-old wife?
A. Causing miscarriage without consent
B. Death of the patient caused by intent to cause miscarriage
C. Causing miscarriage with consent
D. Death of the child during intent to cause miscarriage
----------------------------------------
3. Jessie Pinkman, a 25-year-old student has been charged under IPC 94 after he was arrested with
his chemistry professor, Mr. Walter White. After a thorough investigation, the authorities were of the
opinion that Mr. White was the chief conspirator of the scam. What charges are being levied on Mr.
Pinkman?
(or)
What does Section 94 IPC define?
A. Consent in emergency situations
B. Criminal responsibility of an act done by a person < 7 years
C. Criminal responsibility of an act done by an intoxicated person
D. Criminal responsibility of an act Compelled by threats
----------------------------------------
4. In-camera trial of a rape case hearing is done under which section?
(or)
In-camera trial of a rape case hearing is done under which section?
A. 376 IPC
B. 327 (2) CrPC

26
C. 53 CrPC
D. 375 IPC
----------------------------------------
5. A police officer has the power to summon a doctor to the police station for recording a statement
under which section?
(or)
A police officer has the power to summon a doctor to the police station for recording a statement under
which section?
A. 160 CrPC
B. 161 CrPC
C. 162 CrPC
D. 163 CrPC
----------------------------------------
6. A 25-year-old college student was rescued from the house of a professor in a sedated state. Further
investigation confirmed that the girl had been physically and sexually assaulted. The suspect has been
arrested under several charges. Which IPC section will he be charged under for giving stupefying drugs
with intent to cause hurt?
(or)
Under which IPC section will the suspect be charged for giving stupefying drugs with intent to cause
hurt to the 25-year-old college student who was physically and sexually assaulted?
A. 325 IPC
B. 326 IPC
C. 327 IPC
D. 328 IPC
----------------------------------------
7. A doctor injected a syringe which was used before on a patient, who turned out to be HIV positive
later. Under which of the following IPCs will the doctor be booked for medical negligence?
(or)
Under which IPC section can a doctor be booked for medical negligence?
A. IPC 166 B
B. IPC 202
C. IPC 203
D. IPC 269
----------------------------------------
8. The cruelty of Mrs. X for dowry by her husband Mr. Y and in Laws is punishable under which
section?
(or)
The cruelty for dowry by husband and in Laws is punishable under which section?

Page 2

27
A. 304 A IPC
B. 304 B IPC
C. 498 A IPC
D. 320 IPC
----------------------------------------
9. What is it when a private doctor refers a patient with a sexual offence to a government hospital
instead of treating them?
(or)
Under which section is a private doctor punishable for referring a patient with a sexual offense to a
government hospital instead of providing treatment?
A. Not an offence
B. Punishable under 166 B IPC
C. Punishable under 357 (c) CrPC
D. Punishable under 327 (c) CrPC
----------------------------------------
10. What is the section of IPC that deals with not attending the court after receiving the summons?
(or)
What is the IPC section for not attending the court after receiving the summons?
A. 174 IPC
B. 176 IPC
C. 178 IPC
D. 191 IPC
----------------------------------------
11. A 40-year-old officer has been arrested after being named by a 27-year-old employee for
harassment. Among the various charges being registered against him, one offence is charged under
IPC 509. What specific offence is he being charged for, under that section?
(or)
Which offence is punishable under IPC 509?
A. Public nuisance under alcohol influence
B. Inquest
C. Exhibitionism
D. Act intended to insult the modesty
----------------------------------------
12. Mr. X fired his gun at Mr. Y who moved and escaped with the bullet only grazing his arm. There was
only a little bleeding and no other significant injury. Mr. X is liable for arrest under which section of IPC?
(or)

Page 3

28
Under which IPC section should Mr. X be arrested for firing a gun at Mr. Y, causing a minor injury with
little bleeding but no significant harm?
A. 302
B. 304
C. 324
D. 326
----------------------------------------
13. When a homicide case comes to a doctor, he has to inform police. This comes under which
section?
(or)
For a homicide case under which section doctor has to inform the police ?
A. 39 CrPC
B. 27 CrPC
C. 174 CrPC
D. 176 CrPC
----------------------------------------
14. The lawyer who is defending the suspect has fought the initial trial, for the suspect to be excused
under IPC 84, which was derived from McNaughton RULES. What is the basis of the argument and
request for excuse according to the section appealed under?
(or)
McNaughton rules stand for?
A. Criminal responsibility of insane
B. Criminal responsibility of drunken person
C. Criminal responsibility of intoxicated person
D. Criminal responsibility of child
----------------------------------------
15. A Leading question is asked during the cross-examination of the witness, during
examination-in-chief and re-examination after the court's permission. Which section of the Indian
Evidence Act defines a Leading question?
(or)
Which section of the Indian Evidence Act defines a Leading question?
A. S.141 IEA
B. S.137 IEA
C. S.138 1EA
D. S.165 IEA
----------------------------------------
16. An inpatient of the hospital suffers burns injury when hot bottles were used for the treatment. In this
case, the hospital incharge was also charged with negligence under the doctrine of which of the

Page 4

29
following?
(or)
Under which doctrine was the hospital incharge charged with negligence when an inpatient suffered
burns from hot bottles used for treatment?
A. Vicarious liability
B. Res ipsa loquitur
C. Novus actus interveniens
D. Contributory negligence
----------------------------------------
17. A 27-year-old man has filed a case of criminal negligence against a doctor after repeat X-rays on a
follow-up visit showed that his fractured right radial bone had malunited and thus created a deformity.
The lawyer defending the doctor attempted to use one of the following doctrines during the trial but was
reminded by the judge that it cannot be used in case of criminal negligence. Which doctrine among the
following was the lawyer citing?
(or)
Which among the following doctrine not used in the case of criminal negligence?
A. Vicarious liability
B. Contributory negligence
C. Res ipsa loquitur
D. Novus actus interveniens
----------------------------------------
18. A 30-year-old man has been arrested by the police on charges of stalking a female friend. In the
complaint, she alleges that he had followed her home after office hours and had also been threatening
to hurt her if she did not accept his marriage proposal. What does the law refer to as stalking if this was
the man's first offence?
(or)
What does the law refer to as stalking if it was the man's first offence?
A. Cognizable & bailable
B. Non-Cognizable & non-bailable
C. Non-Cognizable & bailable
D. Cognizable & non-bailable
----------------------------------------
19. The emblems of the International Red Cross and Red Crescent Movement, under the Geneva
Conventions, are to be placed on humanitarian and medical vehicles and buildings and to be worn by
medical personnel and others carrying out humanitarian work, to protect them from military attack on
the battlefield. All the following statements regarding the red cross sign are true except?
(or)
Which statement is false regarding the use of the Red Cross emblem under the Geneva Conventions
for protecting humanitarian and medical personnel, vehicles, and buildings from the military attack on
battlefield?

Page 5

30
A. Can be used by army medical services
B. Punishable to use it without permission
C. Used by members of Red Cross
D. Can be used by private doctors and ambulances.
----------------------------------------
20. Forceps inside the abdomen postoperatively amount to which of the following criminal act?
(or)
Forceps inside the abdomen postoperatively amount to which of the following criminal act?

A. Res ipsa loquitur


B. Res judicata
C. Respondent superior
D. Novus actus interveniens
----------------------------------------
21. In the case of civil negligence against a doctor, the patient has to prove all of the following elements
of negligence except?
A. Duty
B. Dereliction
C. Damage
D. None of the above
----------------------------------------
22. If a private practitioner is convinced that the patient is suffering from homicidal poisoning, he is
bound to inform the police or magistrate under:
(or)
Under which Section ,private practitioner should report a case of homicidal poisoning to the police or
magistrate?
A. Section 39 CrPC
B. Section 176 CrPC
C. Section 175 CrPC

Page 6

31
D. Section 41 CrPC
----------------------------------------

Correct Answers
Question Correct Answer

Question 1 2
Question 2 2
Question 3 4
Question 4 2
Question 5 1
Question 6 4
Question 7 4
Question 8 3
Question 9 2
Question 10 1
Question 11 4
Question 12 3
Question 13 1
Question 14 1
Question 15 1
Question 16 1
Question 17 2
Question 18 1
Question 19 4
Question 20 1
Question 21 4
Question 22 1

Solution for Question 1:


Correct Option B - S.354 IPC:
• Assault or use of criminal force on to a woman with intent to outrage her modesty.
• A female patient can accuse a male doctor of indecent assault (intimate touching or fondling the
breast or private parts etc.) during the consultation.

Incorrect Options:
Option A - S.228 A IPC:

Page 7

32
• Disclosure of the identity of the victim of certain offences like 376, section 376A, 376B, 376C or 376D.
• Printing or publication of a proceeding without prior permission of the court.
Option C - S.354 C IPC:
• Under Sec. 354-C IPC, voyeurism is a punishable offence.
• It states that ‘any man who watches or captures the images of a woman engaged in a private act
(changing clothes, using lavatory or doing a sexual act) in circumstances where it is expected of not
being observed or circulate such image is punishable with imprisonment from 1–3 years and fine
(cognizable and bailable).
Option D - S.493 IPC:
• A man by deceit causes a woman not lawfully married to him to believe that she is lawfully married to
him and to cohabit with him in that belief.

Solution for Question 2:


Correct Option B - Death of the patient caused by intent to cause miscarriage:
• Sec. 314 IPC: If a pregnant woman dies from an act intended to cause miscarriage, the offender is
liable to be punished with rigorous imprisonment which shall not be less than ten years and also a fine
of up to two lakh rupees.

Incorrect Options:
Option A - Causing miscarriage without consent:
• Sec. 313 IPC: If the miscarriage is caused without the consent of the woman, imprisonment may be
up to ten years.
Option C - Causing miscarriage with consent:
• Sec. 312 IPC: Whoever voluntarily causes criminal abortion is liable for imprisonment up to three
years and/or a fine, and if the woman is quick with the child the imprisonment may extend up to 7 years.
• It is necessary that the woman should be pregnant and that abortion should be carried out with her
consent.
• Both the person causing the abortion and the woman are liable for punishment.
Option D - Death of the child during intent to cause miscarriage:
• Sec. 316 IPC: Causing the death of a quick unborn child by any act amounts to culpable homicide,
and the punishment may extend up to ten years imprisonment.

Solution for Question 3:


Correct Option D - Criminal responsibility of an act compelled by threats:
• Section 94: Act to which a person is compelled by threats.
Incorrect Options:

Page 8

33
Option A - Consent in emergency situations:
• 92 IPC: In emergency/ life-threatening situations, consent is not required (law-given consent).
Option B - Criminal responsibility of an act done by a person < 7 years:
• 82 IPC: Nothing is an offence which is done by a child <7 years of age.
Option C - Criminal responsibility of an act done by an intoxicated person:
• 86 IPC: Offence caused after voluntary intoxication.
• 85 IPC: Offence caused after involuntary intoxication is not an offence.

Solution for Question 4:


Correct Option B - 327 (2) CrPC:
• Magistrate if available, and allowed the printing or publication of proceedings in rape cases subject to
maintaining the anonymity of the parties; Sec. 327 (2) & (3) CrPC.
• 327 (2): In-camera trial
• 327 (3): Proceedings not to be published without permission of the court

Incorrect Options:
Option A - 376 IPC: 376 IPC: Punishment for rape.
Option C - 53 CrPC: 53 Cr PC: Examination of accused by medical practitioner at the request of the pol
ice officer.
Option D - 375 IPC: Rape is unlawful sexual intercourse by a man with a
woman, and is defined under Sec. 375 IPC.

Solution for Question 5:


Correct Option A - 160 CrPC:
• S. 160, Cr.P.C.: A police officer has the power to summon any witness (doctor) to the police station for
recording a statement.

Incorrect Options:
Option B - 161 CrPC:
• Examination of witnesses by police.
• Any police officer making an investigation may examine orally any person supposed to be acquainted
with the facts and circumstances of the case.
Option C - 162 CrPC:
• Statements to police not to be signed; Use of statements in evidence.

Page 9

34
• No statement made by any person to a police officer in the course of an investigation, shall, if reduced
to writing, be signed by the person making it.
Option D - 163 CrPC:
• No inducement is to be offered.
• No police officer shall offer any such inducement, threat or promise.
• But no police officer or other person shall prevent any person from making any statement which he
may be disposed to make of his own free will.

Solution for Question 6:


Correct Option D - 328 IPC:
• 328 IPC: Administering stupefying drugs with intent to cause hurt, etc.

Incorrect Options:
Option A - 325 IPC:
• 325 IPC: Voluntarily causing grievous hurt; Punishment: Up to 7 years and a fine.
Option B - 326 IPC:
• 326 IPC: Voluntarily causing grievous hurt by dangerous weapons/means. Punishment: Up to 10
years and a fine.
• 326-A IPC: Voluntarily causing grievous hurt by use of acids 10 years to life; imprisonment and fine
(paid to the victim).
• 326-B IPC: Voluntarily throwing or attempting to throw acid. Punishment: 5–7 years and a fine.
Option C - 327 IPC:
• 327 IPC: Voluntarily causing hurt to extort property or valuable security, or to constrain to do anything
which is illegal or which may facilitate the commission of an offence.

Solution for Question 7:


Correct Option D - IPC 269:
• IPC 269: Negligently doing any act known to be likely to spread infection of any disease dangerous to
life.

Incorrect Options:
Option A - IPC 166 B: IPC 166 B: "Punishment for" non-treatment of victim by the hospital.
Option B - IPC 202: IPC 202: Intentional omission to give information about an offence by a
person legally bound to inform.
Option C - IPC 203: IPC 203: Giving false information respecting an offence committed.

Page 10

35
Solution for Question 8:
Correct Option C - 498 A IPC:
• Section 498A: Husband or relative of the husband of a woman subjecting her to cruelty.

Incorrect Options:
Option A - 304 A IPC: Sec 304 A IPC: Death caused by rash and negligent act –
2 years or fine or both.
Option B - 304 B IPC: Sec 304 B IPC: Dowry death.
Option D - 320 IPC: S
320 IPC defines grievous hurt and lists only eight kinds of hurt that it labels as “grievous”.

Solution for Question 9:


Correct Option B - Punishable under 166 B IPC:
• Section166 B: Punishment for non-treatment of the victim.
Incorrect Options:
Option A - Not an offence:
• A private Doctor refusing to treat a case of sexual offence and referring to govt hospital is an offence.
• Section 166 B: Punishment for non-treatment of the victim.
Option C - Punishable under 357 (c) CrPC:
• CrPC 357 (c): Treatment of rape victims and information to the police.
Option D - Punishable under 327 (c) CrPC:
• CrPC 327: The place in which any criminal Court is held for the purpose of inquiring about any offence
shall be deemed to be an open court to which the public generally may have access.

Solution for Question 10:


Correct Option A - 174 IPC:
• S. 174 IPC: Not obeying a legal order to attend at a certain place in person or by agent, or departing
therefrom without authority. If the order requires personal attendance, etc., in a court of Justice.
• If the order requires personal attendance, etc., in a court of Justice.
• If the order requires personal attendance, etc., in a court of Justice.

Incorrect Options:

Page 11

36
Option B - 176 IPC: Omission to give notice or information to public servant by a
person legally bound to give it.
Option C - 178 IPC: Punishment for refusing oath.
Option D: 191 IPC: Giving false evidence.

Solution for Question 11:


Correct Option D - Act intended to insult the modesty:
• S. 509 IPC: Whoever intends to insult the modesty of any woman, utters any word, makes any sound
or gesture, exhibits any object, or intrudes upon the privacy of a such woman.
Incorrect Options:
Option A - Public nuisance under alcohol influence:
• It is an offence and punishable under Sec. 290 IPC: Punishment for public nuisance in cases not
otherwise provided for (fine of 200) and Sec. 291 IPC: Continuance of nuisance after injunction to
discontinue (imprisonment for 6 months and/or fine) for creating a public nuisance (Frotteurism
/Toucherism).
• 86 IPC: Offence caused after voluntary intoxication.
Option B - Inquest:
• An inquest is an Inquiry into the cause of unnatural death.
• S.174 CrPC: Police inquest (most common in India).
• S.176 CrPC: Magistrate inquest (best inquest).
Option C - Exhibitionism:
• It is a desire and intentional exposure of genitalia in public places while in presence of others (mostly
in front of unsuspecting children or females) to obtain sexual pleasure.
• Legal aspect: It is an obscene act punishable under Sec. 294 IPC with imprisonment up to 3 months
and/or a fine.

Solution for Question 12:


Correct Option C - 324:
• In the given condition injury is simple and is caused by a dangerous weapon, hence 324 IPC.
• 3 yrs imprisonment
Incorrect Options:
Option A - 302:
• 302 IPC: Punishment for murder – death or imprisonment for life.
Option B - 304:
• 304 IPC: Culpable homicide not amounting to murder. Imprisonment for life for 10 yrs, and also fine.

Page 12

37
• 304 A: Death caused by rash and negligence act- 2 yrs +/- fine.
• 304 B: Dowry death.
• Punishment - 7 yrs to Lifetime imprisonment
Option D - 326:
• 326 IPC: Punishment for grievous hurt by dangerous weapons- Imprisonment for life, or up to 10 yrs,
and fine.

Solution for Question 13:


Correct Option A - 39 CrPC:
• CrPC 39: Every person, aware of the Commission of certain offences shall give information to the
nearest Magistrate or police officer.
Incorrect Options:
Option B - 27 CrPC: CrPC 27: Jurisdiction in the case of juveniles.
Option C - 174 CrPC: CrPC 174: Police to inquire and report on suicide, etc. (Police inquest into the ca
use of death).
Option D 176 CrPC: CrPC 176: Inquiry by Magistrate into cause of death.

Solution for Question 14:


Correct Option A - Criminal responsibility of insane:
• Section 84 IPC: Criminal responsibility of insane.
• According to this section, nothing is an offence done by a person who is incapable of knowing that
what he is doing is wrong due to mental illness.
Incorrect Options:
Option B - Criminal responsibility of drunken person:
• Section 86 IPC: Criminal responsibility of voluntary intoxication
Option C - Criminal responsibility of intoxicated person:
• Section 85 IPC: Offence caused after involuntary intoxication
Option D - Criminal responsibility of child:
• Section 82 IPC: Nothing is an offence which is done by a child < 7 years of age.
• Section 83 IPC: Nothing is an offence which is done by a child between 7-12 years seven years of age
who has not attained sufficient maturity to judge the nature of the act.

Solution for Question 15:

Page 13

38
Correct Option A - S. 141 IEA:
• S.141 IEA: Definiton of Leading question.
Incorrect Options:
Option B - S.137 IEA: S.137 IEA: Examination in chief.
Option C - S.138 1EA: S.138 IEA: Order of trial in the court of the law.
Option D - S.165 IEA: S.165 IEA: Questions by Judge.

Solution for Question 16:


Correct Option A - Vicarious liability:
• Vicarious Liability/Respondent Superior: A doctrine that holds an employer legally responsible for the
wrongful acts of an employee, if such acts occur within the scope of the employment.
• An employer is responsible not only for his own negligent act but also for the negligent act of his
employees by the principle of ‘respondent superior’ or ‘Master-Servant Rule’.

Incorrect Options:
Option B - Res ipsa loquitur:
• Res ipsa loquitur (Latin for “the thing speaks for itself”).
• The very nature of an accident or injury itself proves the negligent action of the physician.
• Specialized knowledge or understanding of technical matters of the medical profession is not
required.
• The application of common knowledge is sufficient to understand negligence.
• Eg: Forceps inside the abdomen postoperatively.
Option C - Novus actus interveniens:
• Novus actus interveniens: "new act intervening" breaking the chain of causation such that even if the
defendant has acted negligently, a subsequent intervening action breaks the chain of causation with
the loss or damage sustained and so the defendant is not liable.
Option D - Contributory negligence:
• Contributory negligence: The plaintiff failed to take reasonable care of his own safety and this factor
contributed to the harm ultimately suffered by the plaintiff.
• The burden of proof lies with the defendant (doctor in this case).

Solution for Question 17:


Correct Option B - Contributory negligence:
• Contributory negligence: The plaintiff failed to take reasonable care of his safety and this factor
contributed to the harm ultimately suffered by the plaintiff.

Page 14

39
• The burden of proof lies with the defendant (doctor in this case).

Incorrect Options:
Option A - Vicarious liability:
• Vicarious Liability or Respondent Superior.
• A doctrine that holds an employer legally responsible for the wrongful acts of an employee, if such
acts occur within the scope of the employment.
Option C - Res ipsa loquitur:
• Res ipsa loquitur: "Things/facts which speak for itself"
• The very nature of an accident or injury itself proves the negligent action of the physician.
Option D - Novus actus interveniens:
• Novus actus interveniens: "New act intervening" breaking the chain of causation such that even if the
defendant has acted negligently, a subsequent intervening action breaks the chain of causation with
the loss or damage sustained and so the defendant is not liable.

Solution for Question 18:


Correct Option A - Cognizable & bailable:
• Stalking involves following a woman or contacting such a woman despite clear disinterest or
monitoring her through the internet, e-mail, or any other form of electronic communication.
• It is punishable under Sec. 354-D IPC with imprisonment for up to 3 years and fine (cognizable and
bailable) and imprisonment up to 5 years and fine for subsequent offence (cognizable and
non-bailable).

Incorrect Options:
Option B - Non-Cognizable & non-bailable:
• A non-cognizable offence is an offence listed under the first schedule of the Indian Penal Code and is
bailable in nature.
• But here, in this case, stalking a female is a cognizable offence.
Option C - Non-Cognizable & bailable:
• In the event of a non-cognizable offence, the police are prohibited from detaining the suspect without
a warrant and from opening an inquiry without the court's approval.
• But here, in this case, stalking a female is a cognizable offence.
Option D - Cognizable & non-bailable:
• Stalking a woman is punishable under Sec. 354-D IPC with imprisonment for up to 3 years and fine
(cognizable and bailable) and imprisonment up to 5 years and fine for subsequent offence (cognizable
and non-bailable).

Page 15

40
Solution for Question 19:
Correct Option D - Can be used by private doctors and ambulances.:
• The use of the emblem by Government medical institutions, like hospitals, clinics and blood banks,
doctors, private nursing homes and also on ambulance vehicles is equivalent to abuse and is
punishable with a fine of ` 500 and forfeiture of the goods or vehicles on which the emblem has been
used.
Incorrect Options:
Option A - Can be used by army medical services: The Red Cross emblem can be used only by memb
ers of army medical services.
Option B Punishable to use it without permission: Usage of the Red Cross emblem by doctors is Punis
hable with fine & forfeiture of Property.
Option C - Used by members of Red Cross: The Red Cross emblem is used only by those belonging to
the Red Cross Movement and Army Medical Services.

Solution for Question 20:


Option A: Res ipsa loquitur
• Res ipsa loquitur: Doctrine of common knowledge.
• The very nature of an accident or injury itself proves the negligent action of the physician.
Option B: Res judicata
• Res Judicata: A case adjudicated by a court, can not be pursued further by the same parties in
another court.
Option C: Respondent superior
• Vicarious Liability or Respondent Superior.
• A doctrine that holds an employer legally responsible for the wrongful acts of an employee, if such
acts occur within the scope of the employment.
Option D: Novus actus interveniens
• Novus actus interveniens: "new act intervening" breaking the chain of causation such that even if the
defendant has acted negligently, a subsequent intervening action breaks the chain of causation with
the loss or damage sustained and so the defendant is not liable.

Solution for Question 21:


Correct Option D - None of the above:
• In the case of civil negligence, the burden of proof is on the negligence; four elements including Duty,
Dereliction, Direct Causation and Damage must be present.
Incorrect Options:
Option A - Duty:

Page 16

41
• In the case of civil negligence against the doctor, the patient has to prove the duty of care by the
doctor.
• A duty of care by the doctor must exit.
Option B - Dereliction:
• Dereliction: The physician must maintain the standard of a prudent physician under similar
circumstances.
• If the physician fails to maintain the standard of care, it is called dereliction.
Option C - Damage:
• Damage: The negligent act must cause damage of a type that would have been foreseen by a
reasonable physician.

Solution for Question 22:


Correct Option A - Section 39 CrPC:
• If a private practitioner is convinced that the patient is suffering from homicidal poisoning, he is bound
under Sec. 39 CrPC to inform the police or Magistrate. Non-compliance is punishable under Sec. 176
IPC (simple imprisonment of 1 month or fine of 500/- or both). Giving false information on such matters
is punishable under Sec. 177 IPC (simple imprisonment for 6 months or fine of 1000 or both).
• Non-compliance is punishable under Sec. 176 IPC (simple imprisonment of 1 month or fine of 500/- or
both).
• Giving false information on such matters is punishable under Sec. 177 IPC (simple imprisonment for 6
months or fine of 1000 or both).
• Non-compliance is punishable under Sec. 176 IPC (simple imprisonment of 1 month or fine of 500/- or
both).
• Giving false information on such matters is punishable under Sec. 177 IPC (simple imprisonment for 6
months or fine of 1000 or both).

Incorrect Options:
Option B - Section 176 CrPC: In any case of death, a
magistrate may conduct an inquest instead of, or in addition to the police inquest
Option C - Section 175 CrPC: If the practitioner is summoned by the investigating officer (IO), he is bou
nd to give all information regarding the case that has come to his notice (Sec. 175 CrPC). If he conceal
s the information, he is liable to be prosecuted under Sec. 202 IPC (imprisonment upto 6 months or fin
e or both). If he gives false information during judicial proceedings, he is liable to be charged under Se
c. 193 IPC.
Option D - Section 41 CrPC: In non-cognisable offences, the injured person may go direct to the doctor
, or he may file an affidavit in the court of a magistrate who will send him to the doctor for examination

Page 17

42
Previous Year Questions
1. Who is responsible for handling the investigation in the case of a custodial death?
A. Police inspector
B. Jail superintendent
C. Superintendent of Police
D. Judicial magistrate
----------------------------------------
2. According to the law, nothing is an offense if a person is of an unsound mind and is not able to
understand that the nature and consequences of his/her act are wrong. Which rule of insanity applies in
this case?
A. Durham's rule
B. Mc Naughton's rule
C. Curren's rule
D. Irresistible impulse rule
----------------------------------------
3. The court summons a member of the public who witnessed the incident to provide testimony and
document their account. What kind of evidence does the witness offer to the court?
A. Oral evidence
B. Hearsay evidence
C. Subpoena
D. Circumstantial evidence
----------------------------------------
4. In which procedural route in the court of law would this type of question be allowed: "In your
presence, did A kill B?"
A. Examination-in-chief
B. Direct examination
C. Cross-examination
D. Re-direct examination
----------------------------------------
5. Cause of Opisthotonus is?
A. Cocaine
B. Codeine
C. Strychnine
D. Curare
----------------------------------------

43
6. the authorities to investigate the death of a 14-year-old girl who was abducted by a man and
subsequently died while in police custody.
A. Police
B. Coroner
C. Medical examiner
D. Judicial magistrate
----------------------------------------
7. Under which doctrine is the doctor not responsible when a patient with life-threatening injuries arrives
at the casualty without time for consent, and the doctor initiates life-saving procedures with utmost care,
but unfortunately, the patient still succumbs to death?
A. Res ipsa loquitor
B. Doctrine of anticipation
C. Doctrine of extended consent
D. Doctrine of conjugated consent
----------------------------------------
8. What is the penalty for causing the death of a patient due to the use of counterfeit medication?
A. 5 years
B. 7 years
C. Life imprisonment
D. 1 year
----------------------------------------
9. A surgeon returns home from a party after many pegs of alcohol and is called to perform an
emergency operation. During the operation, the assisting staff noticed the surgeon’s handshaking and
the instruments falling. He eventually nicks an artery, and the patient collapses. Under which of the
following terms will this incident be tried?
A. Criminal negligence
B. Civil negligence not amounting to criminal negligence
C. Therapeutic misadventure
D. Dichotomy
----------------------------------------
10. In relation to the punishment of perjury, which specific section of the IPC addresses this matter?
A. 191
B. 192
C. 193
D. 197
----------------------------------------
11. What should the doctor do when faced with an unconscious 19-year-old female patient who
presented to the emergency department, exhibiting signs of hydrocephalus on a non-contrast CT scan,

Page 2

44
and requiring immediate neurosurgical intervention, while also lacking any accompanying individual or
means of identification?
A. Do not operate without consent
B. Wait for arrival of relatives
C. Operate without consent
D. Take consent by calling the relatives
----------------------------------------
12. Which section of the Indian Penal Code (IPC) is associated with the penalization of perjury?
A. 191
B. 192
C. 193
D. 197
----------------------------------------
13. Which article in the constitution does not pertain to children?
A. 21A
B. 24
C. 45
D. 42
----------------------------------------
14. Please arrange the following clauses of IPC 320 in sequential order. Destruction or permanent
impairing of the power of any member or joint Permanent disfiguration of the head or face Emasculation
Fracture or dislocation of a bone or tooth
A. 1,2,3,4
B. 3,2,1,4
C. 3,1,2,4
D. 1,4,2,3
----------------------------------------
15. What type of consent is obtained when a 45-year-old female patient is informed about the
advantages and potential complications of a hysterectomy, and she gives her agreement for the
procedure?
A. Informed consent
B. Implied consent
C. Opt-out
D. Passive consent
----------------------------------------
16. In the court proceedings, the defense lawyer presents a leading question, to which the prosecutor
objects. The appeal made by the prosecutor is accepted by the judge. Leading questions are prohibited
in which of the following scenarios?

Page 3

45
A. Re-examination
B. Cross-examination
C. Examination in chief
D. Dying declaration
----------------------------------------
17. Which of the following statements is not applicable during cross-examination in a court of law?
A. Witness is examined by the defense lawyer
B. Leading questions are not allowed
C. Objective of cross-examination is to weaken case of opponent
D. There is no time limit to cross examination
----------------------------------------
18. In the scenario where an individual has perpetrated a criminal act and is apprehended by the police,
they are subsequently presented before a criminal court. Upon establishment of the accused's age as
being less than a certain threshold, the court instructs the police to transfer the case to the juvenile
courts.
(or)
In the scenario where an individual has perpetrated a criminal act and is apprehended by the police,
they are subsequently presented before a criminal court. Upon establishment of the accused's age as
being less than a certain threshold, the court instructs the police to transfer the case to the juvenile
courts
A. 21
B. 17
C. 20
D. 24
----------------------------------------
19. The first-hand knowledge rule is applicable to which of the following?
A. Common witness
B. Hand writing expert
C. Doctor
D. Hostile witness
----------------------------------------
20. When an individual commences employment with a second employer, the new employer assumes
responsibility for the individual's actions.
A. Vicarious liability
B. Doctrine of negligent choice
C. Contributory negligence
D. Borrowed servant doctrine
----------------------------------------

Page 4

46
21. If a healthcare professional is found guilty of professional misconduct, they must provide the
patient's records promptly upon request, within:
A. 72 hours
B. 48 hours
C. 24 hours
D. 7 days
----------------------------------------
22. What is the appropriate course of action following the observation of cigarette burn marks on the
forearm of a 14-year-old girl who presents to a doctor with a fractured forearm, claiming to have fallen
and hit her hand?
A. To inform higher authorities
B. To do a complete physical examination
C. To tell or discuss with colleagues that she is a case of abuse
D. To call local social worker for help
----------------------------------------
23. What is the responsibility of the doctor in recording the dying declaration of a victim affected by a
fire that destroyed six huts in a neighboring slum?
A. To assess compos mentis
B. To record statement even in presence of magistrate
C. Cross-examine the person
D. Put person under oath before declaration
----------------------------------------
24. Death registration in India done within?
A. 21 days
B. 28 days
C. 30 days
D. 40 days
----------------------------------------
25. Who will conduct the investigation in the case where a woman, who had been married for 4 years,
died and left a suicide note stating that she was subjected to dowry-related torture?
A. Police
B. Village headman
C. Judge
D. Magistrate
----------------------------------------
26. An 11 year old girl was found to be quiet and shy in her classroom with intermittent crying, for the
past few days. It was discovered that her uncle was touching her genitalis inappropriately. The duty of
the principal is to report this to ?

Page 5

47
A. Magistrate
B. Police
C. Child welfare committee
D. Parents
----------------------------------------
27. According to the amendment in the medical termination of pregnancy (MTP) Bill of 2020, until what
gestational age is it permissible to induce MTP in mentally ill rape victims?
A. 20 weeks
B. 22 weeks
C. 24 weeks
D. 28 weeks
----------------------------------------
28. As an internal medicine consultant at a private clinic, you are handling the case of a patient who has
chronic renal disease and needs hemodialysis. You provide dietary and fluid management
recommendations and closely monitor the patient. Unfortunately, the patient decides to take ayurvedic
supplements and occasionally disregards your instructions without informing you. Consequently, the
patient's condition deteriorates, leading to respiratory distress. Subsequently, the patient's attendant
files a legal complaint, accusing the doctor of negligence. Which of the following can serve as a
defense for the doctor in court?
A. Vicarious liability
B. Res Ipsa loquitur
C. Therapeutic misadventure
D. Contributory negligence
----------------------------------------
29. In a civil negligence case against a doctor, who is responsible for providing evidence?
A. Patient
B. Police not under rank of sub-inspector
C. Magistrate
D. Doctor
----------------------------------------
30. What is the term used to describe the unethical practice wherein a doctor receives payment in
exchange for referring a patient to a radiologist for a CT scan?
A. Dichotomy
B. Medical maloccurence
C. Criminal negligence
D. Commission
----------------------------------------

Page 6

48
31. A surgeon performs a hysterectomy for uterine fibroids after obtaining relevant informed consent
from the patient. Despite adequate precaution, the ureter was injured intraoperatively. Under which of
the following principles will the doctor not be held responsible for this act ?
A. Novus actus interveniens
B. Medical maloccurrence
C. Res ipsa loquitur
D. Physician error
----------------------------------------
32. A 16-year-old girl was rescued from a village in Bihar where she had been sold off by her father for
Rs. 25000 to a 45-year-old man. On examination, she was found to be 12 weeks pregnant. She was
granted permission to abort the pregnancy, taking into consideration the age of the girl and the
circumstances of her pregnancy. A doctor who will be eligible to perform the abortion for this girl should
have performed at least how many MTP prior to this?
A. 10
B. 15
C. 25
D. 35
----------------------------------------
33. Which section of the Indian Penal Code (IPC) pertains to cases involving medical negligence?
A. Sec 301
B. Sec 304 B
C. Sec 304 A
D. Sec 302
----------------------------------------

Correct Answers
Question Correct Answer

Question 1 4
Question 2 2
Question 3 1
Question 4 3
Question 5 3
Question 6 4
Question 7 2
Question 8 3
Question 9 1
Question 10 3

Page 7

49
Question 11 3
Question 12 3
Question 13 4
Question 14 3
Question 15 1
Question 16 2
Question 17 2
Question 18 2
Question 19 1
Question 20 4
Question 21 1
Question 22 2
Question 23 1
Question 24 1
Question 25 4
Question 26 2
Question 27 3
Question 28 4
Question 29 1
Question 30 1
Question 31 2
Question 32 3
Question 33 3

Solution for Question 1:


Correct answer Option D - Judicial magistrate
• Section 176 (I) of the Code of Criminal Procedure (CrPC) states that if a person in custody dies or
disappears or a woman is raped in custody, the Judicial Magistrate (Executive Magistrate) has the
power to order an inquiry.
• Conditions for magistrate inquest: A – Death in Asylum B – Death in Borstals C – Custodial
death/rape D – Dowry death E – Exhumation
• A – Death in Asylum
• B – Death in Borstals
• C – Custodial death/rape
• D – Dowry death
• E – Exhumation

Page 8

50
• A – Death in Asylum
• B – Death in Borstals
• C – Custodial death/rape
• D – Dowry death
• E – Exhumation
Other options are incorrect

Solution for Question 2:


• McNaughton’s rules state that “Nothing is an offense done by a person who is incapable of knowing
that he is doing is wrong due to mental illness”.
Incorrect Options:
• Option a. Durham’s Rule states that a criminal cannot be convicted of a crime if the act was a result of
a mental disease or defect the criminal had at the time of the incident.
• Option c. Curren’s Rule “an accused person will not be criminally responsible if, at the time of
committing the act, he did not have the capacity to regulate his conduct to the requirements of law, as a
result of mental disease or defect.”
• Option d. According to the Irresistible Impulse rule in jurisdictions defendants typically must present
sufficient evidence to prove the existence of mental illness and that the mental illness caused the
inability to control one's actions or conform one's conduct to the law.

Solution for Question 3:


ANSWER OPTION A
• “All statements which the court permits or requires to be made before it by a witness, in relation to
matters of fact under inquiry, such statements are called as oral evidence”. The word oral in itself
describes its meaning as something spoken or expressed by mouth. Hence anything which is spoken
or expressed by the mouth of a witness who is called for trial in the Court is referred to as oral
evidence.
Incorrect options:
• Option b. Hearsay evidence refers to a testimony that is provided by a witness and is not based on
any personal communication. It could be something that the witness overheard or was told about from a
third party, making it second-hand information.
• Option c. Subpoena is a formal written order that requires a person to appear before a court, or other
legal proceedings and testify, or produce documentation.
• Option d. Circumstantial evidence Circumstantial evidence is evidence of facts that the court can draw
conclusions from. For example, if an assault happened on O'Connell Street at 6.15pm, you can give
evidence that you saw the accused walking down O'Connell Street at 6pm. In that situation, you are
giving the court circumstantial evidence.

Page 9

51
Solution for Question 4:
Correct Option C:
Cross-examination is the correct procedural route in the court of law for this type of question. During cr
oss-examination, the opposing lawyer has the opportunity to question the witness to test their credibilit
y, challenge their statements, and uncover any inconsistencies or biases. The question "Did A kill B in
your presence?" seeks to obtain crucial information about the incident directly from the witness, allowin
g the opposing lawyer to challenge or clarify the witness's account of the events. Cross-examination pl
ays a critical role in the adversarial system, allowing both sides to present their case and test the evide
nce presented by the opposing party.
Incorrect options:
Option A. Examination-in-chief: Examination-in-chief refers to the questioning of a witness by the lawye
r who called them to testify. This phase is typically used to elicit direct evidence and establish the witne
ss's version of events.
Option B. Direct examination: Direct examination is similar to examination-in-chief and involves the law
yer questioning their own witness to present evidence and establish their case. Like examination-in-chi
ef, it aims to elicit direct evidence and the witness's version of events.
Option D. Re-direct examination: Re-direct examination occurs after cross-examination and allows the l
awyer to clarify or rebut issues that arose during cross-examination. It is typically limited to matters rais
ed during cross-examination.

Solution for Question 5:


Correct option: C Strychnine
• Alakloid from seeds of strychnosnux-vomica causes opisthotonus.
• The convulsions are most marked in antigravity muscles, so that the body typically arches in
hyperextension.
Other options are incorrect

Solution for Question 6:


Correct Option: D
The inquest in the given scenario would typically be conducted by a Judicial magistrate.
Explanation of the options:
Option A: Police: The police are responsible for investigating crimes, apprehending suspects, and main
taining law and order. While they may be involved in gathering evidence and providing information duri
ng the inquest, they are not typically responsible for conducting the inquest itself.

Page 10

52
Option B: Coroner: A Coroner is a government official, often a medical professional or lawyer, who is re
sponsible for investigating deaths that are sudden, unexpected, or suspicious. They determine the cau
se and circumstances of death and may hold an inquest to gather evidence and information related to t
he death.
Option C: Medical examiner: A Medical Examiner is a forensic pathologist or medical doctor who perfor
ms autopsies and examines the bodies of deceased individuals to determine the cause and manner of
death. They may provide valuable information and evidence during an inquest, but they are not typicall
y responsible for conducting the inquest itself.
Option A: In some jurisdictions, a judicial magistrate may indeed be responsible for conducting an inqu
est. The role and responsibilities of a
judicial magistrate can vary based on the legal system and country.
In certain legal systems, a judicial magistrate may be appointed to oversee specific cases and conduct
inquiries into deaths that occur under certain circumstances, such as deaths in police custody. They m
ay have the authority to gather evidence, question witnesses, and determine the cause and circumstan
ces of the death.

Solution for Question 7:


Correct Option B:
Doctrine of anticipation: This doctrine allows a
physician to take necessary action without explicit consent in emergency situations where there is a hig
h risk of harm or death to the patient. It acknowledges that in life-threatening situations, obtaining form
al consent may not be possible, and immediate intervention is required to preserve the patient's life. In
this case, the doctor acted in accordance with the doctrine of anticipation by initiating life-saving proced
ures without consent.
Incorrect Options:
Option A: Res ipsa loquitur: This doctrine holds that the occurrence of an accident implies negligence.
However, in the given situation, the doctor was faced with a life-threatening emergency where immedia
te action was necessary to save the patient's life. The doctor acted responsibly and with all precautions
, despite the unfortunate outcome. Therefore, the doctrine of Res ipsa loquitur would not hold the docto
r responsible.
Option C: Doctrine of extended consent: This doctrine pertains to situations where additional procedure
s or interventions beyond the immediate emergency may be required, and consent must be obtained fo
r those procedures. It is not directly applicable to the given scenario, as the doctor was focused on add
ressing the life-threatening injuries and preserving the patient's life.
Option D: Doctrine of conjugated consent: This is not a
recognized legal doctrine in the context of medical practice. It is not applicable to the given scenario.

Solution for Question 8:


The punishment for the death of a patient by spurious drug:
Life imprisonment:

Page 11

53
The death of a patient caused by the administration of a spurious drug is a serious offense. In many jur
isdictions, the punishment for such an act is life imprisonment. This reflects the gravity of the crime and
the potential harm caused by the use of counterfeit or adulterated drugs, which can have severe cons
equences for the health and lives of individuals. It serves as a deterrent and emphasizes the importanc
e of ensuring the safety and quality of drugs in healthcare systems.

Solution for Question 9:


Correct Option A.
• The incident depicted within the question is an example of criminal negligence, which refers to a
disregard for the safety of others that constitutes a criminal act.
• In this case, the surgeon was under the impact of liquor, which impaired his capacity to perform the
operation securely, and he eventually caused hurt to the patient.
• The helping staff took note of the surgeon’s hand shaking and instruments falling, showing that he
was not in a fit state to perform the operation.
• Criminal negligence includes a breach of the duty of care owed to the patient and can result in
criminal charges being brought against the responsible party.
Incorrect Options:
Option B:
• Civil negligence not producing criminal negligence: Civil negligence refers to a breach of the duty of
care owed to a patient or other party that comes about in harm or damage.
• Simple lack of skills and care while treating the patient, resulting in damage to the patient is known as
civil negligence. Eg:doctor has given the wrong prescription. Doctor has given wrong doses. Doctor has
given wrong information.
• Eg:doctor has given the wrong prescription.
• Doctor has given wrong doses.
• Doctor has given wrong information.
• These cases are tried under civil or consumer courts.
• However, if the breach of duty is so extreme that it constitutes a criminal act, the occurrence would be
attempted as criminal negligence, not civil negligence.
• Eg:doctor has given the wrong prescription.
• Doctor has given wrong doses.
• Doctor has given wrong information.
Option C. Therapeutic misadventure:
• This term refers to an antagonistic outcome or harm during medical treatment if the treatment was
given with due care and consideration.
• It does not necessarily imply any negligence on the part of the healthcare provider.
Option D. Dichotomy:
• This term refers to a division or contrast between two things that are or are spoken to as being
restricted or completely distinctive.

Page 12

54
• It isn't significant to the incident portrayed within the question.

Solution for Question 10:


Correct Option: C.
• Section 193 of the IPC is titled "Punishment for false evidence." It states that whoever intentionally
gives false evidence in any stage of a judicial proceeding, or fabricates false evidence for the purpose
of being used in a judicial proceeding, shall be punished with imprisonment of either description for a
term that may extend to seven years, and shall also be liable to pay a fine.
Incorrect options:
Option A. Section 191: This section deals with the punishment for giving false evidence. It states that w
hoever, being legally bound by an oath or affirmation to state the truth, makes a
false statement on any subject, shall be punished with imprisonment of either description for a
term which may extend to three years, or with a fine, or with both.
Option B. Section 192: This section deals with the punishment for fabricating false evidence. It states t
hat whoever causes any circumstance to exist or makes any false entry in any book or record, with the
intention of causing it to be believed that such circumstance exists or such entry is genuine, shall be pu
nished with imprisonment of either description for a
term which may extend to seven years, and shall also be liable to pay a fine.
Option D. Section 197: This section deals with the punishment for giving false certificates or false infor
mation. It states that whoever issues or signs any certificate required by law to be given or signed, or gi
ves any false information with the intention of causing injury to any person, shall be punished with impri
sonment of either description for a term which may extend to six months, or with a fine, or with both.

Solution for Question 11:


Correct Choice: C
Explanation:
• Since, the patient requires urgent medical intervention and she is unconscious. Also, she has no
relatives accompanying her, so their consents couldn’t be taken in this case scenario. She might die if
the doctor waits for her relatives to come and give consent.
• In such a scenario (emergency), section 92 IPC would be taken into consideration.
• 92 IPC states that: Nothing is an offence which is done in good faith and for the benefit of person (or)
the guardian in a situation when taking the consent is not possible (like in emergencies)
Incorrect choices:
Option A. Patient was presented with an emergency requiring urgent neurological intervention. Till the r
elatives are informed and they come to give consent, the patient may acquire some irreversible damag
es and may also lead to the death of the patient. Hence, the correct step would be to operate without c
onsent.
Option B. If the doctor waits for the relatives to come to give consent, some irreversible damages to the
patient might occur and the condition might become life-threatening.

Page 13

55
Option D. Taking consent by calling the relatives might be a good step, but since the patient is in an e
mergency situation, section 92 IPC permits the doctor to operate without consent. Hence, the best opti
on here should be option c

Solution for Question 12:


The correct option is Option C: Section 193 of the Indian Penal Code (IPC).
Option C: Section 193- Section 193 of the IPC deals with the offense of giving false evidence in a
judicial proceeding or fabricating false evidence for the purpose of being used in a
judicial proceeding. It specifies that whoever intentionally gives false evidence in any stage of a
judicial proceeding, or fabricates false evidence intending it to be used in a
judicial proceeding, shall be punished with imprisonment and/or a fine. This section specifically addres
ses the punishment of perjury, which involves knowingly making false statements while under oath duri
ng a court proceeding.
Incorrect options:
Option A: Section 191 -Section 191 of the IPC deals with the offense of giving false evidence. It pertain
s to cases where a person gives false evidence either during any stage of a
judicial proceeding or during any inquiry conducted by a public servant authorized to hold such inquiry.
However, Section 191 does not specifically deal with the punishment of perjury.
Option B: Section 192- Section 192 of the IPC relates to the offense of fabricating false evidence. It ap
plies to situations where a person causes the existence of false evidence to be known with the intent to
cause it to be believed as true. While this offense is related to the act of creating false evidence, it doe
s not specifically address the punishment of perjury.
Option D: Section 197 - Under Section 197, if a person in an official capacity, such as a
public servant or a person authorized by a public servant, knowingly issues or signs a
false certificate, they can be punished with imprisonment and/or a
fine. The false certificate referred to in this section can be in various forms, such as a false birth certific
ate, false educational certificate, or any other document that provides false information or verification.

Solution for Question 13:


Correct Option D: 42
• 42," is the correct answer as it is not directly related to children.
• 42: Article 42 of the Indian Constitution focuses on the provision of just and humane conditions of
work and maternity relief.
Correct Options
Option A - Article 21A: Article 21A of the Indian Constitution relates to the Right to Education. It states t
hat the State shall provide free and compulsory education to all children in the age group of 6 to 14 yea
rs. This article specifically addresses the rights of children and ensures their access to education.
Option B - Article 24: Article 24 of the Indian Constitution deals with the prohibition of child labor. It stat
es that no child below the age of 14 years shall be employed in any hazardous occupation or engaged

Page 14

56
in any form of forced labor. This article is directly related to the protection of children's rights and their
well-being.
Option C - Article 45: Article 45 of the Indian Constitution pertains to the provision of early childhood ca
re and education for children below the age of six years. It emphasizes the importance of early childho
od education and states that the State shall endeavor to provide free and compulsory education for all
children until they complete the age of six years.

Solution for Question 14:


Correct Answer: C
• Section 320 of the Indian Penal Code (IPC) deals with grievous hurt. The sequential arrangement of
the clauses is as follows: Emasculation: This clause refers to the act of castrating or removing the
testicles. It is considered a severe form of injury. Destruction or permanent impairing of the power of
any member or joint: This clause refers to causing significant damage or permanent impairment to any
limb or joint, rendering it non-functional or severely impaired. Permanent disfiguration of the head or
face: This clause refers to causing permanent disfigurement to the head or face, resulting in a
significant alteration in appearance. Fracture or dislocation of a bone or tooth: This clause refers to
causing a fracture or dislocation of a bone or tooth.
• Emasculation: This clause refers to the act of castrating or removing the testicles. It is considered a
severe form of injury.
• Destruction or permanent impairing of the power of any member or joint: This clause refers to causing
significant damage or permanent impairment to any limb or joint, rendering it non-functional or severely
impaired.
• Permanent disfiguration of the head or face: This clause refers to causing permanent disfigurement to
the head or face, resulting in a significant alteration in appearance.
• Fracture or dislocation of a bone or tooth: This clause refers to causing a fracture or dislocation of a
bone or tooth.
• Emasculation: This clause refers to the act of castrating or removing the testicles. It is considered a
severe form of injury.
• Destruction or permanent impairing of the power of any member or joint: This clause refers to causing
significant damage or permanent impairment to any limb or joint, rendering it non-functional or severely
impaired.
• Permanent disfiguration of the head or face: This clause refers to causing permanent disfigurement to
the head or face, resulting in a significant alteration in appearance.
• Fracture or dislocation of a bone or tooth: This clause refers to causing a fracture or dislocation of a
bone or tooth.
Incorrect option:
Option A (1, 2, 3, 4): This option is incorrect because it does not follow the correct sequential arrangem
ent. It starts with clause 1 instead of clause 3.
Option B (3, 2, 1, 4): This option is incorrect because it reverses the order of clauses 1
and 2. The correct order is 1, 2, and not 2, 1.
Option D (1, 4, 2, 3): This option is incorrect because it swaps the positions of clauses 2
and 4. The correct order is 2, 4, and not 4, 2.

Page 15

57
Solution for Question 15:
Correct Option A:
• Informed consent refers to an intentional assertion made by a patient to experience a medical or
surgical treatment or procedure after being provided with data about the proposed treatment's benefits,
dangers, and options.
• Informed consent requires that the persistent is competent to form the choice, has been given
adequate data to form the choice, and has unreservedly given their assent without constraint or undue
influence.
Incorrect Options:
Option B. Implied consent:
• Consent is given by means of a gesture/ an act .
• It is when a patient's activities recommend that they have consented to a treatment or method if they
have not given express verbal or written consent.
• This type of assent is typically utilized in emergencies or when the understanding is incapable of
supplying assent.
• Within the scenario given, the patient was given data about the strategy and agreed to undergo it, so it
isn't
Option C: Opt-out:
• Opt-out assent is consent that assumes a patient has agreed to a treatment or strategy unless they
expressly decay it.
• This consent is regularly utilized for scheduled medical strategies such as blood tests or
immunizations.
• Within the situation given, the patient was given information about the procedure and agreed to
experience it, so it isn't opt-out consent.
Option D: Passive consent
• It is when a patient does not effectively give consent, but their silence or need of complaint is taken as
implied consent.
• This consent is ordinarily utilized for routine therapeutic procedures such as school vaccinations. In
the situation given, the understanding effectively gives consent after being given data about the
procedure, so it isn't passive consent.

Solution for Question 16:


Correct Option B.
• Leading question is a question where the answer is yes or no.
• Leading questions are asked only in cross examination
Incorrect Options:

Page 16

58
Option A. Re-examination:
• Re-examination is where the lawyer who called the witness for examination in chief can inquire further
questions to clarify any focuses raised amid cross-examination.
• Leading questions are generally not permitted amid re-examination, so this alternative isn't correct.
Option C. Examination in chief:
• Examination in chief is when the lawyer who called the witness questions them to inspire their
evidence in chief.
• Leading questions are by and large not permitted during examination in chief, so this option isn't
correct.
Option D. Dying declaration:
• Given by 32(1) IEA
• It is Written/ oral statement made by a dying person related to the crime.
• Dying declarations are for the most part acceptable in court and leading questions may be permitted
in case the individual making the explanation is incapable of talking clearly.
• Anyone of the following can record Dying declaration Victim (if possible) Magistrate Doctor Police
Public
• Victim (if possible)
• Magistrate
• Doctor
• Police
• Public
• Recording is done in the presence of 2 witnesses
• However, Validity decreases in the above order
• Procedures for recording dying declaration Oath is not needed Leading questions are not permitted
Verbatim (has to be recorded word by word)
• Oath is not needed
• Leading questions are not permitted
• Verbatim (has to be recorded word by word)
• Dying declaration is invalid if: Victim is not in composed mentis state If patient survives after dying
declaration
• Victim is not in composed mentis state
• If patient survives after dying declaration
• Incomplete dying declaration is also valid
• In any case, this choice is incorrect since the question particularly inquires which choices do not
permit leading questions.
• Victim (if possible)
• Magistrate
• Doctor

Page 17

59
• Police
• Public
• Oath is not needed
• Leading questions are not permitted
• Verbatim (has to be recorded word by word)
• Victim is not in composed mentis state
• If patient survives after dying declaration

Solution for Question 17:


• Leading questions are not allowed is an incorrect statement regarding cross-examination in a court of
law. In fact, leading questions are often used during cross-examination to elicit specific information or to
challenge the witness's testimony,under section 143 IEA. Therefore, the statement "Leading questions
are not allowed" does not apply during cross-examination in a court of law.
Incorrect Option:
• Option a: That statement applies during cross-examination in a court of law. The defense lawyer has
the right to cross-examine the witness in order to test their credibility and the reliability of their
testimony.
• Option c: The objective of cross-examination is to challenge the credibility, accuracy, and
completeness of the witness's testimony, and to weaken the case presented by the opposing party. The
cross-examining lawyer aims to extract information that may undermine the witness's testimony or
support their own case.
• Option d: The specific time limits may vary depending on the jurisdiction and the specific rules of the
court, but they are generally in place to ensure that the trial proceeds in an efficient and timely manner.

Solution for Question 18:


Correct Choice. B
Explanation:
• To transfer the case to juvenile court, the person has to be a juvenile. Any person, male or female,
who is below 18 years of age is a juvenile (Age: <18 years). Hence, the correct option here would be
option b - 17 years of age.
• Any crime or an offence done by a juvenile would be tried in the juvenile justice board.
Incorrect choices:
• All the choices except option b is more than juvenile age. Hence, they are incorrect.

Page 18

60
Solution for Question 19:
Correct Option : A
• The first-hand knowledge rule, also known as the firsthand or personal knowledge rule, is applicable
to a common witness. A common witness is an individual who has observed or experienced an event
directly and can provide testimony based on their own personal knowledge of the event.
• The first-hand knowledge rule requires that a witness's testimony be based on facts or information
that they themselves have perceived through their own senses. In other words, the witness must have
directly witnessed or experienced the event in question in order to provide reliable and credible
testimony.
• For example, in a legal trial, if a person was present at the scene of a crime and witnessed the crime
taking place, they would be considered a common witness. They would be able to provide firsthand
knowledge and testify about what they saw, heard, or experienced during the incident.
Incorrect options:
Option B. Handwriting expert: A handwriting expert is not subject to the first-hand knowledge rule beca
use their expertise is based on analyzing and comparing handwriting samples. They rely on their speci
alized knowledge and training rather than personal observation or direct experience of the events relat
ed to the handwriting.
Option C. Doctor: A doctor may provide expert testimony based on their professional knowledge and e
xperience rather than personal observation or direct experience of a specific event. Their testimony is u
sually based on medical records, test results, and their interpretation of the patient's condition.
Option D. Hostile witness: A hostile witness is a witness who is unwilling or uncooperative in providing t
estimony. While their credibility and reliability may be questioned, the first-hand knowledge rule is not s
pecifically applicable to hostile witnesses. The rule applies to the personal knowledge or direct observa
tion of any witness, regardless of their attitude or cooperation during the proceedings.

Solution for Question 20:


Correct Option: D
• When a person starts working for a second employer, the new employer may assume responsibility
for their actions under the Borrowed Servant Doctrine. This doctrine applies when an employee is
temporarily loaned or "borrowed" by another employer. In such cases, the borrowing employer
assumes control over the borrowed employee and becomes responsible for their actions within the
scope of their employment during the loaned period.
Incorrect Options:
Option A. Vicarious liability: Vicarious liability refers to a situation where one person or entity is held res
ponsible for the actions of another person or entity. While it can apply in cases where an employer is h
eld liable for the actions of its employees, it does not specifically address the scenario of a
person starting to serve a second employer.
Option B. Doctrine of negligent choice: The Doctrine of negligent choice refers to a
legal principle that holds a party liable for choosing a negligent individual or entity to perform a
task. It typically applies to cases where a
party knowingly selects an incompetent or unqualified person to perform a
specific job. It is not directly applicable to the situation of a person starting to work for a
second employer.

Page 19

61
Option C. Contributory negligence: Contributory negligence is a
legal concept that assigns partial blame to a plaintiff for their own injuries or damages due to their failur
e to exercise reasonable care. It is not directly relevant to the actions of the new employer when a
person starts working for them.

Solution for Question 21:


Correct Option: A
• In cases of professional misconduct, when a patient requests their medical records, the healthcare
provider is obligated to provide the records within a specific timeframe. The correct timeframe for
providing patient records on demand in the case of professional misconduct is typically within 72 hours
Incorrect Options:
Option B. 48 hours: This choice is incorrect because the timeframe of 48 hours is not the standard requ
irement for providing patient records in cases of professional misconduct. It is important to provide the r
ecords within the correct timeframe to ensure timely access to relevant information for the patient.
Option C. 24 hours: This choice is incorrect as well because the timeframe of 24 hours is not the stand
ard requirement for providing patient records in cases of professional misconduct. While it is crucial to r
espond promptly to patient requests, the standard timeframe for providing records in such cases is typi
cally longer than 24 hours.
Option D. 7 days: This choice is incorrect because the timeframe of 7 days is not the standard require
ment for providing patient records in cases of professional misconduct. Waiting for 7 days may lead to
unnecessary delays and hinder the patient's ability to address the misconduct or access relevant infor
mation in a timely manner.

Solution for Question 22:


Correct Option: B
• When a 14-year-old girl presents with a fractured forearm and reports that she fell and hit her hand, it
is essential for the doctor to conduct a thorough physical examination to assess the extent of the injury
and evaluate any associated findings.
Incorrect Option:
Option A: To inform higher authorities: While it is important to address suspicions of abuse, in this case
, there is no explicit indication or immediate evidence of abuse beyond the presence of the fractured for
earm and cigarette burn marks. Before involving higher authorities, it is crucial to gather more informati
on through a comprehensive examination.
Option C: To tell or discuss with colleagues that she is a case of abuse: While the presence of cigarett
e burn marks raises suspicions of possible abuse, it is important to gather more information and eviden
ce before concluding that abuse is involved. Discussing the case with colleagues can provide additiona
l insights, but it is necessary to gather more information through a
comprehensive examination before making any definitive conclusions.
Option D: To call a local social worker for help: In cases where abuse is suspected, involving a social w
orker is often necessary to provide support and intervention. However, it is crucial to gather more infor
mation through a

Page 20

62
thorough physical examination and, if needed, reporting to higher authorities before contacting a
social worker

Solution for Question 23:


Correct Option: A
• When a dying declaration is being recorded, the role of the doctor is to assess the mental capacity of
the person making the statement, also known as "compos mentis."
• The doctor evaluates whether the person is of sound mind and capable of providing a coherent and
reliable statement.
• This assessment helps determine the admissibility and credibility of the dying declaration as evidence
in legal proceedings.
Incorrect options:
Option B. To record the statement even in the presence of a
magistrate: It is not the role of the doctor to record the statement in the presence of a
magistrate. The presence of a magistrate during the recording of a
dying declaration may be required depending on the jurisdiction and legal procedures, but it is a
separate role usually fulfilled by a judicial or executive magistrate.
Option C. Cross-examine the person: Cross-examination is a legal procedure conducted by legal profe
ssionals, such as lawyers or prosecutors, to question and challenge the statements made by a
witness or a person providing a
declaration. It is not the role of the doctor to cross-examine the person giving the dying declaration.
Option D. Put the person under oath before declaration: While it is common practice to administer an o
ath or affirmation to ensure truthfulness during various legal proceedings, the doctor's role in recording
a dying declaration does not involve putting the person under oath. The doctor's primary responsibility i
s to assess the person's mental capacity to provide a
reliable statement based on their medical expertise

Solution for Question 24:


Correct option A
• In India, death registration is required to be done within 21 days (Option A) of the occurrence of the
death. The Registrar of Births and Deaths, usually located at the local municipality or panchayat, is
responsible for recording deaths and issuing death certificates.
Incorrect options
Option B: 28 days, Option C: 30 days, Option D: 40 days: These options do not correspond to the pres
cribed time frame for death registration in India, which is 21 days.

Page 21

63
Solution for Question 25:
Correct Option D: Magistrate
• Inquest is a procedure defined under CrPC, in which certain investigations are carried out in cases of
death.
• Inquest is usually done either by police or the magistrate.
• Usually, the death of a married lady within 7 years of marriage is presumed to be a case of dowry
death.
• In this case, even though it is not mentioned that she was tortured for dowry forcing her to suicide, it is
still assumed a dowry death.
• Death/suicide from torture/ requesting or pressures of dowry is investigated (inquest) by an executive
magistrate.
Incorrect Options:
Option A: Police would have done the inquest if death occurred from unnatural causes like suicide, mur
der, homicide, etc. If the marriage was more than 7 years old when suicide happened, a
case of dowry death would not have been assumed and police would’ve made the inquest report.
Option B: A village headman does not carry out magistrate inquests.
Option C: A judge does not carry out an inquest in the above scenario. Certain classes of judges carry
out inquests in cases of death under judicial custody.

Solution for Question 26:


Correct Ans: B
• Police: Reporting the incident to the police is crucial because they have the authority and
responsibility to investigate cases of child abuse, including inappropriate touching or sexual abuse. The
police have the resources and expertise to handle such cases, gather evidence, and take necessary
legal action against the perpetrator. By involving the police, the principal ensures that a formal
investigation is initiated, and that the child's safety is prioritized.
Incorrect option:
Option A. Magistrate: While a magistrate or judge may be involved in the legal proceedings related to t
he case, the immediate responsibility of the principal is to report the incident to the police. The police ar
e the appropriate authority to handle the initial investigation and take immediate action to protect the ch
ild.
Option C. Child welfare committee: Child welfare committees play a crucial role in child protection, but t
he immediate action in cases of abuse is to report to the police. The police will work in collaboration wit
h child welfare committees and other relevant authorities to ensure the child's safety and provide neces
sary support.
Option D. Parents: While it is important to inform the parents or guardians of the child about the inciden
t, the primary responsibility of the principal in such cases is to report to the police. The involvement of t
he police is necessary to initiate an official investigation and take appropriate legal action to protect the
child.

Page 22

64
Solution for Question 27:
Answer option C- 24 weeks
• As per the Medical Termination of Pregnancy (Amendment) Bill, 2020, the limit for termination of
pregnancy in a mentally ill woman or a woman who is a survivor of rape has been increased from 20
weeks to 24 weeks of gestation.
Incorrect Option:
• Option a: 20 weeks -The Medical Termination of Pregnancy (Amendment) Bill, 2020, which was
passed by the Indian Parliament, has increased the gestational limit for termination of pregnancy in
certain cases from 20 weeks to 24 weeks.
• Option b: 22 weeks - As per the Medical Termination of Pregnancy (Amendment) Bill, 2020, the
gestational age limit for termination of pregnancy in cases of rape, incest, and vulnerable women,
including mentally ill women, has been increased from 20 weeks to 24 weeks.
• Option d: Beyond 24 weeks, termination of pregnancy is only allowed in cases of substantial fetal
abnormalities or if the continuation of the pregnancy poses a risk to the life of the mother.

Solution for Question 28:


• Contributory negligence can be used as a defense in this situation, as the patient failed to follow the
doctor's instructions and chose to take alternative therapies without disclosing it. This contributed to his
worsening condition and difficulty in breathing. Contributory negligence refers to the patient's own
actions or negligence that contributed to their injuries, and it can reduce the liability of the doctor or
healthcare provider in a legal claim.
Incorrect Option:
• Option a: Vicarious liability refers to the legal responsibility of a person in employee-employer
contract. There are two people: Employer & Employee. The employer is responsible for his negligence
as well as the negligence of the employee. Eg: You are HOD & surgeon of the department and under
power, you are doing surgery and some other doctor causes negligence. The surgeon is also
responsible.
• Option b: Res Ipsa loquitur is a legal doctrine that allows a plaintiff to establish a presumption of
negligence on the part of the defendant based on the facts of the case. In this things/facts which speak
for itself
Ex- You left a swab in the body during surgery. This swab is a
fact in the body. There was an act of omission and there was a lack of standard of care & treatment. ·
It proves 100% negligence. · It proves both civil as well as criminal negligence.
• Option c: Therapeutic misadventure is a term used to describe an unintended outcome of a medical
treatment or procedure that results in harm to the patient.

Solution for Question 29:


Correct Option: A

Page 23

65
• In a civil negligence case against a doctor, the burden of proof lies with the patient. The patient is the
one making the claim against the doctor and alleging negligence. Therefore, it is the patient's
responsibility to provide evidence and prove that the doctor's actions or omissions fell below the
standard of care expected, resulting in harm or injury.
Incorrect options:
Option B. Police not under the rank of sub-inspector: The police are not directly involved in civil neglige
nce cases against doctors. Their role primarily pertains to criminal cases and maintaining law and orde
r, rather than determining liability in civil cases.
Option C. Magistrate: Magistrates are judicial officers who preside over legal proceedings and make de
cisions based on the evidence presented. However, in civil negligence cases against doctors, the burd
en of proof does not lie with the magistrate. Their role is to oversee the proceedings and apply the rele
vant laws.
Option D. Doctor: The doctor is the defendant in a civil negligence case, meaning they are the party bei
ng accused of negligence. It is not the doctor's burden to prove their innocence but rather the patient's
burden to prove the doctor's negligence.

Solution for Question 30:


Correct option A
• The dichotomous method, in which one doctor refers a patient to another for a fee.
Option B. Medical Maloccurence: The phrase "medical maloccurence" refers to any instances of a doct
or's behaviour going wrong that have an impact on their relationships, talents, and professional abilities
.
Option C. Criminal negligence: Criminal negligence occurs when someone acts in a
way that clearly endangers the security of human life. It is a behaviour that goes beyond a simple error
or excused accident, and the person should be aware of the harm that the behaviour could bring about.
It refers to performing something that is improper or failing to carry out an obligation.
Option D. Commission: A commission is a fee a
broker or other financial professional charges for their services in facilitating the purchase or sale of a
financial asset.

Solution for Question 31:


Correct Option B:
Medical maloccurrence refers to an unfortunate event or outcome that occurs during medical treatment
despite the exercise of due care and skill. In this case, the ureter injury during a
hysterectomy, despite adequate precaution, can be considered a medical maloccurrence. The surgeon
took appropriate precautions and obtained informed consent, but complications can still arise during s
urgical procedures.
Incorrect Option:
Option A. Novus actus interveniens: "new act intervening" breaking the chain of causation such that ev
en if the defendant has acted negligently, a subesquent intervening action breaks the chain of causatio

Page 24

66
n with the loss or damage sustained and so the defendant is not liableTherefore, novus actus interveni
ens is not applicable in this context.
Option C. Res ipsa loquitur: Res ipsa loquitur is a legal doctrine that translates to "the thing speaks for i
tself." It applies when an injury occurs under circumstances that indicate negligence without direct evid
ence of it. However, in the given scenario, the injury to the ureter is a known potential complication of a
hysterectomy, and the surgeon took appropriate precautions. Therefore, res ipsa loquitur is not applica
ble.
Option D. Physician error: Physician error refers to mistakes or failures in the actions or decisions of th
e physician that result in harm to the patient. In this case, if the injury to the ureter was due to a specific
error or negligence on the part of the surgeon, then physician error may be applicable. However, the q
uestion states that the injury occurred despite adequate precaution, indicating that the error was not att
ributable to the surgeon's actions or decisions

Solution for Question 32:


Correct option C
• In the given scenario, a 16-year-old girl who has been sold off and is pregnant seeks permission for
an abortion. The eligibility of the doctor to perform the abortion depends on the number of Medical
Termination of Pregnancy (MTP) procedures they have performed prior to this case. According to the
Medical Termination of Pregnancy Act in India, a doctor must have performed at least 25 MTPs (Option
C) to be eligible to perform the abortion for this girl. This criterion ensures that the doctor has sufficient
experience and expertise in performing safe and legal abortions.
Incorrect options:
Option A: 10, Option B: 15 & Option D: 35 - These options do not meet the requirement of having perfo
rmed at least 25 MTPs as per the law.

Solution for Question 33:


Correct Option C: Sec 304 A
• 304A IPC defines punishment of criminal medical negligence, as rash, careless or gross negligence.
Punishment includes up to 2 years imprisonment with a fine.
Incorrect Options:
Option A: 301 IPC defines punishment for culpable homicide causing the death of a
person, other than the person whose death was intended.
Option B: 304B IPC defines punishment for dowry death (death of a
female by suicide, accident, burns - within 7 years of marriage). Punishment is not less than 7
years and up to life imprisonment.
Option D: 302 IPC defines punishment for murder. Imprisonment for life, death penalty along with a
fine.

Page 25

67
Sexual Offences & Trace Evidences
1. A 30-year-old man has been accused of rape by his 26-year-old female friend. The woman allegedly
was lured by the man on a pretext of helping with an assignment but had later spiked her drink and
raped her while she was unconscious. The team has taken swabs of his urethral lining to test for the
presence of vaginal cells. The test that would be done for the same is?
(or)
What test is performed on the swabs of the accused man's urethral lining to detect the presence of
vaginal cells in a rape investigation involving his 26-year-old female friend?
A. Lugol’s iodine test
B. Takayama test
C. Florence test
D. Precipitin test
----------------------------------------
2. An investigation of a missing 25-year-old woman has led the police and the investigating agency to
an abandoned bungalow near the edge of the woods. The woman has been rescued alive but had
allegedly been sexually and physically assaulted. During the questioning, the police got a lead to
excavate the area around the farm shed. They have recovered the bodies of 2 young women, which
showed signs of early putrefaction. The police have decided to slap multiple charges against the owner
of the bungalow including sexual sadism. What offense is termed as such?
(or)
What offense is referred to as "sexual sadism"?
A. Sexual gratification from acts of physical cruelty
B. Having intercourse with an animal
C. Anal intercourse
D. Sexual intercourse with dead bodies
----------------------------------------
3. A husband engaged in sexual intercourse with his wife during their separation without her consent.
Which among the Sections given, deals with it?
(or)
Husband had intercourse with wife during separation without consent. Section which deals with it:
A. 376-A IPC
B. 376-B IPC
C. 376-C IPC
D. 376-D IPC
----------------------------------------
4. Rape is defined under:
(or)
Rape is defined under:

68
A. Sec. 320 IPC
B. Sec. 375 IPC
C. Sec. 376 IPC
D. Sec. 351 IPC
----------------------------------------
5. A 35-year-old man has been arrested by the police after 3 women filed charges against him for
harassing them by indulging in Frotteurism at the park. The police have made the arrest after
questioning multiple witnesses from the park that evening. Which act is the man being charged for?
(or)
What does FrotteurismFrotteurism stand for ?
A. Sexual pleasure is obtained by witnessing the act of urination
B. Sexual gratification by touching or rubbing one’s genitals against another non-consenting individual
C. Sexual gratification by wearing clothes of opposite sex
D. Sexual gratification by acts of physical cruelty
----------------------------------------
6. A 25-year-old man was apprehended by the college officials after multiple students witnessed him
forcefully pushing a girl to the floor and also attempting to remove her saree. He was rejected by the
same girl after he proposed her for a relationship. The family of the girl and the college officials have
decided to file charges under a section that deals with "Assault or use of criminal force against a
woman with intent to disrobe". Which section is the man being charged under?
(or)
Which section deal with "Assault or use of criminal force to woman with intent to disrobe".
A. 354 D
B. 354 A
C. 354 B
D. 354 C
----------------------------------------
7. A 52-year-old man who had been diagnosed with Young Onset Parkinson's disease reported back
for a follow-up visit. He informed the doctor that he had taken the medication regularly and has
experienced an improvement in his symptoms. His tremors and rigidity had noticeably decreased but
one complaint he had was that his sexual desire had increased. What is an excessive sexual desire in
men termed as?
(or)
What is an excessive sexual desire in men termed as?
A. Nymphomania
B. Satyriasis
C. Frigidity
D. Fetishism
----------------------------------------

Page 2

69
8. The 35-year-old man who had been reported missing was found dead in a room of another house he
owned. He was found naked in an incomplete hanging position. A bedsheet had been used and he was
seen suspended from the bedpost. His laptop was found at the scene with a playlist of obscene videos.
Prima facie, the cause of death appeared to be sexual asphyxia which is usually seen in cases of?
(or)
In the case of a 35-year-old man found dead in an incomplete hanging position with obscene videos
playing, what condition is this suggestive of, often associated with sexual asphyxia?
A. Masochism
B. Voyeurism
C. Sadism
D. Fetichism
----------------------------------------
9. Exhibitionism is punishable under
(or)
Exhibitionism is punishable under
A. 294 IPC
B. 293 IPC
C. 292 IPC
D. 304 IPC
----------------------------------------
10. A 35-year-old man has consulted the psychologist seeking help. He describes to the psychologist
his excessive sexual desire and addiction to porn viewing. He decided to seek help after he found
himself getting sexual gratification by hearing sounds of sexual intercourse that he hears from the
neighboring room which is occupied by another tenant. What will the psychologist document the
symptom as?
(or)
Sexual gratification by hearing sounds of sexual intercourse is defined as ?
A. Mixoscopia
B. Scatologia
C. Sexual oralism
D. Ecoutage
----------------------------------------
11. Mark the incorrect matched section for punishment of rape
(or)
Mark the incorrect matched section for punishment of rape
A. 376 A IPC: Punishment for causing death or resulting in persistent vegetative state of victim
B. 376 D(a): Punishment for gang rape on woman under 16 years of age
C. 376 D(b): Punishment for gang rape on woman under 14 years of age

Page 3

70
D. 376 E IPC: Punishment for repeat offenders
----------------------------------------
12. Married couple ‘A & B’, have got divorce legally. The woman ‘B’ has delivered a child ‘C’ after a few
months. As per relevant sections of Indian Law, the child ‘C’ would be considered legitimate if born after
_____ of dissolution of marriage.
(or)
After how many days of dissolution of marriage will a new born child considered Legitimate ?
A. 210 days
B. 270 days
C. 280 days
D. 235 days
----------------------------------------
13. A 35 year old man is brought to you dressed in woman’s clothing. On further questioning you find
that he gets sexual pleasure from doing so. It is a case of:
(or)
What is the likely diagnosis when a 35-year-old man derives sexual pleasure from cross-dressing in
women's clothing?
A. Eonism
B. Ipsation
C. Peeping tom
D. Mixoscopia
----------------------------------------
14. Mooning is a form of:
A. Exhibitionism
B. Transvestism
C. Voyeurism
D. Frotteurism
----------------------------------------
15. A 40-year-old man was arrested after police raided his house following an intimation by his
neighbor. A young girl child was rescued unhurt from his apartment. He is a known sexual offender and
had previously been arrested on charges of harassment and had been diagnosed with Ephebophilia.
What is the condition being termed Ephebophilia?
(or)
The term "Ephebophilia" refers to ?
A. Sexual attraction of an adult with pubescent adolescents
B. Sexual attraction of an adult with children
C. Sexual attraction of an adult with adults
D. Sexual attraction of an adult with elderly

Page 4

71
----------------------------------------
16. According to 357C CrPC, all hospitals (Govt or private) shall provide the first aid medical care free
of cost to victims of offences covered by the following section except:
(or)
All hospitals (Govt or private) shall provide the first aid medical care free of cost to victims of offences
covered by the following section, except:
A. 326 A IPC
B. 376 IPC
C. 302 IPC
D. 376 A IPC
----------------------------------------
17. Sin of gomorrah is another name for:
(or)
Sin of gomorrah is another name for:
A. Anal intercourse
B. Lesbianism
C. Buccal coitus
D. Tribadism
----------------------------------------
18. After a session with a 20-year-old man, the psychologist was able to document the concerns of the
young man as excessive onanism and addiction to viewing porn. What is the doctor referring to with the
term Onanism?
(or)
Term Onanism refers to?
A. Masturbation
B. Sodomy
C. Buccal coitus
D. Fetishism
----------------------------------------
19. Repeat offence of stalking is:
(or)
Repeat offence of stalking is:
A. Cognizable & bailable
B. Non-Cognizable & bailable
C. Cognizable & non bailable
D. Non-Cognizable & non bailable
----------------------------------------

Page 5

72
20. Which among the following is punishable in India?
(or)
Which among the following is punishable in India?
A. Tribadism
B. Consensual sodomy between 2 adults
C. Incest
D. Voyeurism
----------------------------------------
21. A 14-year-old rape victim was brought to the hospital at 22 weeks of pregnancy. All of the following
are correct statements regarding the case, except:
(or)
A 14-year-old rape victim was brought to the hospital at 22 weeks of pregnancy. All of the following are
correct statements regarding the case, except:
A. Vaginal swab need not be taken
B. The fetus can be aborted after her consent
C. Examination can be done by a male doctor with a female attendant
D. Urine pregnancy test is not necessary
----------------------------------------
22. Seminal stain can be detected by?
(or)
Seminal stain can be detected by?
A. Phenolphthalein test
B. Reinsch's test
C. Barberio’s test
D. Paraffin test
----------------------------------------
23. In genetic profiling of seminal enzyme markers, Phosphoglucomutase (PGM) can be detected till
(or)
Seminal enzyme markers, Phosphoglucomutase (PGM) can be detected till.
A. 6 hours
B. 12 hours
C. 18 hours
D. 24 hours
----------------------------------------
24. Semen of different individuals in case of gang rape is identified by:
(or)

Page 6

73
Semen of different individuals in case of gang rape is identified by:
A. Acid phosphatase test
B. Christmas tree stain
C. PSA
D. Single photon fluorimetry
----------------------------------------
25. A 23-year-old woman was rescued from near the highway in the early hours of the morning. She
was nearly unconscious but was able to record her statement while being treated by the doctors. She
was allegedly abducted and raped by 2 unidentified men. The doctor was able to demonstrate
presence of Spermine. Which of the following test was done by the doctor?
(or)
Which of the following test can detect spermine?
A. Barberios test
B. Florence test
C. CPK test
D. Ammonium molybdate test
----------------------------------------
26. Spermatozoa contain a high concentration of this enzyme which is more than double than found in
any other body fluid. Which enzyme among the following is stable and can be demonstrated even in old
stains of 6 months?
A. Acid Phosphatase test
B. ALP test
C. LDH
D. CPK enzyme
----------------------------------------
27. DNA fingerprinting was first developed by?
(or)
DNA fingerprinting was developed by?
A. Alec Jeffreys
B. Harold Cummins
C. Edward Henry
D. Francis Galton
----------------------------------------
28. Exchange of trace evidence occurs, when a person comes in contact with other person. This
principle is called:
(or)
What is the principle of exchange of trace evidence in forensic science known as?

Page 7

74
A. Locard’s principle
B. Quetelet’s rule
C. Cavett method
D. None of the above
----------------------------------------
29. The medullary index of a human hair is
(or)
The medullary index of a human hair is
A. 0.2
B. 0.4
C. 0.5
D. 0.6
----------------------------------------

Correct Answers
Question Correct Answer

Question 1 1
Question 2 1
Question 3 2
Question 4 2
Question 5 2
Question 6 3
Question 7 2
Question 8 1
Question 9 1
Question 10 4
Question 11 3
Question 12 3
Question 13 1
Question 14 1
Question 15 1
Question 16 3
Question 17 3
Question 18 1
Question 19 3

Page 8

75
Question 20 4
Question 21 2
Question 22 3
Question 23 1
Question 24 4
Question 25 2
Question 26 4
Question 27 1
Question 28 1
Question 29 1

Solution for Question 1:


Correct Option A - Lugol’s iodine test:
• Lugol’s iodine test is to detect vaginal epithelial cells containing glycogen.
Incorrect Options:
Option B - Takayama test:
• Takayama test is to detect blood stains.
Option C - Florence test:
• Florence test is a test for seminal stain.
Option D - Precipitin test:
• Species identification is done by the precipitin test.

Solution for Question 2:


Correct Option A - Sexual gratification from acts of physical cruelty:
• Sadism: sexual gratification is obtained or increased from acts of physical cruelty or infliction of pain
upon one's partner.
Incorrect Options:
Option B - Having intercourse with an animal:
• Bestiality/Zoophilia: It is sexual intercourse with animal, either vaginal, anal or oral. This includes all
animals, including birds, the usual victims being pets and farm animals.
Option C - Anal intercourse:
• Sodomy: It is anal intercourse between two males (homosexual sodomy) or between a male and a
female (heterosexual sodomy). It is also called buggery.
• Pederasty is intimate sexual relations, especially anal intercourse with a boy outside his immediate
family as the passive partner (the boy is known as a catamite, and the adult man as a pederast).

Page 9

76
Option D - Sexual intercourse with dead bodies:
• Necrophilia: Desire for sexual intercourse with dead bodies.

Solution for Question 3:


Correct Option B - 376-B IPC:
• Husband having intercourse with wife during separation without consent is punishable under 376 B
IPC.
Incorrect Options:
Option A - 376-A IPC:
• S 376A, I.P.C: Punishment for causing the death of the woman or causing the woman to be in a
persistent vegetative state shall be punished with rigorous imprisonment for a term which shall not be
less than twenty years, but which may extend to imprisonment for life, which shall mean imprisonment
for the remainder of that person's natural life, or with death.
Option C - 376-C IPC:
• 376-C IPC: Custodial rape/ Sexual intercourse by a person in authority.
Option D - 376-D IPC:
• S 376D, I.P.C: Where a woman is raped by one or more persons constituting a group or acting in
furtherance of a common intention, each of those persons shall be deemed to have committed the
offense of rape and shall be punished with rigorous imprisonment for a term which shall not be less
than twenty years, but which may extend to life which shall mean imprisonment for the remainder of
that person's natural life, and with fine.

Solution for Question 4:


Correct Option B - Sec. 375 IPC:
• Rape: Is defined under Sec. 375 IPC. A man is said to commit “rape” if he:
Incorrect Options:
Option A - Sec. 320 IPC:
• S.320 IPC: grievous injury.
• Any kind of severe hurt - Grievous hurt.
Option C - Sec. 376 IPC:
• Sec. 376 IPC: Punishment for rape.
Option D - Sec. 351 IPC:
• S. 351 IPC: It is the criminal force by which you produce fear in the opposite party, by. Gesture
(Raising voice, starring), or any, Preparation (Bullet in the weapon, knife), Word (Threatening).
• Gesture (Raising voice, starring), or any,

Page 10

77
• Preparation (Bullet in the weapon, knife),
• Word (Threatening).
• S.352 to 358 IPC deal with punishments for various types of assaults.
• Gesture (Raising voice, starring), or any,
• Preparation (Bullet in the weapon, knife),
• Word (Threatening).

Solution for Question 5:


Correct Option B
- Sexual gratification by touching or rubbing one’s genitals against another non-consenting individual
• It is the act of obtaining sexual arousal and gratification by rubbing of one’s genitals against a
non-consenting person in public places : Frotteurism
Incorrect Options:
Option A - Sexual pleasure is obtained by witnessing the act of urination:
• Sexual pleasure is obtained by witnessing the act of urination: Urolagnia
Option C - Sexual gratification by wearing clothes of the opposite sex:
• Sexual gratification by wearing clothes of the opposite sex: Transvestism or Eonism.
Option D - Sexual gratification by acts of physical cruelty:
• Sexual gratification by acts of physical cruelty: Sadism.

Solution for Question 6:


Correct Option C - 354 B:
• Any man who assaults or uses criminal force on any woman or abets such act with the intention of
disrobing or compelling her to be naked is punishable with imprisonment for 3–7 years and a fine (Sec.
354-B IPC). It is a cognizable and non-bailable offense.
Incorrect Options:
Option A - 354 D:
• Stalking involves following a woman or contacting a woman in spite of clear disinterest or monitoring
her through the internet, e-mail, or any other form of electronic communication. It is punishable under
Sec. 354-D IPC with imprisonment for up to 3 years and fine (cognizable and bailable) and
imprisonment up to 5 years and fine for subsequent offense (cognizable and non-bailable).
Option B - 354 A:
• Sexual harassment is defined as physical contact and advances involving unwelcome and explicit
sexual overtures, demanding sexual favors, showing pornography against her will, or making sexually
colored remarks. It is punishable with (rigorous) imprisonment for 1–3 years with/without a fine (Sec.
354-A IPC). The offense is cognizable and bailable.

Page 11

78
Option D - 354 C:
• Under Sec. 354-C IPC, voyeurism is a punishable offense. It states that ‘any man who watches or
captures the images of a woman engaged in a private act (changing clothes, using lavatory or doing a
sexual act) in circumstances where it is expected of not being observed or circulate such image is
punishable with imprisonment from 1–3 years and fine (cognizable and bailable), and for 2nd offense,
imprisonment is for 3–7 years and fine (cognizable and non-bailable).

Solution for Question 7:


Correct Option B - Satyriasis:
• Morbid, insatiable sexual need or desire in a man.
Incorrect Options:
Option A - Nymphomania:
• Nymphomania: Abnormal, excessive, insatiable desire in a woman for sexual intercourse.
Option C - Frigidity:
• Frigidity: It is the inability to initiate or maintain the sexual arousal pattern in females (absence of
desire for sexual intercourse or incapacity to achieve orgasm).
Option D - Fetishism:
• Fetishism
• It is a disorder characterized by satisfaction with inanimate objects (bra and underpants) of the
opposite sex.

Solution for Question 8:


Correct Option A - Masochism:
• Masochism: Recurrent urge or behavior of wanting to be humiliated, beaten, bound, or otherwise
made to suffer physical pain or humiliation.
Incorrect Options:
Option B - Voyeurism:
• Voyeurism: Recurrent urge or behavior to observe an unsuspecting person who is naked, disrobing or
engaging in sexual activities.
Option C - Sadism:
• Sadism: Recurrent urge or behavior involving acts in which the pain or humiliation of the victim is
sexually exciting.
Option D - Fetichism:
• Fetishism: Use of non-sexual or non-living objects or part of a person’s body to gain sexual
excitement. Partialism refers to fetishes specifically involving nonsexual parts of the body.

Page 12

79
Solution for Question 9:
Correct Option A - 294 IPC:
• A man achieves sexual arousal by exposing his private to another non-consenting person, which is
termed Exhibitionism.
• Punishable under section 294 IPC.
Incorrect Options:
Option B - 293 IPC:
• 293 IPC: Sale of obscene objects to a young person.
Option C - 292 IPC:
• 292 IPC: Sale of obscene books, etc.
Option D - 304 IPC:
• 304 IPC: Punishment for culpable homicide not amounting to murder.

Solution for Question 10:


Correct Option D - Ecoutage:
• Getting sexual gratification by hearing sounds of sexual intercourse.
Incorrect Options:
Option A - Mixoscopia:
• GA type of voyeurism in which sexual grati■cation is obtained by the sight of others engaged in sex..
Option B - Scatologia:
• Scatologia involves making obscene phone calls.
Option C - Sexual oralism:
• Sexual oralism is referred to as fellatio or cunnilingus, which involves oral stimulation of a partner’s
genitals or anus.

Solution for Question 11:


Correct Option C - 376 D(b): Punishment for gang rape on woman under 14 years of age:
• Section 376 D(b): Punishment for gang rape on woman under twelve years of age.
Incorrect Options:
Option A - 376 A IPC: Punishment for causing death or resulting in a
persistent vegetative state of victim:

Page 13

80
• TRUE Option, Section 376 A: Punishment for causing death or resulting in persistent vegetative state
of victim.
Option B - 376 D(a): Punishment for gang rape on woman under 16 years of age:
• TRUE Option, Section 376 D(a): Punishment for gang rape on woman under sixteen years of age.
Option D - 376 E IPC: Punishment for repeat offenders:
• TRUE Option, Section 376 E: Punishment for repeat offenders.

Solution for Question 12:


Correct Option C - 280 days:
• Legitimate child: A child born during the continuance of a legal marriage or within 280 days after the
dissolution of the marriage by divorce or death of the husband and the mother remaining unmarried
(Sec. 112 IEA).
Other options are incorrect

Solution for Question 13:


Correct Option A - Eonism:
• Transvestism/ Eonism: by wearing the dressing of the opposite sex, mostly seen among males.
Incorrect Options:
Option B - Ipsation:
• Onanism/ Ipsation / masturbation: Self-stimulation which affects sexual arousal.
Option C - Peeping tom:
• Voyeurism: There is a morbid desire of the individual to observe unsuspecting people undress or
naked, taking bath, see the genitalia or watch intercourse to get erotic excitement and sexual
gratification. It is commonly seen in males.
• Voyeurs frequently peep into the bedrooms of others, and are called ‘Peeping Toms’.
Option D - Mixoscopia:
• Mixoscopia: A type of voyeurism in which sexual grati■cation is obtained by the sight of others
engaged in sex.

Solution for Question 14:


Correct Option A - Exhibitionism:
• Mooning is a form of Exhibitionism.
• Exhibitionism: It is a desire and intentional exposure of genitalia in public places while in presence of
others (mostly in front of unsuspecting children or females) to obtain sexual pleasure.

Page 14

81
Incorrect Options:
Option B - Transvestism:
• Transvestism/Eonism: By wearing the dressing of the opposite sex, mostly seen among males.
Option C - Voyeurism:
• Voyeurism: There is a morbid desire of the individual to observe unsuspecting people undress or
naked, taking bath, see the genitalia or watch intercourse to get erotic excitement and sexual
gratification. It is commonly seen in males.
• Voyeurs frequently peep into the bedrooms of others, and are called ‘Peeping Toms’.
Option D - Frotteurism:
• Frotteurism/Toucherism: Sexual arousal and gratification by rubbing one's genitals against a
non-consenting person in public places. Usually seen in males in crowded trains, buses, and elevators.

Solution for Question 15:


Correct Option A - Sexual attraction of an adult with pubescent adolescents:
• Ephebophilia: Aka hebephilia, is the sexual attraction of an adult to pubescent or post-pubescent
adolescents.
Incorrect Options:
Option B - Sexual attraction of an adult with children:
• Infantophilia: Is a subcategory of pedophilia in which the victims are < 5 years.
Option C - Sexual attraction of an adult with adults:
• Sexual attraction of an adult with pubescent adolescents is known as ephebophilia.
Option D - Sexual attraction of an adult with elderly:
• Gerontophilia: Refers to the sexual preference for the elderly.

Solution for Question 16:


Correct Option C - 302 IPC:
• According to 357C CrPC, all hospitals (Govt or private) shall provide first aid medical care free of cost
to victims of offenses covered by the following section except 302 IPC.
• S. 302 IPC: Punishment for murder; Death, or imprisonment for life, and also fine.
Incorrect Options:
Option A / B / D - 326 A IPC / 376 IPC / 376 A IPC:
• 357C CrPC: All hospitals, public or private, whether run by the Central Government, the State
Government, local bodies or any other person, shall immediately, provide first-aid or medical treatment,
free of cost, to the victims of any offense covered under (voluntarily causing grievous hurt by use of
acid, etc.) section 326A, 376, 376A, 376B, 376C, 376D or section 376E (Rape) of the Indian Penal
Code, and shall immediately inform the police of the such incident.

Page 15

82
Solution for Question 17:
Correct Option C - Buccal coitus:
• Sin of gomorrah is another name for buccal coitus.
Incorrect Options:
Option A - Anal intercourse:
• Sodomy/ Greek love/ buggery: Penile-anal intercourse
Option B - Lesbianism:
• Lesbianism/ sapphism/ Tribadism: Female homosexuals.
Option D - Tribadism:
• Female homosexuality is known as tribadism/ lesbianism/ sapphism.

Solution for Question 18:


Correct Option A - Masturbation:
• Onanism/ Ipsation/ Masturbation: Self-stimulation which affects sexual arousal.
Incorrect Options:
Option B - Sodomy:
• Sodomy: Anal intercourse between 2 males or a male and a female. With mutual consent, it is not
punishable.
Option C - Buccal coitus:
• Buccal coitus/ Sin of Gomorrah: Denotes penile or vaginal oral sexual intercourse and can be
performed by both males and females.
• Fellatio means oral stimulation of the penis either by the female or male.
• Cunnilingus means oral stimulation of female genitalia.
Option D - Fetishism:
• Fetichism: Getting sexual gratification with inanimate objects. Usually, the fetish objects are
handkerchiefs, dresses, and undergarments.

Solution for Question 19:


Correct Option C - Cognizable & non bailable:
• Stalking involves following a woman or contacting a woman in spite of clear disinterest or
monitoring her through internet, e-mail or any other form of electronic communication.

Page 16

83
• It is punishable under Sec. 354-D IPC with imprisonment for upto 3 years and fine (cognizable and
bailable) and imprisonment upto 5 years and fine for subsequent offence (cognizable and non-bailable).

Solution for Question 20:


Correct Option D - Voyeurism:
• Voyeurism: Observing people getting undressed/ bathing/ watching intercourse.
• It is a punishable offense.
Incorrect Options:
Option A - Tribadism:
• Tribadism: Female homosexuality by mutual acts of sexual indulgence to achieve sexual gratification.
• Valid ground for divorce but not punishable in India.
Option B - Consensual sodomy between 2 adults:
• Sodomy: Anal intercourse between 2 males or a male and a female.
• With mutual consent, it is not punishable.
Option C - Incest:
• Sexual intercourse by a man with a woman who is closely related to him by blood or by marriage
(prohibited degrees of relationship).
• It is not a criminal offence.

Solution for Question 21:


Correct Option B - The fetus can be aborted after her consent:
• According to MTP act since the girl is not a major i.e < 18 years the fetus cannot be aborted after her
consent.
• The consent of a parent or guardian is necessary.
Incorrect Options:
Option A & D - Vaginal swab need not be taken & Urine pregnancy test is not necessary:
• Since it is 22 weeks old pregnancy, so a vaginal swab is not necessary and urine pregnancy test is
not necessary.
Option C - Examination can be done by a male doctor with a female attendant:
• Examination can be done by a male doctor with a female attendant.

Solution for Question 22:


Option C: Barberio’s test

Page 17

84
• Barberio's Test: A saturated aqueous or alcoholic solution of picric acid when added to spermatic fluid
produces yellow needle-shaped rhombic crystals of spermine picrate. The reaction probably depends
on the presence of prostatic secretion.
Option A: Phenolphthalein test
• The Phenolphthalein test is acatalytic test for the detection of blood. It is also known as the
Kastle-Meyer or KM test for presumptive blood.
• Phenolphthalein Test (Kastle-Meyer Test): To a solution extracted from the stain with distilled water,
add ten to twenty drops of phenolphthalein reagent (phenolphthalein 2g. + sodium hydroxide 20g. +
zinc+ distilled water 100 ml), and then a drop or two of 10 volumes hydrogen peroxide. If blood is
present, a pink or purple color develops immediately. The test is more specific for blood than the
benzidine test, but comparatively less sensitive. Traces of copper give a positive reaction.
Option B: Reinsch's test
• The Reinsch test is an initial indicator to detect the presence of one or more of the following heavy
metals in a biological sample. The method is sensitive to antimony, arsenic, bismuth, selenium, thallium
and mercury.
Option D: Paraffin test
• Paraffin test or Dermal nitrate test: It detects gunpowder residue (nitrates and nitrites). Melted paraffin
is brushed on the surface of the hand. The wax is removed and the inner surface of the wax cast is
treated with diphenylamine or diphenyl benzidine reagent. A blue color develops where the residue is
present. It is absolute.

Solution for Question 23:


Correct Option A - 6 hours:
• In genetic profiling of seminal enzyme markers, PGM can be detected till 6 hours.

Solution for Question 24:


Correct Option D - Single photon fluorimetry:
• Single photon fluorimetry has been used to differentiate between different semen in a gang rape case
Incorrect Options:
Option A - Acid phosphatase test:
• Acid Phosphatase test: Conclusive test for semen.
• When no sperm are observed, part of each of the swabs from the vagina, rectum, and mouth can be
used for presumptive tests for acid phosphatase. If however, sexual intercourse is still strongly
suspected or if the acid phosphatase test was weakly positive, an assay for prostate-specific antigen
(p30) should be performed. Occasionally, p30 is positive in the face of negative acid phosphatase.
Option B - Christmas tree stain:

Page 18

85
• Christmas tree stain: This staining technique was developed by Oppitz and consist of nuclear fast red
(red stain for sperm head) and picroindigocarmine (green counter-stain for the tail and other
cytoplasmic material) and is sometimes referred to as ‘Christmas tree’ stain because of the red-green
combination.
Option C - PSA:
• Prostate specific antigen or PSA (p30): The glycoprotein p30 is derived from prostrate and is found in
seminal plasma, male urine and blood, and has not been found in any female body tissue or fluid.
• Semenogelin (Sg), a protein originating in the seminal vesicles and a substrate for PSA, is also a
useful marker for the identification of semen.

Solution for Question 25:


Option A: Barberios test
· Detects - Spermine crystals (yellow needle-shaped crystals)
· Reagent - Picric acid.
· Type of crystals: Spermine Picrate.
Incorrect options
Option B:Florence Test
• · Detects - Choline crystals (Dark brown rhombic crystals)
• · Reagent – KI is used.
• ·· Type of crystals: Choline iodide.

Option C:CPK (Creatinine Phospho Kinase)


• Mnemonic:CPK - Old (Pakahua - means old in Hindi).
• Detects: Old seminal stains (6 months old as well)
• Normal range: 660 IU/ml.
Option D:Ammonium molybdate test -
• Detects Phosphorus content of Semen.

Solution for Question 26:


Option D: CPK enzyme
• Spermatozoa contains a high concentration of creatine phosphokinase which is more than double
than found in any other body fluid. The enzyme is stable and can be demonstrated even in old stains of
6 months.
• The test will be negative in case of aspermia.
Option A: Acid Phosphatase test

Page 19

86
• Acid Phosphatase test: Conclusive test for semen.
• When no sperm are observed, part of each of the swabs from the vagina, rectum, and mouth can be
used for presumptive tests for acid phosphatase. If, however, sexual intercourse is still strongly
suspected or if the acid phosphatase test was weakly positive, an assay for prostate-specific antigen
(p30) should be performed. Occasionally, p30 is positive in the face of negative acid phosphatase.
Option B: ALP test
• An ALP test (alkaline phosphatase) is a blood test. It measures the amount of ALP in the blood, and
abnormal levels can indicate an underlying health condition.
Option C: LDH
• Lactate dehydrogenase (LDH) isoenzyme: Polyacrylamide gel electrophoresis is used to separate the
various isoenzymes. This method gives a specific biochemical detection of spermatozoa in semen in
the presence of vaginal fluid, blood, urine, and saliva.

Solution for Question 27:


Correct Option A - Alec Jeffreys:
• DNA fingerprinting was developed by Alec Jeffrey.
Incorrect Options:
Option B - Harold Cummins:
• Dermatoglyphics is the study of ridge patterns in the skin. Harold Cummins, the father of
Dermatoglyphics. Dr. Harold Cummins first established the direct relationship between the patterns
formation and development on the palm and the development of the brain in the year 1926.
Option C - Edward Henry:
• Edward Henry devised a fingerprint-classification system that was adopted in British India. He
presented it in the UK in 1899.
Option D - Francis Galton:
• Dermatoglyphics is the study of ridge patterns in the skin. This system was first used in India in 1858,
by Sir William Herschel in Bengal. Francis Galton systematized this method in 1892.

Solution for Question 28:


Correct Option A - Locard’s principle:
• Locard’s Principle of Exchange: It states that “When two objects come into contact with each other,
there is always a transfer of some material between them”.
Incorrect Options:
Option B - Quetelet’s rule:
• Body weight in kilograms equals height in centimeters minus 100.
Option C - Cavett method:

Page 20

87
• Kozelka and Hine method or Cavett method: It involves aeration/distillation or diffusion of alcohol
under low pressure. It utilizes the principle that alcohol is easily oxidized to acetic acid by oxidizing
agents, such as potassium dichromate and sulfuric acid.
Option D - None of the above:
• Option A is the right answer.

Solution for Question 29:


Correct Option A - 0.2:
• The medullary index of human hair is 0.2

Page 21

88
Virginity, Impotency, Pregnancy and Abortion
1. Which of the following is not used to conduct criminal abortion?
(or)
Which of the following is not used to conduct criminal abortion?
A. Ripe fruit of papaya
B. Saffron
C. Seeds of carrot
D. Unripe fruit of pineapple
----------------------------------------
2. Which among the following is a method of direct local violence to conduct criminal abortion?
(or)
Which among the following is a method of direct local violence to conduct criminal abortion?
A. Abortion stick
B. Electricity
C. Syringing
D. Cupping
----------------------------------------
3. Punishment for seeking prenatal diagnostic facilities for purpose of sex selection is:
(or)
Punishment for a person seeking prenatal diagnostic facilities for purpose of sex selection is:
A. 1 year imprisonment and fine upto Rs.10000
B. 3–5 years imprisonment and fine Rs.50000 – 1 lakh
C. 1 year imprisonment and fine Rs.50000 – 1 lakh
D. 3 year imprisonment and fine Rs.10000
----------------------------------------
4. A medical officer is examining the genitalia of a 23-year-old college student who has filed a complaint
against a fellow student. She alleges that she was invited by him on the pretext of meeting his family
but was raped at the hotel they were staying in.Which area will be described by the doctor in her report,
when she examines the triangular space bounded by the clitoris above, hymen below, and the labia
minora laterally?
(or)
What specific area is the doctor examining when describing the triangular space bordered by the clitoris
above, hymen below, and the labia minora laterally in a rape victim?
A. Fossa navicularis
B. Vestibule
C. Fourchette

89
D. Vulva
----------------------------------------
5. In cases of child sexual assault the hymen is usually:
(or)
In cases of child sexual assault, the hymen is usually:
A. Ruptured as it is thin
B. Ruptured as it is underdeveloped
C. Intact as it is elastic
D. Intact as it is deep-seated
----------------------------------------
6. A 15-year-old girl is brought to the doctor by her mother. The mother is concerned as the girl has not
yet had her menarche. The doctor takes a detailed history and performs a physical examination
followed by measurements of serum follicle-stimulating hormone, luteinizing hormone, estradiol, thyroid
function testing, and bone age radiography all of which are within normal limits. What is the
characteristic of the hymen, if the doctor documents the finding as "False Virgin"?
(or)
What is the characteristic of the hymen, if the doctor documents the finding as "False Virgin"?
A. Admits tip of little finger through orifice painfully
B. Thin, Intact
C. Elastic, Tough
D. Inelastic
----------------------------------------
7. A 22-year-old lady died. The postmortem finding that will indicate that she has delivered a child in the
past are all, except:
(or)
The postmortem finding that will indicate that female has delivered a child in the past are all, except:
A. Walls of uterus are convex from inside
B. Cervix is irregular and external os is patulous
C. Body of uterus is twice the length of cervix
D. Uterus is bulky, large and heavier than nullipara
----------------------------------------
8. Which of the following is true regarding superfecundation?
(or)
Which of the following is true regarding superfecundation?
A. Fertilization of a second ovum in a woman who is already pregnant from a different ovarian cycle
B. Occurs only in bipartite uterus
C. Both ova do not always develop to maturity

Page 2

90
D. The second foetus is born later as a mature child
----------------------------------------
9. A person who may be impotent with one particular woman is called:
(or)
A person who may be impotent with one particular woman is called:
A. Quoad hoc
B. Frigidity
C. Atavistic form
D. None of the above
----------------------------------------
10. A 25-year-old woman who had eloped with her lover has returned to her home after being
abandoned by him. The parents have asked the family doctor to examine her who then examines of her
hymen. Which is the most common site for rupture of the hymen in a virgin in case of penile
penetration?
(or)
Which is the most common site for rupture of the hymen in a virgin in case of penile penetration?
A. Anterior
B. Antero-lateral
C. Posterior
D. Postero-lateral
----------------------------------------
11. The lung float test, also called the hydrostatic test or docimasia, has historically been employed in
cases of suspected infanticide to help determine whether or not an infant was stillborn. In the test, lungs
that float in water are thought to have been aerated, while those that sink are presumed to indicate an
absence of air. A false negative hydrostatic test for a live fetus is seen in which one of the following
conditions?
(or)
A false negative hydrostatic test for a live fetus seen in which one of the following conditions?
A. Atelectasis
B. Artificial respiration
C. Putrefaction
D. None of the above
----------------------------------------
12. A mother died after an illegal abortion. The section of IPC under which a doctor could be
prosecuted:
(or)
Under which IPC section could a doctor be prosecuted for a female's death following an illegal
abortion?

Page 3

91
A. IPC 312
B. IPC 314
C. IPC 315
D. IPC 318
----------------------------------------
13. A 16-year-old girl was rescued from a village in Bihar where she had been sold off by her father for
Rs.25000 to a 45-year-old man. On examination, she was found to be 12 weeks pregnant. She was
granted permission to abort the pregnancy, taking into consideration the age of the girl and the
circumstances of her pregnancy. A doctor eligible to perform the abortion for this girl should have
performed at least how many MTPs prior to this?
(or)
How many MTPs should a doctor have performed under supervision to be eligible to perform
abortions?
A. 10
B. 15
C. 25
D. 35
----------------------------------------
14. A marriage can be declared as null and void in all of the following situations except:
A. If the partner’s mental illness was present at the time of marriage
B. If the partner’s mental illness was present after the time of marriage
C. If the partner’s mental illness was present only before the time of marriage
D. None of the above
----------------------------------------

Correct Answers
Question Correct Answer

Question 1 1
Question 2 4
Question 3 2
Question 4 2
Question 5 4
Question 6 3
Question 7 1
Question 8 3
Question 9 1
Question 10 4

Page 4

92
Question 11 1
Question 12 2
Question 13 3
Question 14 3

Solution for Question 1:


Correct Option A - Ripe fruit of papaya:
• Unripe fruit of papaya is not used for procuring criminal abortion.
Incorrect Options:
Option B/ C/ D - Saffron/ Seeds of carrot/ Unripe fruit of pineapple:
• Saffron, Seeds of carrot, and unripe fruit of pineapple are used for procuring criminal abortion.

Solution for Question 2:


Correct Option D - Cupping:
• CUPPING: A form of direct local violence, A mug is turned mouth downwards over a lighted wick and
placed on the hypogastrium and the mug is pulled, which results in partial separation of the placenta.
This is usually practiced in advanced pregnancy.
Incorrect Options:
Option A/ B/ C - Abortion stick/ Electricity/ Syringing:
• Higginson's syringing (AIR / Soap water in the uterus), rupturing the membrane (pointed object),
abortion sticks, dilation of the cervix (slippery elm bark and drugs), Dilatation and Curettage, air
insu■ation.
• All these are local methods used in criminal abortion.

Solution for Question 3:


Correct Option B - 3-5 years imprisonment and fine Rs.50000 – 1 lakh:
• First offence – Jail time for up to 3 years, along with a fine of up to Rs. 50,000. Subsequent offences –
Jail time for up to 5 years, along with a fine of up to Rs. 1,00,000.

Solution for Question 4:


Correct Option B - Vestibule:
• The vestibule is a triangular surface that extends from the clitoris above to the anterior margin of the
hymen below, and laterally to the labia minora.

Page 5

93
Incorrect Options:
Option A - Fossa navicularis: The depression between the fourchette and the vaginal orifice is called fo
ssa navicularis.
Option C - Fourchette: The lower portions of labia minora fuse in the midline and form a
fold called fourchette.
Option D - Vulva: The vulva includes the mons veneris (pad of fat lying in front of the pubis), clitoris, lab
ia majora and minora, Vestibule, hymen, and urethral opening.

Solution for Question 5:


Correct Option D - Intact as it is deep-seated:
• In sexual assault cases of children, the hymen is usually intact as it is deep-seated.

Solution for Question 6:


Correct Option C - Elastic ,Tough :
• Hymen is intact, but loose and elastic or thick, tough and fleshy in a 'false virgin'
Incorrect Options:
Option A - Admits tip of little finger through orifice painfully
• Seen in true virgin
Option B - Thin , Intact
• Hymen is Intact, but loose, elastic or thick, tough and fleshy in False Virgin.
Option D - Inelastic :
• Intact, rigid, inelastic hymen in True Virgin.

Solution for Question 7:


Correct Option A - Walls of uterus are convex from inside:
• Walls are concave from the inside forming a wider and rounded cavity, while In a nulliparous woman,
the uterine walls are convex, forming a smaller, triangular cavity.
Incorrect Options:
Option B - Cervix is irregular and external os is patulous:
• The cervix is irregular in form and shortened, its edges show cicatrices.
• External os is enlarged, and Internal os is not so well defined.
Option C - Body of uterus is twice the length of cervix:

Page 6

94
• The body of the uterus is twice the length of the cervix (same length in virgin).
Option D - Uterus is bulky, large and heavy than nullipara:
• Uterus is larger, thicker, and heavier.

Solution for Question 8:


Correct Option C - Both ova do not always develop to maturity:
• Superfecundation is an extremely rare phenomenon that occurs when a second ova released during
the same menstrual cycle is additionally fertilized by the sperm cells of a different man in separate
sexual intercourse taking place within a short period of time from the first one
• Both ova do not always develop to maturity is true regarding superfecundation.
Incorrect Options:
Option A - Fertilization of a second ovum in a woman who is already pregnant in different cycle:
• Fertilization of a second ovum in a woman who is already pregnant is superfoetation.
Option B - Occurs only in bipartite uterus:
• Superfoetation can occur in a bipartite uterus.
• It may occur in a regular uterus
Option D - The second foetus is born later as a mature child:
• In superfecundation, one fetus may get aborted early or die and get retained until labour.

Solution for Question 9:


Correct Option A - Quoad hoc:
• Quoad hoc: A person who may be impotent with one particular woman.
Incorrect Options:
Option B - Frigidity:
• Frigidity: Failure of a female to respond to a sexual stimulus; aversion on the part of a woman to
sexual intercourse; failure of a female to achieve an orgasm (anorgasmia) during sexual
intercourse. Also known as female impotence
Option C - Atavistic form:
• Atavistic form is an historical approach used to explain criminal behavior, which is based on biological
factors. It determines whether the person has criminal tendencies based on his physical appearance.

Solution for Question 10:


Correct Option D - Postero-lateral:

Page 7

95
• The hymen is a thin membrane that partially covers the opening of the vaginal canal. The hymen can
vary in thickness and elasticity among individuals, and its position can also differ. The most common
site for rupture of the hymen during sexual intercourse, including penile penetration, is the
postero-lateral region.
Incorrect Options:
Option A - Anterior: While the hymen can have variations in its structure, the anterior region is generall
y not the most common site for rupture during penile penetration.
Option B - Antero-lateral : The antero-lateral region is not typically the most common site for hymen rup
ture during penile penetration.
Option C - Posterior: The posterior region of the hymen is less common for rupture during penile penetr
ation, as it is located towards the back of the vaginal opening.

Solution for Question 11:


Correct Option A - Atelectasis:
• A false negative hydrostatic test for a live fetus be may be seen in Atelectasis.
Incorrect Options:
Option B/ C - Artificial respiration/ Putrefaction
• In artificial respiration and putrefaction: A false positive hydrostatic test can be seen.

Solution for Question 12:


Correct Option B - IPC 314:
• According to 314 IPC, doctors could be prosecuted if the female died after an illegal abortion.
Incorrect Options:
Option A - IPC 312: 312 IPC states that a
doctor can be prosecuted for criminal abortion even if consent is taken.
Option C - IPC 315: Killing child or foetus about >28 weeks of gestation..
Option D - IPC 318: Concealment of birth..

Solution for Question 13:


Correct Option C - 25:
A doctor should have performed at least 25 MTPs, with at least 5
being independent cases, to be eligible to provide abortions

Page 8

96
Solution for Question 14:
Correct Option C - If the partner’s mental illness was present only before the time of marriage:
• If the partner’s mental illness was present only before the time of marriage, the marriage cannot be
declared null and void.
Incorrect Options:
Option A/ B - If the partner’s mental illness was present at the time of marriage/ If the partner’s mental
illness was present after the time of marriage: The mental condition of both the parties at the time of m
arriage is taken for deciding nullification.

Page 9

97
Child Abuse & Infant Deaths
1. A baby girl was declared dead on arrival at the hospital. The husband and his family allege that the
baby was born dead, but the woman alleges that the baby was fine at birth, but the local midwife had
conspired with her in-laws to kill the child as they didn't want a girl child. The forensic doctor will
perform all of the following tests to detect live birth, except?
(or)
All of the following tests can be done to detect live birth, except?
A. Ploucquet’s test
B. Fodere’s test
C. Gettler’s test
D. Raygat’s test
----------------------------------------
2. The school requested the help of a psychologist to understand why a particular girl was on leave
frequently, citing health issues. The school was unable to deny her leave when the request was
submitted by her own mother. After thoroughly questioning the student and her mother, the
psychologist contacted the police under her Privileged communication rights. All of the following are
included in rosenberg criteria, except?
(or)
All of the following are diagnostic criteria of Munchausen’s syndrome by proxy, except?
A. Illness produced or alleged or both by a parent
B. Repeated requests for medical care of a child
C. Parental acceptance of knowledge of cause of symptoms
D. Regression of symptoms on separation from parents
----------------------------------------
3. A 2-year-old girl child has been declared dead on arrival at the hospital. Her dad claims that she had
fallen off the balcony when left unsupervised by them for a few hours. The doctor, after a thorough
examination of the victim's body, is suspecting possible abuse.He has arrived at that conclusion after
noticing all of the following, except?
(or)
Battered baby syndrome can be diagnosed by all of the following ,Except ?
A. Subdural haemorrhage occurs in 40% of fatal cases
B. Nobbing fractures on the anterior angles of the ribs
C. Vitreous and subhyaloid hemorrhages in the eyes
D. Torn frenulum on the lower lip is a characteristic lesion
----------------------------------------
4. The umbilical cord of the baby shows mummification changes. The time since birth is ____
(or)
The umblical cord of the baby shows mummification changes. The time since birth is ____

98
A. 10 days
B. 3 days
C. 4 days
D. 7 days
----------------------------------------
5. After concluding the autopsy of a baby boy, the doctor documents the presence of air in the
abdomen and documents the same as Robert's sign, seen in which one of the following conditions?
(or)
Robert's sign is seen in which one of the following ?
A. Dead born in 12 hours
B. Live born in 12 hours
C. Dead born in 24 hours
D. Live born in 24 hours
----------------------------------------
6. If the fetus is born dead at 5 months, the signs of which one of the following is not identifiable during
autopsy?
(or)
If the fetus is born dead at 5 months, the signs of which one of the following is not identifiable during
autopsy?
A. Putrefaction
B. Mummification
C. Adipocere
D. Maceration
----------------------------------------
7. The case that is being followed by the entire community is that of a young man who has claimed his
rightful property. He alleges that he is the posthumous child of the late owner of a bungalow in that
town. The court has begun an investigation into the matter. While claiming to be a posthumous child, he
is claiming to be?
(or)
Posthumous child refers to ?
A. Child delivered after death of his mother.
B. Child delivered after death of biological father
C. Born after death of both parents
D. Has been abandoned by parents
----------------------------------------
8. A 5-month-old baby has been declared dead on arrival. The parents claim that the baby was found
breathless and unresponsive in the morning when the mother had tried to wake him up for
breastfeeding. The baby had been healthy and had not shown signs of any discomfort or disease.

Page 2

99
Which statement among the following, is not true about this condition??
(or)
Which statement among the following, is not true about "Sudden Infant Death Syndrome"
A. Also known as cot/crib death
B. Threefold increase in twins
C. Incidence common in females
D. Cigarette smoking by pregnant mothers increase the risk
----------------------------------------
9. In macerated fetus, the organ used for DNA profiling is:
(or)
In macerated fetus, the organ used for DNA profiling is:
A. Lungs
B. Spleen
C. Heart
D. Skeletal muscle
----------------------------------------
10. What does the image show?
(or)
Identify the below Image ?

A. Suppositious child
B. Surrogate child
C. Fetus Papyraceous
D. Macerated fetus
----------------------------------------

Correct Answers
Question Correct Answer

Page 3

100
Question 1 3
Question 2 3
Question 3 2
Question 4 2
Question 5 1
Question 6 3
Question 7 2
Question 8 3
Question 9 1
Question 10 3

Solution for Question 1:


Correct Option C - Gettler’s test:
• Gettler’s test: To check chloride content in heart. It is not a recognized or commonly used test for
detecting live birth. It is not a standard method employed in forensic medicine or pathology for this
purpose.
Incorrect Options:
Option A - Ploucquet’s test:
• The ratio of weight of lung to weight of body almost doubled after respiration, from 1/70 before
respiration to 1/35 after respiration.
Option B - Fodere’s test:
• Weight of the unrespired lung is 30 gm, which becomes 60gm after respiration due to increase in
blood flow after respiration.
Option D - Raygat’s test:
• Dissect out the fetal lungs & put them into water and observe (keep liver used as control).
• Specific gravity of the lung before respiration is 1040–1050 and it becomes 940-950 after respiration.
This makes the respired lung float.

Solution for Question 2:


Correct Option C - Parental acceptance of knowledge of cause of symptoms:
• It is not a diagnostic criteria of Munchausen’s syndrome by proxy.
• Munchausen syndrome by proxy: A psychiatric disorder that causes an individual to self-inflict injury
or illness or to fabricate symptoms of physical or mental illness in order to receive medical care or
hospitalization.
Incorrect Options:
Option A - Illness produced or alleged or both by a parent: Rosenberg criteria for Munchausen’s syndro
me includes illness produced or alleged, or both by a parent.

Page 4

101
Option B - Repeated requests for medical care of a
child: Rosenberg criteria for Munchausen’s syndrome includes repeated requests for medical care of a
child, leading to multiple medical procedures.
Option D - Regression of symptoms on separation from parents: Rosenberg criteria for Munchausen’s
syndrome included: Regression of symptoms on separation from parents.

Solution for Question 3:


Correct Option B - Nobbing fractures on the anterior angles of the ribs:
• NNobbing fractures occur on the posterior angles of the ribs in the paravertebral gutter region due to
lateral compression. Gives a string of beads appearance due to callus formation.
Incorrect Options:
Option A - Subdural haemorrhage occurs in 40 %
of fatal cases: Subdural hemorrhage occurs in 40% of fatal cases.
Option C - Vitreous and subhyaloid hemorrhages in the eyes: In the eyes, retinal detachment, retinal h
aemorrhages, Vitreous and subhyaloid haemorrhages are seen.
Option D - Torn frenulum on the lower lip is a characteristic lesion: Torn frenulum on the lower lip is a
characteristic lesion.

Solution for Question 4:


Correct Option B - 3 days:
• The umbilical cord of the baby shows obliteration and mummification changes: 3 days.

Solution for Question 5:


Correct Option A - Dead born in 12 hours:
• RRobert’s sign: Appearance of gas shadow in chambers of heart and great vessels, may appear as
early asby 12 hr.

Solution for Question 6:


Correct Option C - Adipocere:
• Adipocere is not seen in fetuses less than 7 months.
• Adipocere is formation of an offensive, sweet rancid smelling, soft, whitish or grayish white, crumbly,
waxy and greasy material (similar to soap) occurring in fatty tissues of a dead body. It is a modification
of decomposition

Page 5

102
• It is seen most commonly in bodies immersed in water or in damp, warm environment.
Incorrect Options:
Option A: Putrefaction
• Internal organs show autolytic decomposition, but the lungs and uterus remain unchanged for a long
time.
• Putrefaction is characterized by an unpleasant odor, greenish discoloration of skin and formation of
foul-smelling gases. Rarely, the fetus may show adipocere formation.
Option B: Mummification
• Mummification: It results from ischemia and scanty liquor amnii. The fetus is dried up and shriveled in
≥2 weeks.
Option D: Maceration
• Maceration: It occurs when the dead child remains in the uterus for about 3–4 days surrounded with
liquor amnii without air.

Solution for Question 7:


Correct Option B - Child delivered after death of biological father:
• Posthumous child is one who is born after the death of his/her father.
Incorrect options:
Option A,C,D are incorrect due to the above given reason

Solution for Question 8:


Correct Option C - Incidence common in females:
• Incidence is more common in males

Incorrect Options:
Option A/B/D - Also known as cot/crib death/ Threefold increase in twins/ Cigarette smoking by pregna
nt mothers increase the risk:
• Sudden infant death syndrome (SIDS) is also known as cot death or crib death.
• There is three fold increased in the twins
• Cigarette smoking and drug abuse by pregnant women increase the risk

Page 6

103
Solution for Question 9:
Correct Option A - Lungs:
• If the fetus is macerated, fetal lungs and brain tissues are more suitable for DNA typing.
Incorrect Options:
Option B/ C/ D - Spleen/ Heart/ Skeletal muscle:
• These organs are not used in DNA profiling if the fetus is macerated.

Solution for Question 10:


Correct Option C - Fetus Papyraceous:
• The term fetus papyraceous is used when intrauterine fetal demise of a twin occurs early in
pregnancy, with retention of the dead fetus for a minimum of 10 weeks resulting in mechanical
compression of the small fetus and loss of fluid such that it resembles parchment paper.
Incorrect Options:
Option A - Suppositious child:
• When a woman pretends to have been pregnant for some time and later produces a child, (Feigned
delivery) alleging that it is her own child to claim property of a deceased husband, such a child is called
a ‘suppositious child’.
Option B - Surrogate child:
• A surrogate mother is a woman who by contract agrees to bear a child for someone else. It is intended
to help a couple, of whom the woman is infertile, but the male has no reproductive deficiency. Artificial
insemination with the semen of the infertile woman's husband is carried out by a hired woman (womb
leasing).
• After surrogate birth, the baby is returned to its biological father and his wife.
• In gestational surrogacy, the mother's egg is used and the child will not be biologically related to the
surrogate mother.
Option D - Macerated fetus:
• Maceration is a process of aseptic autolysis and is the usual change. This occurs when the dead child
remains in the uterus for about three or four days surrounded by liquor amnii without air. If air enters the
liquor amnii after the death of the fetus, putrefaction occurs instead of maceration.

Page 7

104
Previous Year Questions
1. What is the most appropriate term to describe a man who consistently derives sexual pleasure from
causing physical harm to his partner, often resulting in cuts and burns on their arms?
A. Voyeurism
B. Fetishism
C. Sadism
D. Masochism
----------------------------------------
2. What test is performed when a 10-year-old boy presents to the emergency department with difficulty
walking and pain around the anus, and yellow needle-shaped rhombic crystals are observed in a
specimen taken from the perianal region when tested with picric acid?
A. Barberio test
B. Florence test
C. Teichmann test
D. Acid phosphatase test
----------------------------------------
3. Section 314 IPC deals with
A. Causing miscarriage with the consent of the mother
B. Causing miscarriage without the consent of the mother
C. Death of the mother by act done with intent to cause miscarriage
D. Causing the death of the quick unborn child by an act amounting to culpable homicide
----------------------------------------
4. A person of eonism derives pleasure from
A. Wearing clothes of opposite sex
B. Kissing and licking of anus by a sexual partner
C. Underclothing of female
D. Seeing a female undressing
----------------------------------------
5. What is the appropriate term to describe a person who sits naked on their balcony, deriving sexual
pleasure from observing others looking at them, while overlooking a park?
A. Voyeurism
B. Fetishism
C. Exhibitionism
D. Masochism
----------------------------------------
6. Typically, what is the condition of the hymen in cases of sexual assault involving a young child?

105
A. Ruptured since it is superficially
B. Ruptured since it is very thin
C. Unruptured since it is deeply situated
D. Unruptured since it is highly elastic
----------------------------------------
7. Which of the following techniques is considered abnormal when used for criminal abortion?
A. Dilatation and curettage
B. Mifepristone
C. Abortion stick
D. Vacuum aspiration
----------------------------------------
8. According to the MTP act of 1971, what is the maximum gestational age at which medical
termination of pregnancy is permitted?
A. 28 weeks
B. 22 weeks
C. 24 weeks
D. 16 weeks
----------------------------------------
9. An 18 year old female was sexually assaulted by a 20 year old male. There was presence of stains
on the clothes of the victim. Barberio’s test was performed on the stains. Which of the following is
detected by this test ?
A. Spermine
B. Acid phosphatase
C. Choline
D. Fructose
----------------------------------------
10. What does Barberio's test detect when yellow needle-shaped rhombic crystals are observed under
a microscope?
A. Semen
B. Blood
C. Sputum
D. CSF
----------------------------------------
11. A man continues to call females. Achieves sexual gratification by talking obscenity & sharing
obscene picture. The condition is?
A. Frotterurism
B. Voyeurism

Page 2

106
C. Scatologia
D. Coprophilia
----------------------------------------
12. Frotteurism is characterized as _____________
A. Obtaining sexual pleasure by wearing clothes of opposite sex
B. Desire to seek surgery to become member of opposite sex
C. Sexual gratification by rubbing his private parts against another person
D. Exposure of one’s genitals to an unsuspecting stranger
----------------------------------------
13. Which option provides the most accurate description of a posthumous child?
A. Delivery of a macerated fetus
B. Child is delivered after death of biological father
C. Child is born to an unmarried couple
D. Child is abandoned by the parents
----------------------------------------
14. As a medical officer, what actions should you take when a 14-year-old girl accompanied by her
mother presents to the outpatient department (OPD) with a history of penovaginal penetration by a
neighbor, with consent given by the girl? The girl expresses her refusal to undergo an examination.
Document informed refusal Do not inform the police Counsel the mother and daughter Examine with
necessary force Inform the police
A. 1,2,3 are true
B. 2,3,4 are true
C. 1,3,5 are true
D. 3,4,5 are true
----------------------------------------
15. During the visit to the outpatient department, a female patient aged 18 years was examined. The
genital examination disclosed the following observations. These observations suggest: Labia majora –
separated Labia minora – flabby Fourchette tear present Roomy vagina seen with intact hymen
A. True virgin
B. False virgin
C. Premenstrual stage
D. Molestation
----------------------------------------
16. A registered medical practitioner is requested by a survivor of sexual violence to examine her.
Within what time must the practitioner take a vaginal swab to look for the presence of spermatozoa?
A. Within 72 hours
B. Within 36 hours
C. Within 5 days

Page 3

107
D. Within 48 hours
----------------------------------------
17. Which organization is related to the Protection of Children from Sexual Offences (POCSO) Act?
A. Child and welfare committees
B. Women and child safety department
C. Children rights protection committee
D. All of the above
----------------------------------------
18. All are Grounds of divorce except :
A. Adultery
B. Incurable Leprosy
C. Poverty
D. Insanity
----------------------------------------
19. What is the appropriate IPC section for the act of inducing abortion without the consent of the
woman?
A. 312
B. 313
C. 314
D. 315
----------------------------------------
20. What is the term for a child born when a woman pretends to be pregnant and later uses the child to
extort a man?
A. Supposititious child
B. Superfecundation
C. Posthumous child
D. Superfetation
----------------------------------------
21. Which of the following defines rape?
(or)
Which of the following defines rape?
A. 375 IPC
B. 376 IPC
C. 377 IPC
D. 498 (A) IPC
----------------------------------------

Page 4

108
22. Which test is employed to distinguish human blood from blood of other species among the options
provided?
A. Teichman test
B. Precipitin test
C. Barberio test
D. Takayama test
----------------------------------------
23. 'A' kills 'B' with a knife. Another person 'Z' saw 'A' with knife in the park few minutes before murder.
'Z' give this statement in court of law. What kind of evidence is it?
A. Direct
B. Indirect
C. Hostile
D. Hearsay
----------------------------------------
24. In the OPD, a 5-year-old child presenting with a past medical record of perianal pain is examined.
Upon testing a sample obtained from the perianal region, it reveals the presence of yellow rhombic
crystals of spermine picrate. What is the specific diagnostic procedure used to identify this condition?
A. Barberio test
B. Florence test
C. Takayama test
D. Teichmann test
----------------------------------------
25. Please describe the test depicted in the image provided.

A. Barberio test
B. Florence test
C. Takayama test
D. Teichmann test
----------------------------------------
26. Identical twins have similarities in all of the following except?

Page 5

109
A. Blood group
B. DNA fingerprinting
C. Fingerprint pattern
D. Iris colour
----------------------------------------
27. Edmond Locard is most famous for?
A. Study of fingerprints
B. Theory of exchange
C. Stature estimation
D. Forensic ballistics
----------------------------------------
28. Which confirmatory test is utilized by forensic experts to establish the presence of a bloodstain at a
crime scene?
A. Spectroscopic test
B. Kastle meyer test
C. Benzidine test
D. Orthotoluidine test
----------------------------------------
29. What is the substance identified by Barbero's test, which was conducted on the stains from a sexual
assault incident involving an 18-year-old female and a 20-year-old male?
A. Spermine
B. Acid phosphatase
C. Choline
D. Fructose
----------------------------------------

Correct Answers
Question Correct Answer

Question 1 3
Question 2 1
Question 3 3
Question 4 1
Question 5 3
Question 6 3
Question 7 3
Question 8 3

Page 6

110
Question 9 1
Question 10 1
Question 11 3
Question 12 3
Question 13 2
Question 14 3
Question 15 2
Question 16 1
Question 17 1
Question 18 3
Question 19 2
Question 20 1
Question 21 1
Question 22 2
Question 23 2
Question 24 1
Question 25 4
Question 26 3
Question 27 2
Question 28 1
Question 29 1

Solution for Question 1:


• The behavior described in the scenario involves inflicting pain and harm upon another person to
derive sexual pleasure. This is the core characteristic of sadism, which is a sexual disorder that
involves gaining pleasure from the experience of inflicting pain, humiliation, or suffering on others.
• The person in the scenario is engaging in sadistic behavior by intentionally inflicting cuts and cigarette
burns on his partner's arms for the purpose of achieving sexual gratification. This behavior can cause
physical and emotional harm to the partner and is not considered acceptable or consensual within a
healthy sexual relationship.
Incorrect Option:
• Option a: Voyeurism is the act of observing or spying on individuals who are naked or engaged in
sexual activity without their knowledge or consent, for the purpose of sexual gratification. This behavior
does not involve inflicting pain or harm on others, as in the scenario described.
• Option b: Fetishism is a type of sexual disorder in which an individual is sexually aroused by
non-human objects or specific body parts that are not typically considered sexually arousing, such as
shoes, leather, or feet.
• Option d: Masochism is a sexual disorder in which an individual derives sexual pleasure from
receiving pain or humiliation. In the scenario described, the individual is inflicting pain and harm on their

Page 7

111
partner rather than receiving it themselves.

Solution for Question 2:


• Barberio created the Barberio test in the year 1905. This test is used to detect spermine. It includes
the microscopic confirmation of spermine picrate crystals, which form when semen is subjected to an
aqueous solution of picric acid, in the form of needle-shaped crystals. Therefore, the test done is the
Barberio test.
Incorrect Choices:
Option b. Florence test: In this test, when the Florence reagent (Potassium Iodide + Iodine +
Water) is put to the slide, choline periodide crystals with a rhomboidal shape start to form. The test doe
s not produce yellow needle-shaped rhombic crystals, hence, it is not the test carried out for the patient
.
Option c. Teichmann test: The Teichmann test is a forensic test that is used to confirm the presence of
blood. It involves the formation of hemin crystals when blood is treated with a glacial acetic acid solutio
n and heated. The test does not produce yellow, needle-shaped, rhombic crystals, thus this option is in
correct.
Option d. Acid phosphatase test: The acid phosphatase test is also used in forensics, particularly to det
ect semen, as acid phosphatase is found in high concentrations in semen. However, this test does not i
nvolve the formation of yellow, needle-shaped, rhombic crystals. It relies on a
color change reaction when acid phosphatase reacts with a specific substrate.

Solution for Question 3:


• Section 314 of the Indian Penal Code (IPC) is very specific in its definition. It deals with the death of
the mother by an act done with the intent to cause a miscarriage. Anyone who does an act that kills a
pregnant woman with the intent to produce a miscarriage is punishable by imprisonment of either kind
for a time that may be as long as 10 years, as well as by a fine if the act was committed without the
woman's permission.
Incorrect Choices:
Option a. Causing miscarriage with the consent of the mother: This is covered by Section 312 of the IP
C. It describes the offense of causing a
miscarriage with the woman's consent, punishable by imprisonment and/or a fine.
Option b. Causing miscarriage without the consent of the mother: This is addressed under Section 313
of the IPC. It describes the act of causing a miscarriage without the woman's consent, which carries a
higher penalty due to the violation of the woman's right to consent.
Option d. Causing the death of the quick unborn child by an act amounting to culpable homicide: This i
s addressed under IPC section 316. Anyone who commits any act that, if it resulted in death, would co
nstitute culpable homicide and who also causes the quick death of an unborn child shall be punished b
y imprisonment of either description for a term that may not exceed ten years, as well as by fine.

Page 8

112
Solution for Question 4:
Correct Option A - Wearing clothes of opposite sex:
• The correct answer is a. Wearing clothes of the opposite sex.
• Eonism is a practice where individuals derive pleasure or satisfaction from wearing the clothes of the
opposite sex. It's not about voyeurism or fetishizing specific items of clothing or sexual practices but
about the act of adopting the appearance of the opposite sex
Incorrect Options:
Option B - Kissing and licking of the anus by a sexual partner: This sexual activity, often referred to as
anilingus, doesn't have any specific connection to eonism.
Option C - Underclothing of a female: While some people who practice eonism might enjoy wearing wo
men's underwear, the pleasure derived from eonism isn't solely about this particular item of clothing. It'
s about the act of dressing in clothing of the opposite sex as a whole.
Option D - Seeing a female undressing: This option describes voyeurism, a
practice where someone obtains sexual pleasure from observing others without their consent. This is a
different concept from eonism

Solution for Question 5:


Correct Option C:
The behavior described in the scenario, where a guy derives sexual gratification by sitting nude on his
balcony and observing people who look at him, is best described as exhibitionism. Exhibitionism is a p
araphilic disorder characterized by the act of exposing one's genitals to unsuspecting individuals in ord
er to achieve sexual arousal or gratification.
Incorrect Options:
Option A. Voyeurism: Voyeurism is the act of gaining sexual pleasure or gratification from observing ot
hers who are naked or engaged in sexual activities without their consent. It involves the act of watching
others without their knowledge or permission. In the given scenario, the individual is the one being obs
erved and deriving pleasure, rather than observing others. Therefore, voyeurism is not the correct term
to describe the behavior.
Option B. Fetishism: Fetishism is a paraphilic disorder in which individuals derive sexual arousal or grat
ification from specific objects, materials, or body parts. It typically involves a fixation on nonliving object
s or non-genital body parts. In the given scenario, the individual's sexual gratification is derived from th
e act of exposing himself and observing the reactions of others, rather than from a specific object or bo
dy part. Therefore, fetishism is not the most appropriate term to describe the behavior.
Option D. Masochism: Masochism refers to the sexual arousal or gratification derived from experiencin
g pain or humiliation. It involves deriving pleasure from being dominated, punished, or experiencing dis
comfort during sexual activities. The given scenario does not involve the individual experiencing pain or
humiliation, but rather derives sexual gratification from the act of exhibitionism. Therefore, masochism
is not the correct term to describe the behavior

Solution for Question 6:

Page 9

113
Correct Option: C
• In cases of rape or sexual abuse of a young child, the hymen is more likely to be unruptured rather
than ruptured. This is because the hymen, which is a thin membrane located at the entrance of the
vagina, is a highly elastic structure that can stretch without tearing. It is situated deeper inside the
vaginal canal and is not easily accessible or visible externally.
Incorrect options:
Option A. Ruptured since it is superficially: This option is incorrect because the hymen is not superficial
ly located. It is positioned deeper inside the vagina.
Option B. Ruptured since it is very thin: While the hymen is thin, its thinness does not necessarily imply
that it will be ruptured in cases of rape or sexual abuse. The elasticity of the hymen allows it to stretch
rather than tear.
Option D. Unruptured since it is highly elastic: This option is incorrect because it contradicts the statem
ent that the hymen is unruptured. The elasticity of the hymen enables it to stretch, but it does not guara
ntee that it will remain unruptured in cases of sexual abuse

Solution for Question 7:


Correct Option: C
Seaweed called laminaria is used as an abortion stick and can be sterilised, rolled, and dried to form a
thin stick or "tent" that can be put into the cervix to stimulate dilation. Criminal abortion methods include
:
• Up to the end of first month: violent exercise
• Up to the end of second month: abortifacient drugs
• 3rd or 4th month: mechanical interference
Incorrect Options:
Option A. Dilatation and curettage: A surgery called dilation and curettage (D&C;) is used to remove tis
sue from within the uterus. The doctor will use small tools or a
drug to open up the lower, narrow region of the uterus during a
dilation and curettage procedure. The uterine tissue is then removed by the doctor using a
surgical tool called a curette, which can be either a suction device or a cutting instrument.
Option B. Mifepristone: Mifepristone is a medication that inhibits progesterone, a
hormone required for a pregnancy to progress. When combined with the drug misoprostol, mifepristone
is used to stop pregnancies up to ten weeks gestation.
Option D. Vacuum aspiration: In the first trimester, curettage by vacuum aspiration is the most common
method of abortion. A flexible or rigid plastic cannula coupled with a syringe can be used to accomplis
h early pregnancy aspiration. Uterine evacuation is accomplished using this method, known as manual
vacuum aspiration, up to a gestational age of 10 weeks.

Solution for Question 8:


Correct Option: C

Page 10

114
• MTP ACT is Linked with Abortion Practices.It was Passed in 1971; and implemented in 1972. It
wasAmended in 2021.According to this at,Abortion can be done till 20-24 weeks with the upper limit of
24 weeks. Acc to this act, consent of women is of utmost priority.

Solution for Question 9:


Correct Choice: A
• Barberio's test is a chemical test used in forensic analysis to detect the presence of spermine.
Spermine is a naturally occurring polyamine found in human seminal fluid, which is released during
ejaculation. Therefore, if Barberio's test is positive for spermine, it indicates the presence of semen and
can be an important forensic evidence in cases of sexual assault.
Incorrect Choices:
Option B. Acid phosphatase: Acid phosphatase is an enzyme found in high concentration in human se
men. While acid phosphatase can also be used as a marker for semen detection, Barberio's test specif
ically targets the detection of spermine, not acid phosphatase.
Option C. Choline: Choline is a nutrient and essential component of various molecules in the human bo
dy. However, it is not specifically associated with the detection of semen or sexual assault. Barberio's t
est is not designed to detect choline.
Option D. Fructose: Fructose is a sugar present in human seminal fluid. Although the presence of fruct
ose in stains can indicate the presence of semen, Barberio's test is not specifically designed to detect f
ructose.

Solution for Question 10:


Correct Choice. A
Explanation:
Barberios test:
• Test reaction depends on the presence of Spermine which comes from the prostate.
• Reaction gives yellow, needle shaped, rhombic crystals of Spermine picrate.
• Reagent - Picric acid.
• Mnemonic: Barbar picks your hair with a needle.

Page 11

115
Incorrect choices:
Option B. Takayama is one of the confirmatory tests for the blood staining. It gives pink-feathery crystal
s (Mnemonic- Takatak is pink in color).
Option C. Sputum analysis is generally done in case of mycobacterial infections such as Tuberculosis.
Option D. CSF analysis is done to help diagnose the brain and spinal diseases. CSF analysis generally
measures the different levels of chemicals in the CSF fluid.

Solution for Question 11:


Correct Option: C
• Based on the provided information, the patient most likely has scatologia.
• Scatologia refers to a paraphilic disorder in which a person achieves sexual arousal and gratification
by engaging in obscene or sexually explicit talk or behavior. It involves the use of obscene language,
sharing obscene pictures, or engaging in explicit conversations with others, often without their consent.
Let's briefly explain the other options for clarity:
Option A: Frotteurism: This is a paraphilic disorder in which a person derives sexual pleasure from rub
bing against or touching non-consenting individuals in crowded places. It typically involves non-consen
sual physical contact.
Option B: Voyeurism: Voyeurism is a paraphilic disorder characterized by the persistent and intense se
xual interest in observing others while they are undressing, naked, or engaged in sexual activities, with
out their knowledge or consent.
Option D: Coprophilia: Coprophilia is a paraphilic disorder in which a person derives sexual pleasure fr
om feces or associated activities, such as watching others defecate, playing with feces, or using feces
during sexual acts.

Page 12

116
Solution for Question 12:
Correct Option: C - Sexual gratification by rubbing his private parts against another person
• Frotteurism refers to the sexual gratification derived from rubbing one's genitals against another
person without their consent. It typically occurs in crowded places or situations where the person can
engage in this behavior discreetly. Frotteurism is considered a paraphilic disorder and is associated
with non-consensual sexual behavior.
Incorrect Options
Option A: Obtaining sexual pleasure by wearing clothes of the opposite sex (Incorrect) This option des
cribes cross-dressing or transvestism, which involves individuals finding sexual pleasure or gratification
from wearing clothes typically associated with the opposite sex. While cross-dressing may be a
source of arousal or pleasure for some individuals, it is not specific to frotteurism.
Option B: Desire to seek surgery to become a member of the opposite sex (Incorrect) This option refer
s to gender dysphoria or the desire to undergo sex reassignment surgery to align one's physical body
with their experienced gender identity. Gender dysphoria is not related to frotteurism, as it is primarily f
ocused on one's gender identity rather than engaging in non-consensual sexual behaviors.
Option D: Exposure of one's genitals to an unsuspecting stranger (Incorrect) This option describes exhi
bitionism, which involves exposing one's genitals to unsuspecting individuals for sexual arousal or grati
fication. Exhibitionism is a separate paraphilic disorder and is distinct from frotteurism, which involves r
ubbing one's genitals against another person without their consent.

Solution for Question 13:


Correct Option B: Child is delivered after death of biological father
• Child is delivered after the death of the biological father: This is the correct option.
• A posthumous child refers to a child who is born after the death of their biological father.
Incorrect Options
Option B - Delivery of a macerated fetus: This option refers to the delivery of a fetus that has undergon
e maceration, which is the softening and disintegration of fetal tissues due to prolonged retention in the
uterus after fetal death. This is not the correct description of a posthumous child.
Option C - Child is born to an unmarried couple: This option refers to a
child born to unmarried parents. While the marital status of the parents does not specifically define a
posthumous child, it is unrelated to the concept.
Option D - Child is abandoned by the parents: This option refers to a child who has been abandoned b
y their parents, where the parents have intentionally left the child without proper care or support. This is
not the correct description of a posthumous child.

Solution for Question 14:

Page 13

117
• As per the given scenario, Consent for the examination of the victim should be taken from the as she
is 14- year old.
• If the victim is less than 12 years of age or unsound mind, The consent needs to be taken from the
guardian.
• Conselling is done if she denies for examination.
• If after the counselling she denies to get examined than informal refusal has to be documented.

Solution for Question 15:


Correct Option : B
• The given findings, including separated labia majora, flabby labia minora, a tear in the fourchette, and
a roomy vagina with an intact hymen, are indicative of a false virgin. A false virgin is a term used to
describe an individual who has experienced sexual intercourse or other activities that have caused
changes to the external genitalia, despite having an intact hymen.
Incorrect options:
Option A. True virgin: This choice is incorrect because the findings described in the question indicate c
hanges in the external genitalia that are consistent with previous sexual activity. Therefore, the term "tr
ue virgin" does not apply in this case.
Option C. Premenstrual stage: This choice is incorrect because the findings mentioned in the question
are not related to the premenstrual stage. The premenstrual stage refers to the phase of the menstrual
cycle that occurs before menstruation, and it does not involve the specific physical changes described i
n the question.
Option D. Molestation: This choice is incorrect because the findings in the question do not directly indic
ate molestation. Molestation refers to unwanted and inappropriate sexual behavior or contact, typically
involving an individual with abusive intentions. The given findings are more suggestive of previous con
sensual sexual activity rather than molestation.

Solution for Question 16:


Correct Option : A
• When a survivor of sexual violence requests a medical examination, it is crucial to collect evidence
promptly to increase the chances of detecting and preserving relevant forensic evidence, such as the
presence of spermatozoa. The recommended timeframe for collecting a vaginal swab to look for the
presence of spermatozoa is within 72 hours of the assault.
Incorrect options:
Option B. Within 36 hours: This choice is incorrect because the recommended timeframe for collecting
a vaginal swab is typically within 72 hours, which is longer than 36 hours. The earlier the examination i
s conducted within the recommended timeframe, the better the chances of obtaining accurate results.
Option C. Within 5 days: This choice is incorrect because the recommended timeframe for collecting a
vaginal swab is within 72 hours, which is shorter than 5 days. Waiting for 5 days may decrease the likel
ihood of detecting viable spermatozoa or other forensic evidence, making it more challenging to gather
conclusive evidence.

Page 14

118
OptionD. Within 48 hours: This choice is incorrect because the recommended timeframe for collecting
a vaginal swab is within 72 hours, which is longer than 48 hours. While collecting the swab within 48 ho
urs is still relatively prompt, the extended timeframe of 72 hours provides a
more reasonable and inclusive window for evidence collection.

Solution for Question 17:


Correct Option A-Child and welfare committees
• Child and welfare committees Plays an important role under POSCO act, cases registered under this
act need to be reported to CWC with in 24 hours of recording of the complaint.
Incorrect Options:
Option B- Women and Child Safety Department : This option is incorrect
Option C- Children rights protection committee : This option is incorrect
Option D- All of the Above: This option is incorrect

Solution for Question 18:


Correct Option C. Poverty
• Poverty - Is not a Grounds of divorce.
Incorrect options
Option A. Adultery: Adultery, Is Grounds of divorce.
Option B. Incurable Leprosy : Incurable Leprosy, Is Grounds of divorce.
Option D. Insanity: Insanity, Is Grounds of divorce.

Solution for Question 19:


Correct Option : B.
• In the Indian Penal Code (IPC), Section 313 deals with the act of causing miscarriage without a
woman's consent. This section makes it a criminal offense to cause a woman to miscarry against her
will. It states that anyone who causes a woman, who is with child, to miscarry without her consent can
be punished with imprisonment that may extend up to ten years, along with a fine.
Incorrect options:
Option A. IPC Section 312: It deals with voluntarily causing a miscarriage and carries a
lesser punishment compared to Section 313.
Option C. IPC Section 314: Whoever, with intent to cause the miscarriage of a woman with child, does
any act which causes the death of such woman, shall be punished with imprisonment of either descripti
on for a term which may extend to ten years, and shall also be liable to fine; If act done without woman'

Page 15

119
s consent
Option D. IPC Section 315: Act done with intent to prevent child being born alive or to cause it to die aft
er birth

Solution for Question 20:


Correct Option A: Supposititious child
• Supposititious child / fictitious child/ substituted child is a fraudulent child that is said to conveniently
appear before the people/ court to blackmail with intentions of property/money etc. Various scenarios
include:
• Female claims the child as her own but brings the child from somewhere else.
• Female may also claim to be pregnant but may not be (feigns pregnancy)
• If a female delivers a dead fetus, and claims another child to be her own child, is also considered a
suppositious child.
• If a child is kidnapped from somewhere and is wrongfully claimed to be female’s own
Incorrect Options:
Option B: Superfecundation- It is when 2 ova are fertilised in 1 menstrual cycles through 2
different acts of sexual intercourse
Option C: Posthumous child- It is the child born after the death of the father.
Option D: Superfetation - It is when 2 ova are fertilised in 2 menstrual cycles through 2
different acts of sexual intercourse (2 Ts)

Solution for Question 21:


Correct Option A: 375 IPC: Gives definition of rape
Incorrect Options:
Option B. . 376 IPC: Gives punishment for rape
Option C . 377 IPC: Gives punishment for unnatural sexual offences
Option D. 498 (A) IPC: Gives Punishment for causing physical/ mental cruelty of a
woman by husband / relatives

Solution for Question 22:


• The precipitin test is used to differentiate human blood from the blood of other species. This test is
based on the principle that antibodies in serum will react with and precipitate their corresponding
antigens. In the case of blood, the precipitin test involves mixing a sample of the unknown blood with a
specific antibody that is known to react only with human blood. If the unknown blood is human, the

Page 16

120
antibody will react with it and form a visible precipitate.

Incorrect Option:
• a. Teichman test: This test is used to detect the presence of hemoglobin in a blood stain. It is not used
to differentiate human blood from the blood of other species.
• c. Barberio test: this test is used to detect seminal stains.
• d. Takayama test: This test is used to detect the presence of blood in a forensic sample, but it does
not differentiate human blood from the blood of other species. The Takayama test is based on the
principle that hemoglobin reacts with pyridine and produces a characteristic brown color.

Solution for Question 23:


Correct Option B - Indirect:
• This type of proof, also known as circumstantial evidence, needs a bit of thinking or an extra step to tie
it to the conclusion. As person 'Z' has not seen the crime but he only saw a weapon in the hand of the
accused.
• It is also known as corroborative evidence (Seen weapon in the hand of a person). This is a kind of
indirect evidence.
Incorrect Options: Option A, C and D are incorrect.

Solution for Question 24:


Correct Option: A
• The Barberio test is used to identify the presence of oxalate crystals in urine or specimens from the
perianal region.

Page 17

121
• It specifically detects the formation of yellow rhombic crystals of spermine picrate when oxalate ions
react with spermine and picric acid.

Incorrect options:
Option B. Florence test: There is no specific medical test known as the Florence test that is relevant to
the given scenario.
• Florence test · Seminal stain extract is taken and Florence reagent is added & observed under
microscope, · It shows dark brown rhombic crystals · These crystals are choline iodide
Florence test
· Seminal stain extract is taken and Florence reagent is added & observed under microscope,
· It shows dark brown rhombic crystals
· These crystals are choline iodide
Option C. Takayama test: A test that was once widely used to test for the presence of blood.
Shows pink-feathery crystals, It is hemo-chromogen
Option D. Teichmann test: The Teichmann test is used for the detection of the presence of blood and t
he identification of bloodstains. Shows Dark-brown rhombic crystals,These crystals are hemin crystals

Solution for Question 25:


Correct Option: D
• Teichmann test is one of the microchemical confirmatory test used to detect the presence of trace
evidence of blood.
• In this test, the sample is made to react with NaCl and glacial acetic acid and then seen under the
microscope.
• Presence of brown rhombic crystals indicate the presence of blood stains in the sample. These type of
crystals are found due to the formation of hematin chloride.

Page 18

122
Incorrect Options:
Option A. Barberio test: It includes confirming yellow crystals that develop when semen is subjected to
an aqueous solution of picric acid under a microscope.

Option B. Florence test: This test is done to detect the presence of choline. When a
semen extract is exposed to iodine in a
potassium iodide solution, the Florence test can identify the presence of choline periodide crystals.
Option C. Takayama test: A confirmatory test used to find blood stains is the Takayama test. An assum
ed blood sample is placed on a slide before the Takayama reagent is added. The Takayama reagent is
added, and the slide is then dried at 115 degrees Celsius. After that, it is examined under a
microscope, where the presence of dark red, feathery crystals is a promising sign.

Page 19

123
Solution for Question 26:
Correct Option is C - Fingerprint pattern
• Identical twins share many genetic similarities due to their identical DNA. However, one area where
they may differ is in their fingerprint patterns. Fingerprint patterns are not solely determined by genetic
factors but also influenced by other developmental factors during the formation of the ridges and
patterns on the fingers. As a result, even identical twins will have distinct fingerprint patterns.
• Finger prints [FP] are · Not inherited · Different even in monozygotic twins [have same DNA pattern] ·
Even if epidermis is lost, FP can be obtained from dermis
Finger prints [FP] are
· Not inherited
· Different even in monozygotic twins [have same DNA pattern]
· Even if epidermis is lost, FP can be obtained from dermis
Incorrect Options
Option A: Blood group (Incorrect) Identical twins typically have the same blood group. Blood group is d
etermined by specific antigens present on the surface of red blood cells, and identical twins inherit the
same genetic information that determines their blood group.
Option B: DNA fingerprinting (Incorrect) DNA fingerprinting, also known as DNA profiling or genetic fing
erprinting, is a method used to identify individuals based on their unique DNA sequences. Identical twin
s will have virtually identical DNA profiles as they share the same genetic information.
Option D: Iris color (Incorrect) Iris color refers to the pigmentation of the iris, the colored part of the eye.
Identical twins often have similar iris colors as it is influenced by genetic factors. While there can be sli
ght variations in shade or intensity, the overall color of the iris is usually similar between identical twins.

Page 20

124
Solution for Question 27:
Correct Option B: Theory of exchange
• Locard is most famous for his theory of exchange, also known as Locard's Exchange Principle.
• This principle states that whenever two objects come into contact, there will be an exchange of
materials between them.
Incorrect Options
Option A - Study of fingerprints: While Edmond Locard did contribute to the field of forensic science, he
is not primarily known for the study of fingerprints.
Option C - Stature estimation: Stature estimation involves determining a
person's height based on skeletal remains.
Option D - Forensic ballistics: Forensic ballistics involves the analysis of firearms, ammunition, and the
examination of ballistic evidence.

Solution for Question 28:


Correct option A
• In forensic investigations, confirmatory tests are used to prove the presence of substances or
evidence. When it comes to identifying a bloodstain, the confirmatory test used is a spectroscopic test
(Option A). Spectroscopic tests, such as infrared spectroscopy or UV-visible spectroscopy, analyze the
absorption or emission of light by the sample to identify specific compounds or substances present in
the bloodstain. These tests can provide precise identification of blood components and distinguish them
from other substances.
Incorrect options:
Option B: Kastle Meyer test (Option B): The Kastle Meyer test is a presumptive test used to detect the
presence of blood based on the reaction between blood and phenolphthalein. It is not a
confirmatory test.
Option C: Benzidine test (Option C): The benzidine test is another presumptive test for detecting blood,
based on the reaction between blood and benzidine reagent. It is not a confirmatory test.
Option D: Orthotoluidine test (Option D): The orthotoluidine test is a
colorimetric test used to detect the presence of blood. It is also a presumptive test and not a
confirmatory test.

Solution for Question 29:


Correct Option A: Spermine
• Barberio’s Test is used to detect spermine crystals, using picric acid. Spermine is an important
component of prostatic secretion and is found in semen. On adding picric acid to the sample, yellow
needle shaped (rhombic) crystals are seen (also called spermine picrate crystals).

Page 21

125
Incorrect Options:
Option B: Acid phosphatase- Acid phosphatase test is done in cases where old semen is suspected to
be present in the crime scene.
Option C: Choline- Choline crystals are detected by florence test.
Option D: Fructose- Although fructose forms an important part of semen, it is not used often in semen
analysis in sexual offenses.

Page 22

126
Strangulation & Hanging
1. What is it called when a victim was strangled with an elbow around the neck?
A. Garroting
B. Mugging
C. Bansdola
D. Burking
----------------------------------------
2. On postmortem examination, a contusion of neck muscles is seen along with a fracture of the hyoid
bone. What is the most probable cause of death?
(or)
What is the cause of death in a postmortem examination showing neck muscle contusion and a
fractured hyoid bone?
A. Smothering
B. Mugging
C. Burking
D. Throttling
----------------------------------------
3. The forensic team has concluded the autopsy on the 35-year-old railway employee's body recovered
from the railway track about 2 km from the nearest station. Among the findings, the doctor has
documented the presence of ipsilateral dilatation of the pupil and partial opening of the eyelids. This is
known in forensic language as 'Le facies sympathique' and is seen in case of death due to which of the
following?
(or)
Condition is indicated by the presence of "Le facies sympathique" in cases of death?
A. Hanging
B. Strangulation
C. Throttling
D. Railway accidents
----------------------------------------
4. Where is the "knot" usually placed during a judicial hanging?
A. Behind the neck
B. Side of the neck
C. Below the chin
D. Choice of hangman
----------------------------------------
5. The body of the 28-year-old television actress has been found in the hotel room where she had been
staying. Initial findings by the forensic team include glove & stocking hypostasis, bruises around the

127
neck, protrusion of the tongue, and dribbling of saliva. What is the dribbling of saliva is suggestive of?
(or)
What does the dribbling of saliva in the 28-year-old actress's case suggest about her cause of death?
A. Antemortem hanging
B. Strangulation
C. Postmortem hanging
D. Throttling
----------------------------------------
6. The body of the 55-year-old businessman has been found in his beach house. Autopsy findings
reveal the presence of Simon's hemorrhage, which provided the forensic team with evidence of the
possible cause of death. Which among the following statement is true about Simon's hemorrhage?
(or)
Which among the following statement is true about Simon's hemorrhage?
A. Seen in cases of drowning
B. Bleeding onto outer layers of intervertebral discs
C. Seen in thoracic region
D. Pathognomic sign of asphyxia
----------------------------------------
7. The body of the 35-year-old businessman has been recovered in the bedroom of his house on the
outskirts of the city. Prima facie, appears to be a case of suicidal hanging. The family of the victim
suspects foul play as the victim did not show any signs of suicidal intentions prior to his death. All of the
following would be observed by the forensic team, in case of suicidal hanging, except?
(or)
All of the following seen in the case of suicidal hanging, except?
A. Dribbling of Saliva
B. Disruption of vertebral column
C. Gap in the skin mark at the point of suspension
D. Ecchymosis and bruises at the edges & floor of the ligature mark
----------------------------------------
8. A 25-year-old woman was rushed to the city hospital but was declared dead on arrival. The husband
had been with the victim at the time of death and hence, on grounds of suspicion, he was held in
custody by the police. The autopsy report submitted by the forensic team has confirmed it to be a case
of Isadora Duncan syndrome which in general terms is death caused by?
(or)
Isadora Duncan syndrome signify?
A. Suicidal Hanging
B. Poisoning
C. Accidental strangulation

Page 2

128
D. Schizophrenia
----------------------------------------
9. The body of the 27-year-old female tourist, who had been missing for a week, has been recovered
from near a waterfall. The forensic team has begun its investigation to determine the cause of death.
Among the findings, the team has documented the presence of Masque Ecchymotique, which indicates
the possible cause of death as?
(or)
What does the presence of Masque Ecchymotique indicate as a possible cause of death?
A. Ligature strangulation
B. Manual strangulation
C. Traumatic asphyxia
D. Burking
----------------------------------------
10. The body of the 48-year-old farmer has been retrieved from the field nearby. An autopsy has been
performed on the body. The face was congested and there were gloves and stocking hypostasis. On
examination of the hyoid bone, the doctor found the bone fractured and displaced outward. This is
commonly observed in a fracture sustained by which of the following?
(or)
In which scenario is a fractured and outwardly displaced hyoid bone commonly observed?
A. Manual strangulation
B. Ligature strangulation
C. Hanging
D. Bansdola
----------------------------------------
11. A teenage girl was found dead in her room. On postmortem examination following appearances
were found. Images show the appearance of the neck on external examination and findings of neck
dissection. Which of the following is the most probable cause of death in this case?
(or)
According to the below finding ,Which of the following is the most probable cause of death ?

A. Throttling

Page 3

129
B. Strangulation
C. Hanging
D. Traumatic asphyxia
----------------------------------------
12. Which cavity should be opened last in the suspicion of hanging?
(or)
Which cavity should be opened last in the suspicion of hanging?
A. Thorax
B. Head
C. Abdomen
D. Neck
----------------------------------------
13. A 50-year-old chronic alcoholic while having dinner suddenly becomes aphonic and is brought to
the casualty with the complaint of respiratory distress. What is the immediate management for this
patient?
(or)
What is the immediate management for a 50-year-old chronic alcoholic with sudden aphonia and
respiratory distress during dinner?
A. Cricothyroidotomy
B. Emergency tracheostomy
C. Humidified oxygen
D. Heimlich maneuver
----------------------------------------
14. The act of Burking includes which one of the following methods?
A. Choking
B. Ligature
C. Overlaying
D. Traumatic asphyxia
----------------------------------------
15. The cause of death of the 30-year-old IT employee, who was found dead in her apartment, was
confirmed to be due to asphyxia. Further investigations have been initiated to determine the method
employed by the killer to cause suffocation. Any of the following acts can lead to suffocation except?
(or)
Any of the following can lead to suffocation ,except?
A. Choking
B. Gagging
C. Smothering

Page 4

130
D. Throttling
----------------------------------------
16. What is the type of hanging in which the body is fully suspended and the feet are not touching the
ground?
(or)
What is the type of hanging in which the body is fully suspended and the feet are not touching the
ground?
A. Partial hanging
B. Complete hanging
C. Homicidal hanging
D. Suicidal hanging
----------------------------------------
17. A woman died in her room. Her room was unlocked. Her alcohol levels were 150 mg%. The image
is shown below. On neck dissection, there was a contusion present. What is the cause of the death?
(or)
During Postmortem on neck dissection, contusion was present on soft tissues . What is the cause of
the death?

A. Throttling
B. Smothering
C. Cafe coronary
D. Alcohol intoxication
----------------------------------------

Correct Answers
Question Correct Answer

Question 1 2
Question 2 4
Question 3 1

Page 5

131
Question 4 2
Question 5 1
Question 6 2
Question 7 4
Question 8 3
Question 9 3
Question 10 3
Question 11 1
Question 12 4
Question 13 4
Question 14 4
Question 15 4
Question 16 2
Question 17 1

Solution for Question 1:


Correct Option B - Mugging:
• Strangulation is caused by holding the neck of the victim in the bend of the elbow or knee of the
assailant.
Incorrect Options:
Option A - Garroting:
• Garroting: Strangulation is caused by compression of the neck by a ligature which is quickly tightened
by twisting it with a lever (rod, stick or ruler) known as Spanish windlass, which results in a sudden loss
of consciousness and collapse.
Option C - Bansdola:
• Bansdola: A bamboo or stick is placed across the back of the neck and another across the front. Both
ends are tied with a rope due to which the victim is squeezed to death.
Option D - Burking:
• It is a combination of homicidal smothering and traumatic asphyxia.
• One man does smothering while another seating over his chest causing traumatic asphyxia.

Solution for Question 2:


Correct Option D - Throttling:
• Throttling is a form of manual strangulation.
• The frequency of hyoid bone fracture is maximum in throttling.

Page 6

132
Incorrect Options:
Option A - Smothering:
• Smothering is a form of asphyxia caused by mechanical occlusion of external air passages, i.e. the
nose and mouth by hand, cloth, plastic bag, or other material.
Option B - Mugging:
• Strangulation is caused by holding the neck of the victim in the bend of the elbow or knee of the
assailant.
Option C - Burking:
• It is a combination of homicidal smothering and traumatic asphyxia.
• One man does smothering while another seating over his chest causing traumatic asphyxia.

Solution for Question 3:


Correct Option A - Hanging:
• The eyes are closed or partially open, and the pupils are usually dilated. If the ligature knot presses on
cervical sympathetic, the eye on the same side may remain open and its pupil dilated (le facie
sympathique).
• It indicates antemortem hanging.
Incorrect Options:
Option B - Strangulation:
• It is a form of violent asphyxial death caused by constriction of air passage at the neck by means of a
ligature or by any means other than suspension of the body.
• Classification: Ligature strangulation, manual strangulation or throttling, mugging, bansdola, garroting,
etc.
Option C - Throttling:
• Throttling or Manual Strangulation: Asphyxia produced by compression of the neck by human hands.
• Inward compression fracture of the hyoid bone is the most diagnostic finding of throttling.
Option D - Railway accidents:
• Le facie sympathique is one of the surest sign of ante mortem hanging, not from railway accidents.

Solution for Question 4:


Correct Option B - Side of the neck:
• Judicial hanging in India, the knot is placed near the side of the neck.
• In India, the site is the left lateral side (sub-aural) at the angle of the mandible.
Incorrect Options:
Option A - Behind the neck:

Page 7

133
• It is an incorrect option.
• In homicidal cut-throat, blood stains are found on both palms in an effort to cover the wound; if lying
down, stains collect behind the neck and shoulder.
Option C - Below the chin:
• Judicial hanging in India, the knot is placed near the side of the neck.
• But the knot below the chin is highly effective.
Option D - Choice of hangman:
• The knot in Judicial hanging does not depend on the choice of hangman.

Solution for Question 5:


Correct Option A - Antemortem hanging:
• Dribbling of saliva: Surest sign of antemortem hanging.
• Excessive salivation occurs when the person is alive, due to pressure and friction caused by ligature
material on the submandibular glands.
• Dribbling of saliva occurs from the angle of the mouth which is at a lower level, i.e. from the angle
opposite to the side of the knot.
• When the knot is on the nape of the neck, it occurs across the middle of the lower lip.
Incorrect Options:
Option B - Strangulation:
• It is a form of violent asphyxial death caused by constriction of air passage at the neck by means of a
ligature or by any means other than suspension of the body.
• Classification: Ligature strangulation, manual strangulation or throttling, mugging, Bansdola,
Garroting, etc.
Option C - Postmortem hanging:
• Dribbling of saliva is the surest sign of antemortem hanging, not postmortem hanging.
Option D - Throttling:
• Throttling or Manual Strangulation: Asphyxia produced by compression of the neck by human hands.
• Inward compression fracture of the hyoid bone is the most diagnostic finding of throttling.

Solution for Question 6:


Correct Option B - Bleeding onto outer layers of intervertebral discs:
• Simon's bleedings are hemorrhages into the anterior aspect of the intervertebral discs of the lumbar
region and are usually considered a classic sign of vitality in cases of hanging with full suspension of
the body.
Incorrect Options:

Page 8

134
Option A - Seen in cases of drowning: Simon’s hemorrhages are seen in the case of hanging.
Option C - Seen in thoracic region: Simon’s hemorrhages are seen in the lumbar region.
Option D - Pathognomic sign of asphyxia: Simon's bleedings, in cases of asphyxiation, most likely occu
r due to agonal convulsions and forced movements in the lumbosacral part of the spinal column.

Solution for Question 7:


Correct Option D - Ecchymosis and bruises at the edges & floor of the ligature mark:
• Prominent ecchymosis and bruises at the edges & the floor of the ligature mark are suggestive of
ligature strangulation.
• Peri-ligature injuries and hemorrhages are seen.
Incorrect Options:
Option A - Dribbling of Saliva:
• M/c finding of AM hanging .
• Not present in every case.
• It is due to stimulation of the Pterygopalatine Ganglion (submandibular gland).
• Opposite side of the knot.
Option B - Disruption of vertebral column:
• Disruption of the vertebral column can be seen in case of suicidal hanging.
Option C - Gap in the skin mark at the point of suspension:
• Direction: It runs obliquely, backward, non-continuous, upwards, and towards the point of suspension.
• Suspension peak: Mark is non-continuous because of a gap at the nape of the neck, and hair
intervening between ligature material and the skin underneath.

Solution for Question 8:


Correct Option C - Accidental strangulation:
• Accidental strangulation due to a scarf getting caught in the wheels of a vehicle or machine was called
“Isadora Duncan Syndrome” or “Long Scarf Syndrome”.
Incorrect Options:
Option A - Suicidal Hanging:
• Isadora Duncan syndrome is caused by accidental strangulation; not suicidal hanging.
Option B - Poisoning:
• Poisoning occurs when any substance interferes with normal body functions after it is swallowed,
inhaled, injected, or absorbed.
• Isadora Duncan syndrome is not caused by poisoning.

Page 9

135
Option D - Schizophrenia:
• Schizophrenia is a serious mental disorder in which people interpret reality abnormally.
• Schizophrenia may result in some combination of hallucinations, delusions, and extremely disordered
thinking and behaviour that impairs daily functioning, and can be disabling.

Solution for Question 9:


Correct Option C - Traumatic asphyxia:
• The presence of Masque Ecchymotique, indicates traumatic asphyxia.

Incorrect Options:
Option A - Ligature strangulation:
• The ligature mark is transverse, complete below the thyroid & cartilage (mostly).
• The ligature mark of hanging due to the slip knot is situated above the thyroid cartilage.

Page 10

136
Option B - Manual strangulation:
• Constriction of the neck by hands/ fingers manually is known as manual strangulation.
Option D - Burking:
• It is a combination of Homicidal smothering + Traumatic asphyxia.
• One person sits on the chest of the victim, while another person covers the mouth & nostrils of the
victim.

Solution for Question 10:


Correct Option C - Hanging:
• In hanging, the fracture is usually due to ligature forcing the hyoid bone backward, which results in
increased divergence of greater horns (anteroposterior compression fracture), but it can be a traction
fracture.
Incorrect Options:
Option A - Manual strangulation:
• Manual strangulation or throttling: When human fingers, palms, or hands are used to compress the
neck.
• Inward compression fracture of the hyoid bone is the most diagnostic finding of throttling.
Option B - Ligature strangulation:
• Ligature strangulation: When ligature material is used to compress the neck.
• Injury of the hyoid bone is not commonly noticed, because the level of constriction is well below, and
traction on the thyrohyoid ligament is negligible.
• Their face is congested, swollen and cyanosed. Tardieu’s spots are present on the forehead, temples,
eyelids and conjunctiva; more abundant than in hanging.

Page 11

137
• Eyes are prominent, wide open, conjunctiva congested, pupils dilated and subconjunctival
hemorrhages are present.
• The tongue is swollen, dark-coloured, may protrude out of the mouth, and be bitten by teeth.
• A ligature mark is a well-defined groove, which is slightly depressed and of the same width as that of
ligature material. The groove may be narrow at parts due to the folding of the ligature.
Option D - Bansdola:
• Bansdola: A bamboo or stick is placed across the back of the neck and another across the front. Both
ends are tied with a rope due to which the victim is squeezed to death.
• When a foot or knee is placed across the front of the throat and pressed while the victim is lying on the
ground, the same condition will follow. If a stick or foot is used, a bruise is seen in the centre, across the
trachea corresponding to the width of the object used.

Solution for Question 11:


Correct Option A - Throttling:
• Externally contusion marks on fingernails are seen and internally severe contusion marks on soft
tissue are present.
• This is characteristic of manual strangulation or throttling.
Incorrect Options:
Option B - Strangulation:
• Neck constriction by any means with no body suspension is known as strangulation.
• Strangulation will cause a ligature mark over the neck. So this option is wrong.
Option C - Hanging:
• It is a form of asphyxia, caused by the suspension of the body by a ligature which encircles the neck.
• Constricting force is the weight of the body.
• Hanging and strangulation will cause a ligature mark over the neck. So it is excluded.
Option D - Traumatic asphyxia:
• Asphyxia resulting from respiratory arrest due to mechanical fixation of the chest, so that the normal
movements of the chest wall are prevented.
• One man does smothering while another seating over his chest causing traumatic asphyxia.

Solution for Question 12:


Correct Option D - Neck:
• The neck should be opened at last so as to drain the blood and get a bloodless field to facilitate proper
examination of injuries.

Page 12

138
Incorrect Options:
Option A - Thorax: Thorax is opened next to the cranium.
Option B - Head: The head (cranium) should be opened first.
Option C - Abdomen: The abdomen is opened right after the thorax.

Solution for Question 13:


Correct Option D - Heimlich maneuver:
• Treatment for choking: If there is difficulty in breathing and cyanosis, give first aid by application of
pressure on the abdomen (Heimlich maneuver) till the patient recovers or loses consciousness.
Incorrect Options:
Option A - Cricothyroidotomy:
• Cricothyroidotomy also called cricothyrotomy, inferior laryngotomy, inter-cricothyrotomy, coniotomy, or
emergency airway puncture.
• It is an incision made through the skin and cricothyroid membrane to establish a patent airway during
certain life-threatening situations, such as airway obstruction by a foreign body, angioedema, or
massive facial trauma.
Option B - Emergency tracheostomy:
• In rare cases, an emergency tracheotomy is performed when the airway is suddenly blocked, such as
after a traumatic injury to the face or neck.
Option C - Humidified oxygen:
• Standard oxygen gases have a drying effect on the mucous membranes that can result in airway
damage as well as heat and fluid loss.
• Humidified oxygen reduces this effect and can assist in breaking down a patient’s respiratory
secretions, making them easier to clear.
• Humidified oxygen is most effective when the gas reaching the alveoli is at body temperature (37ºC)
with a relative humidity of 100%.
• Oxygen is passed through a humidifying device producing sterile vapour before travelling in elephant
tubing to a face mask covering the patient’s nose and mouth.

Solution for Question 14:


Correct Option D - Traumatic asphyxia:
• Burking is a combination of homicidal smothering and traumatic asphyxia.
Incorrect Options:
Option A - Choking:
• Choking is a form of asphyxia caused by an obstruction within the air passages by a foreign object,
like a coin, fruit seed, toffees, candies, fish, or any other material.

Page 13

139
Option B - Ligature:
• Ligature strangulation: When ligature material is used to compress the neck.
• A ligature mark is a well-defined groove, which is slightly depressed and of the same width as that of
ligature material. The groove may be narrow at parts due to the folding of the ligature.
Option C - Overlaying:
• Overlaying or compression suffocation results from compression of the chest, nose, and mouth, so as
to prevent breathing.

Solution for Question 15:


Correct Option D - Throttling:
• Throttling is a form of manual strangulation, not a form of suffocation.
Incorrect Options:
Option A - Choking:
• Choking is a form of asphyxia caused by an obstruction within the air passages by a foreign object,
like a coin, fruit seed, toffees, candies, fish, or any other material.
Option B - Gagging:
• Gagging is a form of asphyxia that results from pushing a gag (rolled-up cloth or paper balls) into the
mouth, sufficiently deep to block the pharynx.
• It combines the features of smothering and choking.
Option C - Smothering:
• Smothering is a form of asphyxia caused by mechanical occlusion of external air passages, i.e. the
nose and mouth by hand, cloth, plastic bag or other material.

Solution for Question 16:


Correct Option B Complete hanging:
• The body is fully suspended and no part of the body touches the ground.
• Constricting force is the weight of the body.
Incorrect Options:
Option A - Partial hanging:
• Incomplete or partial hanging: Lower part of the body is touching the ground (toes or feet touching the
ground) or in a sitting, kneeling, lying down, or prone position.
• The weight of the head acts as the constricting force.
Option C - Homicidal hanging:
• Homicidal hanging is very rare. Not ordinarily possible in an adult victim, unless intoxicated or made
unconscious or the victim is either a child or a debilitated person.

Page 14

140
• Homicide should be suspected where: There are signs of violence/disorder in furniture. The clothing
of the deceased is torn or disarranged. There are injuries, either offensive or defensive.
• There are signs of violence/disorder in furniture.
• The clothing of the deceased is torn or disarranged.
• There are injuries, either offensive or defensive.
• There are signs of violence/disorder in furniture.
• The clothing of the deceased is torn or disarranged.
• There are injuries, either offensive or defensive.
Option D - Suicidal hanging:
• The point of the suspension remains approachable to the suicider.
• Partial hanging is almost always suicidal in nature.
• A history of a previous attempt may be present and generally committed in a secluded place (the
victim’s home is the most frequent site).
• A suicidal note may be left behind.
• There should be a motive for committing suicide.
• Fibers of ligature material may be present in the clenched hand.

Solution for Question 17:


Correct Option A - Throttling:
• On neck dissection, there was a contusion present which can also be seen in the given picture;
suggestive of throttling or manual strangulation.
Incorrect Options:
Option B - Smothering:
• Smothering: Death from mechanical occlusion of the mouth and nose, excluded.
Option C - Cafe coronary:
• Café coronary: Sudden and unexpected death occurring during a meal due to accidental occlusion of
the airway by food.
• No signs of airway obstruction are given in the question moreover contusion is present over the neck
which cannot be explained by café coronary, so excluded.
Option D - Alcohol intoxication:
• She is under alcohol intoxication but the level is not sufficient to cause death.
• Moreover, contusion present over the neck cannot be explained by alcohol intoxication, so excluded.

Page 15

141
Asphyxial Deaths & Drowning
1. The body of the 27-year-old woman has been recovered from the site where it was buried after the
suspect confessed to the crime. Autopsy findings suggest that the woman has been physically and
sexually assaulted but death has occurred secondary to traumatic asphyxia. Traumatic asphyxia is a
type of which of the following?
(or)
Traumatic asphyxia is a type of which of the following?
A. Hanging
B. Ligature strangulation
C. Mechanical asphyxia
D. Manual strangulation
----------------------------------------
2. What is the mechanism of death when a victim is wedged between a wall and the corner of a bed?
(or)
What is the mechanism of death when a victim is wedged between a wall and the corner of a bed?
A. Gagging
B. Choking
C. Smothering
D. Wedging
----------------------------------------
3. The body of the 24-year-old German female tourist in Goa has been recovered. Prima facie, appears
to be a case of abduction, sexual assault and death by suffocation. Both the conjunctiva showed signs
of Bayard's spots. Which one statement among the following is true about Bayard's spots?
(or)
Which one statement among the following is true about Bayard's spots?
A. Due to the rupture of arterioles
B. Pathognomic of asphyxia
C. Appear with putrefaction
D. Well-defined, dark red spots seen over conjunctiva, subpleural surface of lungs, heart etc.
----------------------------------------
4. A 32-year-old construction worker is brought to the emergency room after a workplace accident. He
accidentally fell down from the first floor by hanging on a rope. What is the amount of force over the
neck required to compress & fracture the cricoid cartilage?
(or)
What is the amount of force over the neck required to compress & fracture the cricoid cartilage?
A. 5 kg
B. 9 kg

142
C. 18.8 kg
D. 30 kg
----------------------------------------
5. Read the following statements and choose the correct answer- a. Smothering is caused by an
obstruction within air passages b. Overlaying is due to compression of the chest c. Choking is caused
by obstruction of external air passages d. Gagging is forcing the cloth into the mouth
A. a, b, c are correct
B. a and c are correct
C. b and d are correct
D. All four (a, b, c, & d) are correct
----------------------------------------
6. What is the outer covering of a diatom made up of?
(or)
what is the outer covering of a diatom made up of?
A. Magnesium
B. Silica
C. Hydrocarbons
D. None
----------------------------------------
7. After 5 days, the search team recovered the body of the 10-year-old boy who had gone missing while
swimming in the lake. The skin of the boy showed signs of cutis anserina and the hands were seen
clenched with weed and gravel in it. Which of the following findings would also be seen in a case of
antemortem drowning?
(or)
Which of the following findings can be seen in a case of antemortem drowning?
A. Water in the small intestine
B. Emphysema aquosum
C. Hemorrhage in the middle ear
D. All of the above
----------------------------------------
8. The usual boat ride off the coast of Chennai turned tragic when the boat capsized with all 7 tourists
and the boatman on board. 5 of them knew how to swim and managed to stay afloat. They were
rescued by the coast guards within 30 mins. The 2 others were separated from the rest by the strong
current. Their bodies were recovered and sent for examination. Among the findings, the forensic doctor
has documented the presence of Emphysema Aquosum. Which one of the following is that finding
associated with?
(or)
Emphysema Aquosum is seen in ?
A. Dry drowning

Page 2

143
B. Wet drowning
C. Immersion syndrome
D. Secondary drowning
----------------------------------------
9. A 55-year-old man who had gone camping in Manali on New year's eve was found dead in a lake the
next morning. He had been part of the celebrations the previous night and was seen drinking and
smoking hookah with his group of friends. Autopsy reports confirm that he died of drowning in cold
water. Among the following, what causes the death in cold water drowning?
(or)
What is the cause of death in cold water drowning ?
A. Vagal inhibition of the heart
B. Pulmonary edema
C. Loss of consciousness
D. Ventricular fibrillation
----------------------------------------
10. After 4 days, the body of the 35-year-old woman had washed ashore on the Mahabalipuram beach
in Chennai. A rigorous search by the coast guard team had proved to be futile as the sea had been
rough for last week. The lungs of the woman were being examined by the forensic doctor to confirm
death by drowning in the sea. What change among the following would one not expect in the lungs if
death was indeed by drowning in the sea?
(or)
Which of the following changes would NOT be expected in the lungs if death was due to drowning in
the sea?
A. Ballooned and heavy lungs
B. Tend to flatten out on removing
C. Crepitus is heard on sectioning
D. Shape of a sectioned portion is not retained
----------------------------------------
11. The forensic team has received the body of the tourist who had drowned in a bathtub in the hotel
room he was staying in. After a thorough investigation, the cause of death was confirmed to be
hydrocution, which happens when the body comes in contact with?
(or)
What is the specific condition that caused hydrocution in the tourist who drowned in the hotel room's
bathtub?
A. Water that is at least 5°C lower than body temperature
B. Water that is at least 5°C higher than body temperature
C. Water that is at least 15°C lower than body temperature
D. Water that is at least 15°C higher than body temperature
----------------------------------------

Page 3

144
12. On examination of the lungs of the 10-year-old boy's body, the forensic doctor noticed large
patches of hemorrhage. After identifying the location of the hemorrhage, the doctor documented the
finding as Paltauf's hemorrhage. All of the following statements are true about it, except?
(or)
All of the following statements are true about Paltauf's hemorrhage, except?
A. Sign of drowning
B. Subpleural hemorrhage
C. Mostly seen in middle lobe of lungs
D. Due to rupture of alveolar walls
----------------------------------------
13. While vacationing in a coastal town, a 25-year-old woman was caught in a strong undertow while
swimming, and despite a lot of efforts she cold not be rescued alive by the lifeguard. Which of the
following is not true regarding sea water drowning?
(or)
Which of the following is not true regarding sea water drowning?
A. Lungs are ballooned and heavy
B. Lungs are pale pink in colour
C. Tend to flatten out after taken from the body
D. On cut section crepitus is not heard
E. Little froth appears on cut section
----------------------------------------
14. A famous actor allegedly committed suicide by hanging recently. On examination, there was
evidence of dribbling of saliva, protrusion of the tongue, and ligature marks around the neck. What kind
of tissue damage do ligature markings represent?
(or)
What kind of tissue damage do ligature marks represent in cases of hanging?
A. Contusion
B. Pressure abrasion
C. Laceration
D. Burn
----------------------------------------

Correct Answers
Question Correct Answer

Question 1 3
Question 2 4
Question 3 4

Page 4

145
Question 4 3
Question 5 3
Question 6 2
Question 7 4
Question 8 2
Question 9 1
Question 10 3
Question 11 1
Question 12 3
Question 13 T,F,T,T,F
Question 14 2

Solution for Question 1:


Correct Option C -Mechanical asphyxia:
• Traumatic asphyxia: Asphyxia resulting from respiratory arrest due to mechanical fixation of the chest,
so that the normal movements of the chest wall are prevented. One man does smothering while
another seating over his chest causing traumatic asphyxia.
Incorrect Options:
Option A - Hanging:
• Hanging is a form of asphyxia caused by the suspension of the body by a ligature that encircles the
neck, the constricting force being at least part of the weight of the body.
Option B - Ligature Strangulation:
• A strangulation is a form of violent asphyxial death caused by constriction of air passage at the neck
by using a ligature or by any means other than suspension of the body.
• Classification: Ligature strangulation, manual strangulation or throttling, mugging, bansdola, garroting,
etc.
• Ligature strangulation: When ligature material is used to compress the neck.
Option D - Manual strangulation:
• Manual strangulation or throttling: When human fingers, palms or hands are used to compress the
neck.

Solution for Question 2:


Correct Option D - Wedging:
• Wedging: Occlusion of the internal airways by external pressure.
• Is a form of mechanical asphyxia in which the face, neck or thorax is compressed between two firm
structures.

Page 5

146
• It is common in 3–6 months old children when they start to move to the corners of beds and cribs, but
they do not have the muscle development to free themselves out of a wedged position.
• They tend to get wedged between the mattress and either wall of the bed
Incorrect Options:
Option A - Gagging: Gagging: Results from forcing a cloth into the nasopharynx.
Option B - Choking: Choking: Caused by an obstruction within the air passages usually between the ph
arynx and bifurcation of the trachea.
Option C - Smothering: Smothering: Caused by closing the external respiratory passages either by han
d or other means.

Solution for Question 3:


Correct Option D
-Well-defined, dark red spots seen over conjunctiva, subpleural surface of lungs, heart etc.
• They are usually round, dark-red, well-defined, pin-head sized spots, found in those parts where
capillaries are least supported, e.g. conjunctiva, face, epiglottis, subpleural surface of lungs, heart,
meninges and thymus.
Incorrect Options - A/B/C
Option A -Due to the rupture of arterioles
• Found in those parts where capillaries are least supported.
Option B - Pathognomic of asphyxia
• They are not pathognomic of asphyxia, and their absence does not exclude asphyxia (rarely seen in
drowning).
Option C-Appear with putrefaction
• Disappear with putrefaction.

Solution for Question 4:


Correct Option C - 18.8 kg:
• The amount of force over the neck required to compress & fracture the cricoid cartilage is 18.8 kg.
Incorrect Options:
Option A - 5 kg: The amount of force over the neck required to occlude the carotid arteries is 5 kg.
Option B - 9 kg: The amount of force over the neck required to compress the trachea is 9 kg.
Option D - 30 kg: The amount of force over the neck required to occlude the vertebral arteries is 30 kg.

Page 6

147
Solution for Question 5:
Correct Option C - If b and d are correct:
b. Overlaying is due to compression of the chest
• Overlaying: Due to compression of the chest to prevent breathing; usually occurs when the mother or
other person shares a bed with an infant.
d. Gagging is forcing the cloth into the mouth
• Gagging: Results from forcing a cloth into the nasopharynx.
Incorrect
a. Smothering is caused by an obstruction within air passages
• Smothering: Caused by closing the external respiratory passages either by hand or other means.
c. Choking is caused by obstruction of external air passages
• Choking: Caused by an obstruction within the air passages usually between the pharynx and
bifurcation of the trachea.

Solution for Question 6:


Correct Option B - Silica:
• A unique feature of diatom anatomy is that they are surrounded by a cell wall made of silica (hydrated
silicon dioxide), called a frustule.
Incorrect Options:
Option A,C and D are incorrect as diatoms are surrounded by a cell wall made of silica.

Solution for Question 7:


Correct Option D - All of the above:
• All the mentioned signs suggest signs of antemortem drowning.
Option A - Water in the small intestine:
• The presence of water in the stomach & small intestine is included in the internal finding of
antemortem drowning.
Option B - Emphysema aquosum:
• Emphysema aquosum develops only when the conscious victim of drowning struggles for survival.
• Froth is present because there is mucus (Frothy water). Lung shows marbled appearance (alveoli
ruptured, damaged,thin).
Option C - Hemorrhage in the middle ear:
• The presence of water and hemorrhage in the middle ear also be seen in the case of antemortem
drowning.

Page 7

148
Solution for Question 8:
Correct Option B - Wet drowning:
• Emphysema aquosum is a sign of wet drowning when conscious people drown.
Incorrect Options:
Option A - Dry drowning:
• In dry drowning, water does not enter the lungs, but death results from immediate sustained laryngeal
spasm and cardiac arrest due to vagal inhibition.
• This results in asphyxia & death
Option C - Immersion syndrome:
• Also known as hydrocution, submersion inhibition or cold water drowning.
• It refers to syncope resulting from cardiac dysrhythmias on sudden contact with water that is at least
5°C lower than body temperature.
• Mechanism: Vagal stimulation leading to asystolic cardiac arrest (‘diving reflex’), or ventricular
fibrillation secondary to QT prolongation after a massive release of catecholamine on contact with cold
water. The resultant loss of consciousness leads to secondary drowning.
Option D - Secondary drowning:
• Near drowning (post-immersion syndrome or secondary drowning).
• Near drowning refers to survival beyond 24 h after a submersion episode.
• Death is caused by complications or sequelae (e.g. ARDS, pneumonia, sepsis, hypoxic-ischemic
encephalopathy, cerebral edema and DIC).
• Secondary drowning sometimes refers to a victim who initially responds well to resuscitation but then
suffers respiratory decompensation.

Solution for Question 9:


Correct Option A - Vagal inhibition of the heart:
• This phenomenon is called as HYDROCUTION.
• Aka Immersion Syndrome/ Vagal inhibition of Heart
• MC seen with cold water, particularly when its temperature is <5°C of the body temperature of the
victim
• When cold water comes in contact with skin, it stimulates skin receptors/ causes direct Epigastric
blow.
• Both of these cause vagal simulation, resulting in bradycardia and cardiac arrest.
• This cardiac arrest is also k/a Vagal Inhibition of heart
• Here the water is not entering into the lung. So, it is also a type of DRY DROWNING

Page 8

149
• Alcohol increases such effects.
Aka Immersion Syndrome/ Vagal inhibition of Heart
MC seen with cold water, particularly when its temperature is <5°C of the body temperature of the victi
m
When cold water comes in contact with skin, it stimulates skin receptors/ causes direct Epigastric blow.
Both of these cause vagal simulation, resulting in bradycardia and cardiac arrest.
This cardiac arrest is also k/a Vagal Inhibition of heart
Here the water is not entering into the lung. So, it is also a type of DRY DROWNING
Incorrect Options:
Option B - Pulmonary edema:
• In both freshwater and saltwater drowning, there is terminal pulmonary edema. It is an incorrect
option.
Option C - Loss of consciousness:
• Vagal inhibition of the heart causes death when drowning in cold water, not loss of consciousness.
• There may be loss of consciousness due to drinking and smoking hooka, but it is not the cause of
death.
Option D - Ventricular fibrillation:
• In freshwater drowning, death results from ventricular fibrillation.
• While in salt water, it is due to cardiac arrest from fulminant pulmonary edema and associated
changes.

Solution for Question 10:


Correct Option C - Crepitus is heard on sectioning:
• On the cut section, crepitus is not heard and a copious amount of fluid and froth oozes out.
Incorrect Options:
Option A - Ballooned and heavy lungs: Sea/saltwater drowning shows ballooned and heavy lungs.
Option B
- Tend to flatten out on removing: In sea/ saltwater drowning: Lungs tend to flatten out on removal.
Option D - Shape of sectioned portion is not retained: The shape is not retained on taking out from the
body and tends to flatten out.

Solution for Question 11:


Correct Option A - Water that is at least 5°C lower than body temperature:
• Hydrocution refers to syncope resulting from cardiac dysrhythmias on sudden contact with water that
is at least 5°C lower than body temperature.

Page 9

150
Incorrect Options:
Option B - Water that is at least 5°C higher than body temperature: Water that is at least 5°C higher tha
n body temperature is hot and does not cause hydrocution.
Option C - Water that is at least 15°C lower than body temperature: Water that is at least 15°C lower th
an body temperature does not cause hydrocution.
Option D - Water that is at least 15°C higher than body temperature: Water that is at least 15°C higher
than body temperature is hotter and can cause hydrocution. But the minimal requirement is 5°C

Solution for Question 12:


Correct Option C - Mostly seen in middle lobe of lungs:
• Paltauf’s hemorrhage is mostly seen in the lower lobes on the anterior surface and margins of the
lungs.
Incorrect Options:
Option A - Sign of drowning: Paltauf’s hemorrhage is seen in drowning.
Option B - Subpleural hemorrhage: Hemorrhages on the anterior surface of the lungs, present subpleur
ally with shining pale pink or bluish-red in colour, usually 1cm to 2cm in diameter are called Paltauf he
morrhages.
Option D - Due to rupture of alveolar walls: Paltauf’s hemorrhages are produced due to the rupture of a
lveolar walls as a result of increased pressure produced during forced expirations.

Solution for Question 13:


Correct Options:
Option A - Lungs are ballooned and heavy:
• Sea/ saltwater drowning show ballooned and heavy lungs.
Option C - Tend to flatten out after taken from the body:
• In sea/ saltwater drowning: Lungs tend to flatten out on removal.
Option D - On cut section crepitus is not heard:
• On the cut section, crepitus is not heard and a copious amount of fluid and froth oozes out.
Incorrect Options:
Option B - Lungs are pale pink in colour:
• It is purplish or bluish in colour.
Option E - Little froth appears on cut section:
• On the cut section, a copious amount of fluid and froth oozes out.

Page 10

151
Solution for Question 14:
Correct Option B - Pressure abrasion:
• A ligature mark seen in the case of hanging is a typical example of pressure abrasion.
Incorrect Options:
Option A - Contusion:
• Bruise/Contusion is the extravasation of blood in the subcutaneous/sub-epithelial tissues due to the
rupture of blood vessels, usually capillaries, as a result of blunt force injury or pressure.
Option C - Laceration:
• A laceration is the tearing or splitting of skin, mucous membranes, muscles, or internal organs caused
by either a shearing or a crushing force, and produced by the application of a blunt force to a broad
area of the body.
• If the blunt force produces extensive bruising and laceration of deeper tissue, it is called a crush
injury.
Option D - Burn:
• Burn Injury caused by heat, or by a chemical or physical agent having an effect similar to heat.

Page 11

152
Previous Year Questions
1. Identify the type of homicide

A. Mugging
B. Garrotting
C. Burking
D. Bansdola
----------------------------------------
2. A 24 year old man who was accused of murder was taken into police custody. As a part of the
interrogation, the prisoner was kept in a prone position with both his wrists and ankles bound behind his
back and secured with a rope. What is this method known as ?
A. Choke/ carotid hold
B. Carotid steeper hold
C. Hog tie
D. Bar arm
----------------------------------------
3. What is the term used to describe the act of a friend causing the death of another person by sitting
on their chest and blocking their nose and mouth, following an argument over a shared girlfriend, while
under the influence of alcohol?
A. Burking
B. Smothering
C. Traumatic asphyxia
D. Overlaying
----------------------------------------
4. In a case where an autopsy is performed on a person who died by hanging, a ligature mark is
observed on the lower one-third of the neck. The person's tongue appears to be protruding, and the
head was found hanging to the left side with saliva dripping from the left corner of the mouth.
Additionally, both pupils are dilated, and the right eye remains open. Which structure is most likely
being compressed, leading to the right eye remaining open in this case?
A. Right internal jugular vein

153
B. Cervical sympathetic chain
C. Left vagus nerve
D. Right internal carotid artery
----------------------------------------
5. What is the specific name of the test performed by the doctor during the autopsy where they tied the
bronchus and submerged the lung in water to determine if it floats or sinks?
A. Gettler’s test
B. Ploucquet’s test
C. Hydrostatic test
D. Diatom’s test
----------------------------------------
6. What is the cause of death in a case where a deceased individual has a ligature fully encircling the
neck at a horizontal level below the thyroid, as observed during post-mortem examination, and no
evidence of saliva dribbling?
A. Throttling
B. Ligature strangulation
C. Gagging
D. Hanging
----------------------------------------
7. What is this finding suggestive of

A. Drowning after Organophosphate poisoning


B. Antemortem drowning
C. Drowning after Arsenic poisoning
D. None of the above
----------------------------------------
8. What type of strangulation method is the Spanish windlass?
A. Bansdola
B. Garrotting

Page 2

154
C. Mugging
D. Throttling
----------------------------------------
9. A girl was found deceased in a lake and her body was retrieved. Which of the following is false
regarding ante mortem drowning?
(or)
Which of the following is false regarding ante mortem drowning?
A. Water in the stomach
B. Froth on nose and mouth
C. No Mud and vegetation in respiratory tract
D. Cadaveric spasm in hand muscle
----------------------------------------
10. In judicial hanging, the cause of death is due to
(or)
In judicial hanging, the cause of death is due to
A. Vagal inhibition
B. Cervical vertebral fracture or dislocation
C. Cerebral hypoxia
D. Asphyxia
----------------------------------------
11. In which type of hanging does the weight of the head act as a constricting force?
A. Complete
B. Partial
C. Typical
D. Atypical
----------------------------------------
12. A person was found dead in the bushes with his hands tied. On examination of the body, there was
cyanosis petechial haemorrhages, abrasions, and bruises around the mouth and lips. There was
associated facial congestion and edema. What is the least likely cause of death here?
A. Asphyxia
B. Suicide
C. Smothering
D. Homicide
----------------------------------------

Correct Answers

Page 3

155
Question Correct Answer

Question 1 3
Question 2 3
Question 3 1
Question 4 2
Question 5 3
Question 6 2
Question 7 2
Question 8 2
Question 9 3
Question 10 2
Question 11 2
Question 12 2

Solution for Question 1:


Answer Option C
• Burking is a type of homicide caused by smothering and traumatic asphyxia. In this method, the
attacker kneels or sits on the chest of the victim and closes the victim’s mouth and nose with his or her
hands.
Incorrect Choices:
• Option a. Mugging is a method of strangulation caused by holding the neck of the victim in the bend of
the elbow. The bend causes exertion of pressure on the larynx. The forearm and upper arm cause
lateral pressure on one or both sides of the neck. The attacker usually attacks from behind.
• Option b. Garrotting is a method of strangulation caused by a ligature being thrown around the neck
and quickly tightening it. There is a sudden loss of consciousness and collapse. The attacker attacks
from behind and this method is usually used in lonely places to kill the victim and rob them.
• Option d. Bansdola is a method of strangulation that uses two bamboo sticks. One bamboo stick is
placed across the back of the neck and another one in front. Both the sticks are then tied together
which causes the squeezing of the victim to death. The method can also be modified when one
bamboo stick is placed in front of the neck and the second is replaced by the hands or the knee of the
assailant.

Solution for Question 2:


Hog tie is a method of restraining an individual by binding their wrists and ankles together behind their
back, typically using ropes or other restraints. This position renders the person immobilized and limits t
heir ability to move or escape. It is commonly used in law enforcement or security situations to prevent
individuals from fleeing or posing a threat.
Incorrect Choices:

Page 4

156
Option A: Choke/carotid hold: A choke or carotid hold involves applying pressure to the neck, specifical
ly targeting the carotid arteries and jugular veins, with the intention of causing temporary unconsciousn
ess or rendering the person incapacitated. It is a controversial technique and is not typically used as a
method of restraining a prisoner in a prone position with bound wrists and ankles.
Option B: Carotid steeper hold: The term "carotid steeper hold" does not refer to a
recognized or commonly used technique. It may be a misspelling or a
term that is not widely known or utilized in relation to law enforcement or restraint methods.
Option D: Bar arm: "Bar arm" does not correspond to a
specific method of restraint or positioning described in the scenario. It is not a
term commonly used in law enforcement or security contexts.

Solution for Question 3:


Correct Choice: A
• Burking is a term derived from the name of notorious murderers William Burke and William Hare. In
the early 19th century, these individuals were involved in a series of murders in Edinburgh, Scotland.
Their method of killing involved sitting on the victims' chests and compressing their chests and
abdomen, thereby preventing proper breathing and leading to asphyxia and death. Burking accurately
describes the act of sitting on the victim's chest and obstructing their airway, resulting in asphyxia and
death.

Incorrect Choices:
Option B. Smothering: Smothering refers to the act of covering or suffocating someone, usually by plac
ing an object or substance over their nose and mouth to prevent them from breathing. While similar to
burking, the key distinction is that burking specifically involves sitting on the victim's chest to compress
their chest and abdomen. Therefore, smothering alone would not accurately describe the described sc
enario.
Option C. Traumatic asphyxia: Traumatic asphyxia is a condition that occurs when there is a
sudden, severe compression of the chest or abdomen, leading to a
disruption of normal breathing. It can result from various causes, such as a crush injury or a forceful blo

Page 5

157
w to the chest. While the mechanism of traumatic asphyxia shares similarities with burking, the term "tr
aumatic asphyxia" is a broader category that encompasses different causes, whereas burking refers sp
ecifically to the act of sitting on the victim's chest to cause asphyxia.
Option D. Overlaying: Overlaying, also known as accidental suffocation, occurs when an adult or object
unintentionally covers an infant's airway during sleep, leading to suffocation. This term is not applicabl
e to the described scenario involving the friend and victim, as overlaying typically pertains to accidental
suffocation of infants rather than intentional acts of homicide.

Solution for Question 4:


Correct Choice: B
• Cervical sympathetic chain: The sympathetic chain, also known as the sympathetic trunk or ganglia, is
part of the autonomic nervous system. It runs along the sides of the vertebral column, and its
compression can have significant effects on various bodily functions. In this case, the compression of
the cervical sympathetic chain is the probable reason for the open right eye. When the sympathetic
chain is affected, it can lead to an interruption in the innervation of the smooth muscles in the eyelids,
resulting in the inability of the right eyelid to close.
Incorrect Choices:
Option A. Right internal jugular vein: The right internal jugular vein is a major blood vessel located in th
e neck. Its compression would not directly cause the opening of the right eye. While the jugular vein's c
ompression might result in congestion or swelling in the neck, it is not responsible for the specific symp
tom mentioned in the question.
Option C. Left vagus nerve: The vagus nerve is a cranial nerve that provides parasympathetic innervati
on to many organs in the body. While the left vagus nerve can be affected during hanging due to press
ure on the neck, its compression would not directly cause the specific symptom mentioned in the questi
on, which is the open right eye.
Option D. Right internal carotid artery: The right internal carotid artery is a major blood vessel that supp
lies oxygenated blood to the brain. Its compression is called ammusat’s sign but would not directly cau
se the opening of the right eye. While the carotid artery's compression could potentially lead to altered
blood flow and neurological effects, it is not responsible for the specific symptom mentioned in the que
stion.

Solution for Question 5:


Correct Option: C
• The test described, where the bronchus is tied and the lung is immersed in water to check whether it
floats or sinks, is known as the Hydrostatic test.
Incorrect options:
Option A: Gettler's test: It is based on the chloride concentration in the blood samples of heart chamber
s [right side & left side]
· Normally, chloride concentration in both right side chambers & left side chambers should be equal
Option B: Ploucquet's test: Ploucquet's test is a test used to detect ratio between

Page 6

158
Option D: Diatom's test: Diatom's test is a forensic examination that involves analyzing the presence of
diatoms, microscopic algae, in various tissues and fluids to determine if a
person was submerged in water.
In the context of the described test during autopsy, the correct answer is the Hydrostatic test. This test i
s performed to assess the buoyancy of the lung, as the presence of air within the lung tissue causes it t
o float in water. It is used as a supportive indication of drowning, as drowning often leads to the aspirati
on of water into the lungs, resulting in increased lung buoyancy.

Solution for Question 6:


Correct Option B:
• Ligature strangulation is the right cause of death in this situation.
• The presence of a ligature that encompassed the neck and was below the thyroid level indicates a
ligature check, which may be a classic sign of ligature strangulation.
• The absence of saliva spilling is additionally consistent with this sort of asphyxia, as the airway is
regularly compressed without hindrance to the oral cavity

Page 7

159
Incorrect Options:
Option A.
• Throttling is a form of asphyxia in which pressure is connected to the neck by the hands, forearm, or
other object.
• This will cause bruises or abrasions on the neck and encompassing areas, but it isn't steady with the
level of ligature mark seen in this case.
Option C.

• Gagging includes airway obstacles with a foreign object, such as a cloth or piece of cloth, put within
the mouth.
• This could cause saliva to spill from the mouth and isn't steady without saliva.
Option D.

Page 8

160
• Hanging is a form of asphyxia in which a ligature is utilized to suspend the body from the next point,
such as a rope or cord tied to a beam or tree.
• The ligature mark, in this case, is level and below the level of the thyroid, which isn't steady, with the
ordinary vertical ligature mark seen hanging.

Solution for Question 7:


Correct Option B - Antemortem drowning:
• Violent respiratory struggle causes churring effect, which mixes Mucus + surfactant+ water+ air &
results in production of Froth.
• In post-mortem drowning as this violent respiratory struggle is missing, froth can't be seen.
• Thus, it is a sign of Antemortem drowning.

Page 9

161
Violent respiratory struggle causes churring effect, which mixes Mucus + surfactant+ water+ air &
results in production of Froth.
In post-mortem drowning as this violent respiratory struggle is missing, froth can't be seen.
Thus, it is a sign of Antemortem drowning.
Incorrect Options:
Option A - Drowning after Organophosphate poisoning: This option is incorrect.
Option C - Drowning after Arsenic poisoning: This option is incorrect.
Option D - None of the above: This option is incorrect.

Solution for Question 8:


Correct Option B - Garrotting:
• The correct answer is b. Garrotting.
• The term "Spanish windlass" refers to a specific type of garrotting, a method of strangulation. In this
method, a cord or similar material is wrapped around a person's neck, and a stick or other similar object
is twisted to tighten the cord, leading to strangulation. This particular type of garrotting is named after a
technique historically used in Spain.
Incorrect Options:
Option A - Bansdola: Bansdola is a technique where a bamboo or wooden block is used to cause pres
sure on the neck. This is different from the mechanism used in the Spanish windlass method.
Option C - Mugging: Mugging usually refers to an act of robbery or assault, often involving physical forc
e. It's not specific to any particular method of strangulation, and thus, it doesn't specifically describe the
Spanish windlass technique.
Option D - Throttling: Throttling refers to strangulation with the hands or fingers. It's a direct method tha
t doesn't typically involve the use of cords or similar materials, which are integral to the Spanish windla
ss technique.

Solution for Question 9:


Correct Option C: No Mud and vegetation in respiratory tract
• This statement is false as in the case of antemortem drowning there is fine froth in the respiratory tract
along with mud, sand and vegetation.
• There is no mud and vegetation in the case of post-mortem drowning.
Incorrect options:
Option A: Water in the stomach: In cases of antemortem drowning, it is common to find water in the sto
mach due to the inhalation or ingestion of water during the drowning process.
Option B: Froth on nose and mouth: Froth on the nose and mouth can be observed in cases of antemo
rtem drowning. It occurs due to the mixing of air and water during the struggle for breath.

Page 10

162
Option D: Cadaveric spasm in hand muscle : Cadaveric spasm refers to the postmortem stiffening of m
uscles that occurs in the position they were in at the time of death. It can be observed in drowning case
s when the victim may grasp onto an object or exhibit a specific hand posture due to a
sudden muscular contraction at the time of death.

Solution for Question 10:


Correct Choice. B
• Explanation:The most common cause of death in judicial hanging is cervical vertebral fracture or
dislocation. The most common cause of death in hanging is a combination of asphyxia and cerebral
venous congestion.

• In India, a legal death sentence is carried out by hanging the criminal. The face of the person is
covered with a dark mask, and he is made to stand on a platform above trap doors which open
downwards when a bolt is drawn.
• The knot in judicial hanging is commonly placed at the side of the neck (on the right side at the right
angle of the mandible). But, the most effective/ ideal site is said to be below the chin/ submental.
• Cause of death: With proper judicial hanging, there will be rupture of brainstem between pons and
medulla. Results in instant and irreversible loss of consciousness and irreversible apnea and heart
continues to beat for 15 minutes due to autorhythmicity. Hangman’s fracture is the main cause of death
in judicial hanging. It is seen when the knot is at the chin. There is fracture dislocation at C2– C3 level.
Bilateral fracture of either of pedicles or laminae of C2 is also Hangman’s fracture.
• Bilateral fracture of either pedicle/ lamina of C1 is known as Jefferson fracture.
• Undertaker fracture – It is an postmortem artefact at the level of C6– C7.
Incorrect choices:
Option A. Vagal inhibition is one of the causes of death in case of ligature strangulation, it is also the ca
use of death in some cases of hanging. In judicial hanging the cause of death is fracture or dislocation

Page 11

163
of cervical vertebrae.
Option C. Cerebral hypoxia/ anoxia is one of the several causes of death in various cases of hanging a
nd ligature strangulation. But it is not the cause of death in case of judicial hanging.
Option D. Hanging is the most common asphyxial method of death. Asphyxia along with venous conge
stion is the most common cause of death in hanging.

Solution for Question 11:


Correct Choice: B
• Explanation: In an incomplete type of hanging also known as partial hanging, some part of the body is
touching the ground. Constricting Force here will be the weight of the head (around 4-5 kg).
Incorrect choices:
Option A. In the complete type of hanging, the constricting force is the whole body.
Option C. In typical, position of the knot is the occiput. But the whole body is suspended, hence the wh
ole body weight is the constricting force.
Option D. In atypical type of hanging, the position of the knot is any place other than the occiput. But in
this type also just like typical, the whole body is suspended and hence the constricting force will be the
whole body weight

Solution for Question 12:


Correct Option B: Suicide
• Although cyanosis may be present in some techniques of suicide, brusis around mouth and lips, along
with petechial hemorrhages and abrasions points towards injury by force.
• Asphyxia, smothering and homicide can be possible in such situations.
• However, suicidal attempts are usually painless, and may not fit in this scenario.
Incorrect Options:
Option A: Asphyxia - Usual signs include Cyanosis, petechial hemorrhages and congestion.
Option C: Smothering- Done by pillow or hand, smothering has features like hematoma, contusions on
the lips, mouth and oral orifices. facial congestion and edema in the description fit in this
Option D: Homicide - It may present various features depending on the mode of homicide. abrasions, b
ruises, petechial hemorrhages are possible in homicidal attempts.

Page 12

164
Forensic Traumatology Part - 1 (Mechanical Injuries)
1. A 28-year-old woman's body has been brought for autopsy. She was allegedly sexually assaulted
and strangulated by the suspect when she attempted to free herself. There are marks of a struggle
including nail marks and bruises on the forearms, thighs and also around the groin. What are the
features that will help the forensic doctor to determine if the bruises were inflicted ante-mortem or
post-mortem?
(or)
What are the features that will help the forensic doctor to determine if the bruises were inflicted
ante-mortem or post-mortem?
A. Well-defined margin
B. Capillary rupture with extravasation of blood
C. Yellow color
D. Gaping
----------------------------------------
2. An autopsy of the 25-year-old man who had been stabbed to death by unidentified men reveals
multiple stab wounds which are elliptical, both ends are pointed and are gaping in the thorax and
abdomen. There are also lacerations ranging from 10-15 cm on the shoulder and the forearms. Which
statement among the following is true about stab wounds?
(or)
Which statement is true about stab wounds when the wounds are elliptical with both ends pointed and
gaping?
A. Depth is greater than breadth
B. Breadth is greater than depth
C. Length is greater than breadth
D. Penetrating stab has a wound of entry and exit
----------------------------------------
3. A 23-year-old man suffered fatal head injuries when he was hit by a lorry while riding his motorbike.
His right leg had got entangled amidst the bike's broken parts. An autopsy of his body revealed a
collection of blood in the subarachnoid space and multiple incision like clean wounds ranging from 5-
10 cm along the shin of his right leg. All of the following statements about incision-like lacerations are
true except?
(or)
Which statement about incision-like lacerations in the autopsy findings is NOT true?
A. Type of split lacerated wound
B. Avulsion lacerated wound caused by blunt object
C. Produced by sharp objects
D. Commonly seen on scalp
----------------------------------------

165
4. A 15-year-old boy was examined by the team doctor after he sustained an injury to his knees and
shin during an ongoing cricket match. He had attempted a difficult fielding where he had instinctively
dived and skidded on his knees for a short distance on the rough unkempt school playground.
Examination of his legs revealed multiple parallel and longitudinal scratches that had some avulsed
skin gathered at the edge of the wound near the apex of the patella. The doctor identified them to be
graze wounds which is a type of?
(or)
What type of wound is a graze wound seen in the 15-year-old cricket player who dived and skidded on
his knees during the match?
A. Contusion
B. Abrasion
C. Lacerated wound
D. Incised wound
----------------------------------------
5. A 35 year old married woman had arrived at the police station complaining of domestic abuse by her
husband. The investigating medical officer noticed bruises on the forearm, shoulder and back on
examination. She also had blackening with swelling and puffiness around her left eye, which made it
difficult for her to open it. The blackening around the eye is caused by?
(or)
What is the cause of the blackening around the woman's left eye, resulting from domestic abuse?
A. Friction abrasion
B. Patterned abrasion
C. Penny bruises
D. Ectopic bruise
----------------------------------------
6. A 55-year-old woman has been brought into the emergency with multiple stab wounds that she had
suffered while attempting to stop a couple of unidentified masked men who were robbing her house. On
examination, the doctor noticed 3 deep incisions with clean edges, pointed on one end and squared off
on the other end with each measuring 3cm in width. The 2 wounds 5 cm above the umbilicus were
narrow and directed medially but the wound 2 cm below the xiphisternum was wider with a breadth of
1.5 cm and was directed more laterally. Which specific characteristic of the stab wound is determined
by langer lines?
(or)
What specific characteristic of the stab wounds is determined by Langer lines?
A. Direction
B. Gaping
C. Shelving
D. Healing
----------------------------------------
7. A person was admitted to the emergency ward with injuries sustained at a protest site. The person
was one among the crowd that was dispersed by the police using lathi charge. Which among the

Page 2

166
following is least likely to be seen secondary to being hit by a lathi?
(or)
Which among the following is least likely to be seen secondary to being hit by a lathi?
A. Fissured fracture
B. Contusion
C. Incised looking lacerated wound
D. Incised wound
----------------------------------------
8. A 23-year-old woman was brought in for examination after she was rescued from a house where she
had been locked up following an argument with her in-laws. On examination she had multiple linear
brownish bruises measuring about 2.5 cms on her back, shoulders and abdomen which most probably
had been inflicted with a belt. She also had a blackened swollen right eye and swollen lower lip with
traces of dried blood at the right edge of her mouth. The medical officer can tell how old the bruises are
by their coloration, which indicates that they are around 5 days old. The presence of which pigment
causes the discoloration?
(or)
What pigment causes the discoloration in the 5-day-old bruises observed on the woman's body?
A. Hemosiderin
B. Reduced hemoglobin
C. Biliverdin
D. Bilirubin
----------------------------------------
9. A 10-year-old boy was brought to the emergency after he had sustained a cut on his right sole after
stepping on a glass piece while running barefoot around the picnic spot. The medical officer on duty
identified the 2x1 cm incision on the lateral aspect of the mid-sole. All of the following characteristics
are true of an incision wound except?
(or)
All of the following characteristics are true of an incision wound except?
A. It has clean-cut margins
B. Bleeding is generally lesser than in lacerations
C. Tailing is often present
D. The length of the injury does not correspond with the length of the blade
----------------------------------------
10. The body of a 28-year-old man has been discovered in the hostel premises with his throat cut. On
immediate observation, there were no signs of struggle inside the room. A half empty bottle of whisky
and a couple of burnt cigarette butts were seen lying next to the body. Information gathered from
friends revealed that the man had been on medication for depression and had also gotten into
arguments with some of the other students in recent times. All of the following may be seen on autopsy
in case of a suicidal throat cut except?
(or)

Page 3

167
All of the following may be seen on autopsy in case of a suicidal throat cut except?
A. Ragged edges
B. Hesitation cuts
C. Tailing
D. Defence wounds
----------------------------------------
11. A 22 year old woman was examined by the medical officer post allegations of being physically and
sexually assaulted by her boyfriend. To assist in confirming the allegation, the doctor began a thorough
physical examination. The doctor preferred to document the abrasion wounds over contusions. All of
the following are true about contusions except?
(or)
All of the following are true about contusions except?
A. They may become visible several hours after injury
B. They may not indicate the site of trauma
C. They do not indicate the direction of force
D. They are visible immediately after an injury
----------------------------------------
12. A completely charred body was brought for autopsy. Examination of the skull and cranial cavity
revealed blood collection in the epidural space. Which among the following findings is suggestive of
hematoma secondary to trauma?
A. Carboxy hemoglobin in the hematoma
B. Dark brown friable clot
C. Radiating fracture lines over the skull
D. Diffuse hematoma
----------------------------------------
13. A person was stabbed to death because of a disagreement over their ancestral land. During the
autopsy of the sustained injuries, the following wounds were observed. The weapon used during this
assault would most likely be?
(or)
Based on the image below , what was the likely weapon used in the fatal stabbing?

Page 4

168
A. Screwdriver
B. Single-edged knife
C. Double-edged knife
D. Ice pick
----------------------------------------
14. The body of the 75 year old lady has been brought for autopsy. The body had traveled a long
distance by an ambulance across very rough roads. The examining officer notices an "Undertaker's
fracture". The ambulance clerks informed the doctor that the body had to be re-positioned multiple
times due to the jerky ride. The fracture of which structure is the doctor documenting as such?
(or)
What structure is the doctor documenting when noting an "Undertaker's fracture" in a 75-year-old lady's
body after a rough ambulance ride with multiple re-positions?
A. Skull
B. Cervical spine
C. Lumbar vertebrae
D. Pelvis
----------------------------------------
15. The body of a 6 year old child was brought in for an autopsy. The child had been on treatment for
anaplastic astrocytoma. He had undergone a resection surgery and also been treated with
Bevacizumab for the last 3 months. During autopsy, the doctor documents the presence of a diastatic
fracture. This refers to fractures that occur through which structure?
(or)
During autopsy, the doctor documents the presence of a diastatic fracture. This refers to fractures that
occur through which structure?
A. Outer table of skull
B. Cranial sutural lines
C. Inner table of skull
D. Base of the skull
----------------------------------------
16. A football player sustained abrasion wounds during an ISL league match held at Chennai. On
examination of the wound during a follow up visit, the following was observed. When did the person
most likely sustain the injury prior to this visit?

Page 5

169
A. 12-24 hrs
B. 1-2 days
C. 2-3 days
D. > 7 days
----------------------------------------
17. The 33 year old man has regained consciousness after being unconscious for almost 10 minutes
post the bike accident. The psychiatrist noted that he was unable to recall recent events including the
moments leading up to the accident though he was able to recall past events like his wedding and first
child's birth with more ease. The doctor was also able to document a characteristic amnesia by
performing further tests. What type of amnesia is seen post traumatic brain injury?
(or)
What type of amnesia is typically seen after a traumatic brain injury when a person can't recall recent
events but can remember past events more easily?
A. Retrograde amnesia
B. Anterograde amnesia
C. Transient global amnesia
D. All are true
----------------------------------------
18. The body of the driver of the car that had collided with the lorry was brought in for examination.
There was a steering wheel shaped bruise showing on the abdomen with fullness and a doughy
consistency on palpation which could indicate intra-abdominal hemorrhage. Instability of the lower
thoracic cage is also noted and may indicate the possibility of splenic or hepatic injuries. Which of the
following statements about blunt abdominal trauma is not true?
(or)
Which of the following statements about blunt abdominal trauma is not true?
A. Solid organ injuries are more common in children than adults
B. Liver injuries are more common than splenic injury
C. Diaphragmatic injuries are rare
D. Intraperitoneal gas shadows are pathognomonic of bowel perforation
----------------------------------------

Page 6

170
19. The forensic doctor was nearing completion of his examination of a 2-year-old child's body that was
brought in for autopsy. The child had allegedly fallen off the balcony while left unattended for a short
time. The doctor decides to have another quick look at certain regions specifically to make sure no
concealed puncture wounds are missed. The doctor would observe all of the following areas except?
(or)
In a post-mortem examination of a 2-year-old girl who fell off a balcony, which body regions should Tthe
forensic doctor should observe to ensure no concealed puncture wounds were missed in all of the
following except?
A. Nape of neck
B. Inner canthus of eye
C. Wrist
D. Vagina
----------------------------------------
20. The medical officer arrives to examine the 22-year-old woman who had been brought in
unconscious with a blood-soaked cloth tied around her left wrist. She was allegedly found by her
roommate in a pool of blood with a kitchen knife lying next to her in the bathroom of their dorm. On
examination, the doctor notes the presence of multiple shallow incision wounds extending from the
lateral to the medial aspect indicating hesitation when making the cuts. These will be documented as
evidence to support which type of wounds??
(or)
What type of wounds do multiple shallow incisions with hesitation marks suggest?
A. Self-inflicted wounds
B. Homicidal wounds
C. Defence wounds
D. Accidental wounds
----------------------------------------
21. Which of the following tests is used to differentiate antemortem bruise from postmortem
hypostasis?
A. Breslau's test
B. Gettler's test
C. Incision test
D. Precipitin test
----------------------------------------
22. What is your diagnosis for a 10-year-old child based on the observed clinical findings, as depicted
in the provided image?

Page 7

171
A. Tram line bruise
B. Ectopic bruise
C. Six penny bruise
D. Butterfly bruise
----------------------------------------

Correct Answers
Question Correct Answer

Question 1 2
Question 2 1
Question 3 2
Question 4 2
Question 5 4
Question 6 2
Question 7 4
Question 8 3
Question 9 2
Question 10 4
Question 11 4
Question 12 3
Question 13 3
Question 14 2
Question 15 2
Question 16 4
Question 17 2
Question 18 2
Question 19 3

Page 8

172
Question 20 1
Question 21 3
Question 22 1

Solution for Question 1:


Correct Option B - Capillary rupture with extravasation of blood:
• Extravasation of blood into surrounding tissues is a characteristic feature of an antemortem bruise.
Incorrect Options:
Option A - Well-defined margin:
• Sharply defined margins are seen in postmortem bruising.
Option C - Yellow colour:
• The age of injury can be determined by the colour changes. Colour change starts at the periphery and
extends inwards to the centre.
• Bruises of the same age may show different colour progression, so that variation in colour does not
necessarily mean that there have been multiple episodes of injury. Not all bruises pass through a
yellow phase before they resolve.
Option D - Gaping:
• Gaping does not distinguish between antemortem or post-mortem bruises.

Solution for Question 2:


Correct Option A - Depth is greater than breadth:
• Stab wound by a knife produces oval, elliptical, triangular, or irregular scars which are depressed. The
stab wound is deeper than its length and width.
Incorrect Options:
Option B - Breadth is greater than depth: In a
stab wound, the depth is more than the breadth of the wound.
Option C
- Length is greater than breadth: Length is the maximum dimension in case of an Incised wound.
Option D
- Penetrating stab has wound of entry and exit: Penetrating stab produces only one wound, i.e., a
wound of entry.

Solution for Question 3:


Correct Option B - Avulsion lacerated wound caused by blunt objects:
• Avulsion of the skin and subcutaneous tissues from the underlying structures can occur when a
weight, such as a wheel of a big vehicle or a heavy piece of machinery, passes over the limb (shearing

Page 9

173
lacerations).
• Known as 'flaying,' this is when a revolving wheel rips off the skin across a large region.
• Extensive abrading and bruising of the ripped skin's edges are possible.
• Internally, the organs might be avulsed or pulled away from their attachments partially or totally.
• Flapping/ Flaying: A flap of tissues is stripped off from the underlying bone.
Incised-looking laceration:
• The laceration looks like an incised wound (margins appear to be regular & clean cut)
• It is a type of split laceration wound
• Sites: Where the soft tissues are ‘sandwiched’ between an underlying bone and the weapon. eg.,
Scalp, shin, eyebrows, zygoma, iliac crest, knee etc.

Solution for Question 4:


Correct Option B - Abrasion:
• Abrasion is the removal of the superficial epithelial layer of the skin, usually the epidermis and
papillary dermis, by friction against a rough surface.
• Graze abrasion is also known as sliding, scrape, or grinding abrasion.
• Grazes (gravel rash) are caused by horizontal or tangential friction between the skin and the hard
rough surface. They show uneven, longitudinal parallel lines, which indicate the direction in which the
force was applied (epidermis being heaped up at the opposite end known as epithelial tag).
Incorrect Options:
Option A - Contusion :
• When a blunt force acts on the skin, blood vessels rupture resulting in accumulation/extravasation of
blood under the skin(not in the epidermis) is known as a Contusion/Bruise.

Page 10

174
• Graze wounds are not a type of contusion.
Option C - Lacerated wound:
• Lacere means to tear.
• In laceration wounds, there are irregular margins with contused edges.
• Graze wounds are not a type of laceration.
Option D - Incised wound:
• It is also known as Cut injury/Slash injury/Slice injury.
• Produced by light-cutting weapons with sharp edges. eg. surgical blades
• Graze is not a type of incised wound.

Solution for Question 5:


Correct Option D - Ectopic bruise:
• An ectopic bruise is responsible for the appearance of bruises at a site other than the site of injury,
e.g. black eyes.
• Contusion over bone takes a long time to become visible and may appear further from the site of
impact. This results from gravity shifting of blood to the tissue which are more superficial and lax. Such
bruises are called ectopic bruises.
Incorrect Options:
Option A - Friction abrasion: Friction abrasion is synonymous with pressure abrasion.
Option B - Patterned abrasion: Patterned Abrasions: If a weapon with a
patterned surface hits the body or the body falls on a
rough hard patterned surface, the abrasions will usually have the pattern of the object.
Option C - Penny bruise: Six penny bruisbruisestiple small bruises on the trunk and or limbs caused by
the fingertips of the assailant, produced while overpowering the victim.

Solution for Question 6:


Correct Option B - Gaping:
• Langer’s lines:Imaginary lines
• Corresponding to collagen ■bers of the body
• Gives an idea about the gaping of the body. If stab injury is parallel to these lines, then gaping would
be less. If stab injury is perpendicular to these lines, then gaping would be more.
• If stab injury is parallel to these lines, then gaping would be less.
• If stab injury is perpendicular to these lines, then gaping would be more.
• If stab injury is parallel to these lines, then gaping would be less.
• If stab injury is perpendicular to these lines, then gaping would be more.

Page 11

175
Incorrect Options:
Option A - Direction:
• Hiltmark is the patterned abrasion/bruise produced by hilt of the weapon on the sides of the stab
injury.
• Direction is not determined by the langer lines.
Option C - Shelving:
• Shelving of the wounds gives the investigator an indication of the direction of the track of the wound.
• Shelving is not determined by the langer lines.
Option D - Healing:
• Healing depends on the site and degree of the stab wounds.
• Healing is not determined by the langer lines.

Solution for Question 7:


Correct Option D - Incised wound:
• Lathi is a blunt weapon. It cannot produce an incised wound, which is a sharp force injury.
Incorrect Options:
Option A - Fissured fracture:
• A fissured fracture is a type of incomplete fracture where the bone has a crack or groove but doesn't
completely break into separate pieces.Also known as linear fracture
• Most common fracture of the skull
• A blow with a blunt weapon like a lathi can cause fissred fracture.
Option B - Contusion:
• Blunt force injuries: Abrasion, contusion, laceration & fracture.
• Bruise/Contusion is the extravasation of blood in the subcutaneous/sub-epithelial tissues due to
rupture of blood vessels, usually capillaries, as a result of blunt force injury or pressure.
Option C - Incised - looking lacerated wound:
• Lacerations produced without excessive skin crushing may have relatively sharp margins. Blunt force
on areas where the skin is close to bone, and the subcutaneous tissues are scanty, may produce a
wound which by linear splitting of the tissues (as the skin is easily stretched during impact), may look
like incised wound.
• The sites are the scalp, eyebrows, cheek bones, lower jaw, iliac crest, perineum, and shin. A wound
produced by a fall on the knee or elbow with the limb flexed, and by a broken glass or sharp stone also
simulates incised wound.

Solution for Question 8:

Page 12

176
Correct Option C - Biliverdin:
• Biliverdin is a green pigment formed as a byproduct of heme breakdown.
• Time Duration: 5-6 days.
Incorrect Options:
Option A - Hemosiderin:
• Hemosiderin staining produces bruise-like marks on the body that can range in color from yellow to
bluish-black/brown, which are around 4 days old.
Option B - Reduced hemoglobin:
• Your red blood cells carry oxygen throughout the body. If you have a condition that affects the body’s
ability to make red blood cells, your hemoglobin levels may drop. Low hemoglobin levels may be a
symptom of several conditions, including different kinds of anemia and cancer.
Option D - Bilirubin:
• As the bruise begins to heal, bilirubin will multiply at the site of the bruise and as the purple or black
color of the bruise begins to fade the bruise will take on a yellow color until it fully heals.
• Time duration: 7-13 days.

Solution for Question 9:


Correct Option B - Bleeding is generally lesser than in lacerations:
• Blood vessels are clean-cut, profuse bleeding present.
Incorrect Options:
Option A - It has clean-cut margins: Margins edges are clean cut, well-defined and usually everted.
Option C - Tailing is often present: Tailing is present, suggesting the direction.
Option D - Length of injury does not correspond with length of blade in a
stab wound: Beveling can be seen if the blade cuts obliquely.

Solution for Question 10:


Correct Option D - Defence wounds:
• Defence wounds like cuts, lacerations, bruises, etc. on or in-between the fingers and palms, back of
hands, wrists, inner aspects of forearms if present, are strongly suggestive of homicide as these are
produced during attempts by the victim to grasp the weapon in instinctively defending himself or toward
off the attack on the head or some other vital part of the body.
Incorrect Options:
Option A - Ragged edges: Edges are likely to be toothed or may show some irregularity of the margins
at places because the skin is thrown into folds before severance (ragged edges).
Option B - Hesitation cuts: Multiple superficial tentative or hesitational cuts may be present around the
commencement of the main wound (shows the divided state of mind of a

Page 13

177
person as it is usual human nature to preserve life).
Option C - Tailing: Depth and direction: Usually deeper at the commencement, except in case of suicid
al cut-throat injuries, with hesitation cuts at the beginning. This is known as the head of the wound. To
wards termination, the cut becomes progressively shallow, known as the tailing of the wound. Consequ
ently, the depth of the incised wound with tailing will suggest the direction in which the force was applie
d.

Solution for Question 11:


Correct Option D - They are visible immediately after an injury:
• It is advisable that a medical officer should re-examine the patient after 24 hours, as by this time the
bruises are clearly visible.
Incorrect Options:
Option A - They may become visible several hours after injury: They may become visible after a
few hours to 1–2 days after the injury has happened.
Option B - They may not indicate the site of trauma: They may appear at a
distance away from the actual site of injury eg. Black eye
Option C - They do not indicate the direction of force: Bruises/Contusions do not indicate the direction
of the applied force.

Solution for Question 12:


Correct Option C - Radiating fracture lines over the skull:
• Artifactual epidural hematomas can occur in fire victims, related to heat-induced postmortem skull
fractures.
• Examination of the skull and cranial cavity revealed blood collection in the epidural space and
Radiating fracture lines over the skull suggest EDH.
Incorrect Options:
Option A - Carboxy hemoglobin in the hematoma: Carboxyhemoglobin in the hematoma is not seen in
EDH due to trauma.
Option B
- Dark brown friable clot: Dark brown friable clot suggests EDH due to burns (heat hematoma).
Option D - Diffuse hematoma: Diffuse hematoma is seen in heat hematoma.

Solution for Question 13:


Correct Option C - Double-edged knife:
• The stab injury in the given image shows "acute angles at both ends", which is suggestive of a
double-edged weapon.

Page 14

178
Incorrect Options:
Option A - Srew driver: In the case of a
screwdriver, the wound is slit-like with squared ends and abraded margins.
Option B - Single-edged knife: In the case of a
single-edged knife, the wound is wedge-shaped; sometimes the blunt edge of a
knife can harm the skin giving it a tail-like appearance which is known as FISH TAIL.
Option D - Ice pick: In the case of ice pick the wound is circular in shape.

Solution for Question 14:


Correct Option B - Cervical spine:
• Sometimes, fractures of the ribs or long bones or cervical spine may occur by rough handling of the
body, especially in the elderly or debilitated. Undertaker’s fracture may be seen which is a subluxation
of the lower cervical spine due to tearing of the intervertebral disc at about C6-C7.
Incorrect Options:
Option A - Skull:
• Skull fracture is uncommon during repositioning of the corpse.
• It is not the Undertaker's fracture .
Option C - Lumber vertebrae:
• Fracture of Lumber vertebrae is uncommon during repositioning of the corpse.
• It is not the Undertaker's fracture .
Option D - Pelvis:
• Pelvic fracture is not the Undertaker's fracture .

Solution for Question 15:


Correct Option B - Cranial sutural lines:
• They are usually seen in the sagittal suture.
• They are particularly common in traffic accidents.
• They xan occur secondary to increased intracranial pressure which results in the splitting of sutures.
Incorrect Options:
Options A, C and D: These are not associated with diastatic fracture.

Solution for Question 16:


Correct Option D - > 7 days:

Page 15

179
• In the given image ,the scab has almost completely peeled off which indicates person most likely
sustain the injury >7 days.
Incorrect Options:
Option A - 12-24 hrs: In 12-24 hours, exudate from a wound dries up to form a
reddish scab that comprises of dried blood, lymph, and injured epithelial cells.
Option B - 1-2 days:
• Abrasion produces minimum bleeding, heals rapidly and leaves no permanent scarring on healing.
• Inflammation begins, bringing immune cells to the wound site to remove debris and defend against
infection..
Option C
- 2-3 days: In 2-3 days: The scab is reddish-brown, less tender and adhering over the abraded area.

Solution for Question 17:


Correct Option B - Anterograde amnesia:
• Anterograde amnesia is characteristically seen in post-traumatic brain injury.
• Loss of memory subsequent to the event that caused the amnesia is called anterograde amnesia.
Incorrect Options:
Option A - Retrograde amnesia:
• Retrograde amnesia is a loss of memory preceding the event.
Option C - Transient global amnesia:
• After performing tests, the doctor diagnosed this case as anterograde amnesia, NOT transient
global amnesia.
• This is a short-lived condition that involves both anterograde and retrograde amnesia
• After performing tests, the doctor diagnosed this case as anterograde amnesia, NOT remote amnesia.
Option D - All are true:
• Here is this case, after performing tests doctor diagnosed this case as anterograde amnesia.
• Anterograde amnesia is characteristically seen in post-traumatic brain injury.

Solution for Question 18:


Correct Option B - Liver injuries are more common than splenic injury:
• Spleen is the most common organ involved in blunt injury abdomen.
Incorrect Options:
Option A - Solid organ injuries are more common in children than adults: Children have comparatively l
arge solid organs and less subcutaneous fat &
musculature, therefore they are more prone to solid organ injuries than adults.

Page 16

180
Option C - Diaphragmatic injuries are rare: In order of frequency, the structures most likely to be dama
ged in blunt abdominal trauma are: the spleen, liver, kidneys, intestines, abdominal wall, mesentery, pa
ncreas, and diaphragm.
Option D - Intraperitoneal gas shadows are pathognomonic of bowel perforation: The presence of free i
ntraperitoneal gas on a routine radiograph usually indicates bowel perforation.

Solution for Question 19:


Correct Option C - Wrist:
• The doctor would observe all of the following areas except wrist.
• The wrist is not a concealed area and therefore need not be observed for concealed wounds.
Incorrect Options - Option A, B and D:
• Sites observed in concealed punctured wounds includes: Nostrils, fontanelles, inner canthus of eyes,
vagina, axilla, rectum, and the nape of the neck.

Solution for Question 20:


Correct Option A - Self-inflicted wounds:
• Hesitation cuts are seen in suicidal wounds/ self-inflicted wounds.
• Hesitation cuts/marks or tentative cuts or trial wound: These cuts are multiple, small and superficial
often involving only the skin, and are seen at the beginning of the incised wound, presumably hesitating
while gaining the courage to make a final decisive cut.
Incorrect Options:
Option B - Homicidal wounds: Hesitation cuts are absent in homicidal wounds.
Option C - Defence wounds:
• Defense wounds are wounds of the extremities, which result from the immediate and instinctive
reaction of the victim to ward off an attack.
• Defense wounds are absent if the victim is: unconscious; taken by surprise; attacked from behind and
under the influence of alcohol/drugs.
Option D - Accidental wounds:
• Hesitation cuts are absent in accidental wounds
• Accidental wounds are rare. They are caused by falling against projecting sharp objects like glass,
nails, etc.,

Solution for Question 21:


Correct Option C - Incision test:

Page 17

181
• The test used to differentiate (antemortem) bruise from (postmortem) hypostasis is the incision test.
Incorrect Options:
Option A - Breslau's test:
• Breslau's test is used for showing whether or not a child has been born alive, i.e,confirms live birth.
• This test is done simply by determining whether the stomach or intestines float in water.
Option B - Gettler's test:
• Gettler's test is a quantitative test used to estimate the chloride content of the blood from both sides of
the heart.
• Tests for Drowning: Gettler Chloride Test Levels less on the right side of the heart: Drowned by
saltwater. Levels less on the left side of the heart: Drowned by freshwater.
• Levels less on the right side of the heart: Drowned by saltwater.
• Levels less on the left side of the heart: Drowned by freshwater.
• Levels less on the right side of the heart: Drowned by saltwater.
• Levels less on the left side of the heart: Drowned by freshwater.
Option D - Precipitin test:
• Species identification can be done by precipitin test
• It is a specific protein test, and the reaction demonstrates the presence of albuminous substances
obtained from any part of human body. The origin of skin,flesh, bone or even secretions, such as saliva,
milk and semen is determined by this test.

Solution for Question 22:


Correct Option A - Tram line bruise:
• The image shows 'tram line' bruising most likely to have been produced by a stick or a broom handle.
• When a person is hit with a stick, a line bruise is produced
• They are also known as railway line bruises
Incorrect Options:
Option B - Ectopic bruise:
• Bruise away from the site of impact.
• Eg: Battle's sign, in a case of middle cranial fossa fracture
Option C - Six penny bruises:
• It is seen in manual strangulation (Throttling) i.e. compression of the neck by hand of the assailant.
Option D - Butterfly bruise:
• It is produced by skin pinching; seen in child abuse.

Page 18

182
Forensic Traumatology Part - 2 (Regional Injuries)
1. Bystanders reported that a lorry had suddenly braked to avoid hitting a cow in the middle of the road,
causing a bike behind the lorry to speed into the lorry's tail board. The impact threw the victim off his
bike as the bike got entangled under the lorry. Such under-running impact injuries may also involve
which among the following?
(or)
What type of injury can be associated other factors might be associated with impact injuries involving
under-running?
A. Crush injury abdomen
B. Run over injury
C. Decapitation
D. Chest injury
----------------------------------------
2. Which of the following statements is true regarding the image?

A. Seen in Electric burns


B. Disappears after 24-48 hours
C. Follows the track of vessels
D. Filigree burn
----------------------------------------
3. A 27-year-old man's body has been brought in for an autopsy. He was allegedly speeding on the
highway when the lorry ahead of him suddenly braked causing the man ran into the tail-board of the
lorry with his bike. On impact, he was thrown backwards and landed on his head. With this information,
where would the forensic doctor suspect the contrecoup fracture to be located most probably?
(or)
In a motorcycle accident where the victim landed on the back of his head after impact, where would the
forensic doctor most likely suspect the location of the contrecoup fracture?
A. Parietal bone
B. Temporal bone
C. Orbital roof

183
D. Ethmoidal bone
----------------------------------------
4. Which of the following scenarios is more likely to cause a whiplash injury?
A. Pedestrian hit from front
B. Pedestrian hit from behind
C. Occupant of a car
D. Pedestrian is hit and moves over the vehicle.
----------------------------------------
5. Autopsy reports of a 45-year-old man found dead in his farmhouse, describe a wound that is
approximately square-shaped and 5x5 cm by dimension. The forensic expert safely assumes that the
shape of the wound indicates the weapon used, which was likely a hammer found with blood stains in
the shed behind the house. Further investigation of the wound confirms that it is indeed a "fracture ala
signature" In simpler terms, how would you document this finding?
(or)
How should the forensic expert document a 5x5cm, roughly square shaped wound that corresponds to
a "fracture ala signature"
A. Gutter fracture
B. Depressed fracture
C. Ring fracture
D. Sutural Fracture
----------------------------------------
6. Gutter fractures of the skull are most often seen with which of the following injuries?
A. Axe injury
B. Stick injury
C. Stone injury
D. Bullet injury
----------------------------------------
7. A forensic doctor documents the injury findings of an RTA victim as 'sparrow foot marks'. The victim
had multiple shallow, punctate wounds that had a bizarre pattern on the face and neck. What is the
likely cause?
(or)
What is the likely cause of sparrow foot marks?
A. Motor cyclist’s fracture
B. Under-running or tail gating
C. Steering wheel impact
D. Wind screen impact
----------------------------------------

Page 2

184
8. The forensic doctor has completed the autopsy of the 30 year old man who has succumbed to
injuries sustained in a road traffic accident. During his initial observation, on noting a bruise over a
particular region, he has mentioned the possibility of a fracture of the middle cranial fossa with
underlying brain trauma. What location of the bruise would have helped the doctor arrive at that
conclusion?
(or)
What bruise location indicates a possible middle cranial fossa fracture with underlying brain trauma?
A. Orbital region
B. Occipital region
C. Mastoid region
D. Neck region
----------------------------------------
9. A 38 year old female met with an RTA. The collision triggered the deployment of the airbags and the
seat belt tensioners in the car. Which of the following is commonly involved in seat belt injuries?
(or)
Which of the following is commonly involved in seat belt injuries?
A. Spleen
B. Mesentery
C. Femoral artery
D. Abdominal aorta
----------------------------------------
10. Examination of the body of a 55-year-old woman who had succumbed to injuries sustained in a
road traffic accident revealed an injury to the spinal cord at a certain level, possibly secondary to a
whiplash force. Which is the likely injured segment of the spinal cord?
(or)
What is the most commonly injured part of the spinal cord in cases of whiplash injuries?
A. Lower cervical
B. Thoracolumbar
C. Upper cervical
D. Lumbosacral
----------------------------------------
11. An autopsy of a 50-year-old man with a grade 3 astrocytoma revealed descending transtentorial
and subfalcine herniations. He also noticed the presence of hernations, with duret hemorrhages. Which
part of the brain is the doctor investigating when he documents this hemorrhage?
(or)
What part of the brain are Duret hemorrhages found in?
A. Cerebrum
B. Thalamus

Page 3

185
C. Brainstem
D. Pituitary body
----------------------------------------
12. Identify the type of fracture in below image:

A. Hinge fracture
B. Ring fracture
C. Comminuted
D. Gutter fracture
----------------------------------------
13. Identify the type of skull fracture in the image given below.

A. Fissure fracture
B. Depressed fracture
C. Mosaic fracture
D. Diastatic fracture
----------------------------------------
14. A 26-year-old male was brought into the casualty with a history of falling from a height onto his feet.
Further inquiry revealed that it was a suicidal attempt secondary to an unstable relationship. Which of
the following fractures is most likely in such a scenario?
A. Hinge fracture
B. Diastatic fracture

Page 4

186
C. Indented fracture
D. Ring fracture
----------------------------------------
15. A 32-year-old who died after a traumatic brain injury is diagnosed with diffuse axonal injury. When
does this patient's earliest histological evidence first appear?
A. 4 hours
B. 8 hours
C. 12 hours
D. 24 hours
----------------------------------------
16. A 24-year-old male presented to you after sustaining blunt abdominal trauma in a street fight. It is
found that he was hit by a cricket bat in the abdomen. Which of the following structure is most
commonly injured in such patients?
A. Liver
B. Kidney
C. Spleen
D. Intestine
----------------------------------------
17. A 30-year-old pedestrian is hit by a car. He is noted to have a tibial fracture on the left leg and a
contusion over the scalp on the right side. The tibial fracture in this patient is a result of?
A. Primary impact injury
B. Secondary impact injury
C. Tertiary impact injury
D. Run-over injury
----------------------------------------

Correct Answers
Question Correct Answer

Question 1 3
Question 2 2,4
Question 3 3
Question 4 3
Question 5 2
Question 6 4
Question 7 4
Question 8 3

Page 5

187
Question 9 2
Question 10 1
Question 11 3
Question 12 1
Question 13 3
Question 14 4
Question 15 3
Question 16 3
Question 17 1

Solution for Question 1:


Correct Option C - Decapitation:
• A unique injury is seen wherein the motorcyclist drives under the rear of the truck, causing head
injuries and even decapitation, which is known as ‘under-running’ or ‘tail-gating’.
Incorrect Options:
Option A - Crush injury abdomen:
• Crush injury refers to extensive bruising and laceration of deeper tissue produced by a blunt
force, which is less likely in an under-running injury.
Option B - Run over injury:
• If a person is run over by a vehicle and dragged, impact injuries may not be found, but the clothes
may be torn, with avulsed skin and compression injuries to internal organs.
• The body or clothing of the victim may show oil and rust from the low parts of the vehicle while the
vehicle may show fibres of clothing, hair, and blood from the victim on some of its lower parts.
• Tyre tread patterns on the clothes or skin are valuable evidence.
Option D - Chest injury:
• Falling from the vehicle, particularly at high speed, may result in injuries to the extremities as well as
the chest and abdomen.Such an injury is more likely if the victim has fallen from the vehicle, not in
under-running

Solution for Question 2:


Correct Option B - Disappears after 24-48 hours / Option D - Filigree burn
• If the person survives, it appears within an hour, disappears after 24-48 hours
Incorrect Options:
Option A
- Seen in Electric burns: Though it is pathognomonic for lightning injury, it is not seen in electric burns.
Option C - Follows the track of vessels: It does not follow the track of vessels.

Page 6

188
Solution for Question 3:
Correct Option C - Orbital roof:
• MC site for contrecoup injury is the frontal lobes (tips of the frontal poles/orbital surface) and may be
symmetrical if a person falls on the occiput.
Incorrect Options:
Option A and B- Parietal bone, Temporal bone:
• In temporal or parietal impacts, the contrecoup lesions are usually on the contralateral surface of the
brain.
• As the person is thrown backwards and landed on his head, the parietal bone does not get
fractured.This may have been possible if he fell on the side of his head.
Option D - Ethmoidal bone:
• Contrecoup fracture occurs exactly opposite to the site of primary impact or ‘coup violence’. This is
due to shear strain.
• It is usually seen in the anterior cranial fossa involving the bones of the orbital or ethmoid plates with
associated periorbital hematoma.
• As the person is thrown backwards and landed on his head, the ethmoidal bone does not get
fractured.Usually seen with injury to upper central part of mid-face

Solution for Question 4:


Correct Option C - Occupant of a car:
• Whiplash injury is sustained commonly by occupants of the front seat in a motor vehicle.
• Cause by violent neck movements. Acceleration -Hyperextension of the neck followed by
hyper■exion. Deceleration -Hyper■exion of the neck followed by hyperextension.( dangerous)
• Acceleration -Hyperextension of the neck followed by hyper■exion.
• Deceleration -Hyper■exion of the neck followed by hyperextension.( dangerous)
• Acceleration -Hyperextension of the neck followed by hyper■exion.
• Deceleration -Hyper■exion of the neck followed by hyperextension.( dangerous)
Incorrect Options:
Option A - Pedestrian hit from front:
• If the victim is struck from the front, he may sustain injuries to the chest and abdomen with fractures of
ribs or vertebrae.
• The victim can also sustain a fracture of the pelvis or fracture-dislocation of the sacroiliac joint from
the impact of a mudguard,
• and a Ffracture of the tibia and fibula of one or both legs can be sustained from impact by a bumper.
Option B - Pedestrian hit from behind:

Page 7

189
• When the pedestrian is knocked down from behind with both feet fixed to the ground: There will be
fractures of the bones of the lower limbs, the buttocks and the back of the pedestrian on being hit by
headlamps or the radiator of the car.
• It may result in fracture dislocation of the lumbar or thoracic spine, and this injury may drive the
femoral head through the acetabulum.
Option D: Pedestrian is hit and moves over the vehicle
• Also known as secondary impact· Victim may hit the Bonnet or Windshield.
• Possible injuries
• Can lead to multiple injuries. Most common: Head injuries. Other Cervical fracture or injury Skeletal
injuries.
• Most common: Head injuries.
• Other
• Cervical fracture or injury
• Skeletal injuries.
• Most common: Head injuries.
• Other
• Cervical fracture or injury
• Skeletal injuries.

Solution for Question 5:


Correct Option B - Depressed fracture:
• When a portion of a fractured bone is driven inwards to a distance equivalent to the thickness of the
skull table, it is known as a depressed fracture.
• It is also called ‘fracture ala signature’ (signature fracture), as the shape often points towards the
shape of the offending weapon.
• They are caused by blows from a heavy weapon having a small striking surface, such as a hammer,
axe, brick or chopper.

Page 8

190
Incorrect Options:
Option A - Gutter fracture:
• A gutter fracture is formed when part of the thickness of the bone is removed so as to form a gutter,
e.g. oblique bullet wounds.
• It is usually accompanied by comminuted depressed fracture of the inner table of the skull, and the
fragments causing injury to the meninges and brain.

Option C - Ring fracture:


• Ring fracture: Fracture around the foramen magnum.
• It is associated with falling from a height & landing on the foot.
• All the force is transmitted from the leg and vertebral column to the base of the skull.
Option D - Sutural Fracture:
• Sutural separation due to blow on the skull [fracture lines run along the sutural line].

Page 9

191
• It is also known as a diastatic fracture.
• Most commonly seen in young adults.
• The most common suture involved is the sagittal suture.

Solution for Question 6:


Correct Option D - Bullet injury:
• Gutter fracture: A furrow in the outer table of the skull,i.e fracture of outer table, typically the result of a
glancing blow by a bullet from a rifled firearm.
• It is usually accompanied by comminuted depressed fracture of the inner table of the skull, and the
fragments causing injury to the meninges and brain.
Incorrect Options:
Option A - Axe injury:
• Chop wounds: Usually, the lower end (heel) of the axe strikes the surface first, which produces a
deeper wound than the upper (toe) end. The deeper end indicates the position of the assailant.
• Elevated fracture: It is caused by a blow from a sharp, heavy object (e.g. an axe) which elevates the
skull fracture by the lateral pull of the weapon while retrieving it.
• Depressed fractures: They are caused by blows with a heavy weapon having a small striking surface,
such as a hammer, axe, brick or chopper.
Option B - Stick injury:
• Patterned Bruise: Blows with a rod, stick or whip produce two parallel, linear hemorrhages (railway
line or tram line type).
Option C - Stone injury:

Page 10

192
• A contusion is an effusion of blood into the tissues, due to the rupture of blood vessels (veins, venules
and arterioles), caused by blunt trauma, such as fist, stone, stick, bar, whip, hammer, axe, wooden
handle, poker, shod foot, boot, etc.

Solution for Question 7:


Option D: Wind screen impact
• As a result of the accident, the windshield glass is broken. The broken windshield leads to shattered
glass. The shattered glass results in the formation of glass particles, which may take on spherical or
cubicle shapes.The driver may be thrown forward during the collision, impacting the broken windshield.
The impact of the driver's face against the broken windshield can lead to specific injuries, described as
"Sparrow Feet Mark”.
• This is due to windshield/windscreen impact
Incorrect Options:
Option A: Motor cyclist’s fracture
• Motor cyclist’s fracture is a type 1 Hinge fracture of the skull. The base is completely split into anterior
and posterior halves, creating a hinge.
Option B: Under-running or tail gating
• A unique injury wherein the motorcyclist drives under the rear of the truck, causing head injuries and
even decapitation, which is known as ‘under-running’ or ‘tail-gating’
Option C: Steering wheel impact
• Steering wheel impact injury: The circular rim of the steering wheel may cause fractures of the jaws
and facial bones, as well as imprint abrasions, minor bruises, and contusions of the chest or bilateral rib
fractures. Transverse fracture of the sternum is usually seen at the 3rd intercostal space. Damaged
steering wheel spokes may penetrate the chest and lacerate the heart and lungs. A flail chest may
occur.

Solution for Question 8:


Correct Option C - Mastoid region:
• Battle’s sign: Bruising behind the ear (over the mastoid process in the line of the posterior auricular
artery) appearing 36 hours after head injury; may be confused with retro-auricular scalp bruise.
• It is commonly seen in middle cranial fossa fractures.
Incorrect Options:
Option A - Orbital region:
• In the orbital region, black eye can be seen which is an ectopic bruise.
• In this, there is an impact at the frontal bone, but contusion is seen at the peri-orbital region.
Option B - Occipital region:

Page 11

193
• In the occipital region, only localized fractures can cause bruises.
• The fracture of the middle cranial fossa does not produce bruises on the occipital region.
Option D - Neck region:
• Bruises in the neck region are due to cervical spine injury or due to strangulation/hanging injuries.

Solution for Question 9:


Option B: Mesentery
• Seat belt injury may involve rupture of the mesentery or intestine or omentum.
• Lap belts can produce tears of the mesentery, omentum or bowel laceration
• The shoulder belt may produce a linear abrasion running downward and medially on the right side of
the driver and left side of the front seat passenger.

Option A: Spleen
• Though the spleen may be injured by a seat belt, it is not the most common organ to get an injury.
Option C: Femoral artery
• The femoral artery does not get injured by the seat belt/steering wheel.
Option D: Abdominal aorta
• The abdominal aorta may be injured by the steering wheel, not by the seat belt.
• Deceleration forces causes shearing and stretching force on the aorta resulting in injury.
• Part involved is the isthmus (distal to the subclavian artery).
• Ladder rung tear: There will be a transverse tear in the intima of the isthmus.
• Osseous punch: There will be compression between the sternum and thoracic vertebrae.

Page 12

194
Solution for Question 10:
Correct Option A - Lower cervical:
• The most common site of injury to the spinal cord: lower cervical.
Incorrect Options:
Option B & C - Thoracolumbar & Upper cervical: The most common sites of injury to the spinal cord in
whiplash injuries, in order of frequency are: Lower cervical region > Thoracolumbar junction >
Upper cervical region.
Option D - Lumbosacral: Whiplash injury does not usually affect the lumbosacral region.

Solution for Question 11:


Correct Option C - Brainstem:
• Duret hemorrhages are delayed, secondary brainstem hemorrhages (seen in the midbrain and pons).
Incorrect Options:
Option A, B and D - Cerebrum:
• HTN is the most common cause for ICH
• Most common location for ICH is the hemisphere
• Pituitary body hemorrhage is rare, and commonly d/t tumors here.

Solution for Question 12:

Page 13

195
Correct Option A - Hinge fracture:
• A fracture of the base of the skull that completely splits it, creating a hinge (‘nodding face’ sign);
frequently occurs with side impacts.(force is from one side and it goes to the other side).
• It is sometimes referred to as ‘motorcyclists fracture’.
• The most common form is the one that extends from the petrous bony ridge through sella turcica to
the lateral end of the contralateral petrous ridge.
Incorrect Options:
Option B - Ring fracture:
• Associated with fall from height due to vertebral column hitting the foramen magnum
• Seen if someone falls from a height or falling on feet or falling on buttocks.
• Can be due a to blow on the chin or blow the on vertex
• Fracture lines run around the foramen magnum like a ring.
• Due to a fracture of the posterior cranial fossa.

Option C - Comminuted:
• Comminuted fracture/spider web fracture: Multiple fracture lines intersecting each other.
• The bone breaks into pieces that may collide or split and get displaced, like a spider's web or mosaic
when there is no fragment displacement.

Page 14

196
Option D - Gutter fracture:
• Caused by an oblique bullet tangentially travelling to the skull.
• Aka glancing bullet.
• Bullet doesn't enter the skull. It just grooves the skull and doesn't produce a gutter-like fracture.

Solution for Question 13:


Correct Option C - Mosaic fracture:
• The image shows a mosaic fracture of the skull. It is also known as comminuted fracture/spider web
fracture.

Page 15

197
• Two or more intersecting lines of fracture divide the bone into three or more fragments resembling a
spider web or mosaic pattern.
• Skull bone gets broken into multiple pieces by fracture lines, which are haphazardly or concentrically
arranged, or stellate if they radiate from the site of impact.
• It is caused by vehicular accidents, falls from a height on a hard surface, or by blows with weapons
having a large striking surface, such as a heavy iron bar, or from a bullet.

Incorrect Options:
Option A - Fissure fracture:
• Fissured fractures are produced by general deformation of the skull.
• About 70% of skull fractures are linear.
• The outer table is capable of rebounding to its normal shape, while the more brittle inner table
fractures.
• It may be present alone or associated with other types.
• They are likely to be caused by forcible contact with a broad resisting surface like the ground, blows
with an agent having a relatively broad striking surface or a fall on the feet or buttocks.

Page 16

198
Option B - Depressed fracture:
• When a portion of a fractured bone is driven inwards to a distance equivalent to the thickness of the
skull table, it is known as a depressed fracture.
• It is also called ‘fracture a la signature’ (signature fracture), as the shape often points towards the
shape of the offending weapon.
• They are caused by blows with a heavy weapon having a small striking surface, such as a hammer,
axe, brick, or chopper.

Option D - Diastatic fracture:


• Diastatic or sutural fractures: Separation of the sutures occurs only in young persons, due to a blow
on the head with a blunt weapon.
• It may occur alone but often is associated with fractures. It is usually seen in the sagittal suture.
• They are particularly common in traffic accidents.

Page 17

199
• They also occur secondary to increased intracranial pressure with resulting splitting of sutures.

Solution for Question 14:


Correct Option D - Ring fracture:
• In such a case with a history of falls from height onto the feet, a Ring fracture is most likely
• Ring or foramen fracture Fracture surrounding foramen magnum. Seen if someone falls from a height
or falling on feet or falling on buttocks. Can be due a to blow on the chin or blow the on verte
• Fracture surrounding foramen magnum.
• Seen if someone falls from a height or falling on feet or falling on buttocks.
• Can be due a to blow on the chin or blow the on verte
• Fracture surrounding foramen magnum.
• Seen if someone falls from a height or falling on feet or falling on buttocks.
• Can be due a to blow on the chin or blow the on verte
Incorrect Options:
Option A - Hinge fracture:
• A fracture of the base of the skull occurs that completely splits it, creating a hinge (‘nodding face’
sign); frequently occurs with side impacts.
• It is sometimes referred to as ‘motorcyclists fracture’.
• The most common form is the one which extends from the petrous bony ridge through sella turcica to
the lateral end of the contralateral petrous ridge.
Option B - Diastatic fracture:
• Diastatic or Sutural Fractures: Separation of the sutures occurs only in young persons, due to a blow
on the head with a blunt weapon.

Page 18

200
• It may occur alone but often is associated with fractures.
• It is usually seen in the sagittal suture.
• They are particularly common in traffic accidents.
• They also occur secondary to increased intracranial pressure with resulting splitting of sutures.

Option C - Indented fracture:


• Pond or indented fracture: This is a smooth concave depression without a fracture line, resulting from
in-buckling of the skull, occurring only in the elastic skull of infants and children (prior to 4 years of age).

Solution for Question 15:


Correct Option C - 12 hours:

Page 19

201
• The earliest histological/microscopic evidence for the diagnosis of diffuse axonal injury is 'retraction
balls” which appear 12-18 hours after injury.
Incorrect Options:
Options A, B and D do not match the time frame for the diagnosis.

Solution for Question 16:


Correct Option C - Spleen:
• Spleen is the most common organ injured in blunt abdominal trauma.
Incorrect Options:
Option A - Liver: The liver is the second most common organ involved in abdominal trauma.
Option B - Kidney: Order of structures most likely to get damaged in blunt abdominal trauma: Spleen →
Liver → Kidneys → Intestines → Abdominal wall → Mesentery → Pancreas → Diaphragm.
Option D - Intestine: In order of frequency, the structures most likely to be damaged in blunt abdominal
trauma are the spleen, liver, kidneys, intestines, abdominal wall, mesentery, pancreas, and diaphragm.

Solution for Question 17:


Correct Option A: Primary impact injury
• Primary impact injuries indicate that part of the body has been struck first by the vehicle, and often
form recognizable patterns.
• Injuries include abrasions, contusion (sometimes patterned), and lacerations on legs, thighs, or
buttocks, along with fractures of the tibia and fibula, and rarely, of the femur and pelvis.
Incorrect Options:
Option B: Secondary impact injury
• These are often seen in cases of a ‘scooped up’ victim being thrown over the bonnet, i.e. further
injuries caused by the vehicle following primary impact.
• He may sustain injuries by hitting his head against the windscreen, its rim or side pillars.
• Extensive abrasions, bruises and lacerations may be seen.
Option C: Tertiary impact injury
• These result from body parts striking the ground following the primary and secondary impact.
• They are more lethal than the primary injuries, especially to the head, chest and pelvis.
Option D: Run-over injury
• Lorry, bus etc produce run-over injuries (i.e vehicle run over the victim).
• Tyre mark (intradermal bruise) on clothing/ body parts is the most specific for run-over injuries.

Page 20

202
Page 21

203
Thermal Injuries & Torture Methods
1. Wischnewski ulcers refer to which of the following?
(or)
Wischnewski ulcers refer to which of the following?
A. Gastric ulcers in hypothermia
B. Gastric ulcers in burns
C. Duodenal ulcers in hypothermia
D. Duodenal ulcers in burns
----------------------------------------
2. Which of these differentiates between antemortem and postmortem burns?
(or)
Which of these differentiates between antemortem and postmortem burns?
A. Heat rupture
B. Heat hematoma
C. Soot particles up to terminal bronchioles
D. Pugilistic attitude
----------------------------------------
3. The body of the 28-year-old woman who had allegedly committed suicide by self-immolation was
observed by the forensic doctor. There were multiple large splits in the skin on the abdomen and in the
buttock regions. There was no bleeding in the wounds. How will the margins of these heat ruptures
appear on observation?
(or)
How would you describe the margins of the heat ruptures observed in the skin of the abdomen and
buttock regions of the deceased woman who had allegedly committed self-immolation?
A. Irregular margin
B. Clear regular margin
C. Contused margin
D. Abraded margin
----------------------------------------
4. Four members of the same family were burnt to death when their hut caught fire, allegedly from
embers that had been blown around from the bonfire they had lit earlier that evening to warm
themselves. The bodies were seen with their hips and knees flexed. Their hands were held in a
defensive pose and with clenched fists. This posture in burn victims is due to which of the following?
(or)
What is the cause of the characteristic posture with hips and knees flexed, and clenched fists often
observed in burn victims?
A. Coagulation of proteins and present in ante-morterm burn

204
B. Coagulation of proteins and present in post-morterm burn
C. Coagulation of proteins and present in both ante-mortem and post-mortem burn
D. None of the above
----------------------------------------
5. The lost tourists have been rescued from the desert. The couple had wandered off from the rest of
the group while halting to click some pictures in private. Their bodies showed all signs of severe
dehydration. All the following are true of hyperthermic anhydrosis, that they would have experienced,
except?
(or)
Which of the following feature is not true about hyperthermic anhydrosis?
A. Also called as dessert syndrome
B. Complete absence of sweating
C. Hyperkeratotic plugging of sweat glands
D. Papular rashes over face and neck
----------------------------------------
6. The body of the young female tourist who had gone trekking in the Himalayas has been found. The
body was recovered in a semi-naked state but there was no signs of struggle or physical abuse. In
which scenario is such paradoxical clothing seen?
A. Seen in accidental drowning cases
B. Due to the failure of peripheral vasodilation
C. The victim may be forced to undress due to threat/influence
D. Seen in hypothermic deaths
----------------------------------------
7. The body of the missing young female escort has been recovered from a freezer in the
slaughterhouse. The skin appeared granular and puckered. The doctor documents this as
cutis-anserina. This is also possible in the living if the person was subject to which of the following?
(or)
Cutis anserina is observed in a living person in which of the following condition or stimulus?

A. Drowning
B. Lightening

Page 2

205
C. Strangulation
D. Fire arm injury
----------------------------------------
8. A Jawaan who had been left stranded at his post in Siachin during an unprovoked attack by the
neighbouring country has been rescued. He had to be immediately treated for frost bites and trench
feet. Some part of his right leg might have to be amputated to save the rest of the limb. All the following
are true about trench foot except?
(or)
All the following statements are true about trench foot except ?
A. Due to dry heat
B. Due to prolonged immersion
C. Can develop necrosis and gangrene
D. Gradual rewarming is done
----------------------------------------
9. Which among the following is/are true statement(s) about Joule burn?
(or)
Which among the following is/are true statement(s) about Joule burn?
A. Exogenous thermal burn
B. Diagnostic of contact with electricity
C. At point of entry and exit of the current.
D. Cause is heat generated in the body from electricity
E. Most commonly found at mucocutaneous junctions
----------------------------------------
10. A 26-year-old male presented with the finding as shown in the image below. All of the following
could be a cause of this finding except?
(or)
All of the following could be a cause of this finding except?

A. Hot liquid
B. Steam

Page 3

206
C. Molten rubber
D. Chemical fire
----------------------------------------
11. A 58-year-old man was working in his agriculture fields on a hot summer afternoon and collapsed.
His son told before collapsing he had Altered Mental Status and Disorientation Which of the following
features is least likely to be seen in this patient?
(or)
Which of the following feature is least likely to be seen in heat stroke?
A. Sweating
B. Hot skin
C. Hypotension
D. Altered consciousness
----------------------------------------
12. What is the form of torture where the legs and thighs are tied very tightly with bamboo sticks?
(or)
What is the term for torture involving the tight binding of legs and thighs with bamboo sticks?
A. Chepuwa
B. Saw horse
C. Black slave
D. Ghotna
----------------------------------------
13. Identify the below image:
(or)
Identify the below image:

A. Torture by ‘Parrot's perch’


B. Black Slave
C. Incaprettamento
D. Torture by ‘Campana’

Page 4

207
----------------------------------------
14. A 35 year old man's body has been brought for examination. His company was showing undue
interest in the autopsy report as the man was a victim of an occupational injury. Among the findings, the
forensic doctor has described in detail the Joule burn the victim had suffered. From this finding, what
can be deciphered as the cause of death of the victim?
(or)
What was the cause of death in the 35-year-old man who suffered a Joule burn as a result of an
occupational injury?
A. Blast injuries
B. Electrocution
C. Firearm wounds
D. Lightning stroke
----------------------------------------

Correct Answers
Question Correct Answer

Question 1 1
Question 2 3
Question 3 1
Question 4 3
Question 5 2
Question 6 4
Question 7 1
Question 8 1
Question 9 2,4
Question 10 4
Question 11 1
Question 12 1
Question 13 3
Question 14 2

Solution for Question 1:


Correct Option A - Gastric ulcers in hypothermia:
• Gastric mucosa may show small hemorrhages and ulceration, which are called Wischnewski ulcers.

Incorrect Options:

Page 5

208
Option B - Gastric ulcers in burns: In severely burned patients, stress-related ulcers of the stomach an
d duodenum, also known as Curling's ulcers, develop as a result of a
breakdown in the mucosal barrier to acid released by the body.
Option C - Duodenal ulcers in hypothermia: Wischnewski ulcers are not seen in the duodenal region.
Option D - Duodenal ulcers in burns: In severely burned patients, stress-related ulcers of the stomach
and duodenum, are also known as Curling's ulcers.

Solution for Question 2:


Correct Option C - Soot particles up to terminal bronchioles:
• Soot particles up to the level of terminal bronchiole is a specific finding in antemortem burns.

Incorrect Options:
Option A - Heat rupture: Heat rupture is a non-specific finding in antemortem and postmortem burns.D
ue to excessive heat, and severe burning/ charring, skin contracts and ruptures.
Option B - Heat hematoma: It is seen in brain. The dura matter contracts and blood excludes out from t
he venous sinus ,
It is above dura so it can be mistaken as an extradural orepidural hematoma.Heat Hematomais a
non-specific finding in antemortem and postmortem burns.
Option D - Pugilistic attitude: It is also known as boxing attitude, fencing attitude, and heat
sti■ening.It is a contraction of muscles that causes a speci■c posture.Pugilistic attitude is a
non-specific finding in antemortem and postmortem burns.

Solution for Question 3:


Correct Option A - Irregular margin:
• Heat rupture has irregular margins with no contusion or abrasion.
Incorrect Options:
Option B - Clear regular margin: Heat rupture is characterized by irregular margins.
Option C - Contused margin:
• Heat rupture is characterized by irregular margins.
• No bruising/abrasion in the margins.
Option D - Abraded margin: No bruising/abrasion in the margins.

Solution for Question 4:


Option C: Coagulation of proteins and present in both ante-morterm and post-morterm burn

Page 6

209
• The posture described in burn victims with hips and knees and their hands held in a defensive pose
with clenched fists is commonly referred to as the "pugilistic attitude" or "pugilistic posture”.
• This posture occurs due to the coagulation of proteins in the muscles and soft tissues.
• It is typically seen in burn victims, both ante-mortem (before death) and post-mortem (after death).
Incorrect choices:
Option A : Coagulation of proteins and present in ante-mortem burn:
This option is incorrect because the pugilistic posture can be present in both ante-mortem and post-mo
rtem burn cases. It is not exclusive to either one.
Option B : Coagulation of proteins and present in post-mortem burn:
This option is also incorrect because the pugilistic posture can occur in both ante-mortem and post-mor
tem burn cases. It is not limited to post-mortem situations.

Solution for Question 5:


Correct Option B - Complete absence of sweating:
• After generalized profuse sweating for days perspiration stops suddenly in all parts of the body below
the neck region but persists in the face and neck.
Incorrect Options:
Option A - Also called as dessert syndrome: Hyperthermic anhydrosis is also called dessert syndrome.
Option C - Hyperkeratotic plugging of sweat glands: There is hyperkeratotic plugging of sweat glands l
eading to functional failure of sweat apparatus.
Option D - Papular rashes over face and neck: Papular rashes appear over face and neck. It was obse
rved in U.S. soldiers having training in deserts.

Solution for Question 6:


Option D: Seen in hypothermic deaths
• Reciprocal or paradoxical undressing is the condition that is seen in hypothermic deaths.
• In deaths due to hypothermia, the body is found either partially or fully undressed.
• This sometimes results in the assumption that the deaths are associated with sexual assault
(homicide).
Incorrect options
Option A:
• Paradoxical undressing is not seen in accidental drowning cases.
Option B
• Due to long exposure to severe cold producing paralysis of the thermal regulatory mechanism. This
results in 'failure of vasoconstriction of arterioles' of the skin which results in a flow of blood from the
core of the body, thus giving an exaggerated sensation of warmth

Page 7

210
Option C
• Paradoxical undressing appears like homicide after sexual assault.
• It is not related to threatening or not done under influence

Solution for Question 7:


Correct Option A - Drowning:
• Cutis anserina, also known as "goosebumps" or "piloerection," is a physiological response that occurs
in response to various stimuli, such as cold temperature, fear, or emotional distress. It involves the
contraction of tiny muscles at the base of hair follicles, causing the hair to stand on end and the skin to
appear granular and puckered. When a person is exposed to cold water or drowning, cutis anserina
can occur as a reaction to the cold temperature and the body's physiological response to submersion in
water.

Incorrect Options:
Option B - Lightening: Lichtenberg flowers are pathognomonic of lightening strike. Lightning strikes can
cause severe burns and injuries to a
person, but cutis anserina (goosebumps) is not typically associated with lightning strikes.
Option C - Strangulation: Strangulation may cause various physical signs and injuries, but it is not direc
tly associated with cutis anserina
Option D - Firearm injury: Firearm injuries, such as gunshot wounds, can cause a range of injuries, incl
uding gunshot wounds to the skin and underlying tissues. However, cutis anserina is a
distinct physiological response and is not typically associated with firearm injuries.

Solution for Question 8:


Correct Option A - Due to dry heat:
• Trench foot is due to exposure to moist Cold.

Incorrect Options:
Option B - Due to prolonged immersion: Trench foot can be seen in persons exposed to prolonged im
mersion. It is caused due to a moist cold
Option C - Can develop necrosis and gangrene: Trench foot can develop necrosis and gangrene. leadi
ng to formation of erythema
Option D - Gradual rewarming is done: Treatment: Air drying, protecting the extremities from trauma an
d secondary infection, and gradual rewarming by exposure to air at room temperature (not heat) withou
t massaging or moistening the skin or immersing it in water.

Page 8

211
Solution for Question 9:
Correct Options
Option B - Diagnostic of contact with electricity:
• Joule burns, also known as electrical burns which is specific and diagnostic of electric burns.
Option D - Cause is heat generated in the body from electricity:
• Joule burn is due to the heat generated in the body from electricity.
Incorrect Options:
Option A - Exogenous thermal burn:
• These are endogenous thermal burns that are due to the heat generated in the body from electricity.
Option C - At point of entry and exit of the current:
• Site: At the point of entry of the current.
• Joule burn at the site of entry is diagnostic.
• The shape and size of the mark may correspond to the shape and size of the source of the current.
Option E - Most commonly found at mucocutaneous junctions:
• Joule burn is commonly found on exposed parts of the body, especially on the palmar aspects of the
hands.

Solution for Question 10:


Correct Option D - Chemical fire:
• The above image shows scald. A chemical fire is going to cause a burn, not a scald.

Incorrect Options:
Option A ,B and C
Scalds may be due to hot oils,steam molten rubber, and other liquid chemicals.

Solution for Question 11:


Correct Option A - Sweating:
• The above scenario is likely a case of heat stroke.
• Sweating is least likely to be seen in a case of heat stroke.
Incorrect Options:
Option B - Hot skin:
• Heat stroke is a life-threatening medical emergency resulting from the failure of the thermoregulatory
mechanism.

Page 9

212
• Skin is hot, and initially covered with perspiration, later it dries. Pulse is strong initially (160–180/min).
Option C - Hypotension:
• Blood pressure may be elevated in early stages, but later hypotension develops.
Option D - Altered consciousness:
• Onset is sudden, with sudden collapse and loss of consciousness.
• Delirium, blurred vision, convulsions, collapse, and unconsciousness occur.

Solution for Question 12:


Correct Option A - Chepuwa:
• The legs and thighs are tied very tightly with bamboo sticks to induce severe pain known as Chepuwa.
Incorrect Options:
Option B - Saw horse: Forced straddling of a bar (saw horse): Perineal or scrotal hematoma.
Option C - Black slave: Heated metal skewer inserted into the anus.
Option D - Ghotna: Rolling a wooden log over the thighs up &
down, while the log is weighed by one or two policemen standing on it.

Solution for Question 13:


Correct Option C - Incaprettamento:
• The above-shown image is a type of ligature strangulation known as Incaprettamento.
Incorrect Options:
Option A - Torture by ‘Parrot's perch’:
• Parrot’s perch: Person is suspended head down from a horizontal pole placed under the knees with
wrists bound to ankles.
Option B - Black Slave: Heated metal skewer inserted into the anus
Option D - Torture by ‘Campana’:
• Campana: It is a type of torture method in which the victim’s head is placed within a metal container or
pail which is then struck repeatedly, causing sudden loud sounds and reverberations.

Solution for Question 14:


Correct Option B - Electrocution:
• Joule Burns are specific and diagnostic of contact with electricity.

Page 10

213
Incorrect Options:
Option A - Blast injuries:
• The most common urgent clinical problem in survivors is usually the penetrating injury caused by
blast-energized debris, and fragments from the casing of the exploding device.
• Many of those exposed will have a blunt, blast and thermal injuries, in addition to more obvious
penetrating wounds (referred to as combined injury).
• The soft-tissue wounds are heavily contaminated with dirt, clothing and secondary missiles, such as
wood, masonry and other materials from the environment (flying missiles).
• Blast injuries are divided into four categories: primary, secondary, tertiary and quaternary.

Option C - Firearm wounds:


• Gunshot wounds are either penetrating or perforating.
• Penetrating wounds: The bullet enters an object and does not exit.

Page 11

214
• Perforating wounds: The bullet passes completely through an object.

Option D - Lightning stroke:


• A flash or bolt of lightning is due to an electrical discharge from a cloud to the earth.
• It may injure or kill an individual by a direct strike, a side flash or conduction through another object.
• Death is caused by the high-voltage direct current due to cardiopulmonary arrest or electrothermal
injuries.
• Lichtenberg flowers are pathognomonic of a lightning strike but are rare.
• It is known by different names like ‘arborization’, ‘feathering’, ‘ferning’, ‘filigree burns’ or
‘keraunographic markings’.

Page 12

215
Forensic Ballistics
1. The police who was on patrol last night was shot by a smuggler on the highway while attempting to
speed past the security booth. Which of the following factors about the bullet will determine its
maximum destruction?
(or)
What factors about the bullet will determine its maximum destruction when fired at the police on patrol
by a smuggler on the highway?
A. Size
B. Shape
C. Weight
D. Velocity
----------------------------------------
2. A 28-year-old man was among the 6 people who succumbed to the injuries sustained during a bus
stand grenade blast. An autopsy of his body would reveal the characteristic features of the Marshal's
triad except?
(or)
what characteristic feature of Marshal's triad would NOT be revealed during the autopsy?
A. Abrasion
B. Contusion
C. Avulsion laceration
D. Punctate lacerations
----------------------------------------
3. The initial impression for the examining doctor of the crime scene and the body was that it was a
case of gunshot injury. It is yet to be seen if it was self-inflicted or homicidal in nature. There was a
visible entry wound at the forehead with an abrasion collar and partial tattooing present. The pooling of
blood around the head made it difficult to view the exit wound. The forensic doctor will safely assume
that all of the following features will be present as exit wound except?
(or)
Which of the following features is NOT typically present as exit wound in a gunshot injury?
A. Bigger than the entry wound
B. Everted edges
C. Protrusion of soft tissues
D. Singeing & Smudging
----------------------------------------
4. Rifling of the bore of a gun improves the accuracy and range of the shot. In a shotgun, the bore is
smooth but the muzzle end is constricted for a few centimeters. This is known as choking. Which of the
following is the primary purpose of this design?
(or)

216
What is the primary purpose of choking in the muzzle end of a shotgun?
A. Prevent the early dispersion of pellets
B. Minimize sound
C. Minimize smoke emission
D. Cause maximum destruction
----------------------------------------
5. The forensic team has begun its investigations at the crime scene. A well-known politician was found
dead from a gunshot wound in his bedroom. The relatives and the political friend circle are alleging that
he was murdered. On initial investigation, the victim was found with the shotgun lying next to his right
hand and there was also a clear handwritten suicide note by the side of the bed. Which among the
following would help the team to confirm it to be suicidal in nature?
(or)
What evidence supports the suspicion of suicide in the case of the deceased politician with a gunshot
wound, a shotgun near the right hand, and a handwritten suicide note by bed side ?
A. Finger print on the gun
B. Blood on the gun
C. Gun in hand
D. Gunshot residues in the hand
----------------------------------------
6. Forensic ballistics involves the examination of evidence from firearms that may have been used in a
crime. When a bullet is fired from a gun, the gun leaves microscopic marks on the bullet and cartridge
case. These marks are like ballistic fingerprints. As part of the evidence, a particular finding was
documented as FG, FFG, FFFG. Which component of the firearm is the doctor documenting?
(or)
What component of the firearm is documented when findings include FG, FFG, FFFG in forensic
ballistics evidence?
A. Cartridge
B. Gunpowder size
C. Base of gun
D. Wadding of cartridge
----------------------------------------
7. While observing the wound on a victim, the doctor documented it as a circular wound with minimal
bleeding, inverted edges and the presence of an abrasion collar around the edges. what conclusions
can the doctor draw for documentation regarding the cause and nature of the injury?
(or)
What does a circular wound with minimal bleeding, inverted edges, and the presence of an abrasion
collar suggest about the cause and nature of the injury?
A. Gunshot injury
B. Stab wound

Page 2

217
C. Drowning
D. Railway track accident
----------------------------------------
8. The 25-year-old man was one of the victims of the recent hate crime. During the autopsy of the
victim, the wound on the forehead was observed to have a patterned abrasion and a stellate laceration.
What clue about the range of shots does that give to the examining doctor?
(or)
What does the presence of a patterned abrasion and stellate laceration on the victim's forehead
suggest about the range of shots in a hate crime incident?
A. Contact shot
B. Close shot
C. Range within 60 cm
D. Distant shot
----------------------------------------
9. The marks on the bullet recovered from the victim has helped the forensic team conclude that it was
fired from a weapon with spiral grooves along its bore. What is this modification which helps to improve
the accuracy and range of the shot called?
(or)
What is the modification to a firearm's barrel, involving the addition of spiral grooves, that improves the
accuracy and range of fired shots called?
A. Rifling
B. Incendiary
C. Choking
D. None of the above
----------------------------------------
10. Autopsy findings of the wound on the forehead of the gunshot victim were documented as a
cruciate wound, with inverted edges, the presence of singing of hairs and tattooing around the edges.
What is the component of the fired shot which is responsible for the findings of tattooing?
(or)
What component of the fired shot causes tattooing in a gunshot wound?
A. Burns
B. Smoke
C. Gunpowder
D. Wads
----------------------------------------
11. The body of a 45-year-old man was brought in for an autopsy. The previous night, the man had got
into an argument with another person at a bar over political opinions. The heated argument had turned
violent and ended with the other person shooting at the victim with his unlicensed rifle. To avoid being
exposed, that person and the bar owner had taken the victim to a doctor who had accepted to remove

Page 3

218
the bullet and suture the wound in private. The victim had unfortunately succumbed to the injuries in the
early hours of this morning. While performing the autopsy, the forensic doctor faced difficulty in
determining the entry and exit of the bullet wound. This phenomenon is known as?
(or)
What is the phenomenon known as when the forensic doctor faces difficulty in determining the entry
and exit of a bullet wound during an autopsy?
A. Formication phenomenon
B. Gordon phenomenon
C. Rayalaseema phenomenon
D. Kennedy phenomenon
----------------------------------------
12. These bullets have a column of pyrotechnic composition in the base that is ignited by the flame of
the propellant; this provides a visible pyrotechnic display during the bullet’s flight. They are special
ammunition that visually trace their path to the target; gunners can adjust their aim in real-time, under
actual conditions, to accurately hit their targets (especially if the target is in motion). What is the most
likely bullet?
(or)
Which type of bullet is designed to provide a visible pyrotechnic display during its flight, enabling
real-time aiming adjustments for hitting moving targets?
A. Tandem bullet
B. Tracer bullet
C. Dum-dum bullet
D. Incendiary bullet
----------------------------------------
13. The forensic team has begun a thorough sweep of the bedroom where the victim's body was found.
On immediate observation, a large exit wound was noticed which most likely was from a close contact
shot. In the direction of the forward spatter, at a certain distance, some intact parts of the victim's brain
were also recovered. this would be documented as which of the following?
(or)
What is the appropriate documentation for a close-contact gunshot wound with an exit wound and the
recovery of intact brain tissue in the forward spatter direction during a forensic investigation?
A. Kronlein shot
B. Back spatter
C. Billiard ball effect
D. Balling of shot
----------------------------------------
14. Which among the following statements is true about the tail wagging of a bullet?
(or)
Which among the following statements is true about tail wagging of a bullet?

Page 4

219
A. Bullet rotates end-to-end during its path
B. Causes the tattooing around the entry wound
C. Seen when the bullet leaves the muzzle at the beginning of its flight
D. Seen when the bullet loses its velocity near the termination of its flight
----------------------------------------
15. A victim of a recent blast injury was brought in for a follow-up check-up. After evaluation, the doctor
was able to conclude that the man requires management of his Quaternary blast injuries. Which among
the following is a possible diagnosis made by the doctor?
(or)
What is a possible diagnosis for the victim of a recent blast injury requiring management of Quaternary
blast injuries?
A. Bone fractures
B. ARDS
C. Penetrating trauma
D. Psychiatric trauma
----------------------------------------
16. A young woman has been fatally shot by a man who had been stalking her for some time with
hopes of starting a relationship with her. The forensic team has had difficulty with the autopsy. Though
an entry wound was detected, an exit wound was not seen and there was no sign of the bullet in its
projection tract in the body. The bullet was not detected on an X-ray either. Which among the following
is the site where such a wandering bullet would most commonly be lodged in?
(or)
Where is the most common site for a wandering bullet ?
A. Aorta
B. Femoral vein
C. Carotid artery
D. IVC
----------------------------------------
17. After a week of being questioned, the suspect revealed the location where he can hide the revolver
used to commit suicide and can use it to commit the homicide. The forensic doctor is aware that a
revolver is effective around the range of?
(or)
What is the effective range of a revolver?"
A. 30-35 m
B. 100-200 m
C. 1000 m
D. 3000 m
----------------------------------------

Page 5

220
18. The gun used by the alleged suspect in the shooting incident has been recovered. The forensic
team has conducted tests on the bullets recovered from the crime scene and has now begun an
examination of the bore of the gun to match the bullet to the gun. Among the following instruments,
which one would be used for the examination?
(or)
Which instrument is typically used to examine the bore of a recovered gun in forensic ballistics to match
bullets to the firearm used in a shooting incident?
A. Broach cutter
B. Helixometer
C. Comparison microscope
D. Simple microscope
----------------------------------------
19. Bullet wipe term is used for which of the following?
(or)
Bullet wipe term is used for which of the following?
A. Gutter fracture of skull
B. Blackening
C. Tattooing
D. Dirt from barrel
----------------------------------------
20. The images below are of a type of bullet used by the army during training. They are made of
compressed metals and disintegrate on contact with the target and are useful in accessing the
accuracy and precision of the soldier. What type of bullet is shown?
(or)
What type of bullet is shown in the given?

A. Souvenir bullet
B. Frangible bullet
C. Piggy back bullet
D. Tracer bullet

Page 6

221
----------------------------------------
21. Which of the following is useful for the identification of the class characteristics of a gun?
A. Bullet fingerprinting
B. Primary markings
C. Secondary markings
D. Metallic fouling
----------------------------------------
22. You receive a body of a 38- years-old male for autopsy. During the procedure, you are seeing the
finding as shown in the image below. What is the most likely firearm that caused this injury?
(or)
What type of firearm is most likely responsible for the injury depicted in the image?

A. Rifle
B. Pistol
C. Revolver
D. Shotgun
----------------------------------------
23. Which of the following is the appropriate precaution to be followed while recovering a bullet from the
crime scene?
(or)
What is the appropriate precaution when recovering a bullet from a crime scene?
A. Bullet should be cleaned with sterile water immediately
B. Bullet should be packed in leakproof packaging made of hard plastic
C. Bullet should be picked with metallic toothed forceps
D. Bullet should be removed with bare hands
----------------------------------------
24. During the autopsy of the body brought in for examination by the medical officer, he documented
some of the findings in association with Puppe's rule. What type of injury is the doctor documenting
about?
(or)

Page 7

222
What type of injury is the doctor documenting when referring to Puppe's rule during the autopsy?
A. Sexual assault
B. Multiple impact injuries
C. Chemical injuries
D. Percentage of burns
----------------------------------------

Correct Answers
Question Correct Answer

Question 1 4
Question 2 3
Question 3 4
Question 4 1
Question 5 4
Question 6 2
Question 7 1
Question 8 1
Question 9 1
Question 10 3
Question 11 4
Question 12 2
Question 13 1
Question 14 3
Question 15 4
Question 16 1
Question 17 2
Question 18 2
Question 19 4
Question 20 2
Question 21 2
Question 22 4
Question 23 2
Question 24 2

Solution for Question 1:

Page 8

223
Correct Option D - Velocity:
• The capacity of a bullet to cause maximum destruction lies in its velocity.
Incorrect Options:Option A /Option B/Option C

Solution for Question 2:


Correct Option C - Avulsion laceration:
• Avulsion laceration is seen in run over injuries in roadside accidents.
Incorrect Options:
Option A - Abrasion:
• Marshall’s triad is seen in bomb blast injuries.
• It is a triad of bruises, abrasions and punctate lacerations.
Option B - Contusion:
• Bruise/contusion, abrasion, and laceration seen in blast injury are known as Marshall's triad.
Option D - Punctate lacerations:
• Marshall’s triad of bruises, abrasions and punctate lacerations with tattooing of the body indicates
bomb explosion.

Solution for Question 3:


Correct Option D - Singeing & smudging:
• Singeing & Smudging are the features of the entry wound; not the exit wound.
Incorrect Options:
Option A
- Bigger than the entry wound: The wound of exit will be slightly bigger than the wound of entry.
Option B - Everted edges: The wound of exit: Through which the tip of the weapon emerges out of the
body. It is usually smaller with everted edges.
Option C - Protrusion of soft tissues: The exit wound when present, it is in the form of a serrated, irregu
lar laceration with everted margins through which some tissues or bone fragments may be seen protru
ding.

Solution for Question 4:


Correct Option A - Prevent the early dispersion of pellets:

Page 9

224
• Choking is a technique utilized in smoothbore or shotgun firearms where the terminal portion of the
bore, located a few centimeters from the muzzle, is intentionally narrowed. This constriction helps
regulate the spread of shot, keeping the shot column more cohesive and improving its effectiveness
over a distance.
• It reduces the dispersion of the pellets (primary purpose).

Solution for Question 5:


Correct Option D - Gunshot residues in the hand:
• Gunshot residues in the hand confirm it to be suicidal in nature.
• GSR on the hands may be visible to naked eyes and can be observed and described.
• If not visible special techniques can be used to demonstrate invisible GSR.
Incorrect Options -Option A/Option B/Option C

Solution for Question 6:


Correct Option B - Gunpowder size:
• The documentation of "FG," "FFG," or "FFFG" likely refers to the size of the gunpowder or gunpowder
grains found in the cartridge case. Black powder size helps to determine the type of ammunition, its
performance characteristics, and potentially link it to a specific firearm or batch of ammunition.
The documentation of "FG," "FFG," or "FFFG" likely refers to the size of the gunpowder or gunpowder
grains found in the cartridge case. Black powder size helps to determine the type of ammunition, its per
formance characteristics, and potentially link it to a specific firearm or batch of ammunition.
Incorrect Options:
Option A - Cartridge: The notation "FG," "FFG," or "FFFG" does not directly refer to the cartridge itself
but rather to the size of the black powder grains within it.
Option C - Base of gun: The critical forensic analysis typically focuses on the bullet, cartridge case, and
the marks left on them, rather than the base of the gun.
Option D
- Wadding of cartridge: The notation "FG," "FFG," or "FFFG" does not directly relate to the wadding.

Solution for Question 7:


Correct Option A - Gunshot injury:
• The gunshot entry wound will be circular in shape, approximately the same size as the bullet, with
lacerated, inverted edges surrounded by a narrow zone of grease and abrasion collar, with no evidence

Page 10

225
of any burning and singeing.
• The gunshot entry wound will be circular in shape, approximately the same size as the bullet, with
lacerated, inverted edges surrounded by a narrow zone of grease and abrasion collar, with no evidence
of any burning and singeing.
Abrasion collar/ring:

• Proof of bullet entry wound.


• Due to spinning motion of bullet.
Incorrect Options:
Option B - Stab wound:
• The force of the stab wound is delivered along the long axis of a pointed object (e.g. knife, scissors,
needle, pencil etc) into the body. Depth is the largest dimension.
• Classification of Stab Wound: Penetrating wound: When the weapon pierces through the body and
enters into a body cavity or viscus; Perforating wound: When the weapon pierces through the whole
thickness of any part of the body thus producing two surface wounds, i.e. wound of entrance and
wound of exit. The entrance wound is bigger than the exit wound due to the tapering of the blade of the
weapon and its margins will be inverted, whereas the margins of the exit wound will be everted. The
track of the stab wound, starting from its entrance towards the termination, is convergent compared to
the track of a gunshot wound caused by a missile at short ranges where it is divergent.
• The entrance wound is bigger than the exit wound due to the tapering of the blade of the weapon and
its margins will be inverted, whereas the margins of the exit wound will be everted.
• The track of the stab wound, starting from its entrance towards the termination, is convergent
compared to the track of a gunshot wound caused by a missile at short ranges where it is divergent.
• The entrance wound is bigger than the exit wound due to the tapering of the blade of the weapon and
its margins will be inverted, whereas the margins of the exit wound will be everted.
• The track of the stab wound, starting from its entrance towards the termination, is convergent
compared to the track of a gunshot wound caused by a missile at short ranges where it is divergent.

Page 11

226
Option C - Drowning:
• External findings in drowning: The face is pale; eyes are found half open or closed; the tongue may be
swollen and protruded. The presence of fine, copious white ‘shaving-lather’ like froth at the mouth and
nostrils is the most characteristic antemortem external finding. Cutis anserina (goose skin/goose
flesh/goosebumps/ horripilation). Washerwomen’s hand. Cadaveric spasm:Cadaveric spasm refers to a
phenomenon where a set of muscles extensively engaged just prior to death undergoes immediate and
pronounced stiffness and rigidity following the cessation of life
• The face is pale; eyes are found half open or closed; the tongue may be swollen and protruded.
• The presence of fine, copious white ‘shaving-lather’ like froth at the mouth and nostrils is the most
characteristic antemortem external finding.
• Cutis anserina (goose skin/goose flesh/goosebumps/ horripilation).
• Washerwomen’s hand.
• Cadaveric spasm:Cadaveric spasm refers to a phenomenon where a set of muscles extensively
engaged just prior to death undergoes immediate and pronounced stiffness and rigidity following the
cessation of life
• Internal findings in drowning: Lungs are voluminous, distended and show ballooning. Tenacious,
lathery froth in the trachea and bronchi is present. In the case of a laryngeal spasm, there will be no
ballooning. Paltauf hemorrhage; emphysema aquosum (emphyseme hydroaerique). When the person
is unconscious at the time of drowning, edema aquosum develops.
• Lungs are voluminous, distended and show ballooning. Tenacious, lathery froth in the trachea and
bronchi is present. In the case of a laryngeal spasm, there will be no ballooning.
• Paltauf hemorrhage; emphysema aquosum (emphyseme hydroaerique).
• When the person is unconscious at the time of drowning, edema aquosum develops.
• The face is pale; eyes are found half open or closed; the tongue may be swollen and protruded.
• The presence of fine, copious white ‘shaving-lather’ like froth at the mouth and nostrils is the most
characteristic antemortem external finding.
• Cutis anserina (goose skin/goose flesh/goosebumps/ horripilation).
• Washerwomen’s hand.
• Cadaveric spasm:Cadaveric spasm refers to a phenomenon where a set of muscles extensively
engaged just prior to death undergoes immediate and pronounced stiffness and rigidity following the
cessation of life
• Lungs are voluminous, distended and show ballooning. Tenacious, lathery froth in the trachea and
bronchi is present. In the case of a laryngeal spasm, there will be no ballooning.
• Paltauf hemorrhage; emphysema aquosum (emphyseme hydroaerique).
• When the person is unconscious at the time of drowning, edema aquosum develops.
Option D - Railway track accident:
• It is a common mode of suicide, but accidents are common in children.
• The body may be severed into many pieces and soiled by axle grease and dirt from the wheels and
track. When passengers fall off the train, multiple injuries along with abrasions are seen due to contact
with coarse gravel along the line ballast.
• Suicidal Injuries: The injuries are extensive and due to primary impact. If a person lies down on the
track, extrusion of organs, traumatic amputations of the limbs or trunk or decapitation may occur. Wheel

Page 12

227
marks and dirt and grease contamination may be found on the body.

Solution for Question 8:


Correct Option A - Contact shot:
• The stellate wound is a characteristic of a contact wound.

Incorrect Options:
Option B - Close shot:
• The body lies within the range of flame, smoke and powder blast, i.e. within 2–3 inches (5–8 cm).
• The entry wound is small and circular in shape having inverted and contused lacerated margins.

Option C - Range within 60 cm:

Page 13

228
• Tattooing becomes discrete as the range increases, no trace of powder marks will be found when the
range is beyond 24 inches, i.e. normally beyond arm’s length.
• For handguns, powder tattooing extends to a maximum distance of 18–24 inches (45–60 cm).

Option D - Distant shot:


• Gunshot entry wounds with no associated soot or gunpowder stippling are referred to as ‘distant’
wounds, i.e. range is beyond 2 feet.
• The entry wound is usually circular in shape, smaller than the bullet, because of the elasticity of the
skin, with lacerated, inverted skin margins, a bigger dirt collar and the usual zone of an abraded collar.

Solution for Question 9:


Correct Option A - Rifling:

Page 14

229
• In rifled firearms, the Interior of the bore has spiral grooves which run parallel to each other but are
twisted spirally from breech to the muzzle end.
• These grooves are called ‘rifling’.
Incorrect Options:
Option B - Incendiary:
• Incendiary bullets: A type of army bullet used to cause a fire in the target.
Option C - Choking:
• Advantages of choking: reduces the rate of dispersion of the shot, increases the explosive force and
increases velocity, and thus increases the range.
• Primary purpose: Reduces dispersion of the pellets.

Solution for Question 10:


Correct Option C - Gunpowder:
• Gunpowder is the component of the fired shot which is responsible for the findings of tattooing.
• Tattooing: It consists of unburnt or partially burnt powder particles that are embedded in and under the
skin through the force of their impact (when the weapon is near enough for the powder grains to strike).
• Tattooing is an antemortem phenomenon and indicates that the individual was alive, and it cannot be
wiped away with wet cotton.
Incorrect Options:
Option A - Burns:
• Burning at entrance wound along with burning or singeing of hairs due to flame/fire.
• Smokeless powder: It is more effective than black powder as it burns more efficiently and produces
much less smoke, resulting in less blackening and tattooing around the entry wound.
Option B - Smoke:
• Soot or smoke soiling/blackening/smudging: These particles being light, do not travel afar (for virtually
all handgun cartridges, soot is absent around the entry wound where the muzzle-to-target distance is
beyond one foot).
Option D - Wads:
• Wad is made of some soft material, like a disc of felt, cardboard, plastic, cork or straw.
• It is placed between powder and shot or over the shot.
• The cardboard disc behind the shot charge prevents the pellets from getting lodged in the felt wad.

Solution for Question 11:


Correct Option D - Kennedy phenomenon:
• Iatrogenic alteration of a gunshot wound.

Page 15

230
• There is Difficulty in range determination.
Incorrect Options:
Option A - Formication phenomenon:
• Formication is the perception of what appears to be tiny insects crawling on (or under) the skin but is
actually nothing.
Option B - Gordon phenomenon:
• The Gordon phenomenon is the degradation of cerebellar Purkinje cells following intracerebral
injection of eosinophil granulocytes or their preparations.
Option C - Rayalaseema phenomenon:
• After the stabbing, the bullet is put inside the wound.
• This is the Rayalaseema phenomenon.

Solution for Question 12:


Option B: Tracer bullet
• Tracer bullets leave a visible mark or "trace' while in flight, so that the path of the bullet can be
observed.
Option A: Tandem bullet
• Tandem bullet (Piggyback bullet): Bullets are ejected one after the other, when the first bullet having
been struck in the barrel, fails to leave the barrel and is ejected by a subsequently fired bullet.
• Cause: Faulty ammunition or loaded firearm unused for years.
Option C: Dum-dum bullet
• Dum-dum bullet: Jacketed bullet which does not cover the entire bullet, an area near the nose is left
uncovered to expose the core.
Option D: Incendiary bullet
• Incendiary bullets are a type of army bullet used to cause a fire in the target.

Solution for Question 13:


Correct Option A - Kronlein shot:
• Seen only in contact wounds.
• Most commonly with high-velocity missile.
• A large exit wound is produced and a large portion of the brain or even the entire brain may be thrown
out of the bursting skull and found relatively intact at a distance.
Incorrect Options:
Option B - Back spatter:
• Once the bullet is fired, a spattering of blood back into the barrel.

Page 16

231
Option C - Billiard Ball Effect:
• It occurs after the shotgun pellets strike an intermediate object like doors, etc.
• When a group of shotgun pellets hits tissue, The first pellets are slowed down by the tissues which on
impact from the subsequent pellets scatter into the tissue, similar to a cluster of billiard balls struck by
the cue ball.
• On X-ray, the dispersal of pellets may suggest a distant shot, while actually, the weapon was fired
from close range.
Option D - Balling of shot:
• Sticking of Lead shots, which can result in a major entry wound that is circular or oval in shape and
multiple smaller punctures that are circular in shape.
• This suggests the usage of two weapons, one rifle and one shotgun.

Solution for Question 14:


Correct Option C - Seen when the bullet leaves the muzzle at the beginning of its flight:
• Tail wobble/Tail wagging occurs for a few microseconds after a bullet exits the muzzle.
• After the bullet has left the muzzle end and for a few microseconds (up to about 50 meters for a pistol
or 150 meters for a rifle), there may be a “tail wobble” or “tail wag”. This is partly responsible for the
great tissue damage and the large atypical entrance wound at short range.
Incorrect Options:
Option A
- Bullet rotates end-to-end during its path: Tumbling bullet: Bullet rotates end-to-end during its path.
Option B - Causes the tattooing around the entry wound Evidence of burning, singeing, blackening, an
d tattooing of the skin in and around the entry wound can be seen in a Close Shot (Flame Range).
Option D - Seen when the bullet loses its velocity near the termination of its flight: Tail wobble/Tail wag
ging occurs for a few microseconds after a bullet exits the muzzle, and does not occur when the bullet l

Page 17

232
oses its velocity near the termination of its flight.

Solution for Question 15:


Correct Option D - Psychiatric trauma:
• Psychiatric injury (due to neurological damage sustained during the blast) is most common, and
post-traumatic stress disorder (PTSD) may affect people who are otherwise completely uninjured.
Incorrect Options:
Option A - Bone fractures:
• Tertiary injuries may present as some combination of blunt and penetrating trauma, including bone
fractures and coup contrecoup injuries.
• Children are at particular risk because of their lesser weight.
Option B - ARDS:
• Primary injuries are caused by blast waves and are characterized by the absence of external injuries.
• They are usually internal injuries that are often unrecognized and their severity is underestimated.
• The ears are most often affected by overpressure, followed by the lungs and the hollow organs of the
GIT.
Option C - Penetrating trauma:
• Secondary injuries are due to people being injured by shrapnel and other objects propelled by the
explosion.
• These injuries may affect any part of the body and sometimes result in penetrating trauma.

Solution for Question 16:


Correct Option A - Aorta:
• The most common sites of entry for a bullet are the aorta, right atrium and ventricle, pulmonary artery,
and inferior vena cava.
Incorrect Options:
Option B - Femoral vein: The wandering bullet does not commonly lodge in the femoral vein.
Option C - Carotid artery: As the carotid artery has a
lesser diameter, the wandering bullet does not commonly lodge in it.
Option D - IVC: A
wandering bullet can lodge in the Inferior vena cava, but it is not the most common site.

Solution for Question 17:

Page 18

233
Correct Option B - 100-200 m:
• The revolver is effective around the range of 100-200 m.
Incorrect Options:
Option A - 30-35 m: The shotgun is effective around the range of 30-35 m.
Option C - 1000 m: The rifle is effective around the range of 1000 m.
Option D - 3000 m: The military rifle is effective around the range of 3000 m.

Solution for Question 18:


Correct Option B - Helixometer:
• An instrument for inspecting the interior of a gun barrel and for measuring the rate of twist of the rifled
bore of a firearm.
Incorrect Options:
Option A - Broach cutter: A broach cutter is used to produce riflings in a gun.
Option C
- Comparison microscope: A comparison microscope is used to match the marks in test bullet &
crime bullet
Option D - Simple microscope: Simple microscopes make use of a
biconvex lens to magnify the image of a
specimen. The closer the object is to the lens, the larger the magnified image becomes.

Solution for Question 19:


Correct Option D - Dirt from barrel:
Grease Collar (Bullet Wipe):
• A black/grey coloured ring is seen lining the entrance wound.
• Due to the deposition of lubricants, gun oil or lead from the bullet.

Page 19

234
• An abrasion collar surrounds the dirt collar.
• Dirt collar rim is more prominent in clothing: ‘Bullet wipe’.
• Abrasion and dirt collars are proof of an entry wound.
Incorrect Options:
Option A - Gutter fracture of skull:
• A gutter fracture is formed when part of the thickness of the bone is removed so as to form a gutter,
e.g. oblique bullet wounds.
• It is usually accompanied by comminuted depressed fracture of the inner table of the skull, and the
fragments causing injury to the meninges and brain.
Option B - Blackening:
• Deposition of powder soot (carbon) produced by combustion of gunpowder. As the range increases,
the size of the zone of blackening will increase, whereas the density will decrease. It can be easily
removed with wet cotton.
• As the range increases, the size of the zone of blackening will increase, whereas the density will
decrease.
• It can be easily removed with wet cotton.
• As the range increases, the size of the zone of blackening will increase, whereas the density will
decrease.
• It can be easily removed with wet cotton.
Option C - Tattooing:
• It consists of unburnt or partially burnt powder particles that are embedded in and under the skin
through the force of their impact (when the weapon is near enough for the powder grains to strike).
• It cannot be wiped away with wet cotton.
• It consists of numerous reddish-brown punctate abrasions surrounding the wound of the entrance.
• The greater the range, the larger and less dense the powder tattooing.

Page 20

235
Solution for Question 20:
Correct Option B - Frangible bullet:
• A frangible bullet is a type of ball bullet made from compressed particles of metal (Zinc/ Copper) and
paint.
Incorrect Options:
Option A - Souvenir bullet:
• Also known as sleeping bullet.
• It is a retained bullet in the body or skin.
• Cause lead poisoning.
Option C - Piggy back bullet:
• In Tandem bullet/ Piggyback bullet: Bullets are ejected one after the other, when the first bullet having
been struck in the barrel, fails to leave the barrel and is ejected by a subsequently fired bullet.
• Cause: Faulty ammunition or loaded firearm unused for years.
Option D - Tracer bullet:
• It leaves a visible mark or ‘trace’ while in flight so that the gunner can observe the strike of the shot.

Solution for Question 21:


Correct Option B - Primary markings:
• Primary markings are also called class characteristics.
Incorrect Options:
Option A - Bullet fingerprinting:
• Both primary and secondary markings are used for bullet fingerprinting. Primary markings: D/t rifling.
Secondary markings: D/t irregularities in the barrel.
• Primary markings: D/t rifling.
• Secondary markings: D/t irregularities in the barrel.
• Primary markings: D/t rifling.
• Secondary markings: D/t irregularities in the barrel.
Option C - Secondary markings:
• Secondary markings are called individual characteristics.
Option D - Metallic fouling:
• Tiny lesions around the entry wound caused by fragments of metal expelled by the discharge.

Page 21

236
Solution for Question 22:
Correct Option D - Shotgun:
• The injury shown in this image is a characteristic of a shotgun wound.
• There is a central round hole with multiple pellets holes scattered around the central hole.
• This wound is most likely to have been caused by a shotgun from a range of 1-5 m.
Incorrect Options:
Option A - Rifle:
• A rifle is a firearm with a rifled barrel that is designated to be fired from the shoulder.
• In a rifle, the bullet is elongated with a pointed end.
Option B - Pistol:
• A handgun is a firearm capable of being carried and used by one hand, such as a pistol or revolver.
Option C - Revolver:
• The revolver has a revolving cylinder that contains several chambers, each of which holds one
cartridge.
• In revolvers and pistols, the bullet is short, and the point is usually round.

Solution for Question 23:


Correct Option B - Bullet should be packed in leakproof packaging made of hard plastic:
• The bullet should be packed in leakproof packaging made of hard plastic.
Incorrect Options:
Option A - Bullet should be cleaned with sterile water immediately: The bullet is gently rinsed only after
collection of any trace evidence on the bullet (not immediately).
Option C - Bullet should be picked with metallic toothed forceps: To prevent marring of the bullet surfac
e, it should be removed by a plastic forceps or a rubber tipped bullet extractor.
Option D - Bullet should be removed with bare hands: Bullets should be handled only by persons weari
ng double heavy-duty gloves.

Solution for Question 24:


Correct Option B - Multiple impact injuries:
• Multiple impact injuries of the skull are associated with Puppe's rule .
Incorrect Options:
Option A - Sexual assault: Sexual assault does not associate with Puppe's rule.
Option C - Chemical injuries: Chemical injuries do not associate with Puppe's rule.
Option D - Percentage of burns: Burn injuries do not associate with Puppe's rule.

Page 22

237
Page 23

238
Previous Year Questions
1. Identify the type of wound from the image.

A. Defense wounds
B. Antemortem wound
C. Postmortem wound
D. Hesitation cuts
----------------------------------------
2. When examining a patient who has been assaulted, you observe a wound that has uneven and
jagged edges. The tears in the wound extend at an angle from the main wound and have distinct
"swallowtail" patterns. The bleeding from this wound is minimal and there are crushed blood vessels
and hair bulbs. How would you classify and document this wound?
A. Incised wound
B. Chopped wound
C. Stab wound
D. Lacerated wound
----------------------------------------
3. What characteristic helps distinguish a genuine bruise from a fabricated one among the options
provided?
A. Positive chemical tests
B. Progression of the color of bruise over time
C. Regular and well-defined margins
D. Presence of vesicles
----------------------------------------
4. Identify the pattern of abrasion shown in the image below.

239
A. Pressure abrasion
B. Graze abrasion
C. Ligature mark
D. Imprint abrasion
----------------------------------------
5. The image given below is a type of

A. Contusion
B. Abrasion
C. Laceration
D. Stab wound
----------------------------------------
6. What is the identification of the image depicted?

Page 2

240
A. Hinderers line
B. Langers lines
C. Blaschko lines
D. Dermatomes
----------------------------------------
7. Please identify the lesion displayed in the image provided.

A. Laceration
B. Incised wound
C. Laceration-looking incised wound
D. Incised looking laceration
----------------------------------------

Page 3

241
8. What type of wound is likely to be present when hair bulbs are observed to be crushed during injury
examination?
A. Abrasion
B. Laceration
C. Stab injury
D. None
----------------------------------------
9. Identify the post-mortem finding in a patient with a stab wound on the left side of the chest.

A. Cardiac tamponade
B. Fat necrosis
C. Hemothorax
D. Pulmonary embolism
----------------------------------------
10. Which of the following is classically associated with a lucid interval?
A. Subdural haemorrhage
B. Parenchymal haemorrhage
C. Epidural hematoma
D. Subarachnoid haemorrhage
----------------------------------------
11. Please match the following weapons with their corresponding types of injuries. A. Axe B. RTA C.
Blade D. Lathi Incised wound Tram track bruise Grazed abrasion Chop wound
A. AxeB. RTAC. BladeD. Lathi Incised woundTram track bruiseGrazed abrasionChop wound

A. A-4, B-3, C-1, D-2


B. A-1, B-3, C-4, D-2
C. A-1, B-2, C-4, D-3
D. A-4, B-1, C-3, D-2
----------------------------------------

Page 4

242
12. Rearrange the following to correspond with alterations in fingerprints. 1. Incomplete atrophy of
ridges 2. Less of pattern with ridge atrophy 3. Altered ridges 4. Permanent loss of fingerprints 5.
Distance between ridges changed but pattern retained A. Radiation B. Scleroderma C. Celiac disease
D. Acromegaly E. Dermatitis
trophy3. Altered ridges4. Permanent loss of fingerprints5. Distance between ridges changed but pattern retained A. RadiationB. Scleroder

A. 1-e, 2-c, 3-b, 4-a, 5-d


B. 1-d, 2-b, 3-c, 4-e, 5-a
C. 1-c, 2-a, 3-d, 4-b, 5-e
D. 1-d, 2-c, 3-a, 4-e, 5-b
----------------------------------------
13. What is the probable etiology of the patient's burn injury who presents to the outpatient department
(OPD) with a dampened, erythematous, blistered lesion on the right thigh, along with a demarcation line
that extends distally?
A. Electric burn
B. Chemical burn
C. Dry heat
D. Moist heat
----------------------------------------
14. Which mechanism is responsible for the injuries sustained by a man brought to the emergency
room with multiple small puncture wounds on the right side of his chest, below the nipple, and multiple
small abrasions and bruising on the left lower abdomen following a blast incident?
A. Flying missiles
B. Blast of air
C. Explosion burns
D. Building collapse
----------------------------------------
15. What is the type of wound found on a dead body with clean-cut edges, crushed tissues, and
disruption of the vertebral bones in the neck region?
A. Incised wound
B. Chop wound
C. Lacerated wound
D. Avulsion
----------------------------------------
16. Please determine the category of abuse depicted in the provided image.

Page 5

243
A. Bagging
B. Snorting
C. Sniffing
D. Huffing
----------------------------------------
17. What is the association between a gutter fracture of the skull bone and impact caused by-
A. A sharp object
B. A blunt object
C. Fall from height
D. A bullet
----------------------------------------
18. Which is the first organ to putrefy?
A. Brain
B. Heart
C. Kidney
D. Prostate
----------------------------------------
19. Please match the following weapons with the potential injuries they may cause: A. Axe B. RTA C.
Blade D. Lathi Incised wound Train track bruise Grazed abrasion Chop wound
A. AxeB. RTAC. BladeD. Lathi Incised woundTrain track bruiseGrazed abrasionChop wound

A. A-4, B-3, C-1, D-2


B. A-1, B-3, C-4, D-2
C. A-1, B-2, C-4, D-3
D. A-4, B-1, C-3, D-2
----------------------------------------
20. Name the type of skull fracture.

Page 6

244
A. Depressed fracture
B. Hinge fracture
C. Pond fracture
D. Comminuted fracture
----------------------------------------
21. Identify the injury the image below :

A. Abrasion
B. Laceration
C. Bruise
D. Imprint
----------------------------------------
22. Paradoxical undressing is seen in :
A. Heart stress
B. Voyeurism
C. Hypothermia
D. LSD overdose
----------------------------------------
23. Which of the following statements is not true regarding the given condition?

Page 7

245
A. Swabs should be taken immediately after moistening with sterile water
B. It is most commonly caused by incisors and canines
C. There are fewer chances of bacterial infection
D. Tetanus vaccination should be given
----------------------------------------
24. What type of wound can be identified in a middle-aged man who was brought to the emergency
room following an attack involving a sharp axe?

A. Incised wound
B. Chop wound
C. Lacerated wound
D. Abrasion
----------------------------------------
25. During a football match, a 12-year-old boy was tackled by an opponent and sustained an injury. As
the attending medical officer, how would you document the wound in the case sheet?

Page 8

246
A. Incised wound
B. Abrasion
C. Laceration
D. Bruise
----------------------------------------
26. Identify the image shown below

A. Shot gun wound


B. Tattooing
C. Arsenic poisoning
D. Graze wound
----------------------------------------
27. A 31-year-old man was found dead at home. He was brought to the mortuary by the police, and the
following were noted during the examination: Which of the following is the likely cause of his death?

Page 9

247
A. Strangulation by ligature
B. Hanging
C. Throttling
D. Drowning
----------------------------------------
28. Inhaling the cloth soaked in drug is
A. Bagging
B. Spraying
C. Huffing
D. Sniffing
----------------------------------------
29. From the dense vegetation near the highway, the police have found the remains of a burnt body.
The location was discovered with the help of a sniffer dog, as they were tracing a 25-year-old IT worker
who had gone missing the night before. To determine the identity of the victim, the team needs to
gather a suitable sample. Which type of tissue from a charred body is most effective for identification
purposes?
A. Hair
B. Bone
C. Teeth
D. Blood
----------------------------------------
30. Please determine the classification of the skull fracture depicted in the provided image.

Page 10

248
A. Ring
B. Depressed
C. Fissured
D. Comminuted
----------------------------------------
31. Identify the gun that caused the wound and the range of the shot.

A. Shotgun, intermediate range


B. Shotgun, close range
C. Pistol, near shot
D. Pistol, close shot
----------------------------------------
32. In a reported case of gunshot injury, there are visible signs of burning, blackening, and tattooing
surrounding the wound. The nature of the injury can be described as:
A. Close shot entry wound
B. Close shot exit wound
C. Distant shot entry wound
D. Distant shot exit wound
----------------------------------------
33. Bullet fingerprinting involve:

Page 11

249
A. Primary markings
B. Secondary markings
C. Both
D. None
----------------------------------------
34. In a case where a gunshot victim was brought for autopsy, the entry wound displayed margins that
were turned inward. Additionally, there was evidence of tattooing around the wound, but no signs of
blackening or singeing of hair were observed. Based on this information, what is the likely firing range
of the gunshot?
A. Contact shot
B. Near shot
C. Close shot
D. Distant shot
----------------------------------------
35. What is bullet wipe ?
A. Residual from barrel of gun
B. Blackening
C. Gutter fracture of skull
D. Tattooing
----------------------------------------
36. Which of the following is true about exhumation?
A. The executive magistrate can order for exhumation
B. Police can order for exhumation
C. Postmortem can be done on exhumated body
D. CrPC 174 is related to exhumation
----------------------------------------

Correct Answers
Question Correct Answer

Question 1 1
Question 2 4
Question 3 2
Question 4 3
Question 5 1
Question 6 2
Question 7 4

Page 12

250
Question 8 2
Question 9 1
Question 10 3
Question 11 1
Question 12 1
Question 13 4
Question 14 1
Question 15 2
Question 16 1
Question 17 4
Question 18 1
Question 19 1
Question 20 2
Question 21 1
Question 22 3
Question 23 3
Question 24 2
Question 25 3
Question 26 1
Question 27 1
Question 28 3
Question 29 3
Question 30 3
Question 31 3
Question 32 1
Question 33 3
Question 34 2
Question 35 1
Question 36 1

Solution for Question 1:


Correct Option A:
• Defense wounds are wounds that are dispensed by an individual who is attempting to defend
themselves from an aggressor.
• These wounds are regularly found on the hands and arms when individuals try to protect themselves
from the attack.

Page 13

251
• If the weapon grasps while defending, the Palmar surface of 1st webspace is the MC site affected.
This is k/a Active defense injury
• If the person just tries to protect himself without grasping the weapon, the Medial/ Ulnar margin of the
forearm is MC affected. It is k/a Passive defense injury
• Though Presence of defense injury is diagnostic to homicide , it is not Mandatory to present in all
cases.
• Defense injuries are absent when pt. is Attacked from behind Unconscious Asleep
• Attacked from behind
• Unconscious
• Asleep
• Attacked from behind
• Unconscious
• Asleep
Incorrect Options:
Option B. Antemortem wound:
• Antemortem wounds happened before passing when the individual was still alive. The picture does
not give sufficient data to decide whether the wound was made before or after passing, so it cannot be
classified as an antemortem wound based on the picture alone.
Option C. Postmortem wound:
• Postmortem wounds are wounds that happen after death. The picture shows a wound that appears to
have been made before death.with signs of dying and tissue harm around the edges. Hence, it cannot
be classified as a postmortem wound.
Option D. Hesitation cuts:
• Hesitation cuts are shallow, provisional cuts made by an individual who is reluctant to self-harm or
commit suicide.
• Features: Multiple cuts, superficial, linear/parallel cuts. Seen in accessible parts of the
body(forearm,thigh and abdomen) Suggestive of suicidal tendency.
• Multiple cuts, superficial, linear/parallel cuts.
• Seen in accessible parts of the body(forearm,thigh and abdomen)
• Suggestive of suicidal tendency.
• The wound within the image is more profound and serious than a hesitation cut.
• Moreover, there's no sign that the wound was self-inflicted, so it cannot be classified as a hesitation
cut.
• Multiple cuts, superficial, linear/parallel cuts.
• Seen in accessible parts of the body(forearm,thigh and abdomen)
• Suggestive of suicidal tendency.

Page 14

252
Solution for Question 2:
• The wound described in the scenario is most likely a lacerated wound due to its ragged, irregular, and
uneven margins with tears diverging at an angle from the end of the main wound and typical
‘swallowtails’. Bleeding from this wound is not significant and shows crushed blood vessels and hair
bulbs. Lacerated wounds are typically caused by blunt force trauma and have an irregular shape with
jagged margins. They can also be caused by tearing, such as in cases of dog bites or machinery
accidents. Therefore, the wound should be documented as a lacerated wound.

Incorrect Option:
• Option a: An incised wound typically has clean and sharp margins and is caused by a sharp-edged
instrument like a knife. The edges of an incised wound are not ragged, irregular, or uneven, and there
are no tears diverging at an angle from the end of the main wound or "swallow tails."

Page 15

253
• Option b: A chopped wound is a type of wound caused by a heavy, sharp-edged object, such as an
axe or a machete. It typically has a clean, straight edge with minimal tissue damage.
• Option c: A stab wound typically has a narrow and deep wound track with straight or slightly curved
margins, and a clean and smooth wound surface. It is caused by a pointed object such as a knife,
dagger, or ice pick penetrating the skin and underlying tissues.

Solution for Question 3:


• Progression of the color of bruise over time aids in the differentiation of a true bruise from an artificial
bruise. A true bruise changes color over time, typically from red to purple to blue to green to yellow as
the body metabolizes the blood. In contrast, an artificial bruise may have a uniform color or may not
change color in the same way over time. Positive chemical tests, regular and well-defined margins, and
the presence of vesicles may indicate an artificial bruise, but the progression of color is the most
reliable feature for differentiating between true and artificial bruises.
Incorrect Option:
• Option a: Positive chemical tests can help in identifying the presence of blood, but they cannot
differentiate between true and artificial bruises.
• Option c: Regular and well-defined margins do not aid in the differentiation of a true bruise from an
artificial bruise. Both types of bruises can have irregular margins.
• Option d: The presence of vesicles is not a feature that aids in the differentiation of a true bruise from
an artificial bruise. Vesicles are small fluid-filled sacs that can appear on the skin in various medical
conditions, such as herpes simplex virus infections or allergic reactions, but they are not specific to
bruising.

Solution for Question 4:


Correct Option C - Ligature mark:
• The given image shows a ligature mark.
• A ligature mark is an injury caused by the application of a ligature or a tightly bound material around
the body, usually the neck.
• Ligature marks can be seen in cases of strangulation or hanging, where the ligature leaves a visible
pattern on the skin.
Incorrect Options:
Option A - Pressure abrasion: A pressure abrasion is an injury caused by continuous or prolonged pres
sure on the skin. It can occur when a body part is pressed against a hard or rough surface for an exten
ded period, resulting in skin damage. Pressure abrasions are commonly seen in cases of prolonged im
mobilization or compression.
Option B - Graze abrasion: A graze abrasion, also known as a superficial abrasion or a brush burn, is a
shallow wound caused by friction or scraping of the skin against a
rough surface. It typically appears as a linear or irregular-shaped scrape on the skin surface.
Option D - Imprint abrasion: An imprint abrasion occurs when an object or patterned surface leaves a d
istinct mark on the skin due to pressure or friction. The resulting abrasion reflects the shape, design, or

Page 16

254
texture of the object that caused it.

Solution for Question 5:


Correct Option A - Contusion:
• A contusion happens when trauma to the skin causes blood vessels to burst and leak blood into the
surrounding area. This results in the typical discoloration we associate with bruises which can range
from a bluish-purple to yellowish-green as it heals. While contusions are generally not serious and heal
on their own they can sometimes be a sign of more severe internal damage.
Incorrect Options:
Option B - Abrasion: The given image does not indicate an abrasion. An abrasion is a partial thickness
wound caused by damage to the skin and can be superficial involving only the epidermis to deep, invol
ving the deep dermis. Abrasions usually involve minimal bleeding. Mild abrasions, also known as graze
s or scrapes, do not scar or bleed because the dermis is left intact, but deep abrasions that disrupt the
normal dermal structures may lead to the formation of scar tissue. A
more traumatic abrasion that removes all layers of skin is called an avulsion.
Option C - Laceration: A laceration is a wound that results from soft bodily tissue being torn. This kind
of wound frequently has uneven and jagged edges. The image is not indicative of laceration.
Option D - Stab wound: A stab wound is a
specific form of penetrating trauma to the skin that results from a knife or a similar pointed object. Whil
e stab wounds are typically known to be caused by knives, they can also occur from a
variety of implements, including broken bottles and ice picks. The image is not indicative of laceration.

Solution for Question 6:


Correct Option: B
• Langer's lines, also known as tension lines or cleavage lines, refer to the natural orientation of
collagen fibers in the skin.
• Langer's lines correspond to the alignment of collagen fibers in the dermis, which is the second layer
of the skin.
• They indicate the direction of the least skin tension and the alignment of the skin's elastic fibers as
shown in the image in the question.
• Thus, making skin incisions parallel to the lines of Langer reduce the wound tension.
Incorrect Options:
Option A: Hinderer’s lines are 2 intersecting lines, one from the ala of the nose to the tragus and anoth
er from the lateral canthus of the eye. This is an incorrect option based on the image given in the quest
ion
Option C: Blaschko's lines, also called the lines of Blaschko, are lines of normal cell development in the
skin. These lines are only visible in those with a mosaic skin condition or in chimeras where different c
ell lines contain different genes. These lines may express different amounts of melanin, or become visi
ble due to a differing susceptibility to disease. In such individuals, they can become apparent as whorls
, patches, streaks or lines in a linear or segmental distribution over the skin.

Page 17

255
Option D: Dermatomes are commonly depicted in dermatome maps, which illustrate the specific region
s of the body associated with each spinal nerve's dermatome. These maps are useful in clinical practic
e for diagnosing and localizing nerve-related conditions.
These maps are useful in clinical practice for diagnosing and localizing nerve-related conditions.

Solution for Question 7:


Correct Option. D
• The most appropriate identification for the lesion shown in the image would be "Incised-looking
laceration." This suggests that the wound has the appearance of a laceration (with irregular edges) but
has been caused by a sharp object, resembling an incised wound.
Incorrect Options:
Option A. Laceration: A laceration is a type of wound that is typically irregular, with jagged or torn edge
s. It is caused by blunt force trauma or tearing of the skin and underlying tissues. In the given image, th
e wound does not exhibit the typical characteristics of a
laceration, such as irregular edges, which makes this option incorrect.
Option B. Incised wound: An incised wound is a type of wound that is caused by a sharp object or instr
ument, resulting in clean, straight edges. In the given image, the wound does not have the characteristi
c clean and straight edges of an incise wound, making this option incorrect.
Option C. Laceration-looking incised wound: This option suggests that the wound appears like a lacera
tion but actually has the characteristics of an incised wound. However, in the given image, the wound d
oes not exhibit the typical features of an incised wound, such as clean and straight edges, making this
option incorrect.

Solution for Question 8:


Correct Option:

Page 18

256
Option B:
Laceration involves the tearing or cutting of tissues, which can crush the underlying hair bulbs. It often r
esults from blunt force trauma, which can cause compression and crushing of the hair bulbs. Crushed
hair bulbs indicate a significant force applied to the area, consistent with a laceration.
Incorrect choices:
Option A: Abrasions typically involve superficial damage to the skin without significant underlying tissue
injury. They do not usually cause crushing of hair bulbs.
Option C: Stab injuries are caused by a pointed object piercing the skin. While they may cause damag
e to the underlying tissues, they do not typically result in crushed hair bulbs.
Option D: This option is incorrect because the description in the question indicates a
specific type of wound characterized by crushed hair bulbs corresponding to a laceration.

Solution for Question 9:


Correct Option: A
• When enough fluid builds up in the pericardial sac, squeezing the heart and causing a decrease in
cardiac output and shock, cardiac tamponade, a medical or traumatic emergency, occurs.
• In this condition, blood or fluid collects in the sac surrounding the heart. This prevents the heart
ventricles from expanding fully. The excess pressure from the fluid prevents the heart from working
properly. As a result, the body does not get enough blood.
Cardiac tamponade can occur due to:
• Dissecting aortic aneurysm (thoracic)
• End-stage lung cancer
• Heart attack (acute MI)
• Heart surgery
• Pericarditis caused by bacterial or viral infections
• Wounds to the heart
Incorrect Options:
Option B. Fat necrosis: Epipericardial fat necrosis (EFN) is an inflammatory condition that affects the fa
t surrounding the heart's mediastinum.
Option C. Hemothorax: A hemothorax is a buildup of blood in the pleural space, which is the region bet
ween the visceral and parietal pleura. Tachypnea and respiratory discomfort are among the clinical indi
cators in such patients.
Option D. Pulmonary embolism: When a blood clot becomes lodged in a
lung artery and prevents blood flow to a portion of the lung, it results in a pulmonary embolism. Most fr
equently, blood clots begin in the legs, move through the right side of the heart, and enter the lungs.

Solution for Question 10:

Page 19

257
Correct Option: C
• Epidural hematoma patients may experience a lucid interval of several hours prior to neurologic
impairment because extradural blood buildup is frequently sluggish.
Incorrect Options:
Option A. Subdural haemorrhage: This option is incorrect.
Option B. Parenchymal haemorrhage: Intraparenchymal haemorrhage is bleeding into the parenchyma
of the brain, which is subsequently followed by a period of lucidity and neurologic decline.
Option D. Subarachnoid haemorrhage: Bleeding due to trauma, or rupture of an aneurysm (such as a s
accular aneurysm ) or arteriovenous malformation. Rapid time course. Patients complain of “worst hea
dache of my life.” Bloody or yellow (xanthochromic) lumbar puncture.

Solution for Question 11:


Correct Choice: A
A. Axe:
• The correct match is option 4 - Chop wound.
• An axe is a cutting tool with a sharp blade on one side and a handle on the other. When an axe is
used forcefully to strike a person, it can cause a deep, wide, and often irregular wound known as a
chop wound. This type of wound is characterized by ragged edges and a combination of sharp and
blunt force injury.

B. RTA (Road Traffic Accident):


• The correct match is option 3 - Grazed abrasion.
• RTA refers to a Road Traffic Accident. In accidents involving vehicles, a person may slide or be
dragged along the road surface, leading to abrasions. Grazed abrasions are shallow injuries where the
outer layer of the skin is scraped off due to friction against the road surface. They usually appear as
superficial scrapes or grazes.
C. Blade:
• The correct match is option 1 - Incised wound.
• A blade is a sharp-edged instrument, typically used for cutting. When a blade cuts through the skin, it
creates an incised wound. Incised wounds have clean, smooth edges, and they are usually longer than
they are deep.
D. Lathi:
• The correct match is option 2 - Tram track bruise.
• A lathi is a long, heavy stick often used as a weapon or tool. When a lathi strikes a person, it can
cause a patterned bruise known as a tram track bruise. Tram track bruises are characterized by parallel
linear marks that resemble the tracks of a tram. They occur due to the impact of the cylindrical shape of
the lathi.

Page 20

258
Solution for Question 12:
1. Incomplete atrophy of ridges:
This characteristic is matched with dermatitis (option e).
Dermatitis refers to inflammation of the skin, which can cause changes in the fingerprint patterns. In ca
ses of dermatitis, there may be incomplete atrophy or flattening of the ridges, leading to altered fingerpr
int patterns.
2. Less pattern with ridge atrophy:
This characteristic is matched with celiac disease (option c).
Celiac disease is an autoimmune disorder that affects the small intestine when gluten is ingested. In in
dividuals with celiac disease, ridge atrophy can occur, leading to a
decrease in the pattern visibility or clarity of fingerprints.
3. Altered ridges:
This characteristic is matched with scleroderma (option b).
Scleroderma is a chronic autoimmune disease that affects the connective tissues, including the skin. In
cases of scleroderma, the skin can become thickened and hardened, leading to alterations in the ridge
patterns of fingerprints.
4. Permanent loss of fingerprints:
This characteristic is matched with radiation (option a).
Radiation therapy, particularly in high doses, can cause damage to the skin and underlying tissues. In
some cases, this can lead to permanent loss or disappearance of the fingerprint patterns.
5. Distance between ridges changed but pattern retained:
This characteristic is matched with acromegaly (option d).
Acromegaly is a hormonal disorder that occurs due to the excessive production of growth hormone in a
dulthood. In individuals with acromegaly, the bones and tissues can enlarge, including the hands and fi
ngers. This growth can lead to changes in the distance between ridges, but the overall pattern of the fin
gerprints remains intact.

Solution for Question 13:


Correct Choice: D
• The most likely cause of the described burn injury, characterized by soddened injury, erythema,
blisters, and a line of demarcation that extends down the limb, is "Moist heat."
• Moist heat is the correct answer because the presence of a soddened injury, along with erythema,
blisters, and a line of demarcation down the limb, suggests exposure to moist heat. Moist heat burns
typically occur when there is prolonged contact with hot liquids, steam, or moist objects. The moisture
can lead to deep tissue damage and delayed healing, which matches the description provided.
Incorrect Choices:
Option A. Electric burn: Electric burns are caused by contact with an electrical source, such as expose
d wires or lightning. These burns often have characteristic entry and exit wounds and may not necessa
rily show soddened injury or a line of demarcation. While electric burns can cause erythema and blister

Page 21

259
s, they are not the most likely cause based on the information given.
Option B. Chemical burn: Chemical burns occur when the skin comes into contact with corrosive subst
ances, such as acids or alkalis. They typically present with localized tissue damage and may show sign
s of erythema, blisters, and a line of demarcation. However, the description of a
soddened injury is not consistent with a
chemical burn, making it less likely to be the cause in this scenario.
Option C. Dry heat: Dry heat burns result from direct contact with hot objects, flames, or hot surfaces.
While they can cause erythema, blisters, and a line of demarcation, they do not typically result in a
soddened injury. Given the specific description of a
soddened injury in the question, dry heat is not the most likely cause.

Solution for Question 14:


Correct Option A :
• Flying missiles refer to objects propelled by the force of an explosion or blast. In the context of the
question, the presence of multiple small puncture wounds on the right side of the chest and the pattern
of injuries suggests that the man was likely struck by fragments or shrapnel propelled by the blast.
These fragments can include metal shards, glass, or other debris that become airborne due to the force
of the explosion. Therefore, considering the described injuries, flying missiles are the most likely
mechanism responsible for these injuries.
Incorrect Choices:
Option B: Blast of air: A
blast of air refers to the rapid displacement and movement of air caused by an explosion. While a blast
of air can cause injuries, such as damage to the lungs, eardrums, or other internal organs, it typically d
oes not cause the specific injuries described in the question, such as puncture wounds or abrasions. T
herefore, this option is less likely to be the correct answer in this case.
Option C: Explosion burns: Explosion burns are injuries caused by exposure to the intense heat genera
ted by an explosion. While burns can certainly occur during an explosion, the question does not mentio
n any specific burn injuries. Instead, it describes puncture wounds, abrasions, and bruising. Therefore,
this option is not the most likely mechanism responsible for the injuries described.
Option D: Building collapse: Building collapse refers to the structural failure of a building due to an expl
osion or other catastrophic events. While building collapse can certainly cause various types of injuries,
including crush injuries, fractures, and internal injuries, it is not consistent with the specific pattern of in
juries mentioned in the question (i.e., puncture wounds, abrasions, and bruising). Therefore, this option
is unlikely to be the correct answer.

Solution for Question 15:


Correct Option: B
Based on the description provided, the type of wound is likely a "Chop wound."
Explanation of the options:
Option A: Incised wound: An incised wound is characterized by clean-cut edges, similar to a
surgical or razor blade cut. It does not involve crushing or disruption of deeper tissues or bones.

Page 22

260
Option B: Chop wound: A chop wound occurs when a heavy and sharp-edged object, such as an axe o
r machete, is used with significant force. It typically results in a combination of a
deep incised wound with associated crushing or fracturing of underlying structures, such as bones.
Option C: Lacerated wound: Lacerated wounds are characterized by irregular, torn, or jagged edges. T
hey are caused by blunt force trauma, often resulting in the tearing and stretching of tissues.
Option D: Avulsion: Avulsion refers to the tearing away of tissue or a body part from its normal position.
It typically involves the separation of tissue or structure due to extreme force or trauma.
In this case, the presence of clean-cut edges, crushed tissues, and disruption of vertebral bones sugge
sts a chop wound, indicating that a
heavy and sharp-edged object was used with significant force to cause the injury.

Solution for Question 16:


Correct Option: A
Correct answer is bagging.
Option A: Bagging: Bagging refers to a method of inhaling substances by placing them inside a bag an
d then inhaling or breathing in the vapors. The bag is typically held over the nose and mouth, allowing t
he person to inhale the substance contained within the bag. This method is often associated with the a
buse of volatile substances or inhalants, such as solvents, aerosols, or gases.
Other options
Option B: Snorting: Snorting involves the inhalation of powdered substances, typically drugs, through t
he nose. The substance is crushed into a fine powder and then snorted through the nostrils using a str
aw, rolled-up paper, or other devices. This method allows the drug to be absorbed through the nasal m
ucosa and quickly enter the bloodstream.
Option C: Sniffing: Sniffing refers to the act of inhaling or smelling substances, often volatile or aromati
c substances, to experience their scent or aroma. It is commonly associated with the evaluation of odor
s, such as in perfumery or wine tasting. In a forensic context, sniffing may be relevant in detecting or id
entifying certain substances or odors related to a crime scene.
Option A: Huffing: Huffing involves the deliberate inhalation of volatile substances or inhalants directly f
rom their container or source. The person typically places their mouth or nose directly on the container
and inhales deeply, allowing the vapors or fumes to enter their respiratory system. This method is asso
ciated with the misuse of chemicals, solvents, or other volatile substances for recreational purposes.

Solution for Question 17:


Correct Choice: D
Explanation:

Page 23

261
• When a bullet just grazes/ glances/ taps/ slaps through the skull (there is no head-on collision), a part
of the outer table is chipped out with an intact inner table, leaving behind a gutter-like groove. Hence,
called the ‘gutter fracture’.
• Gutter fracture is a long and narrow fracture. It is seen with a bullet glance/ graze/ slap of the skull. A
part of the outer table will be chipped off/ grooved – Ist degree Gutter fracture.
• IInd degree Gutter fracture – Fracture of the inner table ( called the Depressed fracture).

Incorrect Choices:
Option A. A sharp object – May lead to compound depressed fracture such as stiletto-heel fracture due
to use of stiletto heel shoes (rare type).
Option B. A blunt object – A heavy blunt blow leads to comminuted fracture. Multiple fragments are for
med. There are intersecting lines and webs, known as spider web fracture. Fragments are displaced.
Option C. Fall from height – A person falling from a
great height and landing on his feet results in ring fracture or foramen fracture. It is a

Page 24

262
fracture surrounding foramen magnum. Seen if someone falls from a
height or falling on feet or falling on buttocks. Can be due to a blow on the chin or a blow on the vertex.

Solution for Question 18:


Correct Option A: Brain
• When it comes to decomposition or putrefaction of the body after death, the brain is generally the first
organ to undergo significant decomposition.
Incorrect Options:
Option B - Heart: Heart may undergo changes after death, such as loss of contractility and blood circul
ation, it does not typically decompose as rapidly as the brain.
Option C - Kidney: Kidneys can be affected by postmortem changes, they generally do not decompose
as quickly as the brain.
Option D - Prostate: Prostate may undergo changes after death, but it does not decompose as rapidly
as the brain.

Solution for Question 19:


Correct Option A:
Option A. Axe:
The correct match is option 4 - Chop wound.
An axe is a cutting tool with a sharp blade on one side and a
handle on the other. When an axe is used forcefully to strike a person, it can cause a
deep, wide, and often irregular wound known as a
chop wound. This type of wound is characterized by ragged edges and a
combination of sharp and blunt force injury.
Option B. RTA (Road Traffic Accident):
The correct match is option 3 - Grazed abrasion.
RTA refers to a Road Traffic Accident. In accidents involving vehicles, a person may slide or be dragge
d along the road surface, leading to abrasions. Grazed abrasions are shallow injuries where the outer l
ayer of the skin is scraped off due to friction against the road surface. They usually appear as superfici
al scrapes or grazes.
Option C. Blade:
The correct match is option 1 - Incised wound.
A blade is a sharp-edged instrument, typically used for cutting. When a blade cuts through the skin, it c
reates an incised wound. Incised wounds have clean, smooth edges, and they are usually longer than t
hey are deep.
Option D. Lathi:
The correct match is option 2 - Tram track bruise.

Page 25

263
A lathi is a long, heavy stick often used as a weapon or tool. When a lathi strikes a
person, it can cause a patterned bruise known as a tram track bruise. Tram track bruises are characteri
zed by parallel linear marks that resemble the tracks of a
tram. They occur due to the impact of the cylindrical shape of the lathi.

Solution for Question 20:


Correct Option : B
• In the given scenario where the image shows the base of the skull divided in half, the specific type of
skull fracture is known as a "hinge fracture." A hinge fracture is a specific subtype of basilar skull
fracture, which involves a separation or splitting of the base of the skull along a hinge-like mechanism.
Incorrect options:
Option A. A depressed fracture is a type of skull fracture where the bone fragments are depressed or p
ushed inward. It does not correspond to the given image.
Option C. Pond fracture is not a recognized term for a specific type of skull fracture.
Option D. Comminuted fracture refers to a fracture in which the bone is fragmented into multiple pieces
. It does not correspond to the given image.

Solution for Question 21:


Correct Option : A
• Based on the image,it is showing an abrasion on the palm. An abrasion is a superficial injury that
occurs when the skin is scraped or rubbed against a rough surface, resulting in the removal of the
superficial layers of skin. It typically appears as a shallow, irregular-shaped wound with a reddish or
pinkish color. Abrasions are commonly caused by falls, contact with rough objects, or friction between
the skin and a surface.
Incorrect options:
Option B. Laceration: A laceration is a type of injury characterized by a deep cut or tear in the skin and
underlying tissues. It typically has clean or irregular edges and may require suturing or medical interve
ntionTherefore, imprint is not the correct identification for the image

Page 26

264
Option C. Bruise: A bruise, also known as a contusion, is a discoloration of the skin caused by the rupt
ure of blood vessels beneath the skin surface. It appears as a
reddish, purple, or blue mark and is often the result of blunt force trauma. A bruise is different from an
abrasion, as it does not involve the removal of skin layers.Therefore, imprint is not the correct identifica
tion for the image.

Option D. Imprint: An imprint refers to a pattern or mark left on the skin as a result of contact with an ob
ject or surface. While an abrasion can leave an imprint of the object that caused the injury, the imprint it
self is not the injury. Therefore, imprint is not the correct identification for the image.

Solution for Question 22:


Correct Option: C

Page 27

265
• Paradoxical undressing is a phenomenon commonly observed in cases of severe hypothermia.
Hypothermia is a condition characterized by a dangerously low body temperature, usually below 35
degrees Celsius (95 degrees Fahrenheit). It can occur when the body loses heat faster than it can
generate heat, leading to a drop in core body temperature.
• During severe hypothermia, a paradoxical response may occur where the affected individual, despite
being in a state of extreme cold, exhibits behavior contrary to what would be expected. Paradoxical
undressing refers to the behavior of hypothermic individuals removing their clothing, even though doing
so would exacerbate heat loss and further endanger their lives. This behavior is thought to result from
the disorientation and confusion caused by hypothermia, as well as the dilation of blood vessels near
the skin's surface, which can create a false sensation of warmth.
Incorrect Option:
Option A. Heart stress: Heart stress, or cardiac stress, refers to the strain placed on the heart during ph
ysical exertion or emotional stress. It is not associated with paradoxical undressing.
Option B. Voyeurism: Voyeurism is a paraphilic disorder characterized by an individual's sexual interest
and pleasure derived from observing others without their consent. It is unrelated to paradoxical undres
sing.
Option D. LSD overdose: LSD (lysergic acid diethylamide) is a
hallucinogenic drug. Overdosing on LSD can cause a
variety of physical and psychological effects, but paradoxical undressing is not a
known symptom or consequence of LSD overdose.

Solution for Question 23:


Correct Option: C There are fewer chances of bacterial infection
• There are fewer chances of bacterial infection this statement is false as there are high chances of
bacterial infection. Human bites carry a high risk of polymicrobial infection by organisms such
as Staphylococcus spp., Streptococcus spp., Corynebacterium spp., Fusobacterium spp., and
Eikenella corrodens. Hence antibiotics are recommended.
Incorrect Options:
Option A. Swabs should be taken immediately after moistening with sterile water: This statement is tru
e. When collecting evidence from a bite mark, it is recommended to moisten swabs with sterile water b
efore sampling. Moistening the swabs helps in better collection of potential biological material, such as
saliva or cells, that may be present in the bite mark.
Option B. It is most commonly caused by incisors and canines: This statement is true. Human bite mar
ks are often caused by incisors and canines, which are the front and sharp teeth located in the upper a
nd lower jaws. These teeth are more likely to leave identifiable patterns and impressions in a bite mark.
Option D. Tetanus vaccination should be given :
This statement is true. Tetanus vaccination should be given after human bite.

Solution for Question 24:


Correct Option B:

Page 28

266
• The given image caused by an axe is likely to be a chop wound. Chop wounds are so-called because
of the action in which they are produced. Depending on the type of instrument, they may
resemble incised or lacerated wounds. The shape of the wound may give clues to the type of weapon.
In the given image, the wound resembles an injury produced by a sharp axe. Such wounds may be
inflicted with homicidal intent on the skull/scalp. , Chop wounds are usually deep gaping wounds
caused by a blow with a sharp cutting edge of a fairly heavy weapon like an axe, sword, hatchet, etc.
The given image caused by an axe is likely to be a chop wound. Chop wounds are so-called because o
f the action in which they are produced. Depending on the type of instrument, they may resemble incis
ed or lacerated wounds. The shape of the wound may give clues to the type of weapon. In the given im
age, the wound resembles an injury produced by a
sharp axe. Such wounds may be inflicted with homicidal intent on the skull/scalp. ,
Chop wounds are usually deep gaping wounds caused by a blow with a sharp cutting edge of a
fairly heavy weapon like an axe, sword, hatchet, etc.
Incorrect Options:
Options A. Incised wound: An incised wound is a clean, smooth, and regular wound caused by a sharp
-edged object. It typically has well-defined, straight edges and minimal tissue damage. Unlike the imag
e of a chop wound, an incised wound would not have the characteristic irregular and ragged edges cau
sed by a blunt object like an axe.
Options C. Lacerated wound: A lacerated wound is a jagged, irregular wound caused by tearing or crus
hing of the skin and underlying tissues. It often has irregular edges and significant tissue damage. How
ever, the image provided specifically resembles a chop wound with its distinct features, rather than a
lacerated wound.
Options D. Abrasion: An abrasion is a superficial wound caused by friction, resulting in the removal of t
he superficial layers of the skin. It typically appears as a
scrape or graze without deep penetration. The image described as a
chop wound caused by an axe does not match the characteristics of an abrasion.

Solution for Question 25:


Correct Option C:
A laceration is a wound characterized by irregular, jagged edges caused by blunt force trauma or tearin
g of the skin and underlying tissues. In this case, the injury appears to have resulted from a
forceful impact or tearing action, leading to a laceration.
Incorrect Option:
Option A. Incised wound: An incised wound is a clean, smooth-edged wound typically caused by a
sharp object. It does not match the appearance of the injury in the image.
Option B. Abrasion: An abrasion is a superficial wound caused by friction, often resulting in the removal
of the superficial layers of the skin. It does not involve a
deep tear or irregular edges like the injury shown.
Option D. Bruise: A bruise, also known as a contusion, is caused by blunt force trauma that damages b
lood vessels underneath the skin, leading to discoloration. It does not involve a
tear or break in the skin, unlike the injury depicted.

Page 29

267
Solution for Question 26:
Correct Option A:
• The image shown depicts a typical pattern of injury caused by a shotgun. The appearance of multiple
pellet or shot holes, irregular wound edges, and tissue destruction is characteristic of a shotgun wound.
Incorrect Option:
Option B. Tattooing: Tattooing refers to the deposition of tattoo ink or pigment into the skin. It is unrelat
ed to the image shown, which depicts a penetrating injury.
Option C. Arsenic poisoning: Arsenic poisoning is a systemic condition caused by the ingestion or inhal
ation of arsenic. It does not cause the specific wound pattern seen in the image.
Option D. Graze wound: A graze wound is a superficial wound caused by a glancing impact, resulting i
n abrasion or scraping of the skin. It does not exhibit the characteristics of tissue destruction and multip
le penetrating holes seen in the image.

Solution for Question 27:


Correct Option A.
• Strangulation by ligature is correct because there is evidence of a ligature mark on the neck.
• The presence of a ligature mark makes strangulation by ligature or hanging as the likely cause.
Incorrect Option:

Option B. Hanging is incorrect because there is no indication of suspension or hanging-related findings


.
Option C:The absence of carotid intimal tear and muscle hematoma do not indicate forceful compressi
on or pressure on the neck, which is consistent with throttling.
Option D. Drowning is incorrect as there are no signs or information suggesting drowning in this case.

Solution for Question 28:


Correct Option C
• Huffing refers to the act of inhaling volatile substances or drugs by saturating a cloth or other material
and then inhaling the fumes.
• The cloth is soaked or impregnated with the substance, and the individual places it over their mouth or
nose to breathe in the vapors.
• Huffing is a form of inhalant abuse, and it can have harmful effects on the body, including intoxication
and potential damage to the respiratory system.
Incorrect options:
Option A. Bagging: Bagging is a term used to describe a
specific method of administering anesthesia during medical procedures. It involves using a

Page 30

268
bag-valve-mask device to deliver a
controlled amount of anesthetic gas to the patient. It is not related to inhaling cloth soaked in drugs.
Option B. Spraying: Spraying typically involves the dispersal of liquid substances in a
fine mist or spray form. It does not specifically refer to inhaling cloth soaked in drugs.
Option D. Sniffing: Sniffing refers to the act of inhaling substances directly through the nose. While it ca
n involve inhalation of drugs or volatile substances, it does not specifically involve the use of a
cloth soaked in drugs

Solution for Question 29:


Correct option C
• When attempting to identify a burnt body, different tissues can be examined to aid in identification.
However, the best tissue for identification purposes is the teeth (Option C). Teeth are highly durable
and resistant to heat, making them more likely to survive the burning process compared to other
tissues. Dental records, including dental charts, X-rays, and dental impressions, can be used to
compare with the teeth recovered from the burnt body to establish a positive identification.
Incorrect options:
Option A: Hair: While hair can provide some information for identification, it is more susceptible to dam
age by fire and may not be as reliable as teeth.
Option B: Bone: Bone can also provide information for identification, but it may be severely damaged o
r fragmented in cases of extensive burning.
Option D: Blood: Blood is not a suitable tissue for identification purposes in a
burnt body, as it would likely be destroyed during the burning process.

Solution for Question 30:


Correct Option C:
The Fissured Fracture of skull image is given.
• Thin, Linear fracture line is seen on the skull vault.
• M/C type of skull fracture
• As it is very thin, 10-15% of these skull fractures are missed even on CT scan
• These are obvious during autopsy only

Page 31

269
Incorrect Options:
Option A: Ring fracture is associated with the fall from height and landing on the foot. Fracture line is s
een around the Foramen magnum like a ring.
Option B: Depressed fracture is caused by heawith a small striking surface. Due to the impact of the w
eapon, bone gets fractured and the fractured segment is depressed inside.

Option D. Comminuted fracture is caused due to repeated blows with heavy force. Presents with multip
le Fracture lines intersecting each other.

Page 32

270
Solution for Question 31:
Correct Option C
• When analyzing a gunshot wound, variables such as the size and shape of the wound, the depth of
penetration, and the nearness of gunpowder buildup can give clues about the sort of gun utilized and
the extent of the shot.
• In this case, the wound was caused by a gun, as shown by its size and shape, which are reliable with
a small-caliber handgun.
• Furthermore, the wound was caused by a near shot, as shown by the presence of powder burns and
the nonappearance of soot or stippling.
• Powder burns happen when gunpowder particles from the muzzle blast are stored on the skin,
whereas sediment and stippling are caused by burning gunpowder and unburned powder particles,
separately, which can, as it were, travel short distances.
• Hence, the need for soot or stippling proposes that the weapon was terminated at a near extent,
inside a number of inches of the victim.
Incorrect Options:
Option A. Shotgun, intermediate range:
• A shotgun wound regularly causes a large and irregularly molded wound, with a more extensive
spread of pellets or shot.
• Besides, at middle ranges, the pattern of the shot can be observed on the skin, and there may be
evidence of both powder burns and stippling.
Option B. Shotgun, close range:
• At close range, a shotgun wound can cause extreme tissue harm and a widespread shot, which would
be obvious on the skin as a design of different holes or craters.

Page 33

271
• Besides, there would likely be evidence of powder burns, soot, and stippling.
Option D. Pistol, close shot:
• At a near range, a gun wound can cause a little entry wound and a larger exit wound, with proof of
powder burns, soot, and stippling.
• In any case, in this case, there was no proof of sediment or stippling, showing that the weapon was
fired from a marginally more distant distance.
• Gun fired inside the range of flame.
• Circular puncture wound is seen.

Solution for Question 32:


Correct Option A: Close shot entry wound.When a gunshot injury occurs at close range, the muzzle of t
he firearm is in close proximity to the body. In such cases, the burning, blackening, and tattooing of the
skin around the wound are commonly observed. These characteristics are due to the presence of unb
urned gunpowder particles and hot gasses discharged from the firearm.
Incorrect Options
Option B: Close shot exit wound: In close shot exit wounds, the burning, blackening, and tattooing effe
cts are usually not present. Close shot exit wounds are characterized by an irregular or stellate appear
ance, as the bullet exits the body and causes tissue disruption.
Option C: Distant shot entry wound: In distant shot entry wounds, there is a significant distance betwee
n the muzzle of the firearm and the body. The effects of burning, blackening, and tattooing are not typic
ally observed in such cases. Distant shot entry wounds may appear as small, circular or oval-shaped h
oles without the characteristics associated with close-range shots.
Option D: Distant shot exit wound: Similar to distant shot entry wounds, distant shot exit wounds do not
exhibit the effects of burning, blackening, and tattooing. These exit wounds are often characterized by
a smaller size compared to close shot exit wounds.

Solution for Question 33:


Correct Option: C
• Bullet fingerprinting is a technique used in forensic investigations to analyze the unique markings left
on bullets or cartridge cases by the firearm used to discharge them. These markings, known as firearm
toolmarks, can provide valuable evidence in linking a bullet or cartridge case to a specific firearm.
• Primary markings are the unique characteristics imprinted on a bullet or cartridge case by the barrel of
the firearm. These include the lands and grooves on the inside of the barrel, which leave distinct
striations on the bullet or cartridge case as it passes through the barrel. These primary markings are
the key elements that make each firearm's toolmarks unique.
• Secondary markings, on the other hand, are additional marks or features that may be present on a
bullet or cartridge case due to factors such as the firing mechanism, extractor, or ejector of the firearm.
These markings can include extractor marks, ejector marks, breech face impressions, and firing pin

Page 34

272
impressions.
• When analyzing a bullet or cartridge case, forensic experts examine both the primary and secondary
markings to compare them with test-fired bullets or cartridge cases from suspect firearms. The goal is
to identify matching or consistent characteristics that indicate a potential match between the evidence
and a specific firearm.
Incorrect Options:
Option A: Primary markings alone do not encompass the complete analysis involved in bullet fingerprin
ting.
Option B: Secondary markings alone are also not sufficient for bullet fingerprinting analysis.
Option D: None is incorrect because bullet fingerprinting does involve the examination of both primary
and secondary markings to establish connections between bullets or cartridge cases and firearms.

Solution for Question 34:


Correct Option B: Near shot
• Although an intermediate distance wound leaves blackening and tattooing but does not leave
burging/singeing marks on the gunshot wound, it is not given in the option.
• The next best answer would be a near gunshot wound.
• This is because near gunshot wounds and intermediate gunshot wounds may or may not have
blackening but definitely have tattooing present.
Incorrect Options:
Option A: Contact shot -
Burning, blackening and tattooing are not found on the surface, but may be present within the wound.
Option C: Close Shot- Burning/ singeing, blackening and tattooing- all 3 are present.
Option D: Distant Shot- Burning/ singeing, blackening and tattooing- all 3 are absent.

Solution for Question 35:


Correct Ans: A
• Bullet wipe refers to the residue or material that is left behind on a target when a bullet passes through
it. This residue is primarily composed of substances such as soot, lubricants, and metal particles that
are present in the barrel of the gun. When the bullet contacts the target, some of these materials can be
transferred onto the surface, leaving a distinct pattern or mark.
Option B. Blackening: Blackening typically refers to the darkening or discoloration of a surface due to v
arious factors such as oxidation, charring, or exposure to heat. While blackening may be observed in c
ertain gunshot-related incidents, it does not specifically refer to bullet wipe.
Option C. Gutter fracture of the skull: Gutter fracture is a term used to describe a
specific type of fracture in the skull, characterized by a
groove or channel-like indentation. It is not directly related to bullet wipe.

Page 35

273
Option D. Tattooing: Tattooing, in the context of firearms, refers to the deposition of gunpowder particle
s or other debris onto the skin when a firearm is discharged at close range. This can result in small, dar
k spots or tattoo-like markings on the skin. While tattooing can be associated with gunshot wounds, it is
not synonymous with bullet wipe.

Solution for Question 36:


Correct Answer: A - The executive magistrate can order for exhumation
• Exhumation - It is digging out of an already buried body legally from the grave
• exhumation is done only when there is written order from executive magistrate.
• The body can be exhumated by any government doctor.
Incorrect Option:
Option B Police can order for exhumation : This statement is incorrect
Option C Postmortem can be done on exhumated body :
Autopsies can be performed on exhumated bodies
Option D CrPC 174 is related to exhumation: CrPC 176 is related to exhumation

Page 36

274
Introduction to Toxicology & Diagnosis of Poisoning
1. Specimens for toxicological (Chemical) analysis are commonly preserved using a solution, which is
easily available and cheap. It can be used for all poisons except corrosive acids. Which among the
following is being described here?
(or)
What common solution is used for preserving toxicological specimens, excluding corrosive acids, for
chemical analysis?
A. Rectified spirit
B. Alcohol
C. 10% Formaldehyde
D. Saturated solution of common salt
----------------------------------------
2. The universal antidote is an outdated concept, where the combination of 3 physical and chemical
antidotes was used in those cases where the nature of ingested poisons was unknown or when it was
suspected that two or more poisons were taken. 3 of the following were added in the ratio of 2:1:1,
except?
(or)
Which of the following was not part of the universal antidote mixture, where three physical and chemical
antidotes were combined in a ratio of 2:1:1 for suspected poisonings?
A. Activated charcoal
B. Magnesium oxide
C. Sodium sulphate
D. Tannic acid
----------------------------------------
3. A 17-year-old girl has been admitted in a semiconscious state. She was found in that state in her
bedroom with a bottle of unprescribed drugs near her bed. On regaining consciousness, she revealed
that she took sleeping pills to end her life.Which statement among the following, is not true about this
scenario?
(or)
Which statement among the following, is not true about Parasuicide?
A. Also known as attempted suicide
B. Mostly seen in psychological disturbances
C. Conscious, impulse, manipulative act to get rid of an intolerable situation
D. Hanging is the most common form
----------------------------------------
4. Iran has returned the body of the 35-year-old man who was executed on charges of drug trafficking.
The body has been embalmed prior to repatriation. Although it is usually still possible to identify the
presence of drugs, the constituent chemicals in embalming fluids affect not only the detection but also
the rate of decay of the majority of both pharmaceutical and illicit drugs in blood and many other bodily

275
tissues, making forensic evidence unsafe and, therefore, usually inadmissible in courts of law. Which
tissue sample will the forensic team prefer in this case for toxicological analysis?
(or)
In the case of an embalmed body, which tissue sample is preferred for toxicological analysis by the
forensic team?
A. Bile
B. Vitreous
C. Spinal cord
D. Skeletal muscle
----------------------------------------
5. Role of sodium fluoride as a preservative for blood in viscera packing are all except?
(or)
The role of sodium fluoride as a preservative for blood is all except
A. It prevents glycolysis
B. Acts as anti-coagulant
C. Inhibits bacterial growth
D. Inhibits enolase enzyme
----------------------------------------
6. Urine is obtained by a catheter via a suprapubic puncture before autopsy or by puncture of the
bladder after evisceration. Around 100-200ml is collected.If less, then the whole quantity will be
collected. Which one among the following is a better preservative of urine in visceral packing?
(or)
Which one among the following is a better preservative of urine in visceral packing?
A. Concentrated Hydrochloric acid
B. Thymol
C. Normal saline
D. Sodium fluoride
----------------------------------------
7. During an autopsy, the forensic pathologist removed the stomach after performing a double ligature.
On opening the stomach, he found a bluish discoloration of the stomach walls. He immediately knew
that it was poisoning by a particular compound that is a nonselective CNS depressant primarily used as
sedative-hypnotics. In sub hypnotic doses, it is also used as an anticonvulsant. Which one among the
following could it be?
(or)
Which poison causes bluish discoloration of stomach walls on autopsy.
A. Sodium amytal
B. Soneryl
C. Oxalic acid

Page 2

276
D. Arsenic
----------------------------------------
8. A 35-year-old woman who was rescued from a fire that had allegedly been started by a short circuit,
showed a decreased level of consciousness, low blood pressure, and high lactic acid. The doctor
considers the possibility of poisoning with one of the following which would have a "bitter almond" odor.
Which one is the doctor suspecting it to be?
(or)
In which poisoning Bitter almond odor is present ?
A. Phenol
B. Datura
C. Cyanide
D. Chloral hydrate
----------------------------------------
9. After taking a detailed history and a thorough examination, the doctor was able to confirm the
diagnosis of Lead poisoning. He decides to treat the same with 10% solution in oil of BAL (British
anti-lewisite, dimercaprol), 3–5 mg/kg IM 4 hourly for 2 days, 6 hourly on 3rd day, and then 12 hourly for
the next 10 days. BAL is contraindicated in all of the following, except?
(or)
BAL is contraindicated in all of the following, except?
A. G6PD deficiency
B. Iron poisoning
C. Cadmium poisoning
D. Gold poisoning
----------------------------------------
10. This " knockdown gas", responsible for numerous cases of toxic exposure in the petroleum industry
is fatal in concentrations over 500-1000 ppm, revealing a bluish-green stain on post-mortem
examination. Prolonged exposure in lower concentrations causes rhinitis to acute respiratory failure.
Which among the following is being described?
(or)
Which substance referred as "knockdown gas" and on post-mortem staining it reveals a bluish-green
discoloration?
A. Hydrogen sulphide
B. Carbon monoxide
C. Cyanide
D. Phosphorus
----------------------------------------
11. A 43-year-old male attempted suicide by drinking 54 g of a solution. He is a chemist by profession
and prepared this amount in his laboratory using a commercial solution of its chloride form. Although
the patient arrived at the hospital 5 hours later with stable vitals, but was pale and diaphoretic. His
gastric aspirate had a garlicky odor. This is characteristic of which of the following?

Page 3

277
(or)
On gastric aspirate, garlic smell is characteristic of which poisoning?
A. Arsenic
B. Cannabis
C. Alcohol
D. Hemlock
----------------------------------------
12. A 30-year-old woman was one among the 5 people who had died after being exposed to some
poisonous fumes at the factory she was working in. By the time the body was brought for an autopsy,
the doctor noticed that there was Chocolate brown hypostasis in the back, over the buttocks, thighs,
and posterior side of the arm. This can be secondary to exposure to any one of the following, except?
(or)
On autopsy ,chocolate brown hypostasis in the back, over the buttocks, thighs, and posterior side of the
arm. This can be secondary to exposure to any one of the following, except?
A. Hydrogen sulphide
B. Aniline
C. Nitrites
D. Potassium chlorate
----------------------------------------
13. What is the preservative of choice in phenol poisoning?
(or)
What is the preservative of choice in phenol poisoning?
A. Rectified spirit
B. Saturated solution of sodium chloride
C. Formalin
D. Sodium flouride
----------------------------------------

Correct Answers
Question Correct Answer

Question 1 4
Question 2 3
Question 3 4
Question 4 4
Question 5 2
Question 6 2

Page 4

278
Question 7 1
Question 8 3
Question 9 4
Question 10 1
Question 11 1
Question 12 1
Question 13 2

Solution for Question 1:


Correct Option D - Saturated solution of common salt:
• Saturated solution of common salt is easily available and cheap and it can be used for all poisons
except corrosive acids.

Incorrect Options:
Option A - Rectified spirit:
• 95% pure ethanol is referred to as the rectified spirit, with 5% water.
• Rectified spirit is the best preservative for chemical analysis.
Option B - Alcohol:
• Rectified spirit contraindicated in: Alcohol, Formaldehyde & Formic acid.
Option C - 10% Formaldehyde:
• 10% Formaldehyde is used as a preservative for chemical analysis

Solution for Question 2:


Correct Option C - Sodium sulphate:
• Sodium sulphate is not added in the universal combination antidote.

Incorrect Options:
Option A - Activated charcoal: Activated charcoal, tannic acid &
magnesium oxide are added in the ratio of 2:1:1.
Option B - Magnesium oxide: Activated charcoal, tannic acid &
magnesium oxide are added in the ratio of 2:1:1.
Option D - Tannic acid: Activated charcoal, tannic acid &
magnesium oxide are added in the ratio of 2:1:1.

Page 5

279
Solution for Question 3:
Correct Option D - Hanging is the most common form:
• Poisoning (drug ingestion) is the most common form of Parasuicide.
• The most common method is taking an overdose of drugs.
Incorrect Options:
Option A - Also known as attempted suicide: Parasuicide also known as attempted suicide.
Option B - Mostly seen in psychological disturbances In Parasuicide, most persons are psychologically
disturbed.
Option C - Conscious, impulse, manipulative act to get rid of an intolerable situation: Parasuicide (atte
mpted suicide, or pseudocide) is a
conscious, often impulsive, manipulative act, undertaken to get rid of an intolerable situation.

Solution for Question 4:


Correct Option D - Skeletal muscle:
• In the embalmed body the best specimen may be skeletal muscle from the buttock.

Incorrect Options:
Option A - Bile: It is best removed by puncturing the gall bladder in situ. Narcotic drugs, cocaine, metha
done, glutathione, barbiturates and some tranquilisers.
Option B - Vitreous: Alcohol, chloroform, cocaine, morphine, tricyclic antidepressants.
Option C - Spinal cord: Spinal cord entire length: strychnine and gelsemium.

Solution for Question 5:


Correct Option B - Acts as anti-coagulant:
• Sodium floride is used as a preservative for blood
• Function: Inhibits glycolysis by targeting the enolase enzyme.
• Effect: This inhibition helps preserve the glucose levels in the blood sample and prevents its utilization
by cells. Additionally, it contributes to inhibiting the growth of bacteria in the sample.
• It does not act as an anticoagulant

Incorrect Options:
Option A/C/D: It prevents glycolysis/ Inhibits bacterial growth/ Inhibits enolase enzyme
• 10 mg/ml of sodium or potassium fluoride prevents glycolysis, inhibits enzyme enolase, and inhibits
bacterial growth.

Page 6

280
Solution for Question 6:
Correct Option B - Toluene:
• Thymol is a better preservative of urine in viscera packing.
• Although, to preserve a urine sample both thymol and NaF can be used
Incorrect Options:
Option A/C/D: Concentrated Hydrochloric acid/ Normal saline/ Sodium fluoride
• 1 ml of concentrated hydrochloric acid or 100 mg of thymol or 100 mg of sodium fluoride can be used
for 10 ml of urine as a preservative.
• Thymol is a better preservative compared to the above preservatives.

Solution for Question 7:


Correct Option A - Sodium amytal:
• Sodium amytal capsules stain the stomach and stomach contents turquoise blue.
Incorrect Options:
Option B - Soneryl:
• Pink discoloration can be seen.
Option C - Oxalic acid:
• Oxalic acid: Gray, blackened by blood
• Mucosa is reddened and punctate due to erosions, giving a velvety red or blackish appearance.
Although the stomach wall is thinned out it is not perforated. The stomach usually contains gelatinous
brown material due to acid hematin formation.
Option D - Arsenic:
• Mucosa is swollen, edematous, desquamated, and red, either generally or in patches, especially in
the pyloric region. Usually, groups of petechiae are seen scattered over the mucosa, but sometimes
large submucosal and subperitoneal hemorrhages may be seen-red velvety appearance.

Solution for Question 8:


Correct Option C - Cyanide:
• Cyanide: Bitter almond odor
• Clinical features:- Bitter acid burning taste, constriction or numbness of throat, clenched jaw,
salivation, froth, nausea, confusion, drowsiness, prostration, opisthotonus, lockjaw, hyperthermia,
epileptiform or tonic convulsions, paralysis, stupor and coma.

Page 7

281
• Bitter acid burning taste, constriction or numbness of throat, clenched jaw, salivation, froth, nausea,
confusion, drowsiness, prostration, opisthotonus, lockjaw, hyperthermia, epileptiform or tonic
convulsions, paralysis, stupor and coma.
• Bitter acid burning taste, constriction or numbness of throat, clenched jaw, salivation, froth, nausea,
confusion, drowsiness, prostration, opisthotonus, lockjaw, hyperthermia, epileptiform or tonic
convulsions, paralysis, stupor and coma.

Incorrect Options:
Option A - Phenol:
• Phenol has a distinct odor that is sickeningly sweet and tarry.
• It has a characteristic 'carbolic' or phenolic smell.
• Burning sweetish taste
• Phenol evaporates more slowly than water, and a moderate amount can form a solution with water.
• Clinical features: pins and needles sensation, Diarrhea, pain in abdomen, laryngeal and pulmonary
edema, Stertorous breathing and cyanosis. Carboluria Ochronosis
• pins and needles sensation, Diarrhea, pain in abdomen, laryngeal and pulmonary edema, Stertorous
breathing and cyanosis.
• Carboluria
• Ochronosis
• pins and needles sensation, Diarrhea, pain in abdomen, laryngeal and pulmonary edema, Stertorous
breathing and cyanosis.
• Carboluria
• Ochronosis
Option B - Datura:
• Odorless.
• All parts of these plants are poisonous - fruit, flowers and seeds.
• Clinical features:- Dryness of the mouth Dysphagia Dysarthria Dilatation of cutaneous blood
vessels Diplopia Dry hot skin Drunken gait Delirium Drowsiness
• Dryness of the mouth
• Dysphagia
• Dysarthria
• Dilatation of cutaneous blood vessels
• Diplopia
• Dry hot skin
• Drunken gait
• Delirium
• Drowsiness
• Dryness of the mouth

Page 8

282
• Dysphagia
• Dysarthria
• Dilatation of cutaneous blood vessels
• Diplopia
• Dry hot skin
• Drunken gait
• Delirium
• Drowsiness
Option D - Chloral hydrate:
• Peculiar pungent odor and a bitter taste/Acrid pear
• Clinical features - Retrosternal burning sensation, vomiting, drowsiness, hypotension, slow irregular
pulse, depression of respiration, deep sleep and coma. Albuminuria, scarlatiniform or urticarial rash.
• Retrosternal burning sensation, vomiting, drowsiness, hypotension, slow irregular pulse, depression
of respiration, deep sleep and coma. Albuminuria, scarlatiniform or urticarial rash.
• Retrosternal burning sensation, vomiting, drowsiness, hypotension, slow irregular pulse, depression
of respiration, deep sleep and coma. Albuminuria, scarlatiniform or urticarial rash.

Solution for Question 9:


Correct Option D - Gold poisoning:
• BAL is not contraindicated in Gold poisoning.
• It is used in arsenic, lead, bismuth, copper, mercury, gold, and other heavy metal poisonings.

Incorrect Options:
Option A/ B/ C - G6PD deficiency/ Iron poisoning/ Cadmium poisoning:
• BAL is contraindicated in liver damage, G6PD deficient individuals, iron poisoning, and cadmium
(since dimercaprol-cadmium and dimercaprol-iron complex is itself toxic).

Solution for Question 10:


Correct Option A - Hydrogen sulphide:
• It is a colorless, heavy, flammable gas, which on Post mortem staining will reveal a Bluish-green
discoloration.
• Hydrogen sulphide in combination with CO2 and methane formed in sewers is known as sewer gas.
• It is also referred to as the “knockdown gas” because inhalation of high concentrations can cause
immediate loss of consciousness and death.

Page 9

283
Incorrect Options:
Option B - Carbon monoxide: Carbon monoxide: Cherry red discoloration
Option C - Cyanide: Cyanide: Bright Red/ Brick red; Pink
Option D - Phosphorus: Phosphorus: Dark brown/yellow discoloration

Solution for Question 11:


Correct Option A - Arsenic:

Fruity/sweet
• Ethanol
• Chloroform
Acrid
• Paraldehyde chloral hydrate
Rotten eggs smell/sewer gas
• H2S
Rotten fish
• Aniline
Kerosene odour
• Organophosphate (Aromex)
Fishy or musty
• Aluminium phosphide
• Zinc phosphide
Bitter almonds
• Cyanide
Burnt rope
• Cannabis
Shoe polish
• Nitrobenzene
Garlic
• Arsenic
• Phosphorus
• Parathion
• Tellurium
• Thallium

Page 10

284
Incorrect Options:
Options B, C, and D are incorrect. Refer to Option A for an explanation.

Solution for Question 12:


Correct Option A - Hydrogen sulphide:
• In H2S poisoning it's bluish green; red discoloration.
Incorrect Options:
Option B/C/D - Aniline/ Nitrites/ Potassium chlorate:
• Chocolate brown hypostasis can be seen in Potassium chlorate, Aniline, and Nitrite poisoning.

Solution for Question 13:


Correct Option B - Saturated solution of sodium chloride:
• The most commonly used preservative for viscera is a saturated solution of common salt because it is
easily available, cheap and effective preservative.
• Contraindications for using salt as a preservative- A – Aconite C – Corrosives [as they chemically
react with salt] except phenol (carbolic acid) poisoning
• A – Aconite
• C – Corrosives [as they chemically react with salt] except phenol (carbolic acid) poisoning
• A – Aconite
• C – Corrosives [as they chemically react with salt] except phenol (carbolic acid) poisoning
C – Corrosives [as they chemically react with salt] except phenol (carbolic acid) poisoning

Incorrect Options:
Option A - Rectified spirit:
• 95% pure ethanol is referred to as the rectified spirit, with 5% water.
• Rectified spirit is the best preservative for chemical analysis.
• In cases of suspected alkali or acid poisoning(except carbolic acid), only rectified spirit is used.
• Contraindications for using rectified spirit as a preservative in alcohol detection false positives as it
gives
• Acetone
• Phosphorus
• Paraldehyde
• Phenol
• Formalin [gives false positive]

Page 11

285
Option C - Formalin:
• Formalin is not used as a preservative for chemical analysis because extraction of poison, especially
non-volatile organic compounds become difficult.
• Only used in histopathological examination.
• Histopathological examination: Sections of various internal organs in case of suspected abnormality
are preserved in 10% formalin or 95% alcohol.
Option D - Sodium flouride:
• Fluoride should also be added to urine, CSF, and vitreous humour if alcohol estimation is required,
and also to samples for analysis for cocaine, cyanide and Co.

Page 12

286
Metallic & Non-metallic Poisons
1. Which among the following metals are found on histochemical techniques in hepatic and pulmonary
lesions in patients with vineyard sprayer lung disease.?
(or)
Which metal is found in abundance in hepatic and pulmonary lesions in "Vineyard sprayer lung
disease" when histochemical techniques are applied?
A. Copper
B. Mercury
C. Lead
D. Phosphorous
----------------------------------------
2. A 30-year-old painter, has been working in the construction industry for several years. She
specializes in painting and renovation projects for both residential and commercial properties. She
starts experiencing fatigue, persistent abdominal pain, and frequent headaches.Blood tests reveal
elevated levels of lead. Which of the following chelating agent will not be not used in this patient?
A. EDTA
B. CUPRIMINE
C. DMSA
D. DESFERROXAMINE
----------------------------------------
3. Which among the following is FALSE regarding Thallium poisoning?
(or)
Which of the following statements about Thallium poisoning is FALSE?
A. Can be used as Ideal homicidal poison
B. Chronic poisoning of Thallium causes alopecia, skin rashes, painful peripheral neuropathy
C. Cannot be detected in ashes of burnt body
D. Mees’ lines can be seen on nails
----------------------------------------
4. In which of the following poisons, injuries are not present at the portal of entry but with considerable
systemic involvement during autopsy?
A. Corrosives
B. Chlorine
C. Nitrobenzene
D. Sulphur dioxide
----------------------------------------
5. A Person was admitted with diarrhoea, vomiting, low-grade fever, breathlessness, abdominal and
lower limb pain and later became semiconscious, cyanotic, tachypneic, tachycardic and hypotensive

287
and died. Autopsy finding revealed inflamed intestines with "rice-water" like contents and "red-velvety
appearance" stomach mucosa. What is the causative agent?
(or)
Autopsy finding revealed inflamed intestines with "rice-water" like contents and "red-velvety
appearance" stomach mucosa. What is the causative agent?
A. Arsenic poisoning
B. Sulphuric acid poisoning
C. Phosphorus poisoning
D. Copper Poisoning
----------------------------------------
6. The WHO permissible limit of arsenic in drinking water is:
(or)
The WHO permissible limit of arsenic in drinking water is:
A. 0.05 mg/litre
B. 0.001 mg/L
C. 0.01 µg/L
D. 0.1 mg/L
----------------------------------------
7. In a case of depressed patient who attempted suicide by ingesting at least 3.0 grams of prescribed
medication and presented with symptoms like nausea, vomiting, stomach discomfort, dizziness, and
weakness, along with severe hypokalemia (K+ - 1.7 mmol/L) and ECG abnormalities. which element
toxicity is most likely responsible for the characteristic feature of hypokalemia?
(or)
Which of the following will cause Rapid onset of marked hypokalemia and neuromuscular blockade by
ingesting 3.0g ?
A. Barium
B. Antimony
C. Copper
D. Iron
----------------------------------------
8. In a 35-year-old man employed at a mobile factory who presents with worsening dyspnea, inability to
dress without pausing for breath, increased salivation, dental discoloration, signs of pulmonary edema
on X-ray, and osteomalacia, a diagnosis of Cadmium poisoning is made. Which statement is not true
regarding Cadmium poisoning?
(or)
Which statement is not true regarding Cadmium poisoning?
A. Shell fish is an important source
B. Causes "boiled lobster" like rashes

Page 2

288
C. Cadmium accumulates in kidney
D. Toxicity causes Itai-Itai disease
----------------------------------------
9. A 35-year-old metalworker has been working in a small manufacturing unit specializing in the
production of alloys for many years. He presented to emergency with complaints of breathlessness and
looked distressed. Further examination reveals the presence of goiter, polycythemia, cardiomyopathy,
and metabolic acidosis. This is typically seen in?
(or)
The toxidrome of goitre, polycythaemia, cardiomyopathy, and metabolic acidosis is diagnostic of:
A. Cobalt
B. Barium
C. Cadmium
D. Antimony
----------------------------------------
10. Groote Eylandt syndrome is due to the toxicity of:
(or)
Groote Eylandt syndrome is due to the toxicity of:
A. Manganese
B. Magnesium
C. Thallium
D. Antimony
----------------------------------------
11. Identify the poison which produces similar symptom?
(or)
Identify the poison which produces similar symptom?

A. Mercury
B. Lead
C. Arsenic

Page 3

289
D. Copper
----------------------------------------
12. A 28-year-old first-time mother gives birth to a baby boy. Everything seems normal during the
pregnancy, and the baby is delivered full-term without complications. However, as he reaches the age
of six months, She notices that he doesn't seem to be reaching typical developmental milestones. He
appears like a rag doll, with hypotonia and poor head control. The substance responsible for this is?
(or)
The toxic substance responsible for floppy baby syndrome is:

A. Lithium
B. Cadmium
C. Antimony
D. Barium
----------------------------------------
13. The following terms denote chronic poisoning syndromes, except:
(or)
The following terms denote chronic poisoning syndromes, except:
A. Iodism
B. Bromism
C. Plumbism
D. Carbolism
----------------------------------------
14. A 36 year old male patient presented with pain abdomen, ataxia and constipation. Peripheral blood
smear examination is given belowshowed basophilic stippling of RBCs. He is suffering from poisoning
due to:
(or)
What type of poisoning is indicated by basophilic stippling of RBCs on peripheral blood smear?
A. Iron
B. Lead
C. Cadmium

Page 4

290
D. Arsenic
----------------------------------------
15. A 45-year-old artisan, has been working in a small jewelry-making workshop. He primarily uses
mercury to create molds for intricate designs. Recently, he has been feeling unwell, experiencing
symptoms such as fatigue, headaches, muscle aches, and difficulty concentrating.All of the following is
associated with this condition except?
(or)
Poisoning with Mercury is characterized by all of the following, except?
A. Danbury Tremor
B. K F ring
C. Anterior lens capsule deposits
D. Pink disease
----------------------------------------
16. Necrosis of PCT is seen in all except:
(or)
Necrosis of PCT is seen in all except:
A. Organophosphate poisoning
B. Lysol
C. Mercury
D. Arsenic
----------------------------------------
17. All are true about burtonian line except:
(or)
All are true about burtonian line except:
A. Bluish deposits
B. Deposition of Lead acetate granules
C. Over decayed teeth only
D. Seen in saturnism
----------------------------------------
18. Which of the following order is true about toxicity of mercury?
(or)
Which of the following order is true about toxicity of mercury?
A. Organic salts > mercuric salts > mercurous salts
B. Mercuric salts > organic salts > mercurous salts
C. organic salts> mercurous salts >Mercuric salts
D. Mercurous salts > organic salts > mercuric salts

Page 5

291
----------------------------------------
19. The following is the symptom seen in:
(or)
The following is the symptom seen in:

A. Arsenic poisoning
B. Mercury poisoning
C. Lead poisoning
D. Gold poisoning
----------------------------------------
20. The condition known as "itai-itai disease" , characterized by severe pain in the spine and joints,
softening of the bones and kidney failure, is caused by which specific element?
(or)
"itai-itai disease" is caused by which specific element?
A. Mercury toxicity
B. Cadmium toxicity
C. Lead toxicity
D. Arsenic toxicity
----------------------------------------
21. A 45-year-old laborer was admitted with complaints of multiple sinuses discharging foul-smelling
pus in the lower jaw bone. The man was employed at a firework factory. While taking a history of his
presenting complaints, the doctor was informed that it had begun with toothache about 6 months prior.
He had treated himself with over-the-counter pain medication. Gradually he had noticed the swelling of
his jaw and also lost 2 teeth in the 6 months since symptoms began. This is caused by which one of the
following?
(or)
"Phossy" jaw is caused by which one of the following?
A. Lead
B. Mercury
C. Phosphorus

Page 6

292
D. Arsenic
----------------------------------------

Correct Answers
Question Correct Answer

Question 1 1
Question 2 4
Question 3 3
Question 4 3
Question 5 1
Question 6 1
Question 7 1
Question 8 2
Question 9 1
Question 10 1
Question 11 1
Question 12 1
Question 13 4
Question 14 2
Question 15 2
Question 16 1
Question 17 2
Question 18 1
Question 19 1
Question 20 2
Question 21 3

Solution for Question 1:


Correct Option A - Copper:
• Vineyard sprayer lung: Due to inhalation of copper sulphate spray, which is characterized by
histiocytic granulomatous lung.
Incorrect Options:
Option B - Mercury:
• Acute exposure to elemental mercury vapor may produce corrosive bronchitis with fever, chills, and
dyspnea. It may progress to pulmonary edema and fibrosis. Sometimes, manifestations similar to
Kawasaki disease (mucocutaneous lymph node syndrome) are seen especially in children.

Page 7

293
• Chronic Mercury Poisoning (Hydrargyrism): Intention tremors (Danbury tremors/shaking palsy).
Mercurial erethism is seen in persons working with mercury in minor manufacturing firms. This term is
used to refer to the psychological effects of mercury toxicity. Mercurialentis: It is a peculiar eye change
due to exposure to mercury vapor. Acrodynia or pink disease (because it is characterized by a
generalized body rash) is thought to be an idiosyncratic hypersensitivity reaction particularly seen in
children. This can be caused by chronic mercury exposure in any form. Minimata disease is a type of
organic mercurial poisoning due to the eating of fish poisoned by mercury.
• Intention tremors (Danbury tremors/shaking palsy).
• Mercurial erethism is seen in persons working with mercury in minor manufacturing firms. This term is
used to refer to the psychological effects of mercury toxicity.
• Mercurialentis: It is a peculiar eye change due to exposure to mercury vapor.
• Acrodynia or pink disease (because it is characterized by a generalized body rash) is thought to be an
idiosyncratic hypersensitivity reaction particularly seen in children. This can be caused by chronic
mercury exposure in any form.
• Minimata disease is a type of organic mercurial poisoning due to the eating of fish poisoned by
mercury.
• Intention tremors (Danbury tremors/shaking palsy).
• Mercurial erethism is seen in persons working with mercury in minor manufacturing firms. This term is
used to refer to the psychological effects of mercury toxicity.
• Mercurialentis: It is a peculiar eye change due to exposure to mercury vapor.
• Acrodynia or pink disease (because it is characterized by a generalized body rash) is thought to be an
idiosyncratic hypersensitivity reaction particularly seen in children. This can be caused by chronic
mercury exposure in any form.
• Minimata disease is a type of organic mercurial poisoning due to the eating of fish poisoned by
mercury.
Option C - Lead:
• Signs and Symptoms: Anemia, Burtonian line/Basophilic stippling, Bone lines, Colic, Constipation (dry
belly ache); Drop (wrist/Foot drop) - Lead palsy, Encephalopathy, Facial pallor (earliest sign), Gout
(saturnine gout), Hypertension, Impotence.
Option D - Phosphorous:
• Signs and Symptoms of acute phosphorus poisoning. Garlicky odour. Luminous vomit and stool
because of the presence of phosphorus. Fumes emanate from the stools (smoking or smoky stool
syndrome). Acute poisoning causes liver cell necrosis, fatty liver and acute yellow atrophy.
• Garlicky odour.
• Luminous vomit and stool because of the presence of phosphorus.
• Fumes emanate from the stools (smoking or smoky stool syndrome).
• Acute poisoning causes liver cell necrosis, fatty liver and acute yellow atrophy.
• Chronic Phosphorus Poisoning (Phossy Jaw/Glass Jaw/Lucifer’s Jaw): Due to inhalation of fumes
over years. Toothache (earliest symptom) & swelling of the jaw. Loosening of the teeth &Necrosis; of
the lower jaw. Sequestration of bone in the mandible & multiple sinuses discharging foul-smelling pus.
• Due to inhalation of fumes over years.
• Toothache (earliest symptom) & swelling of the jaw.

Page 8

294
• Loosening of the teeth &Necrosis; of the lower jaw.
• Sequestration of bone in the mandible & multiple sinuses discharging foul-smelling pus.
• Garlicky odour.
• Luminous vomit and stool because of the presence of phosphorus.
• Fumes emanate from the stools (smoking or smoky stool syndrome).
• Acute poisoning causes liver cell necrosis, fatty liver and acute yellow atrophy.
• Due to inhalation of fumes over years.
• Toothache (earliest symptom) & swelling of the jaw.
• Loosening of the teeth &Necrosis; of the lower jaw.
• Sequestration of bone in the mandible & multiple sinuses discharging foul-smelling pus.

Solution for Question 2:


Correct Option D - DESFERROXAMINE:
• Desferoxamine is used in iron overload.
• It is not used in lead poisoning.
Incorrect Options:
Option A/ B/ C - EDTA/ CUPRIMINE/ DMSA:
• Options A, B, and C are the chelating agents used in lead poisoning.
• The chelating agents used in lead poisoning are EDTA(Ethylenediamine tetraacetic acid)
• , CUPRIMINE, DMSA(dimercapto succinic acid), BAL(British anti-Lewisite), and
DMPS(2,3-Dimercapto-1-propanesulfonic acid).

Solution for Question 3:


Correct Option C - Cannot be detected in ashes of burnt body:
• Thallium poisoning can be detected even in ashes of burnt body.
Incorrect Options:
Option A - Can be used as Ideal homicidal poison:
• Thallium poisoning is usually an Ideal homicidal poison.
Option B - Chronic poisoning of Thallium causes alopecia, skin rashes, painful peripheral neuropathy:
• Chronic poisoning of Thallium causes alopecia, skin rashes, painful peripheral neuropathy.
Option D - Mees' lines can be seen on nails:
• Mees' lines are also found in Arsenic, thallium poisoning, & chemotherapy.

Page 9

295
Solution for Question 4:
Correct Option C - Nitrobenzene:
• Poisons producing systemic lesions without injury at the portal of entry include acute hemolytic
poisons like Arsine and Nitrobenzene.
Option A/B/D - Corrosives/ Chlorine/ Sulphur dioxide:
• Poisons producing lesions at the portal of entry without systemic injuries includes corrosives, chlorine,
and sulphur dioxide.

Solution for Question 5:


Correct Option A - Arsenic poisoning:
• Gastrointestinal congestion is a feature of acute arsenic poisoning.
• Focal hemorrhages giving rise to a flea-bitten appearance are said to be characteristic. Intestines may
be inflamed and may contain “rice water” contents.
Incorrect Options:
Option B - Sulphuric acid poisoning:
• Sulphuric acid poisoning: Colorless, Odorless, Non-fuming.
• Signs & symptoms: Brown to black streak from the angle of mouth, Black tongue, Chalky white teeth,
Black necrotic stomach or black charred stomach, Blotting paper stomach, Gastric perforation, The
mind remains clear until death.
• Fatal dose: 5-10 ml, Fatal period: 12-24 hours.
• Sulphuric acid is the most common substance associated with vitriolage.
Option C - Phosphorus poisoning:
• Signs and Symptoms of acute phosphorus poisoning: Garlicky odor, Luminous vomit and stool
because of the presence of phosphorus. Fumes emanate from the stools (smoking or smoky stool
syndrome). Acute poisoning causes liver cell necrosis, fatty liver, and acute yellow atrophy.
• Garlicky odor, Luminous vomit and stool because of the presence of phosphorus.
• Fumes emanate from the stools (smoking or smoky stool syndrome). Acute poisoning causes liver cell
necrosis, fatty liver, and acute yellow atrophy.
• Chronic Phosphorus Poisoning (Phossy Jaw/Glass Jaw/Lucifer’s Jaw): Due to inhalation of fumes
over years. Toothache (earliest symptom) & swelling of the jaw; Loosening of the teeth & Necrosis of
the lower jaw. Sequestration of bone in the mandible & multiple sinuses discharging foul-smelling pus.
• Due to inhalation of fumes over years. Toothache (earliest symptom) & swelling of the jaw; Loosening
of the teeth & Necrosis of the lower jaw. Sequestration of bone in the mandible & multiple sinuses
discharging foul-smelling pus.
• Garlicky odor, Luminous vomit and stool because of the presence of phosphorus.

Page 10

296
• Fumes emanate from the stools (smoking or smoky stool syndrome). Acute poisoning causes liver cell
necrosis, fatty liver, and acute yellow atrophy.
• Due to inhalation of fumes over years. Toothache (earliest symptom) & swelling of the jaw; Loosening
of the teeth & Necrosis of the lower jaw. Sequestration of bone in the mandible & multiple sinuses
discharging foul-smelling pus.
Option D - Copper Poisoning
• Most of the time, poisoning is accidental. Pain in the Throat, Abdomen Ptyalism (increased salivation).
Vomiting (bluish green). Hemolysis in RBC, liver, and kidney. Chalcosis (Copper depositions in tissue)
o Chalcosis oculi (Copper deposition in eye) o Defective copper metabolism is Wilson Disease
• Pain in the Throat, Abdomen
• Ptyalism (increased salivation).
• Vomiting (bluish green).
• Hemolysis in RBC, liver, and kidney.
• Chalcosis (Copper depositions in tissue) o Chalcosis oculi (Copper deposition in eye) o Defective
copper metabolism is Wilson Disease
• o Chalcosis oculi (Copper deposition in eye)
• o Defective copper metabolism is Wilson Disease
• Pain in the Throat, Abdomen
• Ptyalism (increased salivation).
• Vomiting (bluish green).
• Hemolysis in RBC, liver, and kidney.
• Chalcosis (Copper depositions in tissue) o Chalcosis oculi (Copper deposition in eye) o Defective
copper metabolism is Wilson Disease
• o Chalcosis oculi (Copper deposition in eye)
• o Defective copper metabolism is Wilson Disease
• o Chalcosis oculi (Copper deposition in eye)
• o Defective copper metabolism is Wilson Disease

Solution for Question 6:


Correct Option A - 0.05 mg/litre:
• Permitted level of arsenic in drinking water is 0.05 mg/litre, as per WHO. Bureau of Indian Standards
gives the acceptable limit of arsenic in drinking water to 0.01 mg/litre and in absence of other
alternative sources to 0.05 mg/litre.

Solution for Question 7:

Page 11

297
Correct Option A - Barium:
• Hypokalemia and neuromuscular blockade: Rapid onset of marked hypokalemia, characteristic of
barium intoxication is due to sequestering of potassium by muscle cells.
Incorrect Options:
Option B - Antimony:
• Fatal Dose: Tartar emetic 0.2 to 0.5gm; Antimony trichloride: 0.1 to 0.2 gm.
• Fatal Period: Usually within 24 hours.
• Treatment: Stomach wash Tannic acid 4 gm. by mouth forms an insoluble antimony tannate B.A.L
Symptomatic
• Stomach wash
• Tannic acid 4 gm. by mouth forms an insoluble antimony tannate
• B.A.L
• Symptomatic
Option C - Copper:
• Copper subacetate: 15 gm.
• Copper sulphate: 20 gm (0.15-0.3 gm/kg).
• Fatal period: 18–24 hrs, but it may extend to 1–3 days.
• Gastric lavage with 1% potassium ferrocyanide, which acts as an antidote by forming cupric
ferrocyanide (insoluble). If not available, plain water can be used.
• Chelating agents: D-penicillamine given in usual doses is very effective.
Option D - Iron:
• Fatal dose: 20–30 gm (> 200 mg/kg).
• Fatal period: 24-48 hrs.
• Treatment is usually whole-bowel irrigation and chelation therapy [Desferrioxamine (antidote) in a
dose of 10–15 mg/kg/h as a continuous infusion to a maximum of 6 gm, till there is a significant
reduction of systemic toxicity].

Solution for Question 8:


Correct Option B - Causes "boiled lobster" like rashes:
• 'Boiled lobster' appearance is seen in poisoning with Boric acid (Hydrogen borate/Orthoboric acid).
Incorrect Options:
Option A/ C/ D -
Shell fish is an important source/ Cadmium accumulates in kidney/ Toxicity causes Itai itai disease:
• Shellfish such as mussels, scallops, and oysters may be a major source of dietary cadmium.
• The kidney accumulates cadmium over a lifetime.

Page 12

298
• Cadmium poisoning causes Itai - Itai disease, aka ouch ouch disease.

Solution for Question 9:


Correct Option A - Cobalt:
• The classic toxidrome of chronic cobalt poisoning is the tetrad of goiter, polycythemia,
cardiomyopathy, and metabolic acidosis.
Incorrect Options:
Option B - Barium:
• Barium acts locally as an irritant poison. After absorption, it acts both on voluntary and involuntary
muscles. Barium seems to act as a potassium antagonist and calcium agonist.
• Mechanism of toxicity of Barium: Hypokalaemia and neuromuscular blockade: Rapid onset of marked
hypokalemia, characteristic of barium intoxication is due to sequestering of potassium by muscle cells.
Barium clogs K+ channels and prevents potassium from exiting the skeletal muscle cells. Barium
stimulates striated, smooth, and cardiac muscle resulting in violent peristalsis, arterial hypertension,
and arrhythmias.
• Hypokalaemia and neuromuscular blockade: Rapid onset of marked hypokalemia, characteristic of
barium intoxication is due to sequestering of potassium by muscle cells.
• Barium clogs K+ channels and prevents potassium from exiting the skeletal muscle cells.
• Barium stimulates striated, smooth, and cardiac muscle resulting in violent peristalsis, arterial
hypertension, and arrhythmias.
• Hypokalaemia and neuromuscular blockade: Rapid onset of marked hypokalemia, characteristic of
barium intoxication is due to sequestering of potassium by muscle cells.
• Barium clogs K+ channels and prevents potassium from exiting the skeletal muscle cells.
• Barium stimulates striated, smooth, and cardiac muscle resulting in violent peristalsis, arterial
hypertension, and arrhythmias.
Option C - Cadmium:
• Cadmium poisoning causes Itai - Itai disease, aka ouch ouch disease.
• It can cause softening of the bones and kidney failure. Proximal tubular necrosis, emphysema, and
anemia can be seen.
• Osteomalacia, bone pain, and pathological fractures.
• Effective treatments involve the use of chelators to promote urinary excretion.
Option D - Antimony:
• Fatal Dose: Tartar emetic 0.2 to 0.5 gm; antimony trichloride: 0.1 to 0.2 gm; Fatal Period: Usually
within 24 hours.
• Treatment: Stomach wash; Tannic acid 4 gm. by mouth forms an insoluble antimony tannate, B.A.L.
• METAL FUME FEVER: It is caused by inhalation of fumes of zinc, copper, magnesium, nickel,
mercury, lead, iron, silver, chromium, cadmium, cobalt, antimony, and manganese.

Page 13

299
Solution for Question 10:
Correct Option A - Manganese:
• Groote Eylandt syndrome is due to the toxicity of Manganese.
• The manganese miners on an Australian island (Groote Eylandt) have been reported to be afflicted
with a peculiar neurological disease characterized by upper motor neuron and cerebellar signs, and
oculomotor symptoms (Angurugu syndrome or Groote Eylandt syndrome).
Incorrect Options:
Option B - Magnesium:
• Magnesium toxicity or hypermagnesemia, generally occurs when magnesium is ingested in large
quantities as a supplement or medication.
• Symptoms including diarrhea, nausea, and abdominal cramping after taking a magnesium-base
laxative or antacid may indicate excessive magnesium or magnesium toxicity.
• People with impaired kidney function are at the greatest risk for magnesium overdose. People with
kidney disease are at risk for dropping blood pressure even with a moderate magnesium overdose.
Option C - Thallium:
• The triad of alopecia, neuropathy, and diarrhea results from Thallium Poisoning.
Option D - Antimony:
• Fatal Dose: Tartar emetic 0.2 to 0.5 gm; Antimony trichloride: 0.1 to 0.2 gm.
• Fatal Period: Usually within 24 hours.
• Treatment: Stomach wash; Tannic acid 4 g. by mouth forms an insoluble Antimony tannate, B.A.L.
• METAL FUME FEVER: It is caused by inhalation of fumes of zinc, copper, magnesium, nickel,
mercury, lead, iron, silver, chromium, cadmium, cobalt, antimony, and manganese.

Solution for Question 11:


Correct Option A - Mercury:
• Acute exposure to elemental mercury vapor may produce corrosive bronchitis with fever, chills and
dyspnoea. It may progress to pulmonary edema and fibrosis. Sometimes, manifestations similar to
Kawasaki disease (mucocutaneous lymph node syndrome) are seen especially in children.
Incorrect Options:
Option B - Lead:
• In Lead Poisoning: Anemia; Burtonian line/Basophilic stippling, Bone lines, Colic, Constipation (dry
belly ache), Drop (wrist/Foot drop) - Lead palsy, Encephalopathy, Facial pallor (earliest sign), Gout
(saturnine gout), Hypertension, Impotence.
Option C - Arsenic:
• In chronic arsenic poisoning: nutritional and gastrointestinal disturbances can be seen. There is
classical "raindrop pigmentation". Hyperkeratosis of palms and soles can occur. Mees' line (white
transverse bands crossing the nails). Chronic exposure can also cause black foot disease.
Option D - Copper:

Page 14

300
• Acute ingestion: Symptoms appear 15–30 minutes after swallowing.
• Chronic copper toxicity may occur from eating acidic foods cooked in uncoated copper cookware, or
from exposure to excess copper in drinking water or food contaminated with verdigris, or other
environmental sources.
• Kayser-Fleischer Ring is seen in Copper poisoning.
• Vineyard sprayer lung: Due to inhalation of copper sulphate spray, which is characterized by
histiocytic granulomatous lung.
• Gastric lavage with 1% potassium ferrocyanide, which acts as an antidote by forming cupric
ferrocyanide (insoluble). If not available, plain water can be used.
• Chelating agents: D-penicillamine given in usual doses is very effective.

Solution for Question 12:


Correct Option A - Lithium:
• Floppy baby syndrome is seen in newborn babies breastfed by lithium-ingesting mothers due to
disruption of afferent inputs from stretch receptors.
Incorrect Options:
Option B - Cadmium:
• Cadmium poisoning causes Itai - Itai disease, aka ouch ouch disease.
• It can cause softening of the bones and kidney failure.
• Proximal tubular necrosis, emphysema, and anemia can be seen.
• Osteomalacia, bone pain, and pathological fractures.
• Effective treatments involve the use of chelators to promote urinary excretion.
Option C - Antimony:
• Fatal Dose: Tartar emetic 0.2 to 0.5g; Antimony trichloride: 0.1 to 0.2 gm.
• Fatal Period: Usually within 24 hours.
• Treatment: Stomach wash; Tannic acid 4 gm. by mouth forms an insoluble antimony tannate, B.A.L.
Option D - Barium:
• Barium acts locally as an irritant poison. After absorption, it acts both on voluntary and involuntary
muscles. Barium seems to act as a potassium antagonist and calcium agonist.
• Fatal dose: About 0.8–1 gm of barium chloride/ sulfide/ nitrate.
• Fatal period: Usually within 12 hrs.

Solution for Question 13:


Correct Option D - Carbolism:
• Acute carbolic acid poisoning is known as Carbolism.

Page 15

301
Incorrect Options:
Option A - Iodism:
• Chronic Iodine poisoning (Iodism).
Option B - Bromism:
• Chronic Bromide poisoning (Bromism).
Option C - Plumbism:
• Chronic lead poisoning (Saturnism or Plumbism).

Solution for Question 14:


Correct Option B - Lead:
• Anemia with punctate basophilia (basophilic stippling) is an early feature of chronic lead poisoning,
also characterized by pain abdomen, ataxia, and constipation.

Incorrect Options:
Option A - Iron:
• Few hours after ingestion, vomiting, abdominal pain, and hemorrhagic gastroenteritis, shock, acidos,
is and coma occur.
• The second stage sets in 6 to 24 hours in which the patient is symptom free.
• In the third stage (24 to 48 hours) metabolic acidosis, jaundice, hypoglycemia, shock, and coma with
hepatic and renal failure occurs.
• After one to two weeks, in the fourth stage late complications such as gastric stricture, and pyloric
stenosis occur.
Option C - Cadmium:
• It causes 'Itai - Itai' disease.
• Teeth: Golden yellow staining
• Proximal tubular necrosis, emphysema, and anemia.
• Osteomalacia, bone pain, and pathological fractures.
Option D - Arsenic:
• In chronic arsenic poisoning: Nutritional and gastrointestinal disturbances can be seen.
• There is classical "Raindrop Pigmentation".
• Hyperkeratosis of palms and soles can occur.
• Mee's line (white transverse bands crossing the nails).
• Chronic exposure can also cause 'Black foot disease'.

Page 16

302
Solution for Question 15:
Correct Option B - K F ring:
• Kayser-Fleischer Ring is seen in Copper poisoning and it is a feature of Wilson's disease.
• KF ring: Brownish–yellow ring visible around the corneo-sclera junction (limbus).
Incorrect Options:
Option A/ C/ D - Danbury Tremor/ Anterior lens capsule deposits/ Pink disease:
• Intention tremors (Danbury tremors/shaking palsy) seen in Poisoning with Mercury.
• Mercurialentis: It is a peculiar eye change due to exposure to mercury vapor. It is due to a brownish
deposit of mercury through the cornea on the anterior lens capsule.
• Acrodynia or pink disease (because it is characterized by a generalized body rash) is thought to be an
idiosyncratic hypersensitivity reaction particularly seen in children. This can be caused by chronic
mercury exposure in any form.

Solution for Question 16:


Correct Option A - Organophosphate poisoning:
• PCT necrosis is not seen in Organophosphate poisoning.
Option B/ C / D - Lysol/ Mercury/ Arsenic:
• The PCT necrosis is seen in Arsenic, Lysol, and Mercury poisoning.
• Also seen in Phenol, carbon tetra chloride and catharidine.

Solution for Question 17:


Correct Option B - Deposition of Lead acetate granules:
• It is due to the formation of lead sulphide by the action of H2S formed by decomposed food in the
mouth.
Incorrect Options:
Option A/ C/ D - Bluish deposits/ Over decayed teeth only/ Seen in saturnism:
• Burtonian line is a stippled blue line seen on the gingival surface in 50–70% of cases.
• Burtonian line is seen in chronic lead poisoning called plumbism or saturnism.
• It appears due to subepithelial deposit of granules at the junction with teeth, only near dirty or carious
teeth, within a week of exposure, especially on upper jaw.

Solution for Question 18:

Page 17

303
Correct Option A - Organic salts > mercuric salts > mercurous salts:
• Organic salts are the most toxic, Mercuric salts are intermediate, and Mercurous salts are the least
toxic.
• So, option A is true; Organic salts > mercuric salts > mercurous salts.

Solution for Question 19:


Correct Option A - Arsenic poisoning:
• Mees line (white transverse bands crossing the nails) seen in Arsenic poisoning.

Incorrect Options:
Option B - Mercury poisoning:
• In mercury poisoning, we can see black, blue lines of gums, intentional tremors (hatters shaking
palsy); erethism; acrodynia or pinks disease, mercuria lentis, minimata disease; membranous
glomerulopathy.
Option C - Lead Poisoning:
• In Lead Poisoning: Anemia; Burtonian line/Basophilic stippling, Bone lines, Colic, Constipation (dry
belly ache); Drop (wrist/Foot drop) - Lead palsy; Encephalopathy; Facial pallor (earliest sign); Gout
(saturnine gout); Hypertension; Impotence.
Option D - Gold poisoning:
• Clinical features include: Most common:Dermatitis, pruritus, urticaria, stomatitis Less common:
alopecia, trophic nails, pseudocyanosis (non-blanching blue-grey skin discoloration; spares mucous
membranes, may be more pronounced where sun-exposed
• Most common:Dermatitis, pruritus, urticaria, stomatitis
• Less common: alopecia, trophic nails, pseudocyanosis (non-blanching blue-grey skin discoloration;
spares mucous membranes, may be more pronounced where sun-exposed

Page 18

304
• Most common:Dermatitis, pruritus, urticaria, stomatitis
• Less common: alopecia, trophic nails, pseudocyanosis (non-blanching blue-grey skin discoloration;
spares mucous membranes, may be more pronounced where sun-exposed

Solution for Question 20:


Correct Option B - Cadmium toxicity:
• Cadmium poisoning causes Itai - Itai disease, aka ouch ouch disease.
• It can cause softening of the bones and kidney failure. Effective treatments involve the use of
chelators to promote urinary excretion.
Incorrect Options:
Option A - Mercury toxicity:
• In mercury poisoning, we can see black, blue lines of gums, intentional tremors (hatters shaking
palsy), erethism; acrodynia or pinks disease, mercuria lentis, minimata disease, membranous
glomerulopathy.
Option C - Lead toxicity:
• In Lead Poisoning: Anemia; Burtonian line/Basophilic stippling, Bone lines, Colic, Constipation (dry
belly ache), Drop (wrist/Foot drop) - Lead palsy, Encephalopathy, Facial pallor (earliest sign), Gout
(saturnine gout), Hypertension, Impotence.
Option D - Arsenic toxicity:
• In chronic arsenic poisoning: Nutritional and gastrointestinal disturbances can be seen. There is
classical "raindrop pigmentation". Hyperkeratosis of palms and soles can occur. Mees' line (white
transverse bands crossing the nails). Chronic exposure can also cause 'Black foot disease'.

Solution for Question 21:


Correct Option C - Phosphorus:
• The frequent inhalation of fumes over a period of years causes necrosis of the lower jaw in the region
of a decayed tooth.
• The vapors act on the jaw through a carious tooth or an interspace due to a missing tooth where
suppurative microorganisms are already present.
• At first, there is a toothache, which is followed by swelling of the jaw, loosening of the teeth, necrosis
of gums, and sequestration of bone in the mandible. This condition is known as 'phossy jaw' (glass jaw)
in which osteomyelitis and necrosis of the jaw occur, with multiple sinuses discharging foul-smelling
pus.

Incorrect Options:
Option A - Lead:

Page 19

305
• A: Anaemia
• B: Burtonian line/Basophilic stippling, Bone lines
• C: Colic, Constipation (dry belly ache)
• D: Drop (wrist/Foot drop) - Lead palsy
• E: Encephalopathy
• F: Facial pallor (earliest sign)
• G: Gout (saturnine gout)
• H: Hypertension
• I: Impotence
Option B - Mercury:
• Chronic exposure to elemental mercury yields a classic triad of gingivitis and salivation, tremors, and
neuropsychiatric changes. The symptoms are increased salivation or ptyalism may occur. Excessive
salivation can contribute to irritation of the gums. and occasionally a blue line at their junction with
teeth, sore mouth and throat, loosening of teeth, gastrointestinal disturbances, anemia, anorexia, loss
of weight, and chronic inflammation of kidneys with progressive uremia. Tremors (sometimes called
Danbury tremors) occur first in the hands, then progress to lips and tongue, and finally involve arms
and legs. The tremor is moderately coarse and is interspersed with jerky movements. The advanced
condition is called hatter's shakes or glass-blower's shakes because they are common in persons
working in glass-blowing and hat industries due to long term exposure of mercury.
• Mercurial erethism is seen in persons working with mercury in minor manufacturing firms. This term is
used to refer to the psychological effects of mercury toxicity,
• Including Tremors Personality changes Emotional lability Insomnia Memory loss Difficulty
concentrating
• Personality changes
• Emotional lability
• Insomnia
• Memory loss
• Difficulty concentrating
• Mercurialent is: It is a peculiar eye change due to exposure to mercury vapor.
• Acrodynia or pink disease (because it is characterized by a generalized body rash) is thought to be an
idiosyncratic hypersensitivity reaction particularly seen in children. This can be caused by chronic
mercury exposure in any form.
• Minimata disease is a type of organic mercurial poisoning due to eating of fish poisoned by mercury.
• Personality changes
• Emotional lability
• Insomnia
• Memory loss
• Difficulty concentrating
Option D - Arsenic:

Page 20

306
• Clinical Features of Chronic Arsenic poisoning: GIT symptoms: Loss of weight, malaise, loss of
appetite, salivation, colicky pain, diarrhea, and vomiting. Liver: Hepatomegaly, jaundice, cirrhosis Skin
symptoms: Usually begin with a persistent erythematous flush leading to hyperkeratotic skin and
desquamation. Hyperkeratosis is most prominent in the distal parts of the body. Pigmentation is patchy
(raindrops pigmentation). Skin cancer (Squamous cell carcinoma and basal cell carcinoma) also
occurs. Nails: Aldrich Mees lines (transverse white striae of the fingernails) may also be seen. Hairs:
Alopecia occurs Hematological: Anemia, leukopenia, Thrombocytopenia, and Bone marrow hypoplasia
are seen. Anemia is normocytic and normochromic Peripheral neuropathy: Sensory and motor
polyneuritis (sensory symptoms usually predominate) manifesting as numbness and tingling in a
‘stocking glove’ distribution. Blackfoot disease: Gangrene of the lower extremities due to platelet
activation and hypercoagulability of blood in peripheral arteries. Encephalopathy Nephritis
• GIT symptoms: Loss of weight, malaise, loss of appetite, salivation, colicky pain, diarrhea, and
vomiting.
• Liver: Hepatomegaly, jaundice, cirrhosis
• Skin symptoms: Usually begin with a persistent erythematous flush leading to hyperkeratotic skin and
desquamation. Hyperkeratosis is most prominent in the distal parts of the body. Pigmentation is patchy
(raindrops pigmentation). Skin cancer (Squamous cell carcinoma and basal cell carcinoma) also
occurs.
• Nails: Aldrich Mees lines (transverse white striae of the fingernails) may also be seen.
• Hairs: Alopecia occurs
• Hematological: Anemia, leukopenia, Thrombocytopenia, and Bone marrow hypoplasia are seen.
Anemia is normocytic and normochromic
• Peripheral neuropathy: Sensory and motor polyneuritis (sensory symptoms usually predominate)
manifesting as numbness and tingling in a ‘stocking glove’ distribution.
• Blackfoot disease: Gangrene of the lower extremities due to platelet activation and hypercoagulability
of blood in peripheral arteries.
• Encephalopathy
• Nephritis
• GIT symptoms: Loss of weight, malaise, loss of appetite, salivation, colicky pain, diarrhea, and
vomiting.
• Liver: Hepatomegaly, jaundice, cirrhosis
• Skin symptoms: Usually begin with a persistent erythematous flush leading to hyperkeratotic skin and
desquamation. Hyperkeratosis is most prominent in the distal parts of the body. Pigmentation is patchy
(raindrops pigmentation). Skin cancer (Squamous cell carcinoma and basal cell carcinoma) also
occurs.
• Nails: Aldrich Mees lines (transverse white striae of the fingernails) may also be seen.
• Hairs: Alopecia occurs
• Hematological: Anemia, leukopenia, Thrombocytopenia, and Bone marrow hypoplasia are seen.
Anemia is normocytic and normochromic
• Peripheral neuropathy: Sensory and motor polyneuritis (sensory symptoms usually predominate)
manifesting as numbness and tingling in a ‘stocking glove’ distribution.
• Blackfoot disease: Gangrene of the lower extremities due to platelet activation and hypercoagulability
of blood in peripheral arteries.

Page 21

307
• Encephalopathy
• Nephritis

Page 22

308
Animal & Plant Irritants
1. A 43-year-old farmer was taken to the only hospital available in the village after he was bitten by a
snake. From the description of the snake, the doctor was sure that it was Russell's Viper and that he
can be administered the Indian polyvalent anti-snake venom which contains the venom of all of the
following, except?
(or)
Indian polyvalent anti-snake venom which contains the venom of all of the following, except?
A. Naja
B. Hypnale
C. Echis carinate
D. Daboia Russelii
----------------------------------------
2. India is among the countries most dramatically affected by snakebite and accounts for almost half
the total number of annual deaths in the world. It is estimated that India had 1.2 million snakebite
deaths (representing an average of 58,000 per year) from 2000 to 2019 with nearly half of the victims
aged 30-69 and over a quarter being children under 15. All of the following are true regarding snake
bite except?
(or)
All of the following are true regarding snake bite except?
A. Neostigmine, ventilatory support should be given to patients along with ASV
B. ASV is the main stay of treatment
C. Humped pit viper snake is excluded from polyvalent ASV
D. Neostigmine and Atropine may be given for Krait bite
----------------------------------------
3. If the head of the snake has small scales covering the whole surface of the head it indicates:
(or)
If the head of the snake has small scales covering the whole surface of the head it indicates:
A. Viper
B. Cobra
C. Krait
D. Non venomous
----------------------------------------
4. A 40-year-old farmer has been admitted to the emergency after being bitten. The patient complained
of abdominal pain, nausea, and difficulty in keeping his head up. During the questioning, he also began
to pause for a breath while speaking, confirming the doctor's doubts of respiratory muscle involvement.
On examination, he also noted bilateral ptosis. The doctor summed up all his findings as ophitoxaemia.
Envenomation from the bite of which one of the following, would be termed Ophitoxaemia?
(or)

309
Ophitoxaemia refers to?
A. Snake venom poisoning
B. Scorpion bite
C. Spider bite
D. Tick bite
----------------------------------------
5. Venom of snakes belonging to the family of elapidae cause which of the following toxin?
(or)
Venom of snakes belonging to the family of elapidae cause which of the following toxin?
A. Vasculotoxic
B. Neurotoxic
C. Musculotoxic
D. Nontoxic
----------------------------------------
6. Match the following 1. Krait a. 2. Cobra b. 3. Python c. 4. Viper d.
(or)
Match the following 1. Krait a. 2. Cobra b. 3. Python c. 4. Viper d.

1. Krait a.

2. Cobra b.

Page 2

310
3. Python c.

4. Viper d.

A. 1-d, 2-c, 3-a, 4-b


B. 1-a, 2-d, 3-b, 4-c
C. 1-c, 2-b, 3-a, 4-d
D. 1-b, 2-c, 3-d, 4-a
----------------------------------------
7. Since snakebite is a rural problem, primarily affecting India’s farmers, rural laborers, and their
families it would make sense for antivenom and associated treatment to be available at Primary Health
Centers and other rural medical facilities. However, this is often not the case and training in snake
identification and snakebite treatment is woefully inadequate. Which of the following statements is not
true of snake bites?
A. Anti snake venom is effective for pit viper
B. Cobra venom is neurotoxic
C. Atropine premedication should be used before administering neostigmine
D. ASV administration does not require test dose
----------------------------------------
8. All the statements regarding the snake shown in the image is true except:
(or)
All the statements regarding the snake shown in the image is true except:

Page 3

311
A. The venom is neurotoxic
B. Indian Anti-snake venom is effective in this bite
C. Viviparous
D. DIC is common
----------------------------------------
9. A 3-year-old boy was brought to the emergency with complaints of persistent crying, weakness, and
inability to stand. The family had been picnicking near the woods and the child was left unattended for
some time so the parents are unsure of what caused the sudden onset of symptoms experienced by
the child. During the time the team waited for the results of the initial investigations, the doctor noticed
that the child was beginning to show difficulty in lifting the arms as well. Poisoning with which one of the
following could cause such Ascending paralysis?
(or)
Which one of the following Poisoning can cause Ascending paralysis?
A. Botulism
B. Hemlock poisoning
C. Zigadenus poisoning
D. Cobra bite
----------------------------------------
10. The toxic principle of this plant is:
(or)
The toxic principle of this plant is:

Page 4

312
A. Tetrahydro cannabinol
B. Thevetin
C. Oduvin
D. Podophyllin
----------------------------------------
11. Which of the following toxalbumin is used as terrorist weapon?
(or)
Which of the following toxalbumin is used as terrorist weapon?
A. Ricin
B. Abrin
C. Croton
D. Chilly
----------------------------------------
12. A 5-year-old girl was brought to emergency department following consumption of flowers and
leaves of calotropis accidentally which was noticed on time by her mother The doctors noticed that she
was hypotensive and her blood pressure was not recordable. She was immediately intubated,
ventilated with an Ambu bag with oxygen, initiated on inotropes, and referred to a tertiary care center
for further management. The active principle in calotropis is all of the following, except?
(or)
The active principle in calotropis is all of the following, except?
A. Calotoxin
B. Calactin
C. Uscharin
D. Bhilawanol
----------------------------------------
13. Marking nuts are black heart-shaped and have a thick, cellular pericarp that contains an irritant juice
that is brownish, oily, and acrid but turns brown on exposure to air. Applied externally, the juice causes
irritation and a painful blister that contains acrid serum which causes eczematous eruptions of the
neighboring skin with which it comes into contact and there is itching. What is the scientific name of this
plant poison which causes such an "Artificial bruise"?
(or)
What is the scientific name of the plant poison which can causes "Artificial bruise"?
A. Strychnos nux vomica
B. Semecarpus anacardium
C. Abrus precatorius
D. Capsicum annum
----------------------------------------
14. Identify the following

Page 5

313
(or)
Identify the following

A. Cannabis
B. Datura
C. Erythroxylum coca
D. Abrus precatorius
----------------------------------------
15. The active principle of this toxic seed is:
(or)
The Active principle of below image:

A. Croton
B. Ricin
C. Abrin
D. Capsicin
----------------------------------------

Correct Answers
Question Correct Answer

Question 1 2

Page 6

314
Question 2 4
Question 3 1
Question 4 1
Question 5 2
Question 6 1
Question 7 1
Question 8 1
Question 9 2
Question 10 4
Question 11 1
Question 12 4
Question 13 2
Question 14 4
Question 15 2

Solution for Question 1:


Correct Option B - Hypnale:
• Hypnale refers to pit viper for which polyvalent ASV is ineffective. Indian Polyvalent anti snake venom
(PAV) is prepared by hyperimmunising horses against the venom of the four common poisonous
snakes, i.e., cobra, common krait, Russell's viper, and saw-scaled viper.
Incorrect Options:
Option A/ C/ D - Naja/ Echis carinate/ Daboia Russelii:
• Indian Polyvalent anti-snake venom is effective in Common cobra (Naja naja), Saw-scaled viper
(Echis carinatus) and Russell’s viper (Daboia russelii), and Common krait (Bungarus caeruleus).

Solution for Question 2:


Correct Option D - Neostigmine and Atropine may be given for Krait bite:
• Neostigmine and atropine are not typically administered for Krait snakebites.
• In krait, toxin inhibits release of acetylcholine from presynaptic membrane. Neostigmine restores
neuromuscular blockage by inhibiting acetylcholine esterase (enzyme which hydrolyse the
acetylcholine) which prolongs the action of acetylcholine (cholinomimetic). therefore neostigmine is
ineffective in krait poisoning
• Antivenom, specifically polyvalent antivenom, is the primary treatment for Krait snakebites.
Incorrect Options:
Option A- Neostigmine, ventilatory support should be given to patients along with ASV: Neostigmine is
sometimes used as an adjunct to antivenom therapy for certain snakebites, such as those caused by c
obras, to address neurotoxic effects. Ventilatory support may also be necessary in severe cases of sna

Page 7

315
kebite, especially when respiratory muscles are affected.
Option B - ASV is the mainstay of treatment: Antivenom (ASV or polyvalent antivenom) is the primary tr
eatment for snakebites, as it helps neutralize the venom's effects. It is crucial in managing snakebite ca
ses.
Option C - Humped pit viper snake is excluded from polyvalent ASV: Polyvalent antivenom is designed
to cover the venoms of various snake species commonly found in a region. Humped pit viper (Hypnale
species) venom may not always be effectively treated with polyvalent antivenom, and specific antiveno
m may be required for this snake's bite.

Solution for Question 3:


Correct Option A - Viper:
• If the head of the snake has small scales covering the whole surface of the head it indicates viper.
• It can be identified by the entire broad plates on the belly, the small scales on the head, and the shield
beneath the tail divided into two rows.
• Russel viper: The Russell’s viper is considered one of the most deadly land-dwelling snakes on earth.

Incorrect Options:
Option B/C - Cobra/ Krait:
• All poisonous snakes have small head scales except cobra, krait & pit viper.
• The COBRA has a hood, which on the dorsal side often bears a double or single spectacle mark, but it
has sometimes an oval spot surrounded by an ellipse. Head scales are large and the third labial
touches the eye and the nasal shield. The portion of the neck surrounding the spectacle mark is darker
than the rest of the back and is often speckled with small golden spots.
• THE COMMON KRAIT is steel-blue, often shining and has single or double white bands across the
back, and a creamy white belly. Its length is one to one-and-half metres. The stripes are not very
distinct in the anterior region. The bead is covered with large shields. Four shields are found on either
side of the lower lip. The scales in the central row down the back are large and hexagonal.

Page 8

316
Option D - Non venomous:
• Non-poisonous snakes, e.g., rat snake, vine snake, sand boa, and mud snake.
• non venomous snake have large scales in the head.

Solution for Question 4:


Correct Option A - Snake venom poisoning:
• Ophitoxaemia characterizes the clinical spectrum of snakebite envenomation.
• Fatal period: Death may occur immediately from shock due to fright.
• Cobra: ½–24 hours; Viper: 1–4 days.
Incorrect Options:
Option B - Scorpion bite:
• Scorpion bite: The venom is a clear, colorless, proteinous toxalbumin, having hemolytic and
neurotoxic effect.
• Its toxicity is more than that of snakes, but only a small quantity is injected.
• Action: The venom is a potent autonomic stimulator, resulting in the release of massive amounts of
catecholamines from the adrenals. It has also some direct effect on the myocardium.
Option C - Spider bite:
• In rare and extreme cases, black widow spider venom poisoning may lead to seizures and even
death. Death generally doesn’t occur in healthy adults. Young people, the elderly, and those with
weakened immune systems are more susceptible to serious complications and death from a black
widow spider bite.
Option D - Tick bite:
• Tick bite occurs in 30% of adult patients and 60% of children who are bitten by an infected tick.
• Symptoms that can occur in the absence of or in addition to a rash after a tick bite include Fever and
chills, Headache, muscle and joint aches, Swollen lymph nodes, Full body rash, and fatigue.
• People who are allergic to ticks can experience the following symptoms after a bite: Pain, swelling, or
a burning sensation at the bite site, Rash or allergic skin irritation, Blisters, and Respiratory difficulties, if
severe.
• A tick bite, in rare cases, can lead to tick paralysis.

Solution for Question 5:


Correct Option B - Neurotoxic:
• Elapidaes are Neurotoxic. This family includes cobras, adders, mambas, and common kraits.
Incorrect Options:
Option A - Vasculotoxic:

Page 9

317
• Viperidae are vasculotoxic/ hemotoxic, e.g., Russel viper and saw-scaled viper
Option C - Musculotoxic:
• Hydrophidae is myotoxic/ musculotoxic and this family includes sea snakes.
Option D - Nontoxic:
• Non-poisonous snakes, e.g., rat snake, vine snake, sand boa, and mud snake.

Solution for Question 6:


Correct Option A - 1-d ,2-c, 3-a, 4-b:
1. Krait – Image D

2. Cobra – Image C

Page 10

318
3. Python - Image A

4. Viper - Image B

Solution for Question 7:


Option A: Anti snake venom is effective for pit viper
• Anti-snake venom is not effective for pit viper. The Pit viper snake is excluded from polyvalent ASV.
• It is effective only for Common cobra, Russel’s viper, Saw-sealed viper, and Krait.
Incorrect options
Option B: Cobra venom is neurotoxic

Page 11

319
• Cobra belongs to elapidae family & is neurotoxic.
Option C: Atropine premedication should be used before administering neostigmine
• Atropine premedication should be used before administering neostigmine (which is used for cobra
bites) to reverse the muscarinic effect of neostigmine
Option D: ASV administration doesn't require test dose
• Anti Snake Venom administration is done only after the test dose.

Solution for Question 8:


Correct Option A - The venom is neurotoxic:
• The picture shows Russel viper and it contains hemotoxic venom.
Incorrect Options:
Option B/ C/ D - Indian Antisnake venom is effective in this bite/ Viviparous/ DIC is common:
• Indian anti-venom snake is effective against common Cobra, Krait, Russel viper and saw-scaled viper.
• Viper is viviparous as it gives birth to young ones.
• DIC is a common manifestation and also causes renal failure. So, options B, C, and D are true.

Solution for Question 9:


Correct Option B - Hemlock poisoning:
• Conium Maculatum (Hemlock): Ingestion causes burning in the mouth and throat, gastric
inflammation, vomiting, diarrhea, slow respiration, increased and later slow pulse, mental confusion,
tremors, ataxia, sometimes blindness, progressive motor paralysis extending upwards from the
extremities, coma, and death from respiratory paralysis.
• Progressive muscular paralysis due to depression of the motor nerves can be seen. The lower limbs
are affected first and the paralysis ascends till the muscles of respiration are affected.

Page 12

320
Incorrect Options:
Option A - Botulism:
• Botulism is a rare and potentially fatal illness caused by a toxin produced by the bacterium Clostridium
botulinum. The disease begins with weakness, blurred vision, feeling tired, and trouble speaking. This
may then be followed by weakness of the arms, chest muscles, and legs. Vomiting, swelling of the
abdomen, and diarrhea may also occur.
Option C - Zigadenus poisoning:
• Zigadenus (commonly known as "death camas" or "mountain camas") is a common plant in the lily
family found throughout the United States. Ingestion may cause hemodynamic instability which has
successfully been treated with atropine.
• Zigadenus poisoning causes vomiting, hypotension, and bradycardia. The hemodynamic instability
may be treated with atropine administration and dopamine infusion.
Option D - Cobra bite:
• Cobra venom is faint transparent yellow and is slightly viscous. When exposed to the sun, it becomes
slightly turbid.
• Local symptoms start within 6 to 8 minutes. A small reddish wheal develops at the site of bite. The
bitten area is tender with slight radiating burning pain and oozing of bloodstained fluid.
• Systemic symptoms appear after about 30 minutes. The patient feels sleepy, slightly intoxicated,
weakness of legs, and is reluctant to stand or move. Nausea and vomiting are sometimes the early
symptoms. Ptosis is the earliest neuroparalytic manifestation followed by ophthalmoplegia. There may
be extraocular muscle weakness and strabismus.
• Weakness of the muscles increases and develops into paralysis of the lower limbs. The paralysis then
spreads to the trunk and affects the head which falls forward.
• In cases of recovery, the skin and cellular tissues surrounding the bite mark undergo necrosis.

Page 13

321
Solution for Question 10:
Correct Option D - Podophyllin:
• Podophyllin is a resin extracted from the roots of Podophyllum peltatum/mayapple plant (American
mandrake) and Podophyllum emodi, which contains numerous compounds, amongst which is
podophyllin (as well as the drug podophyllotoxin).
• Podophyllin is the principal active component.

Other options
Option A - Tetrahydro cannabinol:
• Cannabis sativa (marijuana/marihuana/ hashish): It is not an alkaloid, but a fat-soluble oleoresin,
cannabinol, the active form being δ-9-tetrahydrocannabinol (THC). It also contains benzopyrene, a
known carcinogen that is also found in tobacco.
• Tetrahydrocannabinol (THC) is the principal psychoactive constituent of cannabis.
Option B - Thevetin:
• Thevetins are a group of poisonous cardiac glycosides. They are obtained especially from the seeds
of a West Indian shrub or small tree (Cascabela thevetia/ Thevetia nereifolia) of the dogbane family
(Apocynaceae).
Option C - Oduvin:
• Oduvan (Cleistanthus collinus): It is a small, deciduous tree with spreading, smooth branches. Leaves
are orbicular, broadly ovate or elliptical, with rounded tips.
• All parts of the plant are poisonous. Extract of the various plant parts yields a multitude of compounds
of which the glycosides, arylnaphthalene lignin lactones are toxic. These lignan lactones include
cleistanthin A and B, collinusin, and diphyllin, which in the past were known collectively as “oduvin”.

Page 14

322
Solution for Question 11:
Correct Option A - Ricin:
• The entire plant is poisonous, containing toxalbumin ricin, a water-soluble glycoprotein, and a
powerful allergen. Seeds contain the highest level. They are also rich in purgative oil. Castor oil is not
poisonous as it does not contain ricin.
• The toxin has been linked with terrorist activity and is commonly used as part of immunotoxins for
clinical tumor research and application in cancer therapy.
• Ricinus communis (Plant and seeds):

Incorrect Options:
Option B - Abrin:
• Abrus Precatorius (Rati, Gunchi, Jequirity): Seeds contain active principles, abrin, a thermolabile
toxalbumin; abrine, an amino acid; hemagglutinin, a lipolytic enzyme; and abralin, a glycoside.

Page 15

323
Option C - Croton:
• Croton Tiglium (Jamalgota): The processed seeds are used in Indian medicine for treating flatulence,
dyspepsia, colic, edema, dyspnea and persistent cough.
• All parts are poisonous, but seeds contain the maximum concentration of the active principles. Crotin,
a toxalbumin and crotonoside, a glycoside are the active principles.
• Croton tiglium (seeds)

Option D - Chilly:
• Capsicum or chilli fruits are universally employed as a condiment, the powdered form being known as
red pepper or lal mirch.
• Active Principles: Capsaicin and capsicin, are exceedingly acrid, volatile, non-alkaloidal, and non-fatal
substances.

Solution for Question 12:


Correct Option D - Bhilawanol:
• Bhilawanol is an active principle of semecarpus anacardium.
• Semicarpus anacardium: The fruit of this plant is known as ‘marking nut’ or ‘bhilawa’ as its juice is
used by washerman/laundries to inscribe identification number on the clothes.
• Active Principles: Semecarpol and bhilawanol.
Incorrect Options:
Option A/ B/ C - Calotoxin/ Calactin/ Uscharin:
• Active principles of calotropis: Uscharin, Calotoxin and Calotropin

Page 16

324
Solution for Question 13:
Correct Option B - Semicarpus anacardium:
• Semicarpus anacardium: The fruit of this plant is known as ‘marking nut’ or ‘bhilawa’ as its juice is
used by washerman/laundries to inscribe identification number on the clothes.
• Active Principles: Semecarpol and bhilawanol.
• Bruise-like lesion with small blisters may be seen near the angle of the mouth or lips. Blisters are also
seen in the mouth and throat.
• Semecarpus anacardium (seeds)

Incorrect Options:
Option A - Strychnos nux vomica:
• Strychnos Nux-vomica (Kuchila): It is used as a respiratory stimulant, rodenticide, and for killing stray
dogs. Strychnine is still available as herbal and homeopathic remedies, as a purgative, appetite
suppressant, and as a constituent of nerve tonics. It can be found as an adulterant in some street drugs
(cocaine, heroin, and amphetamines).
Option C - Abrus precatorius:
• Abrus Precatorius (Rati, Gunchi, Jequirity): Seeds contain active principles, abrin, a thermolabile
toxalbumin; abrine, an amino acid; hemagglutinin, a lipolytic enzyme; and abralin, a glycoside.
• Abrus precatorius is commonly used as cattle poison in Indian villages. The toxic principle is injected
into the animal in the form of fine needle-shaped structures called ‘suis'. Seeds are also used as
abortifacient and as an arrow poison.
Option D - Capsicum annum:
• Capsicum or chilli fruits are universally employed as a condiment, the powdered form being known as
red pepper or lal mirch.

Page 17

325
• Active Principles: Capsaicin and capsicin, are exceedingly acrid, volatile, non-alkaloidal, and non-fatal
substances.
• It may be thrown into the eyes to facilitate robbery. The powder is used as a means of torture to extort
money or a confession of some guilt by introducing it into the eyes, urethra, vagina or rectum, burning it
under the nose or rubbing it on the female breasts.
• Superstitious people use the fumes from burning chillis to scare away devils and ghosts.

Solution for Question 14:


Correct Option D - Abrus precatorius:
• Given image is of ABRUS PRECATORIUS, which resembles viperine snake venom.
• Active principle: Abrine (most potent).
• Abrus precatorius (red & white seeds)

Incorrect Options:
Option A - Cannabis:
• Cannabis sativa (marijuana/marihuana/ hashish): It is not an alkaloid, but a fat-soluble oleoresin,
cannabinol, the active form being δ-9-tetrahydrocannabinol (THC). It also contains benzopyrene, a
known carcinogen that is also found in tobacco.
• It is used in the following forms: Bhang, Majoon, Ganja, and Charas or hashish.
Option B - Datura:
• Datura, a member of the Solanaceae family, the fruits are spherical and have sharp spines known as
thorn-apple.
• All parts of these plants are poisonous - Fruit, Flowers and Seeds (highest concentrations of alkaloids
are found in roots and seeds).

Page 18

326
• Active Principles: The plant contains belladonna alkaloids whose primary actions are anticholinergic.
Hyoscine (scopolamine). The major active principle is hyoscine. Hyoscyamine Atropine
• Hyoscine (scopolamine). The major active principle is hyoscine.
• Hyoscyamine
• Atropine
• Hyoscine (scopolamine). The major active principle is hyoscine.
• Hyoscyamine
• Atropine
Option C - Erythroxylum coca:
• Cocaine is obtained from the leaves of Erythroxylum coca, the leaves contain about 0.5 to 1%
cocaine. It is a colourless, odourless, crystalline substance with a bitter taste. It contains alkaloids
ecgonine, hygrine and cinnamyl cocaine. It is used as a local anaesthetic.

Solution for Question 15:


Correct Option B - Ricin:
• The given image is castor seed also known as Ricinus communis.
• It contains an active principle- Ricin, which leads to intense gastroenteritis and bloody diarrhea.
• Ricinus communis (Plant and seeds)

Page 19

327
Incorrect Options:
Option A - Croton:
• Croton Tiglium (Jamalgota): The processed seeds are used in Indian medicine for treating flatulence,
dyspepsia, colic, edema, dyspnea and persistent cough.
• All parts are poisonous, but seeds contain the maximum concentration of the active principles. Crotin,
a toxalbumin and crotonoside, a glycoside are the active principles.
Option C - Abrin:
• Abrus Precatorius (Rati, Gunchi, Jequirity): Seeds contain active principles, abrin, a thermolabile
toxalbumin; abrine, an amino acid; hemagglutinin, a lipolytic enzyme; and abralin, a glycoside.
Option D - Capsicin:
• Capsicum or Chilli fruits are universally employed as a condiment, the powdered form being known as
Red Pepper or Lal Mirch.
• Active Principles: Capsaicin and capsicin, are exceedingly acrid, volatile, non-alkaloidal, and non-fatal
substances.

Page 20

328
Corrosives
1. A 45-year-old man with a history of Schizophrenia presented to the emergency after ingesting a large
glass of a pesticide diluted with water in response to auditory hallucinations. He was alert and in no
apparent distress but had persistent hypotension and tachycardia that was unresponsive to fluid
resuscitation. On day 2 a reddish rash appeared on palms, soles, buttocks, and scrotum. It was
documented as "Boiled lobster rash". This is seen in cases of poisoning with?
(or)
In which poisoning Boiled lobster rash is seen ?
A. Sulphuric acid
B. Boric acid
C. Carbolic acid
D. Formic acid
----------------------------------------
2. The forensic pathologist is examining the body of a 43-year-old man who has allegedly committed
suicide by drinking Lysol, which the doctor is aware is a corrosive poison. They act by any of the
following methods, except?
(or)
Corrosive poisons act by any of the following methods, except?
A. Free radical oxidative injury
B. Extracting water from tissues
C. Coagulation of proteins
D. Convert haemoglobin to hematin
----------------------------------------
3. Which among the following is false about the toxicity of this poison is:
(or)
Which among the following is false about the hydrofluoric acid toxicity ?

A. Severe pain on contact is the characteristic feature


B. Causes Liquefactive necrosis
C. Causes Decalcification and destruction of bone

329
D. Result in hypercalcemia
----------------------------------------
4. In this poisoning, the vomitus is in which colour?
(or)
What is the colour of vomitus in boric acid poisoning?

A. Bluish green
B. Black
C. Brown
D. Red velvety
----------------------------------------
5. A 45-year-old patient presents to the emergency department with symptoms of weakness, tetany,
and convulsions after ingesting a substance containing Oxalic acid. Their ECG shows a prolonged QT
interval and variable conduction defects. What antidote should be used to counteract the effects of
Oxalic acid poisoning and its associated hypocalcemia?
(or)
What is the antidote for Oxalic acid poisoning ?
A. B.A.L
B. Animal charcoal
C. Calcium gluconate
D. Magnesium
----------------------------------------
6. A 23-year-old woman has succumbed to the injuries she sustained when sulfuric acid was splashed
on her face and into her mouth by a stalker. During the autopsy, the stomach was removed after a
double ligature. The doctor noticed that the stomach was perforated and had a black charred
appearance. All the following can also be seen in this case, except?
(or)
Which of the following findings would NOT be expected in the case of a 23-year-old woman who
succumbed to sulfuric acid ingestion after being attacked by a stalker?
A. Dryness of mouth
B. Damaged tongue

Page 2

330
C. Chalky white teeth
D. Swollen tongue with white coating
----------------------------------------
7. Which one of the following poisons can be detected even from burnt bones.
(or)
Which one of the following poisons can be detected from burnt bones.
A. OPC
B. LSD
C. Cyanide
D. Arsenic
----------------------------------------
8. A 50-year-old woman was admitted to critical care due to a drug overdose leading to coma, with the
overdose substance being a long-acting drug causing central nervous system depression, respiratory
failure, and hemodynamic instability. An IV infusion of sodium bicarbonate is administered as a part of
treatment plan. which of the following drug would be the causative agent for this condition?
(or)
In which type of poisoning "Alkaline diuresis" is a necessary management strategy?
A. Phencyclidine
B. Phenobarbitone
C. Amphetamine
D. Morphine
----------------------------------------
9. 20-year-old woman who ingested a rust-removing solution in a suicidal attempt presents with severe
pharyngeal pain, hoarse voice, swollen and blackish lips and tongue, and chalky white teeth .what is
the suspected causative agent?
(or)
What is the suspected causative agent in a 20-year-old woman who ingested a rust-removing solution,
leading to severe pharyngeal pain, swollen and blackish lips and tongue, and chalky white teeth?
A. Carbolic acid
B. Oxalic acid
C. Nitric acid
D. Sulphuric acid
----------------------------------------
10. The following plant contains which of the following the toxic substance?
(or)
Indian spinach contains which of the following the toxic substance?

Page 3

331
A. Atropine
B. Cannabis
C. Ricin
D. Oxalic acid
----------------------------------------
11. A 35-year-old man, while etching glass in a glass factory, accidentally was exposed to acid which
was not readily noticed or painful. The patient began to experience pain that gradually increased in
intensity at the site of exposure but there were still no visible signs of a burn. When he described this to
the doctor on duty at the factory, the doctor immediately applied 2.5% Calcium gluconate gel on the
site. Which acid has the man been exposed to?
(or)
Which acid exposure is characterized by delayed pain without visible burns and 2.5% calcium
gluconate gel used as a treatment?
A. Sulphuric acid
B. Hydrofluoric acid
C. Nitric acid
D. Carbolic acid
----------------------------------------
12. Gastric lavage is contraindicated in all the following poisonings except:
(or)
Gastric lavage is contraindicated in all the following poisonings except:
A. OPC
B. Sulphuric acid
C. Kerosene
D. Nitric acid
----------------------------------------

Correct Answers

Page 4

332
Question Correct Answer

Question 1 2
Question 2 1
Question 3 4
Question 4 1
Question 5 3
Question 6 4
Question 7 4
Question 8 2
Question 9 4
Question 10 4
Question 11 2
Question 12 1

Solution for Question 1:


Correct Option B - Boric acid:
• Boiled lobster’s rash is seen in poisoning with Boric acid.
Incorrect Options:
Option A - Sulphuric acid:
• Sulphuric acid is most commonly used for Vitriolage (Vitriol Throwing).
• Discoloration and staining of the skin and clothing (brown or black in sulphuric acid, and yellow in
nitric acid).
• Poison having local action: Sulphuric acid.
Option C - Carbolic acid:
• Poisoning by carbolic acid is known as carbolism.
• Chronic poisoning (phenol/carbol marasmus): It is characterized by anorexia, progressive weight loss,
excess production of saliva, headache, vertigo, dark urine, and pigmentation of skin and sclera
(ochronosis). It was a common occupational disorder of physicians and their assistants during the mid
19th century when carbolic acid sprays were commonly used for antisepsis in operating rooms.
• Gastric lavage can be done in poisoning with Carbolic acid.
• Fatal dose: 1–2 gms of phenol, or 25–50 ml of household phenol. Fatal period: 3–4 hrs.
Option D - Formic acid:
• Action: It has a corrosive action on (coagulation necrosis) on G.I. mucosa. It causes
hemolysis leading to acute renal failure. ATP synthesis is diminished.
• Fatal Dose: 50 to 200 ml.
• Postmortem Appearances: Corrosion and blackening of gastric mucosa and pulmonary oedema.

Page 5

333
• Poisoning is suicidal or accidental.

Solution for Question 2:


Correct Option A - Free radical oxidative injury:
• Corrosive poisons do not act by Free radical oxidative injury.

Incorrect Options:
Option B/ C/ D
- Extracting water from tissues/ Coagulation of proteins/ Convert hemoglobin to hematin:
• A corrosive poison fixes, destroys, and erodes the surface with which it comes in contact.
• They act by extracting water from the tissues, coagulating cellular proteins, and converting
hemoglobin into hematin.

Solution for Question 3:


Correct Option D - Result in hypercalcemia:
• After absorption, fluoride ions bind calcium ions and to some extent potassium and magnesium ions
and cause hypocalcaemia, hypokalaemia and hypomagnesaemia.
Incorrect Options:
Option A - Severe pain on contact is the characteristic feature: Hydroflouric acid burns results in bone
destruction, and excruciating pain (on contact is the characteristic feature).
Option B - Causes Liquefactive necrosis: Hydrofluoric acid causes liquefaction necrosis; they have no r
emote action.
Option C - Causes Decalcification and destruction of bone: Hydroflouric acid burns cause decalcificatio
n, bone destruction, and excruciating pain.

Solution for Question 4:


Correct Option A - Bluish green:
• Emesis and diarrhea may be bluish green in Boric acid poisoning.
Other options
Option B
- Black -The appearance comes from old and coagulated blood in your gastrointestinal tract. It's a
sign of internal bleeding.
Option C - Brown - This option is incorrect

Page 6

334
Option D - Red velvety - It is the redness caused by an excess of blood in the mucous membrane of ca
rdiac end of stomach. It is a deep crimson coloration, sometimes may be patchy or diffused and color o
f mucus membrane of stomach is velvety in arsenic poisoning.

Solution for Question 5:


Correct Option C - Calcium gluconate:
• The antidote for oxalate poisoning is any preparation of calcium that converts the poison into insoluble
calcium oxalate. e.g. lime water, calcium lactate, calcium gluconate, calcium chloride, or a suspension
of chalk in water or milk.

Incorrect Options:
Option A - B.A.L: B.A.L: Antidote for mercury, lead, and arsenic poisoning.
Option B - Animal charcoal: Mercury Poisoning: Egg whites, milk, or animal charcoal to precipitate mer
cury. Emesis is not induced because of the risk of serious corrosive injury.
Option D: Magnesium: Calcium gluconate is the antidote for oxalate poisoning, not magnesium.

Solution for Question 6:


Correct Option D - Swollen tongue with white coating:
• The lips are usually swollen and excoriated and brown or black streaks may be found.
• There is corrosion of mucous membranes of mouth, throat and esophagus, immediate burning pain,
stridor, drooling, odynophagia and dysphagia.
• Tongue becomes swollen, sodden, and black.
• The abdomen becomes distended and very tender.

Incorrect Options:
Option A/ B/ C - Dryness of mouth/ Damaged tongue/ Chalky white teeth:
• Dryness of mouth, the damaged tongue can be seen, teeth are chalky-white in Sulphuric acid
poisoning.

Solution for Question 7:


Correct Option D - Arsenic:
• Bone, hair, and nails are preserved in Heavy metals poisonings like arsenic, antimony, and thallium.

Incorrect Options:

Page 7

335
Option A/B/C: OPC/ LSD/ Cyanide
• These poisons cant be detected in the burnt bones.

Solution for Question 8:


Correct Option B - Phenobarbitone:
• The clinical scenario point toward diagnosis of phenobarbitone overdose for which alkaline diuresis is
a part of treatment strategy
• Alkaline diuresis enhances the excretion of chlorpropamide, fluoride, methotrexate, phenobarbital,
and salicylate.
Incorrect Options:
Option A - Phencyclidine:
• Phencyclidine/PCP is a white, crystalline powder or a clear, yellowish liquid. It is used recreationally
as a psychedelic and hallucinogen.
• Effects are usually dose-dependent, and onset is rapid when smoked or injected (1-5 min) and are
delayed when snorted or ingested (30 min).
Option C - Amphetamine:
• Liquid gold (slang for urine of amphetamine addicts sold on the streets): A significant proportion of
ingested amphetamine is excreted unchanged in the urine, consumption of which is an economical way
of amphetamine intake.
• Recently, there has been a resurgence of amphetamine use with the availability of ‘designer’
amphetamines, like MDMA (3, 4-methylenedioxy-methamphetamine; street name: Ecstasy or XTC).
Combining ‘ecstasy’ with psilocybin mushrooms is called ‘hippy flipping’.
• Amphetamine is one of the drugs included in the ‘dope test’ for athletes.
Option D - Morphine:
• Routes of administration: It can be taken by snorting, smoking or chasing (chasing the dragon),
intravenously (mainlining), and subcutaneously (skin popping). It can be mixed with cocaine (known as
speed balling) and then taken by addicts.
• Peak effects are seen in 10 minutes (min) with IV route, 10–15 min after nasal insufflations, 30–45
mins with IM, 90 mins after taking orally and 2–4 hours after dermal application.
• Fatal dose: Opium: 2 g; Morphine: 200 mg.11; Codeine: 50 mg.
• The most important dependence-producing derivatives are morphine and heroin. Heroin is more
addicting than morphine and can cause dependence after a short period of exposure. Tolerance to
heroin occurs rapidly and can be increased to more than 100 times the first dose needed.

Solution for Question 9:


Correct Option D - Sulphuric acid:

Page 8

336
• In Sulphuric acid poisoning: The voice becomes hoarse and husky. The lips are usually swollen and
blackish. Teeth are chalky-white. The tongue becomes swollen, sodden, and black.
• Burns in the mouth and throat
Incorrect Options:
Option A - Carbolic acid:
• Poisoning by carbolic acid is known as carbolism. Phenol being fat-soluble, attacks the nervous
system and causes paralysis of respiratory and CVS centers leading to death.
• Grayish or brownish corrosions at the angle of the mouth, chin, front of the body, arms and hands with
phenolic odor.
Option B - Oxalic acid:
• Fulminating poisoning: Intake of large dose (>15 g) produces immediate symptoms and death within
minutes. There is a burning, sour or bitter taste in the mouth with a sense of constriction around the
throat and burning pain from the mouth to the stomach, radiating all over the abdomen.
• Acute poisoning: All findings are due to hypocalcemia-tingling and numbness of fingers and limbs,
weakness, paresthesia, carpopedal spasm, hyperirritability of peripheral nerves (Chvostek/Weiss sign),
tetany, convulsions, coma, and death. There may be dilated pupils, metabolic acidosis, ventricular
fibrillation, and renal failure.
• Delayed poisoning: It is characterized by nephritis-uremia, scanty urine, hematuria, albuminuria,
oxaluria.
Option C - Nitric acid:
• They are those of poisoning by sulphuric acid. It causes yellow discoloration of the tissues, including
the crowns of the teeth, and yellow stains on the clothing. Inhalation of fumes causes lachrymation,
photophobia, irritation of air passages and lungs producing sneezing, coughing, dyspnoea, and
asphyxia.
• The Colour of urine is brown.

Solution for Question 10:


Correct Option D - Oxalic acid:
• The above Image is a plant, Indian spinach, which contains oxalic acid.
• Oxalic acid and Indian Spinach Plant:

Page 9

337
Incorrect Options:
Option A - Atropine:
• A poisonous compound found in deadly nightshade and related plants. It is used in medicine as a
muscle relaxant, e.g., in dilating the pupil of the eye.
• Atropine occurs naturally in a number of plants of the night shade family, including deadly nightshade
(belladonna), Jimson weed, and mandrake.
• The antidote to atropine is physostigmine or pilocarpine.

Page 10

338
(A) Dhatura fruit (thorn apple) and leaves; (B) Datura and capsicum seeds
Option B - Cannabis:
• CANNABIS SATIVA OR INDICA is also known as Indian hemp. The active principles are contained in
its resin.
• Bhang is the mildest of cannabis concoctions. It consists of dried cannabis leaves that are ground to a
fine paste, mixed with a combination of sugar, spices and fruit.
• Hashish is a highly potent, concentrated cannabis resin that has been collected, dried and pressed
into bricks.
• Charas is the handmade form of hashish.
• Marijuana is the dried leaves and flowers of the cannabis plant, and most common form of drug in the
US. It is usually smoked, although it is occasionally baked into foods such as brownies or brewed as
tea for drinking.
• Majum: Sweetmeat made with bhang.
• Cannabis (fresh and dried leaves):

Page 11

339
Option C - Ricin:
• Ricinus Communis (Castor): The entire plant is poisonous, containing toxalbumin ricin, a
water-soluble glycoprotein, and a powerful allergen. Seeds contain the highest level.
• They are also rich in purgative oil. Castor oil is not poisonous as it does not contain ricin.
• Action: Ricin blocks protein synthesis through inhibition of RNA polymerase. It belongs to a group of
poisons known as A-B toxins. Ricin has a special binding protein that gains access to the endoplasmic
reticulum in the GIT mucosal cells causing diarrhea.
• Ricin blocks protein synthesis through inhibition of RNA polymerase. It belongs to a group of poisons
known as A-B toxins.
• Ricin has a special binding protein that gains access to the endoplasmic reticulum in the GIT mucosal
cells causing diarrhea.
• Ricin is commonly used as part of immunotoxins for clinical tumor research and application in cancer
therapy.
• Ricinus communis (seeds):
• Ricin blocks protein synthesis through inhibition of RNA polymerase. It belongs to a group of poisons
known as A-B toxins.
• Ricin has a special binding protein that gains access to the endoplasmic reticulum in the GIT mucosal
cells causing diarrhea.

Page 12

340
Solution for Question 11:
Correct Option B - Hydrofluoric acid:
• Local application of 2.5% calcium gluconate gel is the treatment of choice in hydrofluoric acid burn.
• For hydrofluoric acid burn, soak the area in benzalkonium chloride solution or apply 2.5% calcium
gluconate gel.

Incorrect Options:
Option A - Sulphuric acid:
• Fatal Dose: 5 to 10 ml; Fatal Period: 12 to 24 hours.
• Complications: Acute: Upper airway obstruction and injury, G.l hemorrhage, Oesophageal and gastric
perforation, Sepsis, Tracheobronchial necrosis, atelectasis, and obstructive lung injury. Chronic:
Oesophageal obstruction, Pyloric stenosis, Vocal cord paralysis with airway obstruction.
• Acute: Upper airway obstruction and injury, G.l hemorrhage, Oesophageal and gastric perforation,
Sepsis, Tracheobronchial necrosis, atelectasis, and obstructive lung injury.
• Chronic: Oesophageal obstruction, Pyloric stenosis, Vocal cord paralysis with airway obstruction.
• Treatment in VITRIOLAGE (vitriol throwing): Wash the affected parts with plenty of water and soap or
sodium or potassium carbonate. Later, a thick paste of magnesium oxide or carbonate is applied. The
eyes are washed with water and irrigated with a dilute sodium bicarbonate solution. Later, a few drops
of olive oil or castor oil are instilled into the eyes.
• Acute: Upper airway obstruction and injury, G.l hemorrhage, Oesophageal and gastric perforation,
Sepsis, Tracheobronchial necrosis, atelectasis, and obstructive lung injury.
• Chronic: Oesophageal obstruction, Pyloric stenosis, Vocal cord paralysis with airway obstruction.
Option C - Nitric acid:

Page 13

341
• In concentrated form it combines with organic matter and produces an yellow discolouration of tissue
due to the production of Picric acid (Xanthoproteic reaction).
• Fatal Dose: 10 to 15 ml; Fatal Period: 12 to 24 hours.
• Treatment is the same as for sulphuric acid.
• Test: The test is for the presence of nitrates. If strong ferrous sulfate solution and sulphuric acid are
added to a solution containing nitric acid, a brown ring is formed at the junction of the two fluids.
Option D - Carbolic acid:
• Fatal Dose: 10 to 15 gms; Fatal Period: Three to four hours.
• Poisoning by carbolic acid is known as carbolism.
• Treatment: Stomach is washed carefully with plenty of lukewarm water containing charcoal, olive oil,
MgSO4 or Na2SO4. Medicinal liquid paraffin or 30 g of MgSO4 may be left in the stomach after the
lavage.

Solution for Question 12:


Correct Option A - OPC:
• Gastric lavage is not contraindicated in OPC Poisoning.

Incorrect Options:
Option B/ C/ D - Sulphuric acid/ Kerosene/ Nitric acid:
• Gastric lavage is absolutely contraindicated in Sulphuric acid, Kerosene and Nitric acid.
• Corrosive poisoning (except carbolic acid) owing to the danger of perforation (absolute
contraindication).
• Volatile poisons and hydrocarbons (petroleum distillate and kerosene oil) which may cause aspiration
pneumonitis.

Page 14

342
Delirients, Cardiac & Spinal Poisons
1. Excess use of cannabis will cause degeneration of the central nervous system leading to significant
symptoms like all of the following, except?
(or)
Cannabis use lead to following symptoms ,except ?
A. Hashish insanity
B. Run amok
C. Flashback phenomenon
D. McEwan's sign
----------------------------------------
2. A 28-year-old woman is rushed to the emergency room after her roommate discovers her
unconscious at home. Her vital signs are unstable, with a depressed respiratory rate, and she exhibits
signs of central nervous system depression.The medial team decides to use scandinavian method for
treatment of this patient. Which of the following substance was ingested by her?
(or)
Scandinavian method is used in treatment of :
A. Barbiturate poisoning
B. Alcohol withdrawal syndrome
C. Benzodiazepine poisoning
D. Cocaine abuse
----------------------------------------
3. A 25-year-old woman has been brought in for admission in a semiconscious state. She was found by
her roommate in her hostel room with some bottles of alcohol and some drugs by her side. On
observation, the patient's hands were seen moving in a pill-rolling pattern. Occasionally she was also
threading imaginary needles.This is seen in which of the following?
(or)
Among which of the following Carphologia is seen ?
A. Datura poisoning
B. Digitalis poisoning
C. LSD overdose
D. Alcohol withdrawal
----------------------------------------
4. In the summer of 2023, a group of friends decide to attend the highly anticipated Enchanted Music
Festival, known for its magical atmosphere and diverse musical lineup.One among them faints and is
brought to the emergency by his friends. On examination, There was a peculiar smell coming from the
patient and his friends like a burned rope" odor. Which one among the following is being described
here?
(or)

343
Which of the following substance have characteristic 'burned rope odor '?
A. Phosphorus
B. Hydrocyanic acid
C. Nitrobenzene
D. Marijuana
----------------------------------------
5. A 22-year-old male was admitted to the Emergency Room in a coma after consuming poison, 2
hours ago. The patient presented with fever, tachycardia and in a comatose state. Rapid sequence
induction and intubation were performed immediately, with sedation and assisted-control mechanical
ventilation, after being transferred to the ICU. The biological assessment revealed rhabdomyolysis and
prevention of renal failure was initiated. The antidote used was Physostigmine. Poisoning by which of
the following is being referred here?
(or)
Physostigmine is used as an antidote for which of the following?
A. Calotropis
B. Ricinus communis
C. Datura
D. Croton
----------------------------------------
6. The HR of a company sent a notice to a 28 year old man who was absent quite frequently over the
past month. He used to be a confident and alert employee until recently, when he started complaining
of insects crawling beneath his skin. One of his friends also noticed that his pupils were constantly
dilated. Which of the following conditions could this employee have?
(or)
Which of the following condition presents with sensation of insects crawling underneath the skin
A. Parkinsonism
B. Chronic depression
C. Chronic cocaine poisoning
D. Acute barbiturate poisoning
----------------------------------------
7. Match the following columns: A. Heroin 1. Gunchi B. Spanish fly 2. Blister beetle C. Abrus 3. Blue
rocket D. Aconite 4. Brown sugar
A. Heroin 1. Gunchi
B. Spanish fly 2. Blister beetle
C. Abrus 3. Blue rocket
D. Aconite 4. Brown sugar

A. A-1, B-2, C-3, D-4


B. A-4, B-2, C-1, D-3

Page 2

344
C. A-4, B-2, C-3, D-1
D. A-3, B-4, C-2, D-1
----------------------------------------
8. In traditional Chinese medicine, the roots of the plant are used only after processing to reduce the
toxic alkaloid content. The use of a larger than recommended dose and inadequate processing
increases the risk of poisoning. A characteristic finding in the case of poisoning with this plant toxin is
the presence of "Hippus sign", where the pupils alternately contract and dilate. Which one of the
following is being described here?
(or)
In which type of poisoning, "Hippus sign" is observed?
A. Opium
B. Curare
C. Aconite
D. Datura
----------------------------------------
9. A 32–year–old woman presents to ER with history of ingestion of crushed plant seed shown in the
picture.crushed plant seed as shown in the picture . She was treated with stomach wash and activated
charcoal.Which of the following is not caused by this?
(or)
In Yellow oleander plant poisoning, Which o the following is not commonly seen

A. Bradycardia
B. Tachycardia
C. Miosis
D. Hyperkalemia
----------------------------------------
10. Choose the incorrect statement regarding the following :
(or)
Choose the incorrect statement regarding Aconite/blue rocket.

Page 3

345
A. Causes AV block
B. Atropine is the antidote
C. Only root is poisonous
D. Blue rocket
----------------------------------------
11. In a rural community, A 45-year-old man is brought to the emergency room by his family. They
describe sudden and severe muscle spasms, jaw stiffness, and generalized convulsions as shown in
the image that started shortly after Jack consumed a liquid The family suspects accidental poisoning.
What of the following is the reason for this condition?
(or)
In which of the following Opisthotonus is seen ?
A. Phosphorus
B. Snake venom
C. Strychnine
D. Arsenic
----------------------------------------
12. Which of the following is not true about the poisoning caused by the seeds shown below?
(or)
Which of the following is not true about Kuchila poisoning?

A. Pleurosthotonus

Page 4

346
B. Post mortem caloricity
C. Uncrushed seeds will not lead to poisoning
D. Inhibits neurotransmitter glycine effect at the pre-synaptic receptor
----------------------------------------

Correct Answers
Question Correct Answer

Question 1 4
Question 2 1
Question 3 1
Question 4 4
Question 5 3
Question 6 3
Question 7 2
Question 8 3
Question 9 3
Question 10 3
Question 11 3
Question 12 4

Solution for Question 1:


Correct Option D - McEwan's sign:
• McEwan's sign is due to alcohol intoxication.
• Other options A, B and C are seen in Chronic cannabis Poisoning.
Incorrect Options:
Option A - Hashish insanity:
• Rarely, cannabis users become insane and may suffer from auditory and visual hallucinations and
delusions of persecution.
Option B - Run amok:
• Running amok or run amok: He develops a psychic disturbance marked by a period of depression,
followed by violent attempts to kill people (impulse to murder).
Option C - Flashback phenomenon:
• Recurrence of experience of cannabis use i.e., a person experiences the features of cannabis use
without actually using it.
• Flashback phenomenon is also seen in the abuse of hallucinogens LSD and psilocybin, psilocin.

Page 5

347
Solution for Question 2:
Correct Option A - Barbiturate poisoning:
• Scandinavian method is used in the treatment of Barbiturate poisoning.
Incorrect Options:
Option B - Alcohol withdrawal syndrome:
• Detoxification: Treatment of alcohol withdrawal symptoms, i.e., symptoms produced by removal of the
'toxin' alcohol. The most common withdrawal syndrome is a hangover.
• Treatment: The drugs of choice are benzodiazepines. Chlordiazepoxide (80–200 mg/day in divided
doses) and diazepam (40–80 mg/day in divided doses) may be used. Vitamin B complex is also added.
Option C - Benzodiazepine poisoning:
• Flumazenil is the antidote for benzodiazepine poisoning.
Option D - Cocaine abuse:
• If it has been taken by mouth, gastric lavage should be performed with warm water containing
potassium permanganate, charcoal, or tannic acid. If applied to the nose or throat: Wash out the
mucous membrane with water.
• If injected, apply a ligature above the part. Cocaine intoxication is usually treated with
benzodiazepines
• Amyl nitrite is the antidote and is given by inhalation.
• Airway and circulatory stabilization.
• Thiamine 100 mg IV.; Naloxone hydrochloride 2 mg IV
• The symptoms should be treated on general lines.

Solution for Question 3:


Correct Option A - Datura poisoning:
• Carphologia is seen in Datura poisoning.
• Typical pill-rolling movements, Patients hallucinate imaginary needles and threads
• Other signs and symptoms: Dry mouth, irrelevant slurred speech, dry and hot skin, dilated pupils.
• Treatment: Physostigmine is the drug of choice.
Incorrect Options:
Option B - Digitalis poisoning:
• Toxic symptoms are due to overdose or by a cumulative action.
• In Digitalis poisoning: Nausea, vomiting, pain in the abdomen, burning sensation, diarrhea,
Bradycardia, CNS Headache, fatigue, confusion, anxiety, depression, disorientation, drowsiness,
hallucinations, delirium, Transient amblyopia, blurring, photophobia, Urticaria can be seen.

Page 6

348
• The patient becomes drowsy and the condition may deepen into coma. Convulsions may precede
death. Death occurs from cardiovascular collapse
Option C - LSD overdose:
• Signs and symptoms (Acute intoxication) of LSD: Somatic or physical: Dizziness, dilated pupils,
nausea, flushing, hyperthermia, paresthesia, hyperactive reflexes and tremors. Perceptual: Altered
changes in vision and hearing, like floating feeling, illusions, sensation of synesthesia, i.e., 'seeing'
smells and 'hearing' colors. Psychic or changes in sensorium: Delusional ideation, body distortion,
suspiciousness to the point of toxic psychosis, depersonalization, and loss of sense of time.
• Somatic or physical: Dizziness, dilated pupils, nausea, flushing, hyperthermia, paresthesia,
hyperactive reflexes and tremors.
• Perceptual: Altered changes in vision and hearing, like floating feeling, illusions, sensation of
synesthesia, i.e., 'seeing' smells and 'hearing' colors.
• Psychic or changes in sensorium: Delusional ideation, body distortion, suspiciousness to the point of
toxic psychosis, depersonalization, and loss of sense of time.
• Somatic or physical: Dizziness, dilated pupils, nausea, flushing, hyperthermia, paresthesia,
hyperactive reflexes and tremors.
• Perceptual: Altered changes in vision and hearing, like floating feeling, illusions, sensation of
synesthesia, i.e., 'seeing' smells and 'hearing' colors.
• Psychic or changes in sensorium: Delusional ideation, body distortion, suspiciousness to the point of
toxic psychosis, depersonalization, and loss of sense of time.
Option D - Alcohol withdrawal:
• Tremulousness or shakes or jitters (most common sign), weakness, pain in the muscle, cold sweat,
insomnia, loss of appetite, vomiting, diarrhea, restlessness, exaggerated reflexes, raised temperature,
fluctuating BP, hallucinations, loss of memory, and delirium tremens.

Solution for Question 4:


Correct Option D - Marijuana :
• Cannabis
• Most common illicit drug abuse.
• Burnt rope - Marijuana (Cannabis)
Cannabis
Incorrect Options:
Option A - Phosphorus:

• It is a protoplasmic poison with a garlicky Odor.


• Also known as Diwali Poison.
• Fatal dose- 60-120 mg.
Option B - Hydrocyanic acid:

Page 7

349
• Produced in apple, apricot, peach, plum, almond, and linseed.
• Liquid cyanide → Hydrocyanic acid or Prussic acid.
• Produces a Bitter almond smell.
• Ideal for suicidal poisoning

Option C - Nitrobenzene:
• It is used as an intermediate in the synthesis of solvents, like paint remover
• Produces a Shoe polish odor - Nitrobenzene
• Treatment:intra-venous 1% methylene blue

Solution for Question 5:


Option C: Datura
• Signs and Symptoms:Dryness in skin and mouth Dysphagia - Di■culty in swallowin, Dilated pupil
(very earliest symptom), Drunken Gait, Dysarthria - Slurred speech,Muttering Delirium,Carphologia
• The antidote for Datura poisoning is Physostigmine.
Option A: Calotropis
• Calotropis (Rubber Bush): The patient is treated symptomatically; gastric lavage is done with warm
water, demulcents, and morphine to relieve pain.
Option B: Ricinus communis
• In case of Ricinus communis ingestion: Gastric lavage/ stomach wash. Emetics and demulcents.
Administration of glucose and saline for dehydration. 2–5 gm of sodium bicarbonate is given 8 hours by
mouth to alkalinize the urine. Blood transfusion may be needed in some patients.
• Gastric lavage/ stomach wash.
• Emetics and demulcents.
• Administration of glucose and saline for dehydration.
• 2–5 gm of sodium bicarbonate is given 8 hours by mouth to alkalinize the urine.
• Blood transfusion may be needed in some patients.
• Gastric lavage/ stomach wash.
• Emetics and demulcents.
• Administration of glucose and saline for dehydration.
• 2–5 gm of sodium bicarbonate is given 8 hours by mouth to alkalinize the urine.
• Blood transfusion may be needed in some patients.
Option D: Croton
• Croton Tiglium (Jamalgota): Gastric lavage/ stomach wash. Administration of demulcent drinks, like
milk or egg white. Morphine with atropine to allay pain and reduce intestinal secretions. Glucose and
saline are given IV to combat collapse and dehydration.

Page 8

350
• Gastric lavage/ stomach wash.
• Administration of demulcent drinks, like milk or egg white.
• Morphine with atropine to allay pain and reduce intestinal secretions.
• Glucose and saline are given IV to combat collapse and dehydration.
• Gastric lavage/ stomach wash.
• Administration of demulcent drinks, like milk or egg white.
• Morphine with atropine to allay pain and reduce intestinal secretions.
• Glucose and saline are given IV to combat collapse and dehydration.

Solution for Question 6:


Correct Option C - Chronic cocaine poisoning:
• Magnan's sign or cocaine bugs is characteristic of Chronic cocaine poisoning, in which there is a
feeling as if grains of sand are lying under the skin or some small insects are creeping on the skin
giving rise to an itching sensation (formication, tactile hallucination).
• The individual exhibits a heightened pulse rate, rapid and deep respirations, dilated pupils, a
headache, pale skin, cyanosis, increased sweating, and an elevated body temperature
Incorrect Options:
Option A - Parkinsonism:
• Parkinson’s disease can cause varying and progressive symptoms throughout its course. Some of the
most common symptoms associated with the disease include: Difficulty showing facial expressions
Muscle stiffness Slowed, affected movements Speech changes Tremor, especially of one hand
• Difficulty showing facial expressions
• Muscle stiffness
• Slowed, affected movements
• Speech changes
• Tremor, especially of one hand
• Difficulty showing facial expressions
• Muscle stiffness
• Slowed, affected movements
• Speech changes
• Tremor, especially of one hand
Option B - Chronic depression:
• Dysthymia (Chronic Depression): A mood disorder consisting of the same cognitive and physical
problems as in depression, with less severe but longer-lasting symptoms.
• patients feel like a gloomy black cloud is hanging overhead and do not remember what it felt like to not
be depressed.

Page 9

351
Option D - Acute barbiturate poisoning:
• Barbiturate poisoning: The affected person demonstrates shallow breathing, a profound state of
unconsciousness, lack of response to painful stimuli, diminished reflexes, below-normal body
temperature, low blood pressure, a weak and faint pulse, and dialted pupils

Solution for Question 7:


Correct Option B - A-4, B-2, C-1, D-3:
A. Heroin: Brown sugar
B. Spanish Fly: Blister beetle
C. Abrus: Gunchi
D. Aconite: Blue rocket

Solution for Question 8:


Correct Option C - Aconite:
• In Aconite poisoning: The pupils alternately contract and dilate (hippus) but remain dilated in the later
stages.
Incorrect Options:
Option A - Opium:
• The contact of morphine with the skin of sensitive persons may cause erythema, urticaria and itching
dermatitis.
• When opium is taken by mouth, symptoms begin within half hour. If the drug is injected, its action is
noted within 3 or 4 minutes. It first stimulates, then depresses and finally paralyses the nerve centres.
Option B - Curare:
• Curareis found in various species of strychnos. Curarine is the active principle. Its action is entirely
peripheral and at the myoneural junction, and blocks the postsynaptic nicotinic acetylcholine receptors
in muscles, thus causing a flaccid paralysis of skeletal muscles.
• It causes gradual paralysis of limbs followed by paralysis of respiratory muscle and death from
asphyxia. There is headache, vertigo, mydriasis, blurred vision and hypotension due to the liberated
histamine.
Option D - Datura:
• Datura: A member of the Solanaceae family, the fruits are spherical and have sharp spines known as
thorn-apple.
• All parts of these plants are poisonous: Fruit, Flowers, and Seeds (highest concentrations of alkaloids
are found in roots and seeds).
• Active Principles: The plant contains belladonna alkaloids whose primary actions are anticholinergic.
Hyoscine (scopolamine). The major active principle is hyoscine. Hyoscyamine Atropine

Page 10

352
• Hyoscine (scopolamine). The major active principle is hyoscine.
• Hyoscyamine
• Atropine
• Dryness of mouth, dysphagia, dilated pupils, dry; hot skin, drunken gait, delirium, drowsiness, death
due to respiratory failure can be seen.
• Hyoscine (scopolamine). The major active principle is hyoscine.
• Hyoscyamine
• Atropine

Solution for Question 9:


Correct Option C:Miosis
• Yellow Oleander Also called Cerebra Thevetia or peela kaner. All parts of the plant are poisonous.
Active Principles Cerebrin Thevetoxin A and B Nerifolin Miosis is not usually seen in this poisoning
• Also called Cerebra Thevetia or peela kaner.
• All parts of the plant are poisonous.
• Active Principles Cerebrin Thevetoxin A and B Nerifolin
• Cerebrin
• Thevetoxin A and B
• Nerifolin
• Miosis is not usually seen in this poisoning
• Also called Cerebra Thevetia or peela kaner.
• All parts of the plant are poisonous.
• Active Principles Cerebrin Thevetoxin A and B Nerifolin
• Cerebrin
• Thevetoxin A and B
• Nerifolin
• Miosis is not usually seen in this poisoning
• Cerebrin
• Thevetoxin A and B
• Nerifolin
Incorrect Options:
OptionA,B D :Hyperkalemia:
• The given plant is yellow oleander(thevetia peruviana), contains thevetin (cardiotoxic): And produces
both tachyarrhythmia and bradyarrhythmia.

Page 11

353
• It causes both hyperkalemia and hypokalemia, but more dangerous and life-threatening is
hyperkalemia which needs to be managed quickly. Hyperkalemia is managed by insulin and dextrose
solution.

Solution for Question 10:


Correct Option C - Only root is poisonous:
• It is also called blue rocket/ monks hood/ meetha zeher
• Roots are conical in shape and tapered at the end with longitudinal ridges.
• All parts are poisonous.
• Roots and seeds are the most poisonous part. Roots of Aconite are generally mistaken as
Horseradish root
Incorrect Options:
Option A - Causes AV Block: Aconite causes tachycardia in early stages: Cardiac arrhythmia with AV b
lock occurs.
Option B
- Atropine is the antidote: Atropine can be given as an antidote if the person develops heart block.
Option D - Blue rocket: The given picture is of Aconite and is also known as blue rocket.
The given picture is of Aconite/blue rocket:

Page 12

354
Solution for Question 11:
Correct Option C - Strychnine:
• The image shows Opisthotonus which is a characteristic feature of Strychnine poisoning.
• Strychnos nux vomica (kuchila): Strychnine is a powerful spinal stimulant.
• Opisthotonus is due to powerful extensor spasm causing the body to be hyperextended with arching
of the back.
Incorrect Options:
Option A - Phosphorus:
• Signs and Symptoms of acute phosphorus poisoning: Garlicky odor; Luminous vomit and stool
because of the presence of phosphorus. Fumes emanate from the stools (smoking or smoky stool
syndrome). Acute poisoning causes liver cell necrosis, fatty liver, and acute yellow atrophy.
• Garlicky odor; Luminous vomit and stool because of the presence of phosphorus.
• Fumes emanate from the stools (smoking or smoky stool syndrome). Acute poisoning causes liver cell
necrosis, fatty liver, and acute yellow atrophy.
• Chronic Phosphorus Poisoning (Phossy Jaw/Glass Jaw/Lucifer’s Jaw): Due to inhalation of fumes
over years. Toothache (earliest symptom) & swelling of the jaw; Loosening of the teeth & Necrosis of
the lower jaw. Sequestration of bone in the mandible & multiple sinuses discharging foul-smelling pus.
• Due to inhalation of fumes over years. Toothache (earliest symptom) & swelling of the jaw; Loosening
of the teeth & Necrosis of the lower jaw. Sequestration of bone in the mandible & multiple sinuses
discharging foul-smelling pus.
• Garlicky odor; Luminous vomit and stool because of the presence of phosphorus.
• Fumes emanate from the stools (smoking or smoky stool syndrome). Acute poisoning causes liver cell
necrosis, fatty liver, and acute yellow atrophy.

Page 13

355
• Due to inhalation of fumes over years. Toothache (earliest symptom) & swelling of the jaw; Loosening
of the teeth & Necrosis of the lower jaw. Sequestration of bone in the mandible & multiple sinuses
discharging foul-smelling pus.
Option B - Snake venom:
• It can be neurotoxic, vasculotoxic, or myotoxic in its action.
• Physical appearance: Faint transparent yellow and viscous, when fresh.
• Onset of symptoms and sudden progression is more common with Elapidae bite rather than
Viperidae. Most cobra, krait, and sea snake bites would show symptoms within the first 6 hrs, the
shortest time is for the sea snakes.
• Local signs and symptoms: Fang marks, pain, bleeding, bruising, lymphangitis, lymph node
enlargement, inflammation, blistering, local infection, abscess formation, and necrosis.
Option D - Arsenic:
• In chronic arsenic poisoning: Nutritional and gastrointestinal disturbances can be seen.
• There is classical "Raindrop Pigmentation".
• Hyperkeratosis of palms and soles can occur.
• Mees' line (white transverse bands crossing the nails).
• Chronic exposure can also cause "Black foot disease".

Solution for Question 12:


Correct Option C - Toxin potentiates neurotransmitter glycine effect at pre-synaptic receptor:
• The above given image is of Strychnine Seeds aka Dog Buttons
• It is used to kill stray dogs, hence called Dog-buttons. Seeds shape is concavo-convex. It is odorless
and bitter in taste. Fatal dose: One crushed seed
• Seeds shape is concavo-convex.
• It is odorless and bitter in taste.
• Fatal dose: One crushed seed
• Toxin potentiates neurotransmitter glycine which does not effect the pre synaptic receptor.
• Seeds shape is concavo-convex.
• It is odorless and bitter in taste.
• Fatal dose: One crushed seed
Incorrect Options:
Option A - Pleurosthotonus:
• Pleurosthotonus: Lateral flexion of the spine, which is a feature of Strychnine poisoning.
Option B - Post mortem caloricity:

Page 14

356
• After death ‘Postmortem caloricity’ phenomenon occurs i.e., initially after death, the patient remains
warm for few hours.
Option C - Uncrushed seeds will not lead to poisoning:
• Unbroken seeds when ingested are not poisonous, as the hard pericarp is not soluble in digestive
juices.

Page 15

357
Cerebral Poisons
1. The police have tracked down a gang that was distributing drugs among the college students in the
city. During the raid, large quantities of heroin were seized. Heroin is the semisynthetic form of which
one of the following?
(or)
Heroin is the semisynthetic form of which one of the following?
A. Cocaine
B. Morphine
C. Datura
D. Cannabis
----------------------------------------
2. A 23-year-old male develops nausea, vomiting, and ascending paralysis on ingestion of a poison.
The poisoning is due to:
(or)
What poison causes nausea, vomiting, and ascending paralysis in a patient?
A. Datura
B. Strychnos nux vomica
C. Conium Maculatum
D. Opium
----------------------------------------
3. Mescaline has been recognized by the regional Indian tribes as a cause of significant somatic
discomfort, including abdominal pain, nausea, vomiting, diarrhea, sweating, as well as pleasant
hallucinations and other psychic phenomena. It has recently been adopted as a recreational chemical
by certain drug users All the following statements are true of Mescaline, except?
(or)
All the following statements are true of Mescaline, except?
A. Obtained from Mexican peyote cactus
B. Contains four alkaloids
C. Used in a drink called mescal buttons
D. Causes "boiled lobster" rash
----------------------------------------
4. The ratio between ethyl alcohol in blood and Vitreous humor is
(or)
The ratio between ethyl alcohol in blood and Vitreous humor is
A. 1:1.1
B. 1:1.2
C. 1:1.3

358
D. 1:1.8
----------------------------------------
5. A 30-year-old male is brought by his wife to the emergency department. On examination, there
respiratory depression, and miosis. His wife also admits that he takes fentanyl without a prescription. All
of the following can be given as a treatment for this patient except?
(or)
All of the following are treatments of opium poisoning, except?
A. Stomach wash
B. Purgatives
C. Naloxone
D. Digitalis
----------------------------------------
6. What is the appropriate treatment for a 39-year-old carpenter who has consumed two bottles of
liquor from a local shop, and now, after about an hour, is experiencing symptoms like confusion,
vomiting, and blurred vision, upon arrival in the emergency department?
(or)
What is the appropriate treatment for a symptoms of confusion, vomiting, and blurred vision after
consuming two bottles of liquor ?
A. Naloxone
B. Diazepam
C. Flumazenil
D. Ethyl alcohol
----------------------------------------
7. A 21-year-old white woman was referred to a hospital after exposure to CO from a faulty furnace.
She developed acute weakness, dyspnea, nausea, and vomiting. An ECG revealed sinus tachycardia,
non-specific ST-T wave abnormalities, and a prolonged QTc interval. The chest X-ray revealed
pulmonary edema and the 2-D echo revealed decreased LV systolic function with an EF of 25%. The
treatment for this woman, among the following, would be?
(or)
What is the appropriate treatment for a 21-year-old woman with CO poisoning presenting with acute
weakness, dyspnea, non-specific ECG changes, pulmonary edema, and reduced left ventricular
function?
A. Ascorbic acid
B. Intravenous methylene blue
C. 100% Oxygen
D. None of the above
----------------------------------------
8. Choose the correct statement regarding the occurrence of respiratory failure: Respiratory failure
occurs with 40–50% carboxyhemoglobin concentration in the blood during carbon monoxide poisoning.
Respiratory failure occurs with 70–80% CO2 concentration in carbon dioxide poisoning.

Page 2

359
A. Both statements are true
B. Both statements are false
C. Statement 1 is true, Statement 2 is false
D. Statement 2 is true, Statement 1 is false
----------------------------------------
9. In cases of cyanide poisoning, with what should the blood specimen be covered to prevent
evaporation?
(or)
What should be used to cover a blood specimen in cyanide poisoning to prevent evaporation?
A. Saturated salt solution
B. Formalin
C. Liquid paraffin
D. Acetone
----------------------------------------
10. First, add a few crystals of ferrous sulfate to 5 ml of gastric aspirate. Then, add 5 drops of 2%
sodium hydroxide. Boil the mixture and then allow it to cool. Afterward, add 10 drops of 10%
hydrochloric acid. If the solution turns a 'greenish-blue color,' this indicates the presence of cyanide in
the aspirate. What is the name of this bedside test?
(or)
What is the name of the bedside test that detects the presence of cyanide in gastric aspirate,
characterized by a 'greenish-blue color' resulting from a series of chemical reactions?
A. Marquis Test
B. Lee Jones Test
C. Marsh Test
D. Mydriatic Test
----------------------------------------
11. A 38-year-old chemist works at a research and development facility that specializes in
manufacturing chemicals for various industries. One fateful day, an industrial incident occurs in the
facility, leading to the release of toxic gases. She was immediately rushed to the emergency
department.On examination, the skin & mucosa appears cherry red in colour along with , bitter almond
smell from breath and frothy discharge from mouth. Which toxic substance would have caused this?
(or)
What could be the cause of death in a patient with cherry red skin and mucosa, bitter almond breath
odor, and frothy discharge?
A. H2S poisoning
B. CO poisoning
C. HCN poisoning
D. OPC poisoning
----------------------------------------

Page 3

360
Correct Answers
Question Correct Answer

Question 1 2
Question 2 3
Question 3 4
Question 4 2
Question 5 4
Question 6 4
Question 7 3
Question 8 2
Question 9 3
Question 10 2
Question 11 3

Solution for Question 1:


Correct Option B -Morphine:
• Heroin is diacetylmorphine: A semisynthetic form of opium.
Incorrect Options:
Option A - Cocaine:
• Cocaine (crack, pasta, bazooka, snuff, coke, snow, or white lady) is a colorless, odorless, crystalline
substance with the bitter taste and slightly soluble in water, but freely soluble in alcohol.
• Cocaine is an alkaloid deliriant, obtained from dried leaves of Erythroxylum coca.
Option C - Datura:
• Datura or Thorn-apple: All parts of these plants are poisonous - Fruit, flowers, and seeds (highest
concentrations of alkaloids are found in roots and seeds).
• Active Principles: The plant contains belladonna alkaloids whose primary actions are anticholinergic.
Hyoscine (scopolamine): The major active principle is hyoscine. Hyoscyamine Atropine
• Hyoscine (scopolamine): The major active principle is hyoscine.
• Hyoscyamine
• Atropine
• Hyoscine (scopolamine): The major active principle is hyoscine.
• Hyoscyamine
• Atropine
Option D - Cannabis:

Page 4

361
• Cannabis sativa (Marijuana/ Marihuana/ Hashish): It is not an alkaloid, but a fat-soluble oleoresin,
cannabinol, the active form being δ-9-tetrahydrocannabinol (THC). It also contains benzopyrene that is
formed due to combustion of organic matter.
• It is used in the following forms: Bhang, Majoon, Ganja, and Charas or Hashish.
• Active principles of cannabis are contained in its resin.
• Tetrahydrocannabinol (THC) is the principal psychoactive substance of cannabis.

Solution for Question 2:


Correct Option C - Conium Maculatum:
• Conium Maculatum (Hemlock): Ingestion causes burning in the mouth and throat, gastric
inflammation, vomiting, diarrhea, slow respiration, increased and later slow pulse, mental confusion,
tremors, ataxia, sometimes blindness, progressive motor paralysis extending upwards from the
extremities, coma, and death from respiratory paralysis.
• Progressive muscular paralysis due to depression of the motor nerves can be seen. The lower limbs
are affected first, and the paralysis ascends till the muscles of respiration are affected.
Incorrect Options:
Option A - Datura:
• Datura/ Thorn apple poisoning (9 D's): Dry hot skin; Dilatation of cutaneous blood vessels causing
facial flush; Dilated pupil, Dryness of the mouth; Delirium; Drowsiness; Drunken gait; Dysphagia;
Difficulty in talking can be seen.
• Earliest symptom is a bitter taste in the mouth.
• Treatment: Physostigmine is the drug of choice.
Option B - Strychnos nux vomica:
• Signs and symptoms of strychnine are seen within 15–30 minutes (min) of ingestion. A 'conscious'
seizure is the characteristic presentation of strychnine poisoning.
• Prodromal symptoms: Restlessness, increased acuity of perception, increased rigidity of muscles and
muscular twitchings.
• Risus sardonicus or risus caninus results from raising of patient’s eyebrows, bulging of eyes and
contraction of the jaw and facial muscles in which the corners of the mouth are drawn back leading to
an evil-looking grin. This expression can be seen in tetanus too.
• Convulsions are most marked in anti-gravity muscles resulting in hyperextension (opisthotonus)
• Sometimes, the spasm of the abdominal muscles may bend the body forward (emprosthotonus) or
sideways (pleurothotonus).
• Barbiturates, like pentobarbital sodium or sodium amytal are antidotes for strychnine poisoning.
Option D - Opium:
• Morphine/ heroin intoxication: Stage of excitement: Hallucination, Euphoria, etc. Stage of stupor:
Headache, Nausea, Vomiting giddiness, Drowsiness, Stupor. Stage of coma: Pin-point non-reacting
pupil, Hypotension, Cyanosis, Bradycardia, etc.
• Stage of excitement: Hallucination, Euphoria, etc.

Page 5

362
• Stage of stupor: Headache, Nausea, Vomiting giddiness, Drowsiness, Stupor.
• Stage of coma: Pin-point non-reacting pupil, Hypotension, Cyanosis, Bradycardia, etc.
• CHRONIC POISONING (Morphinism; Morphinomania): The mechanism of tolerance is not known but
is thought to reside at a cellular level. The habit is acquired by young people as it is believed to be an
aphrodisiac and as it produces a sense of euphoria. Opium addicts can tolerate 3 to 6 grams per day.
• IV Naloxone is the drug of choice, followed by oral naltrexone for maintenance therapy.
• Stage of excitement: Hallucination, Euphoria, etc.
• Stage of stupor: Headache, Nausea, Vomiting giddiness, Drowsiness, Stupor.
• Stage of coma: Pin-point non-reacting pupil, Hypotension, Cyanosis, Bradycardia, etc.

Solution for Question 3:


Correct Option D - Causes "boiled lobster" rash:
• Mescaline is not associated with causing a skin rash resembling a "boiled lobster" rash. Skin rashes
are not typically reported as a common effect of mescaline use.
• Boric acid Poisoning causes a Boiled Lobster syndrome Erythema formation and desquamation of
skin.
• Erythema formation and desquamation of skin.
• Erythema formation and desquamation of skin.
Incorrect Options:
Option A- Obtained from Mexican peyote cactus: Mescaline is primarily obtained from the Mexican pey
ote cactus (Lophophora williamsii) and is one of the key psychoactive compounds found in the cactus.
Option B - Contains four alkaloids: Mescaline is the primary alkaloid found in peyote cactus, but there a
re also other alkaloids present in the cactus, such as hordenine.
Option C - Used in a drink called mescal buttons: Mescaline is typically consumed by ingesting the pey
ote cactus itself, not in a drink called "mescal buttons." The term "mescal buttons" may be a
reference to the small, button-like growths on the peyote cactus that contain mescaline.

Solution for Question 4:


Correct Option B - 1:1.2:
• The ratio between ethyl alcohol in the blood and Vitreous humor is 1:1.2

Solution for Question 5:


Correct Option D - Digitalis:

Page 6

363
• Digitalis is not included in the treatments of opium poisoning. Digitalis refers to medications derived
from the foxglove plant and is used to treat heart conditions like heart failure and certain arrhythmias. It
is not indicated for the treatment of opioid (opium) poisoning.
Incorrect Options:
Option A - Stomach wash:
• Stomach wash frequently with 1:5000 KMnO■ leaving some solution in the stomach to oxidize the
alkaloid that might be secreted in the stomach after absorption. Lavage should be carried out even after
IV/IM injection of a drug, as it is secreted in the stomach.
Option B - Purgatives:
• Enema with 30 gm of sodium sulfate twice daily.
• Whole-bowel irrigation in body packers.
Option C - Naloxone:
• Antidote: Narcotic antagonist naloxone in an initial dose of 0.4–2 mg IV/IM repeated every 2–3 min up
to 10 mg, if no response occurs.
• Narcotic antagonists: Naltrexone, Naloxone, Haloperidol, Clonidine, and Cyclazocine.

Solution for Question 6:


Correct Option D - Ethyl alcohol:
• The clinical features, especially ocular features like blurring of vision are suggestive of methyl alcohol
poisoning. Moreover, Methyl alcohol is commonly used to adulterate ethyl alcohol.
Incorrect Options:
Option A - Naloxone:
• Naloxone rapidly reverses an opioid overdose. It is an opioid antagonist.
• It attaches to opioid receptors and reverses and blocks the effects of other opioids.
• Naloxone can quickly restore normal breathing to a person if their breathing has slowed or stopped
because of an opioid overdose.
Option B - Diazepam:
• It is a benzodiazepine that works by enhancing the activity of certain neurotransmitters in the brain.
• Diazepam is used to treat anxiety disorders or alcohol withdrawal symptoms. It is sometimes used
with other medications to treat muscle spasms and stiffness, or seizures.
Option C - Flumazenil:
• Flumazenil is a specific benzodiazepine antagonist to treat intentional overdose of benzodiazepine.
• To reverse sedation and respiratory depression after a surgery or medical procedure in which a
benzodiazepine was used as a sedative.

Solution for Question 7:

Page 7

364
Option C: 100% Oxygen
• Maintain patent airway, fresh air, and orthobaric oxygen (high-flow or 100% oxygen at atmospheric
pressure) Oxygen therapy is started if COHb > 10% and should be given for 4–6 hours.
Incorrect options
Option A: Ascorbic acid
• It is given as an antidote for poisoning caused by a fatal dose of mercury
Option B: Intravenous methylene blue
• It is given as a treatment for cyanide poisoning

Solution for Question 8:


Correct Option B - Both statements are false.:
1. 40–50 % of carboxy haemoglobin in blood in carbon monoxide poisoning
• In CO poisoning, respiratory failure occurs at 70–80% of carboxyhemoglobin in blood.
2. 70–80 % CO2 Concentration in carbon dioxide poisoning
• In CO2 poisoning, respiratory failure occurs at 40–50% of CO2 concentration in blood.
Incorrect Option - A/C/D

Solution for Question 9:


Correct Option C - Liquid paraffin:
• In cyanide poisoning, the specimen of blood must be covered with liquid paraffin to avoid evaporation.
Incorrect Options:
Option A - Saturated salt solution: Saturated salt solution cannot be used as preservative for corrosive
acids, alkalis, corrosive sublimate and aconite poisoning
Option B - Formalin: Formalin exposure can lead to fragmentation of DNA.
Option D - Acetone: Acetone is a better preservative For DNA analysis than Ethanol.

Solution for Question 10:


Correct Option B - Lee Jones Test:
• The Lee Jones Test is a bedside test used to detect the presence of cyanide in gastric aspirate.
• Proceedure: Add a few crystals of ferrous sulfate to 5 ml of gastric aspirate. Add 5 drops of 2%
sodium hydroxide. Boil the mixture and then allow it to cool. Finally, add 10 drops of 10% hydrochloric
acid.
• Add a few crystals of ferrous sulfate to 5 ml of gastric aspirate.

Page 8

365
• Add 5 drops of 2% sodium hydroxide.
• Boil the mixture and then allow it to cool.
• Finally, add 10 drops of 10% hydrochloric acid.
• If the solution turns a "greenish-blue color," it indicates the presence of cyanide in the gastric aspirate.
This test is a rapid and simple way to assess whether cyanide poisoning may be a factor in a patient's
condition.
The Lee Jones Test is a bedside test used to detect the presence of cyanide in gastric aspirate.
Proceedure:
• Add a few crystals of ferrous sulfate to 5 ml of gastric aspirate.
• Add 5 drops of 2% sodium hydroxide.
• Boil the mixture and then allow it to cool.
• Finally, add 10 drops of 10% hydrochloric acid.
If the solution turns a
"greenish-blue color," it indicates the presence of cyanide in the gastric aspirate. This test is a
rapid and simple way to assess whether cyanide poisoning may be a factor in a patient's condition.
Incorrect Options:
Option A - Marquis Test:
• The Marquis Test is a different chemical test used to identify the presence of various drugs, including
opiates and amphetamines, by using specific reagents. It is not related to the detection of cyanide.
Option C - Marsh Test:
• The Marsh Test is a method used to detect and quantify arsenic in a substance or solution. It is not
used for cyanide detection.
Option D - Mydriatic Test:
• It is used for testing for the presence of toxic alkaloids found in datura plants

Solution for Question 11:


Correct Option C - HCN poisoning:
• Cherry-red color of skin and mucous membranes is said to be characteristic of HCN poisoning.
• Pure acid is a colorless gas with a bitter almond odor.
• Cyanide poisoning signs and symptoms Inhalation→Dizziness, vertigo, and constriction of the throat.
Ingestion→ Nausea, headache, loss of muscular power, hypotension, cardiovascular failure, and
convulsion. Death occurs from respiratory failure.
• Inhalation→Dizziness, vertigo, and constriction of the throat.
• Ingestion→ Nausea, headache, loss of muscular power, hypotension, cardiovascular failure, and
convulsion.
• Death occurs from respiratory failure.

Page 9

366
• Odor of bitter almonds in breath, initially tachypnea and dyspnea, followed by rapid slowing of
respiratory rate with severe respiratory depression and cyanosis.
• Inhalation→Dizziness, vertigo, and constriction of the throat.
• Ingestion→ Nausea, headache, loss of muscular power, hypotension, cardiovascular failure, and
convulsion.
• Death occurs from respiratory failure.
Incorrect Options:
Option A - H2S poisoning:
• H2S is a colorless, transparent gas with smell of rotten eggs.
• Headache, vertigo, nystagmus, weakness, coma, Rhinitis, pneumonia, pulmonary edema, Arrhythmia,
myocardial depression, Lacrimation, photophobia, and conjunctivitis can be seen.
Option B - CO poisoning:
• Most frequent acute symptoms are headache (dull, frontal and continuous), dizziness, weakness,
nausea and confusion.
• On examination, there may be tachycardia, hypertension or hypotension, hyperthermia, flame-shaped
retinal hemorrhages and bright red retinal veins. Classic cherry red skin is rare; pallor is present more
often.
• PM staining in carbon monoxide (CO) is Cherry red discolorization.
Option D - OPC poisoning:
• OPCs and carbamates are one of the most common causes of self-poisoning seen in India. They are
used as insecticides, herbicides, antihelminthics, ophthalmic agents, in chemical industry, and as nerve
gas in chemical warfare.
• OPCs are usually mixed with a solvent aromax, which is responsible for a kerosene-like smell in the
breath and body secretions.

Page 10

367
Agricultural & Miscellaneous Poisons
1. Which of the following substances is not typically characterized by a rotten egg odor?
A. Disulfiram
B. Hydrogen sulphide
C. N–Acetyl cysteine
D. Nitrobenzene
----------------------------------------
2. Choose the correct statement regarding Bipyridyl herbicides: 1. Diquat is only half as toxic as
paraquat. 2. Diquat does not produce pulmonary toxicity
A. Statement 1 is true and statement 2 is false
B. Both statements are false
C. Both statements are true
D. Statement 2 is true and statement 1 is false
----------------------------------------
3. In June 2007, India banned 25 pesticides. On the 'banned' pesticides and formulations list, are aldrin,
Benzene hexachloride, Calcium cyanide, Chlordane, Copper cetoarsenite, Cibromochloropropane,
Endrin, and 19 others. All the following statements about Endrin are true, except?
A. Least toxic organochlorine
B. Also known as plant penicillin
C. Has kerosene like smell
D. Resists putrefaction
----------------------------------------
4. A 50-year-old farmer has been brought unconscious to the emergency after being discovered in the
fields with an empty bottle of an organophosphate compound. The mainstay of medical therapy
includes atropine, pralidoxime (2-PAM), and benzodiazepines . In order to achieve adequate
atropinization quickly, a doubling approach is typically used. Poisoning with an OP compound is
characterized by all of the following, except?
(or)
Poisoning with an OP compound is characterized by all of the following, except?
A. Pulmonary oedema
B. Pinpoint pupil
C. Constipation
D. Vomiting
----------------------------------------
5. Poisoning with Paraquat can cause which one of the following?
A. Skin irritation and rash
B. Gastrointestinal discomfort

368
C. Mild headache
D. Multiple organ failure
----------------------------------------
6. ASSERTION (A): Oximes are the specific Antidotes in Carbamate Poisoning REASON (R): Oximes
reactivate inhibited cholinesterase Which of the following conclusions about assertion and reason are
true?
A. Both the assertion and the reason are correct, and the reason correctly explains the assertion
B. Both Assertion and Reason are correct; Reason is not the correct explanation of Assertion
C. Assertion is ncorrect, Reason is correct
D. Both Assertion and Reason are not correct
----------------------------------------
7. A 30-year-old welder has been admitted with complaints of shortness of breath for the last 73 hours.
Features look like Parkinson's disease and symptoms resemble that of malaria. The zinc level was
elevated. His colleagues informed him that they had all experienced these symptoms early in their
careers but were now free of symptoms. Which statement among the following is not true regarding the
diagnosis?
(or)
Which statement among the following is not true about Metal fume fever?
A. It is not due to Manganese
B. Preventive measures such as working in well-ventilated areas and using appropriate personal
protective equipment (PPE) can help reduce the risk of metal fume fever.
C. The syndrome resembles a flu-like illness which starts 6 to 8 hours after exposure of fumes, with
fever, chills, cough, dyspnoea, cyanosis, myalgia, salivation sweating and tachycardia.
D. Symptoms subside in 36 hours after stoppage of exposure
----------------------------------------
8. An intern has been asked to prepare the list of antidotes for common poisonings. Find the Incorrectly
matched antidote:
A. Copper compounds – Ferric oxide
B. Ethylene glycol – Ethanol
C. Cyanide – Amyl nitrite
D. Opioid – Naloxone
----------------------------------------
9. The day after the Lohri (Boghi) festival, a 45-year-old woman was admitted to the hospital with
complaints of persistent fever with shivering, and a sore throat. After a detailed history and
examination, the doctor diagnosed her with Polymer fume fever. Which of the following causes Polymer
fume fever?
A. Burning of Polytetrafluoroethylene
B. Nitrobenzene poisoning
C. Bagging abuse

Page 2

369
D. Huffing abuse
----------------------------------------
10. Identify the image given below?

A. Opium
B. Nigella sativa
C. Datura
D. Tropican black cana
----------------------------------------
11. Which of the following is a common effect associated with LSD use?
A. Flashback phenomenon
B. Increased dopamine levels
C. Euphoria
D. Tactile Hallucinations
----------------------------------------

Correct Answers
Question Correct Answer

Question 1 4
Question 2 1
Question 3 1
Question 4 3
Question 5 4
Question 6 3
Question 7 1
Question 8 1
Question 9 1
Question 10 2

Page 3

370
Question 11 1

Solution for Question 1:


Correct Option D - Nitrobenzene:
• Rotten egg odour is not characteristic of Nitrobenzene.
• In cases of aniline and nitrobenzene poisoning, a shoe polish or bitter almond odor is typically
detected.
Incorrect Options:
Option A/ B/ C - Disulfiram/ Hydrogen sulphide/ N–Acetyl cysteine: A rotten egg odor can be detected i
n substances such as disulfiram, mercaptans, N-acetyl cysteine, and hydrogen sulphide

Solution for Question 2:


Correct Option : Option C is correct answer as both statements are true
1. Diquat is only half as toxic as paraquat
• Paraquat and diquat belong to the bipyridal class of herbicides; of which paraquat is the most widely
used and has significant toxicity. Diquat is only half as toxic as paraquat.
2. Diquat does not produce pulmonary toxicity
The features of diquat ingestion are similar to that of paraquat. However, an important difference i
s that unlike paraquat, it does not produce pulmonary toxicity

Solution for Question 3:


Correct Option A - Least toxic organochlorine:
• Endrin is the MOST toxic organochlorine.
• It is often referred to as 'plant penicillin' due to its broad-spectrum activity against various insect pests.
• A kerosene-like smell emanating from the mouth and nostrils is a sign of Endrin poisoning. This odor
may be detectable even in decomposed bodies.
• Endrin resists putrefaction and can be
• detected in the viscera quite sometime after death.
Incorrect Options:
Option B/ C/ D - Also known as plant penicillin/ Has kerosene-like smell/ Resists putrefaction:
• Endrin is also called 'plant penicillin' because of its broad spectrum of activity against various insect
pests.

Page 4

371
• Kerosene-like smell from the mouth and nostrils, may be found even in decomposed bodies and
resists putrefaction.
• These statements are true

Solution for Question 4:


Correct Option C - Constipation:
• Poisoning with an organophosphate (OP) compound is not characterized by constipation; rather, it
typically causes diarrhea.
Incorrect Options:
Option A/ B/ D - Pulmonary oedema/ Pinpoint pupil/ Vomiting:
• GIT: Increased salivation, nausea, vomiting, retro-sternal pain, abdominal cramps, diarrhea, fecal
incontinence.
• CVS: Bradycardia, hypertension.
• RS: Rhinorrhea, bronchospasm, bronchorrhea, cough, wheezing, dyspnea.May resemble an attack of
asthma.
• Ocular: Blurred vision, miosis (Pinpoint pupil).

Solution for Question 5:


Correct Option D - Multiple organ failure:
• When Paraquat is ingested, it can cause severe damage to various organs in the body, including the
lungs, kidneys, heart, and liver. These toxic effects can lead to the failure of multiple organ systems.
• Multiple organ failure is the primary cause of death in cases of severe Paraquat poisoning, and it often
results in a very high case-fatality rate.
Incorrect Options :
Option A - Skin irritation and rash: Paraquat primarily affects internal organs when ingested and is not t
ypically associated with significant skin irritation or rash. Dermal exposure may lead to localized skin ef
fects but is generally less severe than ingestion.
Option B - Gastrointestinal discomfort: Gastrointestinal symptoms, such as nausea, vomiting, abdomin
al pain, and diarrhea, are common early signs of Paraquat poisoning after ingestion. However, the con
sequences of severe Paraquat poisoning go beyond gastrointestinal discomfort and progress to life-thr
eatening organ damage.
Option C - Mild headache: Paraquat poisoning is not typically associated with mild headaches. Its toxic
effects primarily target vital organs rather than causing mild or localized symptoms like headaches.

Page 5

372
Solution for Question 6:
Correct Option C - Assertion is incorrect, Reason is correct
ASSERTION (A): Oximes are the specific Antidotes in Carbamate Poisoning
• In cases of carbamate poisoning, atropine is used as the specific antidote
REASON (R): Oximes reactivate inhibited cholinesterase
• Oximes binds to anionic site and reactivate inhibited cholinesterase, remove the block at
neuromuscular junction, prevent formation of phosphorylated enzyme and directly detoxify
organophosphates.

Solution for Question 7:


Correct Option A - Option A - It is not due to Manganese:
• Metal fume fever, also known as "welder's fever" or "brass founders' ague," is an occupational health
condition caused by the inhalation of fumes of zinc, copper, magnesium, nickle, mercury, lead. iron,
silver, chromium, cadmium, cobalt, antimony and manganese. The syndrome resembles a flue-like
illness which starts 6 to 8 hours after exposure of fumes, with fever, chills, cough, dyspnoea, cyanosis,
myalgia, salivation, sweating and tachycardia. Symptoms subside in 36 hours after stoppage of
exposure.
Incorrect Options:
Option B - Preventive measures such as working in well-ventilated areas and using appropriate person
al protective equipment (PPE) can help reduce the risk of metal fume fever: Working in well-ventilated
areas and wearing appropriate PPE, such as respirators, can help reduce the risk of exposure to metal
fumes.
Option C - The syndrome resembles a flu-like illness which starts 6 to 8 hours after exposure of fumes,
with fever, chills, cough, dyspnoea, cyanosis, myalgia, salivation sweating and tachycardia: Metal fum
e fever typically presents with flu-like symptoms which usually occur within a
few hours after exposure to metal fumes.
Option D - Symptoms subside in 36 hours after stoppage of exposure: The symptoms of metal fume fe
ver usually resolve within a day or so after exposure to the metal fumes is discontinued.

Solution for Question 8:


Correct Option A - Copper compounds – Ferric oxide
• Potassium ferrocyanide is used as an antidote in cases of poisoning caused by copper compounds
Incorrect Options:
Option B/ C/ D - Ethylene glycol – Ethanol/ Cyanide – Amyl nitrite/ Opioid – Naloxone: Options B, C
and D are correctly matched.

Page 6

373
Solution for Question 9:
Correct Option A- Burning of Polytetrafluoroethylene:
• Polymer fume fever results from inhalation of gases produced by burning of polytetrafluoroethylene
(Teflon).
Incorrect Options:
Option B - Nitrobenzene poisoning: It causes gastric irritation, nausea, vomiting, cyanosis, drowsiness,
seizures, coma and finally respiratory failure culminating in death
Option C/D - Bagging abuse/ Huffing abuse:
• Inhalant drugs are frequently abused by adolescents from poor socioeconomic backgrounds. These
substances are mostly volatile hydrocarbons which are used as solvents, propellants, thinners,and
fuels.
• The hydrocarbon is typically inhaled by
• pouring into a container for “sniffing”, a rag or sock for “huffing”, or a plastic/paper bag for “bagging” .
• Abusers often begin with “sniffing” (lower concentrations), and progress subsequently to “huffing” and
“bagging” (higher levels of exposure).
• The most commonly abused inhalants include toluene from paints and glues; petrol; butane from
cigarette lighter fluids; butyl and isobutyl nitrite; and halogenated hydrocarbons from typewriter
correction fluids, propellants, and dry cleaning fluids.

Solution for Question 10:


Option B: Nigella sattiva
• Nigella sativa is a small black seed that has been used for centuries in herbal medicine.

Option A: Opium

Page 7

374
• Opium (afim) is the dried juice of the poppy (Papaver somniferum).
• Classification: Natural: Morphine, codeine. Semi-synthetic: Heroin, hydromorphone, oxymorphone,
oxycodone. Synthetic: Meperidine, methadone, levarphanol tartrate, paregoric, diphenoxylate, fentanyl,
propoxyphene.
• Natural: Morphine, codeine.
• Semi-synthetic: Heroin, hydromorphone, oxymorphone, oxycodone.
• Synthetic: Meperidine, methadone, levarphanol tartrate, paregoric, diphenoxylate, fentanyl,
propoxyphene.
• The contact of morphine with the skin of sensitive persons may cause erythema, urticaria and itching
dermatitis.
• When opium is taken by mouth, symptoms begin within half hour. If the drug is injected, its action is
noted within 3 or 4 minutes. It first stimulates, then depresses and finally paralyses the nerve centres.
• Natural: Morphine, codeine.
• Semi-synthetic: Heroin, hydromorphone, oxymorphone, oxycodone.
• Synthetic: Meperidine, methadone, levarphanol tartrate, paregoric, diphenoxylate, fentanyl,
propoxyphene.
Option C: Datura
• Datura, a member of the Solanaceae family, the fruits are spherical and have sharp spines known as
thorn-apple.
• All parts of these plants are poisonous - Fruit, Flowers and Seeds (highest concentrations of alkaloids
are found in roots and seeds).
• Active Principles: The plant contains belladonna alkaloids whose primary actions are anticholinergic.
Hyoscine (scopolamine). The major active principle is hyoscine. Hyoscyamine Atropine
• Hyoscine (scopolamine). The major active principle is hyoscine.
• Hyoscyamine
• Atropine
• Hyoscine (scopolamine). The major active principle is hyoscine.
• Hyoscyamine
• Atropine
Option D: Tropican Black Canna
• Botanical Name: Canna ‘Tropicanna Black’
• An extraordinary variety with foliage that is a magnificent combination of dark purple and black
shades; crowned with dazzling scarlet-orange blooms from spring to fall; excellent display when
massed along borders, or as a garden accent.

Solution for Question 11:


Correct Option A -Flashback phenomenon - Flashback phenomenon is seen in LSD use. (re-experienci
ng the past experience).

Page 8

375
Incorrect Options:
Option B - Increased dopamine levels - Increased dopamine levels not associated with LSD use. In LS
D serotonin levels are increased.
Option C -Euphoria - Euphoria is not typically associated with LSD use; instead, it is commonly seen in
individuals using MDMA
Option D -Tactile Hallucinations - Tactile hallucinations are commonly seen in cocaine use, but not in L
SD use. In cases of LSD use, visual hallucinations are typically observed.

Page 9

376
Previous Year Questions
1. Which of the following diagnoses are associated with the hematological picture described below?

A. Minamata disease
B. Saturnism
C. Chronic iron toxicity
D. Arsenic poisoning
----------------------------------------
2. What should be the next step in managing the case of a 23-year-old farm worker from a remote
village who arrived at the hospital 8 hours after being bitten by a snake at 5 in the morning? The patient
presented with a headache, blurred vision, tingling sensation around the mouth, dizziness, and vertigo.
Additionally, he experienced multiple episodes of vomiting since the bite. The patient exhibited signs of
respiratory depression and bradycardia (heart rate less than 50 beats per minute). The local wound
displayed mild swelling and blistering, with no evident bleeding. The patient received 10 vials of
antivenom.
A. Check the blood coagulation profile
B. Neostigmine with atropine
C. Epinephrine injection
D. I.V hydrocortisone
----------------------------------------
3. What is the likely diagnosis for a disoriented 28-year-old male with a heart rate of 50 bpm, blood
pressure of 80/60 mmHg, respiratory rate of 7 breaths/min, temperature of 95 F, and pinpoint pupils?
A. Benzodiazepine overdose
B. Cocaine intoxication
C. Opioid withdrawal
D. Opioid intoxication
----------------------------------------
4. In what poisoning is the presence of cherry red hypostasis observed during a post-mortem
examination?
A. CO Poisoning

377
B. Hydrogen sulphide
C. Yellow phosphorus
D. Hydrogen cyanide
----------------------------------------
5. Which of the following symptoms is NOT typically associated with chronic cocaine abuse?
A. Nasal/palatal perforation.
B. Dilated pupils, rapid pulse and sweating.
C. Black tongue and teeth.
D. Hallucination of insects crawling on the skin (formication).
----------------------------------------
6. What medication is commonly prescribed for the treatment of a 50-year-old factory worker who
presents to the emergency room with symptoms of headache, vomiting, and blurred vision following the
consumption of local alcohol?
A. Fomepizole
B. Flumazenil
C. N-acetyl cysteine
D. Naloxone
----------------------------------------
7. A child before playing consumed fruit from the garden. After some time he developed a high fever,
confusion, photophobia, and unable to urinate. What are the likely causative agent and the appropriate
antidote used in this case?
A. Datura, Pralidoxime
B. Datura, Physostigmine
C. Yellow oleander, Pralidoxime
D. Yellow oleander, Physostigmine
----------------------------------------
8. What could be the possible reason for a patient experiencing seizures, with multiple episodes and
taking herbal medications, who demonstrates arching of the back during these episodes, and remains
conscious in between them?
A. Strychnine
B. Ricinus
C. Nerium odorum
D. Datura
----------------------------------------
9. A farmer was sleeping in the field, and he felt a sting on his leg. He saw something moving away
quickly. He then got drowsy and was taken to the hospital. He developed pain around the site and
continued to bleed profusely from the wound site. The wound became red with blisters. Which of the
following is the most likely cause?

Page 2

378
A. Viper
B. Cobra
C. Wasp bite
D. Scorpion
----------------------------------------
10. Identify the given plant.

A. Datura alba
B. Argemone mexicana
C. Nerium odorum
D. Papaver somniferum
----------------------------------------
11. What is the preferred treatment for acute arsenic poisoning?
A. Ipecac
B. Penicillamine
C. Activated charcoal
D. Dimercaprol
----------------------------------------
12. Which of these is not a cardiac poison?
A. Aconite
B. Cerbera thevetia
C. Nicotiana tabacum
D. Atropa belladonna
----------------------------------------
13. Hooch tragedy is related to:
A. Phosphorus
B. Mercury
C. Ethanol

Page 3

379
D. Methanol
----------------------------------------
14. A male patient was presented with a history of consumption of an unknown substance. He had
increased sweating, lacrimation, diarrhea and bradycardia. What's the management?
A. Atropine
B. Calcium gluconate
C. Pralidoxime
D. Obidoxime
----------------------------------------
15. What manifestations can be observed in individuals suffering from chronic arsenic poisoning? 1.
Raindrop pigmentation 2. Peripheral neuritis 3. Mees line 4. Luminous vomit
A. 1,2,3 are correct
B. 2,3 are correct
C. 1,4 are correct
D. Only 1 is correct
----------------------------------------
16. Which of the following is correct regarding sample collection in as case of alcohol intoxication?
A. Sample area cleaned with spirit
B. Sample area cleaned with soap and water
C. Sample should be preserved without preservative
D. Only one urine sample is required
----------------------------------------
17. A female was given morphine sulfate during labor for pain following which she developed
respiratory distress. Which of the following will be the correct antidote?
A. Naloxone
B. Atropine
C. Pralidoxime
D. Epinephrine
----------------------------------------
18. In which type of metal poisoning is black foot disease accompanied by peripheral neuropathy
observed?
A. Arsenic
B. Lead
C. Mercury
D. Cadmium
----------------------------------------

Page 4

380
19. During a leisure activity in a garden, a man experienced a sudden collapse and was promptly taken
to the hospital. His companion, who had joined him in the garden, informed the medical professionals
that he consumed one of the fruits from the garden, after which he began to exhibit these symptoms.
Additionally, he displayed signs of irritability, restlessness, dry and hot skin, and was unable to urinate
or have a bowel movement. Your task is to determine the type of poison involved and identify the
suitable antidote for it.
A. Datura, Pralidoxime
B. Datura, Physostigmine
C. Yellow oleander, Digoxin
D. Yellow oleander, Physostigmine
----------------------------------------
20. Which of the following symptoms observed in a young teenage boy, who has a history of drug
abuse and received an intravenous cocaine injection at a party, would contradict the probable
diagnosis of cocaine poisoning? The symptoms reported include excessive sweating, headache, and
sudden constriction of the coronary arteries.
A. Myocardial ischemia
B. Bradycardia
C. Hyperthermia
D. Agitation
----------------------------------------
21. What is the primary active component of the poison depicted in the image?

A. Ricin
B. Bhilawanol
C. Abrin
D. Calotropin
----------------------------------------
22. A 72-year-old farmer was presented in the hospital with pinpoint pupil & increased secretions. The
diagnosis is
A. Alcohol poisoning
B. Opioid poisoning

Page 5

381
C. Organophosphorus poisoning
D. Datura poisoning
----------------------------------------
23. A patient displaying a pink hue in their skin and mucosa, emitting a bitter almond odor from their
breath, and experiencing frothy discharge has succumbed to the following cause of death:
A. H2S poisoning
B. CO poisoning
C. HCN poisoning
D. OPC poisoning
----------------------------------------
24. Which poisoning is caused by a bullet retained inside the body ?
(or)
Which poisoning is caused by a bullet retained inside the body ?
A. Iron
B. Phosphorus
C. Nitrocellulose
D. Lead
----------------------------------------
25. Which organ is preserved during an autopsy to investigate suspected poisoning, alongside the
stomach, a portion of the intestine, and the liver?
A. Kidney
B. Lungs
C. Brain
D. Heart
----------------------------------------
26. What is the drug that commonly leads to physical dependence?
A. Ketamine
B. Heroin
C. LSD
D. Phencyclidine
----------------------------------------
27. Which poison can still be detected in skeletal remains of the body?
A. Arsenic
B. Carbon monoxide
C. Hydrogen sulphide
D. Phoshorus

Page 6

382
----------------------------------------
28. Which of the following choices best characterizes the typical symptoms observed in a patient who
has been bitten by the snake depicted in the accompanying image?

A. Non-poisonous, dry bite


B. Severe edema, necrosis and bleeding
C. Severe muscle necrosis, rhabdomyolysis
D. Bleeding, paralysis and amputation
----------------------------------------
29. What is the plant that is regarded as a noxious weed in America, Asia, Africa, and Australia? This
plant is known to cause allergic respiratory problems, contact dermatitis, and mutagenicity in both cattle
and humans. Furthermore, its allelopathic nature greatly decreases crop yield, and its aggressive
dominance poses a threat to biodiversity.
A. Parthenium hysterophorus
B. Datura stramonium
C. Digitalis purpurea
D. Calotropis gigantea
----------------------------------------
30. A young adolescent male arrives at the emergency department after recently engaging in drug
abuse during a party. He reports experiencing symptoms such as excessive sweating, a headache,
feelings of unease, and increased restlessness. Upon examination, the patient displays an elevated
heart rate and high blood pressure and his pulpils are dilated and fixed. Which of the following
substances is likely to have been ingested by the patient?
A. Alcohol
B. Cocaine
C. Opioid
D. Cannabis
----------------------------------------
31. Please rearrange the following options to reflect the correct sequence of symptoms observed in
cases of iron toxicity. Healing with scaring Latent phase Gastrointestinal Irritation Liver failure and
hypoglycemia
A. 2,3,1,4

Page 7

383
B. 2,3,4,1
C. 3,2,4,1
D. 3,4,1,2
----------------------------------------
32. In this case, which poisoning do you suspect if a person employed in a dye factory complains of
symptoms such as nausea, vomiting, dark bloody stools, conjunctivitis, and a burning sensation in the
throat and stomach?
A. Potassium permanganate
B. Arsenic
C. Thallium
D. Lead
----------------------------------------
33. Which of the following withdrawal symptoms can be observed in alcoholic patients after 72 hours of
alcohol cessation?
A. Seizures
B. Delirium tremens
C. TremorsTremors
D. Hallucinations
----------------------------------------
34. A child who consumed an unidentified fruit is brought to your emergency department presenting
with the following indications
(or)
What is the most probable diagnosis for a child with dry mouth, slurred speech, hot and dry skin, dilated
pupils, tachycardia, and hyperthermia after consuming an unknown fruit?
A. Poppy seeds poisoning
B. Datura poisoning
C. Mushroom poisoning
D. Cannabis poisoning
----------------------------------------
35. During the autopsy of a deceased individual who died of starvation, which of the following
observations are likely to be present?
A. Distended gallbladder
B. Enlarged liver
C. Expanded lungs
D. Hypertrophied heart
----------------------------------------
36. In which condition is cherry red post mortem lividity observed?

Page 8

384
A. Opioid poisoning
B. Carbon monoxide poisoning
C. Hydrogen sulphide poisoning
D. Phosphorus poisoning
----------------------------------------
37. A man was found to be lying dead on the ground. His friend saw a snake passing in the area and
took a picture of it and showed it to the doctor (image below). What is the most probable cause of death
of this man?

A. Shock
B. Respiratory failure
C. Coagulopathy
D. Renal failure
----------------------------------------
38. Which medication is administered as a remedy for poisoning when a child mistakenly ingests a
container filled with iron tablets?
A. Luspatercept
B. Deferoxamine
C. Deferiprone
D. DMSA
----------------------------------------
39. What is the likely cause of poisoning in a patient who presents to the emergency department with a
greenish skin color and a foul smell resembling rotten eggs?
A. Carbon monoxide
B. Nitrite
C. Cyanide
D. Hydrogen sulfide
----------------------------------------
40. What poisoning should be considered in the case of a 40-year-old patient who was brought to the
emergency department in an unconscious state by the police after a suspected incident of poisoning?

Page 9

385
The patient presents with needle track marks on the upper limbs, elevated blood pressure, increased
heart rate, and high body temperature.
A. Heroine
B. Cocaine
C. Cannabis
D. LSD
----------------------------------------
41. In the event of poisoning, gastric lavage may be conducted for cases involving:
A. Sulphuric acid
B. Strychnine
C. Carbolic acid
D. Kerosene oil
----------------------------------------
42. Which of the following acts as a physiological antidote to scorpion venom?
A. Physostigmine
B. Norepinephrine
C. Prazosin
D. Atropine
----------------------------------------
43. Which of the following is associated with luminescent stools?
A. Mercury
B. Cadmium
C. Phosphorus
D. Oxalic acid
----------------------------------------
44. On post mortem examination, which of the following toxins result in a bluish discoloration of the
stomach mucosa?
A. Oxalic acid
B. Sodium amytal
C. Soneryl
D. Arsenic
----------------------------------------
45. Saturnine gout is seen in:
A. Lead poisoning
B. Cadmium poisoning
C. Beryllium poisoning

Page 10

386
D. Mercury poisoning
----------------------------------------
46. Which metallic poisoning Mimics cholera
A. Arsenic
B. Chromium
C. Mercury
D. Lead
----------------------------------------
47. Which of the following statements is not true regarding the management of snakebite in India?
A. Atropine and neostigmine are very useful in the management of krait bite
B. Neostigmine and ventilatory support are used in treatment along with ASV
C. ASV is the mainstay of treatment
D. Current ASV in India is not effective against humped pit viper
----------------------------------------
48. Please match the following toxins with their likely mode of action. Zinc chloride Chloral hydrate
Quinine Potassium carbonate Abortifacient Irritant Corrosive Stupefying
Zinc chlorideChloral hydrateQuininePotassium carbonate AbortifacientIrritantCorrosiveStupefying

A. A-2, B-4, C-1, D-3


B. A-4, B-3, C-2, D-1
C. A-4, B-2, C-1, D-3
D. A-2, B-3, C-4, D-1
----------------------------------------
49. Which of the following is incorrect regarding the image shown?

A. Only the roots of this plant are poisonous


B. Causes AV block
C. Atropine is not specific for aconite poisoning
D. Grow in sub Himalayan range
----------------------------------------

Page 11

387
50. Match the following: i. Heroin ii. MDMA iii. Joint iv. LSD Ganja Acid Brown sugar Amphetamine
i. Heroinii. MDMAiii. Jointiv. LSD GanjaAcidBrown sugarAmphetamine

A. i -2, ii-1, iii-4, iv-3


B. i -3, ii-2, iii-1, iv-4
C. i -4, ii-2, iii-1, iv-3
D. i -3, ii-4, iii-1, iv-2
----------------------------------------
51. What is the underlying factor leading to fatality when smoke is inhaled?
A. Histotoxic hypoxia
B. Ischemic hypoxia
C. Anemic hypoxia
D. All of the above
----------------------------------------
52. Please identify the seeds depicted in the image provided.

A. Tropicana canna
B. Nigella sativa
C. Datura seeds
D. Opium seeds
----------------------------------------
53. Can you determine the active toxin that is derived from the toxic plant displayed in the image
below?

Page 12

388
A. Calotropis gigantea
B. Digitalis
C. Ricin
D. 1-hyoscyamine
----------------------------------------
54. What is the probable diagnosis for a 30-year-old man, without any known health issues, who
experienced abdominal pain at the airport and had an x-ray revealing the image displayed below, while
carrying laxatives and an enema apparatus?

A. Bezoar syndrome
B. Pica due to anaemia
C. Body packer syndrome
D. Constipation due to fecalith
----------------------------------------
55. Identify the plant which produces a deliriant toxin.

Page 13

389
A. Erythroxylum coca
B. Datura
C. Hyoscine
D. Digitalis purpura
----------------------------------------
56. Match the following : Krait Cobra Python Viper A. B. C. D.

Page 14

390
A. 1-D; 2-C; 3-A;4-B
B. 1-A; 2-B; 3-C;4-D
C. 1-D; 2-C; 3-B;4-A
D. 1-D; 2-A; 3-B;4-C
----------------------------------------
57. Assertion: Most common hematological finding in case of lead poisoning is eosinophilia Reason:
Lead inhibits ALA dehydratase
A. Both assertion and reason are true and the reason is the correct explanation for the assertion
B. Both assertion and reason are true, but the reason is not the correct explanation for the assertion
C. Assertion is true, but the reason is false
D. Both assertion and reason are false
E. The assertion is false but reason is true
----------------------------------------
58. Which of the following types of toxins has the maximum damage potential?
A. Irritant poison
B. Corrosive poison
C. Alcohol
D. Opioid
----------------------------------------

Page 15

391
59. Match the following poisons and test : Column A Column B Marsh test Silver nitrate paper test
Cavett test Marquis test Opium Aluminium phosphide Cyanide Alcohol Arsenic
Column A Column B
Marsh testSilver nitrate paper testCavett testMarquis test OpiumAluminium phosphideCyanideAlcoholArsenic

A. 1-E; 2-B; 3-A;4-D


B. 1-E; 2-C; 3-B;4-A
C. 1-E; 2-B; 3-D;4-A
D. 1-E; 2-C; 3-B;4-D
----------------------------------------
60. What substance is most likely responsible for the symptoms observed in a 28-year-old man who
presented with dry mouth, urinary retention, and constipation? His pulse rate was 110 beats per minute,
and his pupils were dilated.
A. Heroin
B. Morphine
C. Malathion
D. Belladona
----------------------------------------
61. Which of the following are toxins ? Atropine Colchicine Curare Arsenic
A. All are correct
B. 1,2,3 are correct
C. 1,3 are correct
D. 3,4 are correct
----------------------------------------
62. whether the following statements about digoxin toxicity are true or false: A. Pain in abdomen,
burning sensation, diarrhea seen in digoxin toxicity B. Vision changes like Transient ambylopia,
photophobia, may be noted in digoxin toxicity C. Specific antidote for cardiac arrhythmias is lignocaine
100 mg IV or novocaine or propranolol. D. Death occurs from Respiratory failure .
A. A,B,C
B. D,A,C
C. A,B,D
D. D ,B
----------------------------------------

Correct Answers
Question Correct Answer

Question 1 2
Question 2 2

Page 16

392
Question 3 4
Question 4 1
Question 5 2
Question 6 1
Question 7 2
Question 8 1
Question 9 1
Question 10 2
Question 11 4
Question 12 4
Question 13 4
Question 14 1
Question 15 1
Question 16 2
Question 17 1
Question 18 1
Question 19 2
Question 20 2
Question 21 2
Question 22 3
Question 23 3
Question 24 4
Question 25 1
Question 26 2
Question 27 1
Question 28 4
Question 29 1
Question 30 2
Question 31 3
Question 32 2
Question 33 2
Question 34 2
Question 35 1
Question 36 2
Question 37 2

Page 17

393
Question 38 2
Question 39 4
Question 40 2
Question 41 3
Question 42 3
Question 43 3
Question 44 2
Question 45 1
Question 46 1
Question 47 1
Question 48 1
Question 49 1
Question 50 4
Question 51 3
Question 52 2
Question 53 4
Question 54 3
Question 55 1
Question 56 1
Question 57 5
Question 58 2
Question 59 3
Question 60 4
Question 61 2
Question 62 1

Solution for Question 1:


Correct answer Option B -
• Saturnism, also known as chronic lead poisoning, can cause a variety of hematological abnormalities,
including anemia, basophilic stippling, and the presence of nucleated red blood cells (RBCs) in the
peripheral blood.
• In severe cases, lead poisoning can also lead to bone marrow suppression, which can result in
pancytopenia.
• However, it is important to note that the hematological picture of lead poisoning can vary depending
on the level and duration of exposure, as well as individual factors such as age and nutritional status.

Page 18

394
Incorrect Choice:
• Option a: While it is possible that mercury toxicity may lead to some degree of hematological
abnormalities, such as anemia, the typical hematological picture of Minamata disease does not include
specific features that would clearly distinguish it from other causes of anemia.
• Option c: Chronic iron toxicity can lead to a hematological picture of microcytic hypochromic anemia,
which is characterized by small red blood cells that contain insufficient hemoglobin. While this is a
specific hematological feature that can be seen in chronic iron toxicity, it is not unique to this condition
and can also occur in other forms of anemia.
• Option d: Arsenic poisoning can lead to a variety of hematological abnormalities, including anemia,
leukopenia, and thrombocytopenia. The specific hematological picture may vary depending on the
severity and duration of exposure. However, the presence of anemia alone is not specific enough to
diagnose arsenic poisoning, as anemia can also occur in many other conditions.

Solution for Question 2:


• The subsequent step of management for this patient with snakebite envenomation would be
Neostigmine with Atropine. The symptoms described suggest that the patient may be experiencing
neurotoxic effects of the snake venom, which can cause respiratory depression, bradycardia, and other
neurological symptoms. Antivenom is an important component of snakebite treatment, but it may not
reverse the neurological effects of the venom, which can persist even after neutralization of the
venom's systemic effects. Neostigmine with Atropine is a treatment option for such cases of
neurotoxicity, as it can help to reverse neuromuscular paralysis caused by snake venom.
Incorrect Option:
• Option a: Checking the blood coagulation profile may be necessary in cases of snakebite
envenomation caused by venom that can cause coagulopathy. However, in this case, there are no
apparent bleeding manifestations described
• Option c: Epinephrine injection is not indicated for this patient because the symptoms described are
not consistent with anaphylaxis, which is the primary indication for epinephrine administration.

Page 19

395
• Option d: I.V hydrocortisone is not indicated for this patient because it is not effective in treating the
neurotoxic effects of snake venom.

Solution for Question 3:


• Option d- Opioid intoxication is characterized by respiratory depression, bradycardia, and pinpoint
pupils.
Incorrect Option:
• Option a - Benzodiazepine overdose: Benzodiazepines are central nervous system depressants that
can cause respiratory depression, hypotension, and decreased level of consciousness. However,
benzodiazepine overdose is less likely to cause bradycardia and pinpoint pupils. The patient's
presentation is more consistent with opioid intoxication.
• Option b - Cocaine intoxication: Cocaine is a stimulant drug that can cause tachycardia, hypertension,
hyperthermia, dilated pupils, and agitation. The patient's presentation is inconsistent with cocaine
intoxication. Cocaine use is more likely to cause hyperthermia, tachycardia, and dilated pupils, rather
than hypothermia, bradycardia, and pinpoint pupils.
• Option c - Opioid withdrawal: Opioid withdrawal is characterized by symptoms such as sweating,
nausea, vomiting, diarrhea, dilated pupils, and agitation. The patient's presentation is inconsistent with
opioid withdrawal. Opioid withdrawal is more likely to cause tachycardia, rather than bradycardia, and
elevated respiratory rate, rather than decreased respiratory rate.

Solution for Question 4:


• Carbon monoxide poisoning causes cherry red discoloration of the skin, mucus membranes,
conjunctiva, nail beds, areas of hypostasis, blood tissues and internal organs. This coloration changes
to dark green and then to brown as decomposition starts to occur.
Incorrect Choices:
• Option b. Hydrogen Sulphide poisoning causes a rotten egg smell in the victim. The general signs of
asphyxia are present along with signs of changing of the blood and viscera colour to greenish-purple
and not cherry-red.
• Option c. Yellow Phosphorus is a potent hepatotoxin. Post-mortem findings include garlicky odour
along with icterus of the body due to liver failure as a part of multiple organ failure which causes the
death of the victim.
• Option d. Hydrogen cyanide is an extremely lethal poison. Post-mortem findings of hydrogen cyanide
poisoning include glistening and prominent dilated pupils, frothy mouth, brick-red blood and cheeks due
to impaired oxygenation of the tissues. The internal organs may be oedematous due to congestion.

Solution for Question 5:


Option b. Dilated pupils, rapid pulse and sweating. seen in acute cocaine poisoning

Page 20

396
· Mechanism of action of cocaine: Inhibits the synaptic reuptake of Epinephrine, Norepinephrine, Dopa
mine, Serotonin. · All these substances will increase synaptic cleft.
· This will cause Sympathomimetic Toxidrome

Incorrect Choices
• Option a.Nasal/palatal perforation.Chronic cocaine abuse can lead to damage and perforation of the
nasal passages and palate. Prolonged use of cocaine can cause the erosion of the delicate tissues in
these areas, resulting in perforation.
• Option c. Black tongue and teeth. Chronic cocaine abuse can cause discoloration of the tongue and
teeth. The blackening of the tongue, known as "black tongue," can occur due to the drying effect of
cocaine on the mouth. Additionally, prolonged cocaine use can lead to tooth decay and staining,
resulting in blackened or discolored teeth.
• Option d. Hallucination of insects crawling on the skin . "Formication" refers to a tactile hallucination
commonly associated with cocaine abuse. Individuals experiencing formication may feel sensations
similar to insects crawling on or under their skin. This hallucination is a result of the stimulant effects of
cocaine on the central nervous system.

Solution for Question 6:


• Fomepizole is used as an antidote in confirmed or suspected methanol poisoning. Fomepizole is a
competitive inhibitor of alcohol dehydrogenase, the enzyme that catalyses the initial steps in the
metabolism of ethylene glycol and methanol to their toxic metabolites.
Incorrect options:
• Option b. Flumazenil is a benzodiazepine antagonist that is used for the complete or partial reversal of
the sedative effects caused by benzodiazepines in various clinical settings, such as induced general
anaesthesia for diagnostic and therapeutic procedures. The patient is suspected of methanol poisoning
and hence, usage of flumazenil is not recommended.
• Option c. N-acetylcysteine (NAC) is an acetylated form of the amino acid L-cysteine. It has
antioxidant, anti-inflammatory, and mucolytic properties. NAC is converted to L-cysteine after ingestion,

Page 21

397
which, in turn, is converted into glutathione, a powerful antioxidant. It is used in paracetamol poisoning,
and not in methanol poisoning.
• Option d. Naloxone belongs to a class of drugs known as opioid antagonists. It works by blocking the
effects of the opioid in the brain. Since the patient has a methanol poisoning, he doesn’t require
naloxone.

Solution for Question 7:


• The symptoms exhibited by the child, namely high fever, confusion, photophobia (sensitivity to light),
and inability to urinate, indicate anticholinergic toxicity. The Datura plant, also known as jimsonweed or
devil's trumpet, produces toxins with strong anticholinergic properties. These substances include
scopolamine and atropine. If ingested, they can cause severe symptoms, much like the ones the child
is experiencing.
• Physostigmine is the appropriate antidote in this case because it is a reversible cholinesterase
inhibitor that increases the levels of acetylcholine in the body. This action effectively counteracts the
anticholinergic effects of the Datura toxins. Physostigmine can cross the blood-brain barrier, so it can
help alleviate central nervous system symptoms like confusion and photophobia.
Incorrect Choices:
Option a. Datura, Pralidoxime: The causative agent is correct, but Pralidoxime is not the appropriate an
tidote for Datura poisoning. Pralidoxime is used for poisoning by organophosphates and nerve gases,
which act by inhibiting acetylcholinesterase. While this seems similar to the action of Physostigmine, th
ese substances have different mechanisms and pralidoxime is not effective against anticholinergic pois
oning caused by plants like Datura.
Option c. Yellow oleander, Pralidoxime: Yellow oleander is a toxic plant, but it does not cause symptom
s similar to the ones experienced by the child. The ingestion of yellow oleander leads to cardiac toxicity
, not anticholinergic toxicity. Moreover, Pralidoxime is not the antidote for yellow oleander poisoning. T
he correct treatment for yellow oleander poisoning usually involves the use of Digoxin-specific antibody
fragments.
Option d. Yellow oleander, Physostigmine: Yellow oleander plant does not lead to the symptoms descri
bed. Although Physostigmine is a
good antidote for certain poisonings, it would not be effective for yellow oleander poisoning.

Solution for Question 8:


• The symptoms described in the question - multiple episodes of seizures with consciousness between
episodes and arching of the back (a condition known as opisthotonus) suggest strychnine poisoning.
Strychnine, a potent neurotoxin, is commonly found in certain types of seeds and has historically been
used in rat poison. Its toxicity primarily affects the nervous system, causing severe muscular
convulsions. These symptoms are congruent with the scenario provided.
Incorrect Choices:
Option b. Ricinus: Ricinus is the plant from which castor oil is derived. The seeds contain the toxic com
pound ricin, which is one of the most poisonous naturally occurring substances. However, ricin poisoni
ng typically causes severe abdominal pain, vomiting, and diarrhea. It does not usually result in seizures
or opisthotonus, which were the primary symptoms described in the question.

Page 22

398
Option c. Nerium odorum: This plant, also known as oleander, contains toxins that affect the heart. Ole
ander poisoning can cause a variety of symptoms, including an irregular or slow heartbeat, stomach pa
in, nausea, and vomiting, but it is not typically associated with seizures or arching of the back.
Option d. Datura: Datura is a plant that contains anticholinergic alkaloids. Anticholinergic poisoning can
cause symptoms such as fever, confusion, photophobia, and difficulty urinating. While it may cause de
lirium and hallucinations, seizures are not a common symptom.

Solution for Question 9:


Answer Option A - Viper
• The scenario described in the question aligns most closely with the typical symptoms of a viper bite.
• Vipers are known for their potent hemotoxic venom, which affects the body's blood clotting ability and
can lead to significant local tissue damage.
• Symptoms of a viper bite can include severe local pain, bleeding, swelling, and blistering at the wound
site, all of which have been mentioned in the scenario. Additionally, systemic symptoms such as
drowsiness could also occur, especially in severe cases.
• Bleeding from bite site is the most important feature, as it indicates viper's bite & is important sign to
start ASV immediately
Bleeding from bite site is the most important feature, as it indicates viper's bite &
is important sign to start ASV immediately
Incorrect Choices:
Option b. Cobra: Cobra venom is primarily neurotoxic, affecting the nervous system and leading to sym
ptoms such as difficulty breathing, paralysis, and potentially death due to respiratory failure. While local
symptoms like pain and swelling can occur, bleeding and blistering are not as common as in viper bite
s.
Option c. Wasp bite: Wasps can cause painful stings, and in some individuals, a severe allergic reactio
n can occur. However, profuse bleeding and blistering at the sting site are not typical of a wasp sting.
Option d. Scorpion: Most scorpion stings cause symptoms such as pain, numbness, or tingling around
the sting area. Some scorpions can cause more severe symptoms, but these are typically systemic (su
ch as difficulty breathing, muscle twitching, or irregular heartbeat) rather than severe local reactions lik
e profuse bleeding or blistering.

Solution for Question 10:


Correct Option B - Argemone mexicana:
• They're large and showy, with six bright yellow petals arranged around a central capsule. Keep in
mind though, the petals sometimes have a slightly crumpled look, which gives the flowers a bit of an
interesting flair.Then there's the seed pod, which is another key identifier. Once the flowers have
bloomed and fallen off, you'll find seed capsules, which are prickly, oblong, and start off green but turn
a dark brown or black as they mature.The foliage of the Argemone mexicana is blue-green in color,
lobed, spiky, and has a somewhat thistle-like appearance. It has a prickly feel because of the presence
of spines or bristles, which is a distinct characteristic of the plant. The plant as a whole has a very

Page 23

399
upright growth habit.
Incorrect Options:
Option A -Datura Alba: Large trumpet-shaped flowers are a distinguishing feature of the Datura specie
s, which are either spreading annuals or short-lived perennials. The fruit, a spiny capsule with many se
eds, is produced by the fragrant blooms, which can be white, yellow, pink, or purple.

Option C - Nerium Odorum: A strongly 5-lobed fringed corolla encircles the central corolla tube, and th
e blooms are white, pink to red, growing in clusters at the ends of each branch and about 2.5–5 cm (1-
2 in) in diameter.

Option D - Papaver somniferum: The flowers can be up to 3-10 cm (1-4 in) in diameter and typically ha
ve four white, mauve, or crimson petals. The base of the petals occasionally has dark patterns.

Page 24

400
Solution for Question 11:
Correct Option D - Dimercaprol:
• In treating acute arsenic poisoning, the primary choice is D. Dimercaprol.
• Dimercaprol, also known as British Anti-Lewisite (BAL), is an agent used in chelation therapy - a
medical procedure that involves the administration of chelating agents to remove heavy metals from the
body. Dimercaprol forms stable, non-toxic complexes with arsenic which are then excreted from the
body making this the preferred treatment for acute arsenic poisoning.
Incorrect Options:
Option A - Ipecac: This is a substance used to induce vomiting, mainly in cases of certain types of pois
oning. However, it's not typically employed in arsenic poisoning due to the risk of causing further dama
ge to the gastrointestinal tract.
Option B - Penicillamine: This is a chelating agent used primarily in the treatment of Wilson's disease (
a rare genetic disorder of copper metabolism) and rheumatoid arthritis. It's not the first-line treatment fo
r arsenic poisoning.
Option C - Activated Charcoal: This is commonly used in the emergency treatment of certain kinds of p
oisoning to prevent the poison from being absorbed from the stomach into the body. However, it's not e
ffective in arsenic poisoning as arsenic is rapidly absorbed and not effectively adsorbed by activated ch
arcoal.

Solution for Question 12:


Correct Option D - Atropa belladonna:
• The deadly nightshade plant, Atropa belladonna, is poisonous. Atropine, hyocyamine, and
scopolamine are alkaloids found in their roots, leaves, and fruits. The alkaloids present are called

Page 25

401
tropane alkaloids, often known as belladonna alkaloids. The anticholinergic syndrome may result from
Atropa belladonna poisoning. High doses of the plant may result in drowsiness and unconsciousness.
Belladonna contains compounds that can affect the nervous system's operation as well as the
production of saliva, sweat, changes in pupil size, urine, and other bodily functions. Additionally,
belladonna might raise blood pressure and pulse rate but is not considered a cardiac poison that can
affect cardiac function.
Signs and Symptoms:
• Dryness in skin and mouth
• Dysphagia - Difficulty in swallowing
• Dilated pupil (very earliest symptom)
• Drunken Gait
• Dysarthria - Slurred speech
• Muttering Delirium
• Drowsiness
• Death
• Carphologia-It feels like it is a kind of hallucination, imaginary threads, and pill-rolling movement.

Incorrect Options:
Option A - Aconite: The sodium channels are persistently activated by Aconitine's high affinity binding t
o their open state in the voltage-sensitive sodium channel at site 2, which results in arrhythmias from tri
ggered activity.
Option B - Cerbera thevetia: This plant's seeds contain extremely toxic cardiac glycosides that can lead
to electrolyte imbalances, vomiting, dizziness, and cardiac dysrhythmias such conduction block that aff
ects the sinus and AV nodes.
Option C - Nicotiana tabacum: Chronic nicotine use can increase the risk of heart attack and stroke as
well as cause cardiovascular disease.

Page 26

402
Solution for Question 13:
Correct Option D - Methanol:
• The term 'Hooch Tragedy' is generally used to refer to mass poisonings that occur due to the
consumption of illicitly distilled or home-made alcohol, colloquially referred to as 'hooch.' These
tragedies typically occur when methanol, a highly toxic substance, is used in the production of this illicit
alcohol. Methanol poisoning can lead to a range of serious health problems, including blindness and
death.
Incorrect Options:
Option A - Phosphorus: While phosphorus is indeed a hazardous substance, it is not typically associat
ed with alcohol production or consumption, and therefore would not be related to a 'hooch tragedy.'
Option B - Mercury: Mercury is a heavy metal that is dangerous if ingested, but it is not used in the pro
duction of alcohol and is not associated with hooch tragedies.
Option D - Ethanol: While ethanol is the type of alcohol that is safe for human consumption and is foun
d in legally produced alcoholic beverages, hooch tragedies are specifically associated with the consum
ption of methanol, not ethanol.

Solution for Question 14:


Correct Option A - Atropine:
• The correct management in this case would be A. Atropine.
• From the symptoms that the patient is displaying - excessive sweating, tearing, diarrhea, muscle
weakness, and a rapid heart rate - we can infer that he may have ingested an organophosphorus
compound. These compounds are found in some pesticides. They work by disrupting the normal
function of the nervous system leading to an overabundance of the neurotransmitter acetylcholine.
• Atropine is normally used as the initial treatment in these types of cases. It's an anticholinergic, which
means it works by blocking the action of acetylcholine which helps to lessen the symptoms caused by
this neurotransmitter's excess. It helps to restore the balance that the organophosphorus compound
has upset.
Incorrect Options: Option B, C and D
are incorrect. Atropine is the initial treatment of choice in patients with organophosphate poisoning.

Solution for Question 15:


Correct Option A - 1, 2, 3 are correct:
The correct answer for the manifestations of chronic arsenic poisoning is A. 1,2 and 3 are correct.
Incorrect Options: Option B, C and D are incorrect. Refer to the explanation of the correct answer.

Solution for Question 16:

Page 27

403
Correct Option B: Sample area cleaned with soap and water
• The Sample area should be cleaned with soap and water instead of spirits or any other substance
that might contain alcohol.
• Also 1:1000 mercuric chloride can be used to clean the area from where sample is to be collected.
Incorrect options
Option A: Sample area cleaned with spirit -
This option is incorrect as the presence of alcohol in spirit might effect the sample alcohol levels.
Option C: Sample should be preserved without preservative- This option is incorrect as 100 mg NaF an
d 30 mg potassium oxalate for 10 ml of blood followed by thorough shaking is used for sample preserv
ation.
Option D: Only one urine sample is required - This option is incorrect as 2
samples are taken one at the time of arrival and the other one 25 to 30 min after 1st sample.
For reliable UAC results:
• Collect two urine samples, spaced about half to 1 hour apart.
• Ensure the bladder is fully emptied before the first void.
• The second urine sample reflects the alcohol concentration formed between the first and second
voids.
• Differences in UAC values indicate the trend of the UAC curve (rising or falling).
• The alcohol content of the second void represents the average alcohol concentration of the urine
between voids.
• Typically, a 1.33:1 ratio of urine alcohol to blood alcohol is utilized.

Solution for Question 17:


Correct Option: A
• Naloxone is an opioid receptor antagonist that rapidly reverses the effects of opioids, including
respiratory depression. It competes with opioids for binding to opioid receptors, displacing them and
reversing the respiratory depression.
Incorrect Options:
Option B: Atropine is not the correct antidote for respiratory distress caused by morphine sulfate. Atropi
ne is an anticholinergic medication used to treat muscarinic effects of organophosphorus compounds (
OPC) . It does not reverse the effects of opioids.
Option C: Pralidoxime is an antidote used to treat poisoning by certain types of organophosphorus nerv
e agents or organophosphate insecticides by reactivation of acetylcholine esterase. It is not effective fo
r opioid overdose.
Option D: Epinephrine is primarily used for the treatment of severe allergic reactions (anaphylaxis), car
diac arrest, and certain other emergency situations. It is not the appropriate antidote for respiratory dist
ress caused by opioid overdose.

Page 28

404
Solution for Question 18:
Correct Option: A
• Black foot disease, also known as Blackfoot or arsenicosis, is a condition associated with chronic
arsenic poisoning.
• It is a severe form of peripheral vascular disease in which blood vessels of the lower limb are severely
damaged eventually resulting in gangrene.
• Prolonged exposure to high levels of arsenic, typically through contaminated drinking water or food,
can lead to peripheral neuropathy and other health issues.
Incorrect Options:
Option B: Lead poisoning can lead to developmental delays, learning difficulties, behavior problems, an
d impaired growth. In adults, it can cause neurological symptoms, such as headaches, memory loss, a
nd concentration problems, as well as abdominal pain and reproductive issues. It is not associated with
black foot disease.
Option C: The symptoms and effects of mercury poisoning depend on the form of mercury involved:
• Elemental mercury vapor exposure primarily affects the respiratory system and can cause lung
damage, respiratory distress, and neurological symptoms such as tremors and cognitive impairments.
• Methylmercury, a form of organic mercury found in contaminated fish and seafood, primarily affects
the nervous system and can cause developmental issues in children, as well as neurological and
cardiovascular problems in adults.
• It is not associated with black foot disease.
Option D: Cadmium primarily affects the kidneys and can cause kidney damage or failure over time. It
can also have adverse effects on the bones, respiratory system, and cardiovascular system. Symptom
s of cadmium poisoning may include fatigue, weakness, cough, chest pain, and gastrointestinal disturb
ances. Prolonged exposure to high levels of cadmium can increase the risk of lung cancer and other ca
ncers. It is not associated with black foot disease.

Solution for Question 19:


Correct Option B:
• Datura is a plant that contains tropane alkaloids, such as scopolamine and atropine, which have
potent anticholinergic effects. Ingestion of Datura can lead to the symptoms described in the scenario,
including irritability, restlessness, dry hot skin, and urinary and bowel retention.
• Physostigmine is a reversible acetylcholinesterase inhibitor that acts by increasing the levels of
acetylcholine in the body.
• It is an appropriate antidote for anticholinergic poisoning, including Datura poisoning. By inhibiting
acetylcholinesterase, physostigmine can counteract the effects of excessive anticholinergic activity and
alleviate the symptoms.
• Pralidoxime, on the other hand, is an antidote used for organophosphate poisoning, not specifically for
Datura poisoning. Organophosphates are a different class of toxic compounds that inhibit
acetylcholinesterase, but they have different mechanisms of action compared to Datura alkaloids.
• Therefore, in the given scenario, the correct option is "Datura, Physostigmine" because physostigmine
is the appropriate antidote for reversing the anticholinergic effects of Datura poisoning.

Page 29

405
Incorrect Options:
• Yellow oleander (Thevetia peruviana) is a plant that contains cardiac glycosides, particularly Thevetin
A and Thevetin B. Ingestion of yellow oleander can lead to cardiac toxicity, including arrhythmias,
conduction disturbances, and cardiac arrest. However, the symptoms described in the scenario, such
as irritability, restlessness, dry hot skin, and urinary and bowel retention, are not typical of yellow
oleander poisoning.
• Digoxin, on the other hand, is a cardiac glycoside commonly used in the treatment of heart failure and
certain cardiac arrhythmias.
• It has a narrow therapeutic index, and overdose or toxicity can result in various cardiac
manifestations, including arrhythmias.
• However, it does not cause the specific symptoms mentioned in the scenario, such as irritability,
restlessness, and dry hot skin.
• Therefore, the option "Yellow oleander, Digoxin" is incorrect because it does not match the symptoms
described in the scenario and does not involve the appropriate poison-antidote relationship.

Solution for Question 20:


Correct Option B:
In the context of cocaine poisoning, the symptoms typically include diaphoresis (excessive sweating), h
eadache, acute coronary spasm leading to myocardial ischemia (reduced blood flow to the heart), and
agitation. However, bradycardia (slow heart rate) is not commonly associated with cocaine poisoning. I
nstead, cocaine use usually leads to tachycardia (rapid heart rate) as one of its effects on the cardiova
scular system. Therefore, the presence of bradycardia would go against the likely diagnosis of cocaine
poisoning.
Incorrect Options:
Option A. Myocardial ischemia: Myocardial ischemia refers to inadequate blood supply to the heart mu
scle, often leading to chest pain or discomfort. In the context of cocaine poisoning, one of the significan
t adverse effects is acute coronary spasm, which can result in myocardial ischemia. Therefore, the pre
sence of myocardial ischemia would actually support the diagnosis of cocaine poisoning, making this o
ption incorrect.
Option C. Hyperthermia: Hyperthermia, or abnormally high body temperature, is a common manifestati
on of cocaine poisoning. Cocaine stimulates the sympathetic nervous system, leading to increased bod
y temperature, diaphoresis (excessive sweating), and hyperthermia. Thus, the presence of hyperthermi
a would be consistent with cocaine poisoning, making this option incorrect.
Option D. Agitation: Agitation is a characteristic symptom of cocaine intoxication. Cocaine acts as a ce
ntral nervous system stimulant, leading to heightened arousal, restlessness, and agitation. Therefore, t
he presence of agitation would support the diagnosis of cocaine poisoning, making this option incorrect
.

Solution for Question 21:


Correct Option B:

Page 30

406
The correct answer for the active principle of the poison shown in the image of the marking nut is Bhila
wanol. Bhilawanol is a toxic phenolic constituent found in the seeds or nuts of the plant Semecarpus an
acardium. It is responsible for the toxic effects associated with the consumption or contact with the mar
king nut.
Incorrect Options:
Option A. Ricin: Ricin is a toxic protein found in the seeds of the castor oil plant (Ricinus communis). It i
s not relevant to the marking nut (Semecarpus anacardium).
Option C. Abrin: Abrin is a toxic protein found in the seeds of the rosary pea (Abrus precatorius). It is n
ot associated with the marking nut.
Option D. Calotropin: Calotropin is a cardiac glycoside found in plants of the Calotropis genus, such as
Calotropis gigantea and Calotropis procera. It is not present in the marking nut.

Solution for Question 22:


Correct option:
Option C:
Organophosphorus poisoning generally occurs after respiratory, oral, or dermal exposure to nerve age
nts or organophosphorus pesticides. The onset symptoms of this specific disease are excessive lacrim
ation and saliva production, tiny pupils(miosis) , vomiting, diarrhea, excessive sweating, confusion, etc.
So, as farmers are experiencing pinpoint pupils and increased secretion, the diagnosis is organophosp
horus poisoning.
Incorrect Options:
Option A: Alcohol poisoning is a severe and deadly condition. It quickly affects heart rate, breathing, ga
g reflex, and body temperature. So, it often leads to coma and even death.
Option B: Opioid poisoning occurs when there is excessive stimulation on the pathway of opiate. It eve
ntually leads to lower respiratory effort and death. It can also cause pinpoint pupils but there is no incre
ased body secretions.
Option D: Datura poisoning after 60 minutes of ingestion. The symptoms are dry skin and mouth, respir
atory arrest, urinary retention, delirium, and tachycardia.

Solution for Question 23:


Correct Option:
Option C:
HCN or Hydrogen Cyanide is a pale-blue or almost colorless gas or liquid with a bitter almond taste. W
hen it interferes with using oxygen in the human body, it directly affects our blood vessels, heart, brain,
and lungs. Additionally, the pink color of mucosa and skin appears because of the excessive infectious
hemoglobin saturation.
Incorrect options:
Option A: H2S poisoning is also a severe and fatal condition. The primary symptoms of this condition a
re headaches, tremors, nausea, delirium, convulsions, disturbed equilibrium, eye and skin irritation, rap

Page 31

407
id unconsciousness, and eventually death.
Option B: CO poisoning can also be deadly. The most common signs of CO poisoning are vomiting, up
set stomach, dizziness, headache, chest pain, and confusion. If anyone inhales excessive CO, it has th
e potential to kill that individual.
Option D: OPC poisoning is a known class of insecticides that occurs when a
person is exposed to organophosphates. The condition's primary symptoms are a runny nose, glassy e
yes, blurry vision, agitation, muscle weakness, narrowed pill, etc. In severe cases, it leads to coma or c
ollapse.

Solution for Question 24:


Correct Option:
Option D:
Lead poisoning can occur due to a bullet in an individual's body. The poisoning generally occurs from t
he bullet lead in the synovial cavity of the chest, hip, and pleural space.
Incorrect options:
Option A: Iron poisoning mostly happens due to consuming iron supplements. Therefore, it is best to ta
ke advice from a healthcare professional regarding dosage.
Option B: Phosphorus is very dangerous for our body, but the retained bullet cannot be the cause of ph
osphorus poisoning. The primary reasons for Phosphorus poisoning are tumor lysis syndrome, rhabdo
myolysis, and impaired renal function.
Option C: When an individual inhales Nitrocellulose, it can cause Nitro cellulose poisoning. So, it is one
of the incorrect choices.

Solution for Question 25:


Correct Option:
Option A:
In an autopsy of suspected poisoning, the kidney is preserved because it is the primary organ responsi
ble for filtering and excreting toxins from the body. The kidney can provide crucial evidence of poisonin
g.
Incorrect options:
Option B: While the lungs may show signs of poisoning in some instances, they are not typically preser
ved as a routine procedure in a poisoning autopsy.
Option C: The brain is not directly involved in the filtration or detoxifying of toxins, so it is not commonly
preserved in a poisoning autopsy.
Option D: The heart's primary function is to pump blood and distribute oxygen, so it is not explicitly pres
erved in a poisoning autopsy unless there is suspicion of cardiac involvement.

Page 32

408
Solution for Question 26:
Correct Option: B
• Heroin, also known as diamorphine, is an opioid drug that is derived from morphine. It is highly
addictive and is known to cause physical dependence. When individuals use heroin regularly, their
body adapts to the presence of the drug, and they develop a tolerance to its effects. This means that
over time, they require higher doses of heroin to achieve the desired effect.
• If a person becomes physically dependent on heroin and abruptly stops using it or significantly
reduces their dosage, they may experience withdrawal symptoms. These symptoms can be severe and
include cravings, restlessness, muscle and bone pain, insomnia, gastrointestinal distress, and flu-like
symptoms.
Incorrect options:
Option A. Ketamine: Ketamine is a dissociative anesthetic that produces hallucinogenic effects. While it
can lead to psychological dependence, it is not typically associated with significant physical dependen
ce.
Option C. LSD: LSD (Lysergic acid diethylamide) is a hallucinogenic drug that does not typically produc
e physical dependence. However, it can lead to psychological dependence or addiction in some individ
uals.
Option D. Phencyclidine (PCP): PCP is a dissociative drug that can cause hallucinations, distorted perc
eptions, and other psychological effects. It is not commonly associated with physical dependence, but i
t can lead to psychological dependence in some cases.

Solution for Question 27:


Correct Option A:
• Arsenic is a poison that can be found even in skeletonized remains of the body. It has the ability to
accumulate in bones over time, making it detectable even after the decomposition of soft tissues. This
is because arsenic is absorbed into the bones during life and remains there after death.
Incorrect options:
Option B: Carbon monoxide: Carbon monoxide is a gas that is produced during the incomplete combus
tion of carbon-containing materials. It is unlikely to be found in skeletonized remains since it is a
volatile gas that dissipates quickly and does not accumulate in the body or bones.
Option C: Hydrogen sulphide: Hydrogen sulphide is a toxic gas that has a strong odor of rotten eggs. S
imilar to carbon monoxide, it is unlikely to be found in skeletonized remains as it is a
volatile gas that dissipates rapidly and does not accumulate in the body or bones.
Option D: Phosphorus: Phosphorus is a chemical element that can be toxic in certain forms, such as w
hite phosphorus. However, it is not typically found in skeletonized remains as it is highly reactive and te
nds to undergo chemical reactions or degradation over time.

Solution for Question 28:


Correct option D Bleeding, paralysis and amputation

Page 33

409
• The snake in the image is a highly venomous species of snake called Bungarus caerulus, also
referred to as the common krait. It causes several neurotoxic symptoms and muscle paralysis.
Neurotoxic symptoms are :
• Ptosis: Most Important and Common Symptom-Dropping of Eyelid
• Diplopia: Paralysis of extra ocular Muscles.
• Dysphagia: Paralysis of pharyngeal muscles.
• Dysphonia: Paralysis of vocal codes.
• Dysarthria: Paralysis of tongue muscles.
• Dyspnea: Paralysis of respiratory muscles.
• Stridor: Respiratory difficulty.
• Descending Paralysis
• Death due to respiratory muscles paralysis leading to respiratory failure.
Option A. Non-poisonous, dry bite: The absence of venom being injected into the victim during a
snakebite incidence is known as a "dry bite" from a snake
Option B. Severe edema, necrosis and bleeding: This is brought on by viper bites, which cause more s
evere local reactions than those from other snakes. Within 15 minutes, swelling may be noticeable, an
d it might become huge within two to three days. Bite marks from some rattlesnakes and Asian pit viper
s indicate necrosis.
Option C. Severe muscle necrosis, rhabdomyolysis: These symptoms are due to bites by Russell's vip
ers, which increase renal ischemia and cause acute kidney injury by haemorrhage, hypotension, disse
minated intravascular coagulation (DIC), intravascular hemolysis, and rhabdomyolysis.

Solution for Question 29:


Correct Option: A
• In America, Asia, Africa, and Australia, parthenium hysterophorus is considered a noxious weed. This
weed is thought to be a contributor to allergic respiratory issues, contact dermatitis, and cattle and
human mutagenicity. Its allelopathy significantly reduces crop output. Additionally, this weed's
aggressive dominance endangers biodiversity.
Incorrect Options:
Option B. Datura stramonium: A common annual plant called Datura stramonium (DS) contains the dru
gs atropine, hyoscyamine, and scopolamine, which can cause severe anticholinergic syndrome and lea
d to poisoning.
Option C. Digitalis purpurea: The hazardous kind of flowering plant known as the foxglove, or Digitalis
purpurea, belongs to the Plantaginaceae family of plantains. The same cardiac glycoside, digitoxin, whi
ch is extracted from the leaves, is used as a treatment for heart failure.
Option D. Calotropis gigantea: Calotropis gigantea is a huge shrub or small tree that is native to India,
southern China, Malaysia, and Indonesia but has been widely cultivated in tropical regions all over the
world. It is also known as a crown flower or giant milkweed. It is distinguished by its upright growth, mil
ky sap, crown-like flowers that are pale purple or white, and white woolly stems and leaves that eventu
ally become subglabrous. No significant issues with insects or diseases. Severe irritation might result fr

Page 34

410
om sap in the eyes. Mucous membranes are irritated by sap as well.

Solution for Question 30:


Correct Option: B
• Cocaine is a strongly addictive stimulant that increases the levels of vigilance, focus, and vigour.
Agitation, paranoia, hallucinations, delusions, violence, and thoughts of suicide and murder are some of
the symptoms.
Incorrect Options:
Option A. Alcohol: Common signs and symptoms include weakness, fatigue, thirst, headache, muscle
aches, nausea, stomach discomfort, vertigo, sensitivity to light and sound, anxiety, irritability, perspirati
on, and elevated blood pressure.
Option C. Opioid: While certain opioid side effects, such as tiredness and constipation, may be modest,
more severe side symptoms, including as shallow breathing, a
slowed heartbeat, and loss of consciousness, may be significant and indicate an overdose.
Option D. Cannabis: Disorientation's negative repercussions include diminished perception, a potential
rise in periodontal disease and caries risk, enhanced risk of oral mucositis-related inflammation and mo
uth drool.

Solution for Question 31:


Answer Option C -
• 3. Gastrointestinal Irritation: As iron toxicity progresses, it can cause damage to the lining of the
gastrointestinal tract and irritation. This can lead to gastrointestinal bleeding, which is characterized by
the presence of blood in the vomit or stool. Gastrointestinal bleeding is a serious complication of iron
toxicity and requires immediate medical attention.
• 2. Latent phase: The initial phase of iron toxicity after GI irritation may often be asymptomatic,
meaning that there are no noticeable symptoms. During this phase, the individual may not experience
any immediate effects or manifestations of iron toxicity. However, it is important to note that this phase
can be transient, and symptoms can develop as the toxicity progresses.
• 4. Liver failure and hypoglycemia: In severe cases of iron toxicity, especially when a large amount of
iron has been ingested, it can lead to liver failure and hypoglycemia. Iron overload can cause significant
damage to the liver, impairing its normal function. This can result in liver failure, which is a
life-threatening condition. Additionally, iron toxicity can also cause hypoglycemia, a condition
characterized by low blood sugar levels.
• 1. Healing with scaring : In the context of iron toxicity, pyloric stricture occurs as a late-stage
complication. It is typically seen after significant ingestion of iron, usually in the form of iron tablets or
supplements. Pyloric stricture can cause symptoms such as vomiting, abdominal pain, and difficulty in
food passage.
Therefore, the correct order of symptomatology seen in iron toxicity is 3 (Gastrointestinal Irritation), 2
(Latent phase), 4 (Liver failure and hypoglycemia), and 1 (Healing with scaring).

Page 35

411
Solution for Question 32:
Correct Choice: B
• Arsenic poisoning can occur through exposure to arsenic compounds. It can lead to various
symptoms, including gastrointestinal issues such as nausea, vomiting, and bloody stools. Additionally,
arsenic exposure can cause conjunctivitis (inflammation of the conjunctiva) and a burning sensation in
the throat and stomach. Therefore, considering the presented symptoms, arsenic poisoning is a
plausible explanation.
Incorrect Choices:
Option A. Potassium permanganate: Potassium permanganate is a
chemical compound commonly used as a disinfectant, oxidizing agent, and in water treatment. Howeve
r, poisoning due to potassium permanganate is rare, especially in a dye factory setting. Symptoms of p
otassium permanganate poisoning typically include irritation and burns on the skin, but the other sympt
oms mentioned in the question, such as bloody stools and conjunctivitis, are not commonly associated
with potassium permanganate poisoning. Therefore, this option is unlikely to be the correct answer.
Option C. Thallium: Thallium is a highly toxic heavy metal that can cause severe poisoning when ingest
ed, inhaled, or absorbed through the skin. However, the symptoms described in the question, such as
dark bloody stools and conjunctivitis, are not typically associated with thallium poisoning. Thallium pois
oning often presents with neurological symptoms, such as peripheral neuropathy and hair loss. Therefo
re, thallium poisoning is unlikely based on the given symptoms.
Option D. Lead: Lead poisoning can occur through the ingestion or inhalation of lead-containing substa
nces. While lead exposure can lead to various health issues, the symptoms mentioned in the question,
such as dark bloody stools, conjunctivitis, and a burning sensation in the throat and stomach, are not ty
pical of lead poisoning. Lead poisoning is more commonly associated with neurological symptoms, dev
elopmental delays in children, and other systemic effects. Therefore, lead poisoning is unlikely based o
n the provided symptoms.

Solution for Question 33:


Correct option: B
• Delirium tremens is a severe and potentially life-threatening withdrawal symptom that can occur in
alcoholic patients after 72 hours of cessation of alcohol. It typically manifests with symptoms such as
severe confusion, disorientation, hallucinations, agitation, rapid heartbeat, high blood pressure, and
fever. Delirium tremens requires immediate medical attention.
Incorrect options:
Option A: Seizures: Seizures can indeed be a withdrawal symptom in alcoholic patients, but they typica
lly occur within the first 48 hours of alcohol cessation rather than after 72 hours.
Option C: Tremors: Tremors are one of the most common early withdrawal symptoms seen within the fi
rst few hours to a couple of days after alcohol cessation. They are characterized by involuntary shaking
of the hands, arms, or other body parts. While tremors are a
common symptom, they are not specific to the 72-hour mark.

Page 36

412
Option D: Hallucinations: Hallucinations can be part of the withdrawal syndrome in alcoholic patients, b
ut they often occur within the first 48 hours rather than after 72 hours. They can be visual, auditory, or t
actile in nature.

Solution for Question 34:


Correct Choice. B
Explanation:
• Datura/ devil’s trumpet is a deliriant poison (other than Cannabis, Cocaine). The symptoms given are
due to Datura poisoning.
• If it is a white flower plant, it is Datura alba. If it is a purple dark color flower it is Datura nigeria. Datura
stramonium AKA Jimson’s weed is also known as thorn apple.
• Entire plant is poisonous. Active principle: Hyoscine (Scopolamine), Truth serum drug, which is used
in narco analysis, Hyoscyamine, Atropine.
• 1 Fruit almost contains 100s of seeds.
Fatal dose is 100- 125 seeds.
• Used as a roadside poison or railway poison. There is no criminal responsibility for a person who has
been given Datura because his actions are involuntary intoxication. 328 IPC - Hurt with poison with
criminal intention.
Anticholinergic

Signs and Symptoms


• Dryness in skin and mouth
• Dysphagia - Difficulty in swallowing
• Dilated pupil (very earliest symptom)
• Drunken Gait
• Dysarthria - Slurred speech
• Muttering Delirium
• Drowsiness
• Death
• Carpologia - It feels like it is a kind of hallucination, imaginary threads, and pill-rolling movement.
• Feature: Blind as a bat. Hot as a hare. Dry as a bone. Red as a bee. Mad as a hen.
Incorrect choices:
Option A. 50 grams (5-6 tablespoons) of Poppy seeds might be sufficient to cause poisoning. Symptom
s include severe muscle spasms, severe muscle pain, seizures, cramps, and stiffness or abnormal mo
vements.

Page 37

413
Option C. Mushroom poisoning might cause severe gastrointestinal upsets, nausea, vomiting and diarr
hea to devastating manifestations which might be neurologic complications, kidney and liver failure.
Option D. Cannabis poisoning leads to these symptoms:
• Small doses: Causes euphoria and disorientation; Large doses: Psychiatric problems- Euphoria,
hallucination, increase self-confidence.

Solution for Question 35:


Correct Choice. A
Explanation:
• Starvation may occur from the actual withholding of food or from the administration of unsuitable food.
• In the case of starvation, all organs will shrink (se in size) except gallbladder, because there is no
food. Bile gets accumulated. The only organ which maintains the size is the brain.
• In the case of starvation (no food and water), the person dies in 10-12 days.
• If the person is provided only with water, the person can survive 6-8 weeks.
Incorrect choices:
Option B. In starvation all organs shrink, so does the liver.
Option C. Only organ which distends is the gallbladder, except that all organs shrink. Hence, the lungs
don't expand.
Option D. Only organ which maintains the size is the brain, the only organ which does not shrink is the
gallbladder. Heart may shrink to 5 to 6 ounces from 11 ounces

Solution for Question 36:


Correct Choice. B
Explanation: Cherry red postmortem staining is associated with carbon monoxide poisoning.
• Mechanism of action of CO: It has 200-300 times more affinity to Hb than O2. CO combines with hb to
form carboxyhemoglobin. Carboxy Hb reduces O2 in blood, tissues.
• Signs and symptoms: Depends on percent of CO present in blood
• <10% - No symptoms
• 10-20% - Breathless on moderate exertion, headache, weakness.
• 20-30% - Severe headache, irritability, emotional instability, defective memory.
• 30-40% - Cherry red discolouration skin, mucous membrane, vision decreased, nausea and vomiting.
• 40-50% - Resemble acute alcoholic intoxication
• 50-60% - Syncope and coma
• >80% - Death

Page 38

414
• Treatment: Antidote 100% O2 at atmospheric pressure given by a tight fitting mask.
Postmortem findings:
• Cherry-red post-mortem staining.
• Blisters in dependent or frictional areas– Axilla, buttocks, inner thigh, calves, wrists, and knees. Also
seen in barbiturates poisoning.
• In delayed deaths Bilateral and symmetrical necrosis and cavitation. Seen in globus pallidus and
putamen i.e., basal ganglia change.
Incorrect Choices:
Option A. Opioid poisoning - black postmortem staining is seen.
Option C. Hydrogen sulphide (H2S) poisoning - it produces greenish- blue/ bluish-green discolouration.
Option D. Phosphorus poisoning - dark brown or yellowish discolouration is seen.

Solution for Question 37:


Correct Option: B
The most probable cause of death in this scenario would be respiratory failure. The common krait (Bun
garus caeruleus) is a highly venomous snake found in South Asia, including India. Its venom contains
neurotoxins that affect the nervous system, particularly the respiratory muscles and the diaphragm.
When a person is bitten by a common krait, the venom can cause paralysis of the respiratory muscles,
leading to respiratory failure. This means that the person is unable to breathe adequately on their own,
resulting in a lack of oxygen supply to the body and vital organs.
While shock, coagulopathy (blood clotting disorder), and renal failure are potential complications associ
ated with snakebites, the rapid onset of respiratory failure and subsequent hypoxia (low oxygen levels)
are the most immediate and life-threatening consequences of envenomation by the common krait.
It is important to seek immediate medical attention and provide supportive care, including artificial venti
lation, in cases of suspected snakebite to prevent or manage respiratory failure and other complication
s. Antivenom administration may also be necessary to neutralize the snake venom and improve the pat
ient's outcome.

Solution for Question 38:


Correct Choice. B
Explanation:
• Deferoxamine also known as Desferrioxamine is used as an antidote for iron.
• Fatal dose of iron is 20 to 30 grams.
• Treatment: (1) Stomach wash with 5% sodium bicarbonate solution. Instil 5 to 10 gm of
desferrioxamine at the end of lavage. (2) Give plenty of egg and milk to form iron-protein complexes.
(3) Magnesium hydroxide 1% solution orally. (4) Desferrioxamine 1g i.m. followed by 500 mg 4th hourly

Page 39

415
for 2 doses and finally 500 mg 4 to 12 hourly up to a maximum of 6 g in 24 hours. It can also be given in
infusion 15 mg/kg/hr in normal saline. (5) Haemodialysis or exchange transfusion in severe cases.
Incorrect choices:
Option A. Luspatercept is used to treat anemia in beta-thalassemia, myelodysplastic syndromes, and p
rimary myelofibrosis.
Option C. Deferiprone chelates iron but is used in the treatment of iron overload in thalassemia (For chi
ldrens 3 years & above and adults).
Option D. DMSA (Dimercapto succinic acid) is a
radioisotope. It is used for metal chelation, and is also used in imaging and therapy

Solution for Question 39:


Correct Option: D
Based on the given symptoms of greenish skin color and a
rotten egg odor, the most probable poisoning, in this case, would be hydrogen sulfide (H2S) poisoning.
Hydrogen sulfide is a toxic gas that is commonly found in various industrial settings, such as petroleum
refining, sewer systems, and certain chemical processes. It has a distinct odor resembling rotten eggs.
Exposure to high levels of hydrogen sulfide can be extremely dangerous and potentially fatal.
The characteristic greenish discoloration of the skin is known as sulfhemoglobinemia, which occurs as
a result of hydrogen sulfide poisoning. Hydrogen sulfide binds to hemoglobin in the blood, forming sulfh
emoglobin, which gives the skin a greenish appearance.
Carbon monoxide (CO) poisoning, on the other hand, typically causes a
cherry red coloration of the skin rather than a greenish color. Nitrite poisoning is associated with sympt
oms such as headache, dizziness, and shortness of breath, but it does not cause greenish skin discolo
ration. Cyanide poisoning may cause a bluish discoloration of the skin, but it does not produce a
rotten egg odor.
Therefore, based on the symptoms described, the most probable poisoning in this case is hydrogen sul
fide (H2S) poisoning.

Solution for Question 40:


Correct Choice: B
Explanation:
• Needle track marks over upper limbs, increased blood pressure, increased heart rate, and increased
temperature (hyperthermia) – all of these signs symptoms indicate the poisoning of a deliriant poison
Cocaine.
• Mechanism of action of cocaine: Inhibits the synaptic reuptake of Epinephrine, Norepinephrine,
Dopamine, Serotonin. All these substances will increase synaptic cleft.

Page 40

416
• This will cause Sympathomimetic Toxidrome.

Sympathomimetic Toxidrome Complications


Small Dose
High Dose
Heart Rate
Increase
Cardiac arrhythmia
Blood vessels
Constrictive
Myocardial function
Blood Pressure
Intracerebral brain stroke
Respiratory rate
Temperature
Crack fever
Sweating
Pupil
Dilated
Fixed
CNS Effects
Excitement
Euphoria
Delirium
Excitation
Incorrect choices:
Option A. Excessive heroin ingestion leads to hypothermia, slowed breathing, blue lips and fingernails,
cold and clammy skin, convulsions and coma.
Option C. Cannabis poisoning leads to these symptoms: Small doses: Causes euphoria and disorientat
ion; Large doses: Psychiatric problems- Euphoria, hallucination, increase self-confidence.
Option D. LSD (Lysergic acid diethylamide – synthetic chemical made from ergot) overdose leads to hy
perthermia, blurred vision, rapid heartbeat, insomnia, dry mouth, etc. Adverse outcomes include featur
es of psychosis, extreme anxiety, aggression towards others.

Solution for Question 41:

Page 41

417
Correct Option: C - Carbolic acid
• Carbolic acid, also known as phenol, is a toxic substance that can cause severe poisoning if ingested.
Gastric lavage, or stomach pumping, can be performed in cases of carbolic acid poisoning. The
procedure involves introducing a tube into the stomach to wash out the toxic substance and reduce its
absorption into the body. Gastric lavage is considered beneficial in removing any remaining carbolic
acid from the stomach and can be an important part of the treatment protocol for carbolic acid
poisoning.
Incorrect Options
Option A: Sulphuric acid (Incorrect) Sulphuric acid is a highly corrosive and dangerous substance. In c
ases of sulphuric acid poisoning, gastric lavage is generally not recommended as it may cause further
damage to the gastrointestinal tract. Immediate medical attention and appropriate management are cru
cial in cases of sulphuric acid poisoning to minimize harm and provide necessary treatment.
Option B: Strychnine (Incorrect) Strychnine is a highly toxic alkaloid that affects the nervous system. G
astric lavage is typically not performed in cases of strychnine poisoning. The primary focus of treatment
for strychnine poisoning involves managing symptoms, such as muscle spasms and seizures, providin
g supportive care, and administering specific antidotes as appropriate.
Option D: Kerosene oil (Incorrect) Kerosene oil ingestion can lead to chemical pneumonia or aspiration
pneumonia if it enters the lungs. Gastric lavage is not routinely recommended for kerosene oil poisoni
ng. Treatment primarily involves supportive care, monitoring for any respiratory complications, and pro
viding appropriate respiratory support if needed.

Solution for Question 42:


Correct option C: Prazosin is a medication classified as an alpha-1 adrenergic antagonist. It works by b
locking the action of certain receptors in the body, specifically the alpha-1 adrenergic receptors. In the
case of scorpion envenomation, scorpion venom typically contains compounds that can stimulate the r
elease of catecholamines, such as norepinephrine, leading to sympathetic overactivity. This can result i
n symptoms such as hypertension and increased heart rate.
• By blocking the alpha-1 adrenergic receptors, prazosin helps to counteract the effects of the
excessive sympathetic stimulation caused by scorpion venom. It helps to relax smooth muscles in
blood vessels, reducing blood pressure and improving symptoms associated with scorpion
envenomation.
Incorrect Options
Option A: Physostigmine is a medication that acts as an acetylcholinesterase inhibitor, increasing the a
vailability of acetylcholine in the body. While it can be used as an antidote for certain types of poisoning
, such as anticholinergic toxicity, it is not typically used as a specific antidote for scorpion venom.
Option B: Norepinephrine is a naturally occurring neurotransmitter and hormone that is involved in the
sympathetic nervous system response. While norepinephrine might be elevated in scorpion envenomat
ion, it is not considered a specific physiological antidote to scorpion venom.
Option D: Atropine is an anticholinergic medication that blocks the action of acetylcholine in the body. It
is commonly used to counteract the effects of excessive acetylcholine stimulation. However, in the cas
e of scorpion envenomation, the primary issue is excessive sympathetic activity rather than excessive
cholinergic activity. Therefore, atropine is not the preferred physiological antidote for scorpion venom.

Page 42

418
Solution for Question 43:
Correct Option C: Phosphorus
Luminescent stools are associated with phosphorus.
• High levels of phosphorus ingestion or phosphorus poisoning can sometimes cause stools to exhibit a
luminescent or glowing appearance.
Incorrect Options
Option A - Mercury: Mercury is a toxic heavy metal that can cause various health issues when ingested
. However, it is not typically associated with luminescent stools.
Option B - Cadmium: Cadmium is another toxic heavy metal that can be harmful to human health. Like
mercury, it is not commonly associated with luminescent stools.
Option D - Oxalic acid: Oxalic acid is a naturally occurring compound found in various foods. Ingesting
oxalic acid in normal amounts does not typically result in luminescent stools.

Solution for Question 44:


Correct Option B: Sodium amytal
• The correct answer is not option B
• Sodium amytal is known to cause bluish discoloration of the stomach mucosa on post-mortem
examination
Incorrect Options:
Option A - Oxalic acid: Oxalic acid is not typically associated with bluish discoloration of the stomach m
ucosa on post-mortem examination.
Option C - Soneryl: Soneryl is the trade name for a medication called amitriptyline, which is a
tricyclic antidepressant. It is not known to cause bluish discoloration of the stomach mucosa.
Option D - Arsenic: It is known to cause red velvety discoloration of the stomach mucosa.

Solution for Question 45:


Correct Option A: Lead poisoning
• Saturnine gout refers to gout-like symptoms that occur as a result of chronic lead poisoning.
• Lead can interfere with the metabolism of uric acid, leading to elevated levels of uric acid in the blood,
which can result in the deposition of urate crystals in joints, causing symptoms similar to gout.
Incorrect Options:
Option B - Cadmium poisoning: Cadmium poisoning primarily affects the kidneys and can cause kidney
damage, but it is not specifically associated with Saturnine gout.

Page 43

419
Option C - Beryllium poisoning: Beryllium poisoning is associated with lung disease (berylliosis) and is
not directly linked to Saturnine gout.
Option D - Mercury poisoning: Mercury poisoning primarily affects the nervous system and can lead to
symptoms such as tremors, cognitive impairments, and sensory disturbances. It is not specifically asso
ciated with Saturnine gout.

Solution for Question 46:


Correct Option A: Arsenic
• A: Aldrich-Mees line/Reynold's line (present over nails), Aphrodisiac (increase libido)
• R: Red velvety mucosa, Raindrop pigmentation, Rashes (fading measles rash)
• S: Subendocardial hemorrhage/Sheehan hemorrhage
• E: Excess hyperpigmentation of palms and soles
• N: Neuritis (sensory > motor)
• I: Iron oxide antidote (hydrated ferric oxide)
• C: Mimics cholera
Incorrect Options:
Option B,C,D do not mimic cholera

Solution for Question 47:


Correct Option A:
The false statement regarding the management of snakebite in India is Atropine and neostigmine are v
ery useful in the management of krait bite.
Atropine and neostigmine are not typically used in the management of krait snake bites. Kraits are ven
omous snakes found in India, and their bites primarily result in neurotoxic effects. The mainstay of treat
ment for snakebites, including krait bites, is administration of anti-snake venom (ASV) to neutralize the
venom's effects. Supportive measures such as ventilatory support may be required in severe cases.
Incorrect options:
Option B. Neostigmine and ventilatory support are used in treatment along with ASV in certain cases of
snakebite, but this is not specific to krait bites.
Option C. ASV is indeed the mainstay of treatment for snakebites, including krait bites.
Option D. The statement regarding the current ASV in India not being effective against humped pit vipe
r is not mentioned in the options, so we cannot evaluate its accuracy based on the given choices.

Solution for Question 48:

Page 44

420
Correct Option A:
• Zinc chloride: 2. Irritant
• b. Chloral hydrate: 4. Stupefying
• c. Quinine: 1. Abortifacient
• d. Potassium carbonate: 3. Corrosive
Explanation:
• Zinc chloride is an irritant and can cause irritation and damage to the gastrointestinal tract.
• Chloral hydrate is a sedative-hypnotic drug that can induce a state of stupor or unconsciousness.
• Quinine has been historically used as an abortifacient, which means it can induce abortion or
termination of pregnancy.
• Potassium carbonate is corrosive and can cause damage to tissues upon contact.

Solution for Question 49:


Correct Option : A
• Aconite, also known as monkshood or wolfsbane, is a highly toxic plant. It contains various alkaloids,
including aconitine, which is responsible for its poisonous properties. The incorrect statement in this
case is that only the roots of the plant are poisonous.
• In reality, all parts of the aconite plant are considered poisonous, including the roots, stems, leaves,
and flowers. The plant contains toxic alkaloids throughout its structure, not just in the roots. Therefore, it
is important to exercise caution and avoid direct contact or ingestion of any part of the aconite plant.
Incorrect options:
Option B: Causes AV block. Aconite is known to have potent cardiac effects. It can cause significant ca
rdiac toxicity, including cardiac arrhythmias and disturbances in conduction, such as AV block. These e
ffects are attributed to the toxic alkaloids present in the plant, particularly aconitine. Therefore, it is corr
ect to state that aconite can cause AV block.
Option C: Atropine is not specific for aconite poisoning- In cases of aconite poisoning, the primary treat
ment is supportive care and immediate medical attention. Activated charcoal may be administered to h
elp prevent absorption of the toxin. Additionally, supportive measures such as cardiac monitoring, intra
venous fluids, and management of arrhythmias may be necessary. Atropine is not specific for aconite p
oisoning and may not effectively counteract the toxic effects of aconite alkaloids.
Option D:Grow in sub Himalayan range
- This statement is correct it grows in Himalayan range and used in various kind of herbal medicine.

Solution for Question 50:


Correct Option: D
• Heroin: Heroin is a highly addictive opioid drug derived from morphine. It is commonly abused for its
euphoric effects. Heroin is correctly paired with option 3.

Page 45

421
• MDMA: "Eve" is a term commonly used to refer to the drug MDMA, also known as Ecstasy. MDMA is
a synthetic psychoactive drug with stimulant and hallucinogenic effects. Therefore, "Eve" is correctly
paired with option 4.
• Joint: A joint refers to a rolled cigarette containing cannabis (marijuana). Cannabis is commonly
consumed for its psychoactive effects. Therefore, "Joint" is correctly paired with option 1.
• LSD: LSD (lysergic acid diethylamide) is a potent hallucinogenic drug known for its psychedelic
effects. It can induce visual and auditory hallucinations. Therefore, LSD is correctly paired with option 2.
• i - Joint (1) ii - LSD (2) iii - Heroin (3) iv - Eve (4)
Therefore, the correct answer is D.
Incorrect Options:
Option A. i-2, ii-1, iii-4, iv-3: This choice incorrectly pairs Heroin with option 2, Eve with option 1, Joint w
ith option 4, and LSD with option 3. The correct pairings have been explained earlier.
Option B. i-3, ii-2, iii-1, iv-4: This choice incorrectly pairs Heroin with option 3, Eve with option 2, Joint w
ith option 1, and LSD with option 4. Again, the correct pairings have been provided above.
Option C. i-4, ii-2, iii-1, iv-3: This choice incorrectly pairs Heroin with option 4, Eve with option 2, Joint
with option 1, and LSD with option 3. The correct pairings have been explained earlier.

Solution for Question 51:


Correct Option: C
• Smoke inhalation can lead to anemic hypoxia as the cause of death. Anemic hypoxia occurs when
there is a reduced oxygen-carrying capacity of the blood. In the case of smoke inhalation, the inhaled
smoke can contain toxic substances such as carbon monoxide (CO) and hydrogen cyanide (HCN).
These substances can interfere with the normal binding of oxygen to hemoglobin in red blood cells,
resulting in reduced oxygen transport throughout the body.
• Carbon monoxide (CO) is particularly problematic as it has a strong affinity for hemoglobin and forms
carboxyhemoglobin, which prevents the binding of oxygen. Hydrogen cyanide (HCN) interferes with
cellular respiration and inhibits the utilization of oxygen by cells.
Incorrect Options:
Option A. Histotoxic hypoxia: Histotoxic hypoxia refers to the inability of the body tissues to effectively
use oxygen, even when an adequate amount of oxygen is available. It typically occurs due to the impai
rment of cellular metabolism or function, such as in cases of certain drug overdoses. Histotoxic hypoxia
is not the primary cause of death in smoke inhalation.
Option B. Ischemic hypoxia: Ischemic hypoxia is caused by a reduced blood supply to tissues, resultin
g in insufficient oxygen delivery. It can occur due to conditions like cardiac arrest or severe blood loss.
While smoke inhalation can lead to tissue damage and potential ischemia in specific cases, it is not the
primary cause of death in smoke inhalation.
Option D. this option is incorrect because all the given options are not correct.

Solution for Question 52:

Page 46

422
Correct Option B: Nigella sativa.
• The seeds shown in the image resemble the seeds of Nigella sativa, also known as black cumin or
black seed. Nigella sativa is a flowering plant that is native to Southwest Asia and has been used for
centuries in traditional medicine and culinary practices.
• The seeds of Nigella sativa are small, black, and triangular in shape. They have a distinct aroma and
taste. These seeds are known for their potential health benefits and are used in various cuisines and
traditional remedies.
• In traditional medicine, Nigella sativa seeds have been attributed with several medicinal properties,
including anti-inflammatory, antioxidant, and antimicrobial effects. They are often used for their
potential benefits in improving digestion, boosting the immune system, and relieving respiratory
conditions.
• Culturally, Nigella sativa seeds are used in cooking and baking as a spice or flavoring agent. They
add a nutty and slightly peppery taste to dishes and are commonly used in bread, curries, and pickles.
Incorrect Options:
Option A. Tropicana canna: Tropicana canna refers to a specific variety of Canna plant, which is known
for its colorful flowers. The seeds of Tropicana canna are typically not used or recognized for their me
dicinal or culinary purposes. Therefore, this option is incorrect.
Option C. Datura seeds: Datura is a genus of flowering plants that contains several species, including
Datura stramonium. While Datura seeds do exist, they have a distinct appearance and are not similar t
o the seeds shown in the image. Datura seeds are larger, rounder, and typically have a
spiky or prickly texture. Therefore, this option is incorrect.
Option D. Opium seeds: Opium seeds come from the opium poppy plant (Papaver somniferum). These
seeds are small and kidney-shaped, quite different from the triangular seeds shown in the image. Opiu
m seeds are mainly recognized for their role in opium production and are not typically used in culinary
or medicinal applications. Therefore, this option is incorrect.

Solution for Question 53:


Correct Option D: 1-hyoscyamine
• The above image shows Atropa Belladonna, also called the deadly nightshade. It produces an active
poisonous component called hyoscyamine.
• The toxin may also be found in datura Stramonium (Jimson weed)
• Hyoscyamine is an isomer of Atropine and has Anticholinergic and Antimuscarinic activity.
• It is used as an antiemetic, antisecretory and antispasmodic agent in the treatment of nausea, motion
sickness, allergic rhinitis etc.
• Side effects include dryness of the mouth and eyes, tachycardia, palpitations, decreased sweating,
and can also produce anxiety and restlessness
Incorrect Options:
Option A: Calotropis Gigantea
• It is a purple colored flower which has an active poisonous principle called Calotropin, Calotoxin etc.

Page 47

423
Option B: Digitalis
• The image belows shows the plant digitalis purpurea. It is also called Purple FoxGlove

Option C: Ricin -It is produced by a castor plant, also called Arandi.

Solution for Question 54:


Correct Option C: Body packer syndrome
• Body packer’s syndrome, also called the surgical mules syndrome is a planned activity to smuggle
drugs. The drug to be smuggled is usually wrapped in polythene or condom, and swallowed/planted
inside vagina or rectum. Keywords such as Laxatives, enema apparatus and airport points towards it.

Page 48

424
• The x-ray shows cylindrical objects which may be containers for the drugs
Incorrect Options:
Option A: Bezoar Syndrome- It is a syndrome in which there is a
tightly packed collection of something inside the stomach. Usually, bezoar occurs for a
collection of hair inside the stomach, but it can be any indigestible natural/unnatural object.
Option B: Pica due to anemia - It does not present as shown in the case history.
Option D: Constipation due to fecalith- It mostly shows excessive bloating, not shown in the x ray

Solution for Question 55:


Correct Ans: A
• Erythroxylum coca, commonly known as the coca plant, contains the alkaloid cocaine. While cocaine
is primarily known as a powerful stimulant, it can induce delirium and hallucinations at high doses or in
certain circumstances. Thus, Erythroxylum coca can produce a deliriant effect when consumed in large
quantities.

Incorrect option
Option B. Datura: Datura plants, such as Datura stramonium, are known for producing deliriant toxins.
They contain tropane alkaloids like scopolamine, hyoscyamine, and atropine, which are well-known for
their deliriant properties. However, Datura is not the correct option in this context.

Page 49

425
Option C. Hyoscine: Hyoscine, also known as scopolamine, is an alkaloid found in various plants, inclu
ding Datura. It is known for its deliriant effects. However, in this question, the correct option is not Hyos
cine itself but rather the plant that produces the deliriant toxin.
Option D. Digitalis purpura : Digitalis purpurea, commonly known as foxglove, is not associated with pr
oducing deliriant toxins. It contains cardiac glycosides that primarily affect the heart.

Solution for Question 56:


Correct Ans: A
• 1-D; 2-C; 3-A; 4-B: This is the correct matching. The images are correctly matched with their
corresponding snake species.
Incorrect option:

Page 50

426
Option B. 1-A; 2-B; 3-C; 4-D: This is not the correct matching. The images are mismatched with their c
orresponding snake species.
Option C. 1-D; 2-C; 3-B; 4-A: This is not the correct matching. The images are not matched correctly wi
th their corresponding snake species.
Option D. 1-D; 2-A; 3-B; 4-C: This is not the correct matching. The images are not matched correctly wi
th their corresponding snake species.

Solution for Question 57:


Correct
Option E. The assertion is false but reason is true:
• This option suggests that the assertion (eosinophilia as the most common hematological finding in
lead poisoning) is false, but the reason (lead inhibits ALA dehydratase) is true. Therefore, option E is
correct.
• Eosinophilia is not the most common hematological finding in lead poisoning, Most common
haematological finding in lead poisoning is microcytic hypochromic anemia with ringed sideroblasts and
basophilic stippling.
• The primary problem in lead poisoning is blockage of enzymes of heme synthesis and these are ALA
and Ferrochelatase.

Incorrect Option
Option A. Both assertion and reason are true, and the reason is the correct explanation for the assertio
n:
• This option suggests that eosinophilia is the most common hematological finding in cases of lead
poisoning because lead inhibits ALA dehydratase. Therefore, option A is incorrect.
Option C. Assertion is true, but the reason is false:
• This option suggests that the assertion (eosinophilia as the most common hematological finding in
lead poisoning) is true, but the reason (lead inhibits ALA dehydratase) is false. Therefore, option C is
incorrect.
Option D. Both assertion and reason are false:
• This option states that both the assertion and reason are false. Therefore, option D is incorrect.
Answer: B
• Both assertion and reason are true, but the reason is not the correct explanation for the assertion:
This option is incorrect
• This option is incorrect
• This option is incorrect

Page 51

427
Solution for Question 58:
Correct Answer: B
• Corrosive poison. Corrosive poisons have the maximum damage potential as they can cause severe
chemical burns and tissue damage.
• Corrosive poisons, such as strong acids and alkalis, can cause extensive damage when they come
into contact with the body. They can lead to severe chemical burns, tissue destruction, and long-term
complications. Irritant poisons, alcohol, and opioids can also cause harm, but they generally do not
have the same level of damage potential as corrosive poisons.
Option A (Irritant poison): While irritant poisons can cause harm, they generally have a
lower damage potential compared to corrosive poisons.
Option C (Alcohol): Alcohol can be toxic in excessive amounts or when combined with certain substanc
es, but it typically does not have the same level of damage potential as corrosive poisons.
Option D (Opioid): Opioids are potent drugs that can have serious health effects, including overdose an
d addiction. However, in terms of damage potential, corrosive poisons generally pose a higher risk.

Solution for Question 59:


Correct Answer: C- 1-E; 2-B; 3-D;4-A
Column A lists the poisons, and Column B lists the corresponding tests.
• Marsh test: Marsh test is used to detect the presence of arsenic. It involves generating arsine gas
from the sample and using it to produce a characteristic brown or black deposit on a cold surface.
• Silver nitrate paper test: The silver nitrate paper test is used to detect the presence of cyanide. When
exposed to cyanide, the silver nitrate paper turns yellow or brown.
• Cavett test: The Cavett test is used to detect the presence of alcohol. It involves using potassium
dichromate and sulfuric acid to oxidize the alcohol, producing a colour change.
• Marquis test: Marquis test is used to detect the presence of opium alkaloids. It involves the addition of
Marqui's reagent to the sample, resulting in a characteristic color change. It can be used to test
cocaine, opiates and phenethylamines.
• It can be used to test cocaine, opiates and phenethylamines.
• It can be used to test cocaine, opiates and phenethylamines.
Incorrect options: Option A/ Option B/ Option D

Solution for Question 60:


Correct Option D.
• Belladonna, also known as deadly nightshade, contains tropane alkaloids such as atropine and
scopolamine. These substances are known to cause anticholinergic effects, which include dry mouth,
urinary retention, constipation, increased heart rate (tachycardia), and dilated pupils (mydriasis).
Incorrect Option:

Page 52

428
Option A. Heroin: Heroin is an opioid drug that acts on opioid receptors in the central nervous system.
While opioids like heroin can cause respiratory depression, they typically do not produce significant ant
icholinergic effects. Common effects of heroin use include euphoria, drowsiness, pinpoint pupils (miosi
s), respiratory depression, and constipation. Dry mouth, urinary retention, and dilated pupils are not typ
ical symptoms associated with heroin use.
Option B. Morphine: Morphine is another opioid drug with similar effects to heroin. Like heroin, morphin
e primarily acts on opioid receptors and can cause respiratory depression and constipation. However, it
does not typically produce pronounced anticholinergic effects such as dry mouth, urinary retention, an
d dilated pupils.
Option C. Malathion: Malathion is an organophosphate insecticide. Ingestion of malathion can lead to s
ymptoms such as excessive salivation, sweating, and constricted pupils (miosis), rather than the antich
olinergic effects described in the case.

Solution for Question 61:


Correct Answer: B
• 1, 2, and 3 are correct. Atropine, colchicine, and curare are all toxins.
• Atropine, colchicine, and curare are substances that possess toxic properties. Atropine is a naturally
occurring substance found in certain plants, colchicine is a toxic alkaloid found in the autumn crocus
plant, and curare refers to a group of natural toxins obtained from South American plants. Arsenic, on
the other hand, is not classified as a toxin, but rather as a toxic element.
Incorrect option:
Option A (All are correct): This option is incorrect because arsenic is not considered a toxin.
Option C (1, 3
are correct): This option is incorrect because it fails to include colchicine (option 2), which is indeed a
toxin.
Option D (3, 4 are correct): This option is incorrect because it includes arsenic (option 4), which is not a
toxin

Solution for Question 62:


Correct Option: Option A -Option A,B,C
Option A. Pain in abdomen, burning sensation, diarrhea seen in digoxin toxicity .This statement is true.
Gastrointestinal symptoms, like Nausea, vomiting, pain in abdomen, burning sensation, diarrhea.
Option B - Vision changes like Transient ambylopia, photophobia, may be noted in digoxin toxicity: This
statement is true. Digoxin toxicity can cause Transient ambylopia, blurring, photophobia, scotoma, dipl
opia, color aberration
Option C Specific antidote for cardiac arrhythmias is lignocaine 100 mg IV or novocaine or propranolol.
This statement is true.
Incorrect Options: Option B/C/D
Option D. Death occurs from Respiratory failure . This is a false statement .

Page 53

429
• Death occurs from cardiovascular collapse.

Page 54

430
Techniques in Postmortem
1. Read the following statements regarding autopsy techniques 1. Small intestine is opened along the
ante-mesenteric border 2. Large intestines are opened along anterior taenia
A. Both are true
B. Both are false
C. 1 is true, 2 is false
D. 2 is true, 1 is false
----------------------------------------
2. A 25-year-old woman has a stillbirth. During the 24th week of pregnancy, she experienced bleeding
from the vagina associated with crampy abdominal pain. She had also not experienced fetal movement
since. On examination, the obstetrician was unable to detect any fetal heart sounds. After delivery, the
baby's body was sent for autopsy to determine the cause of death. Which part of the body will the
doctor open first during the autopsy?
(or)
In the autopsy of a stillborn baby following a 24-week pregnancy with vaginal bleeding and absent fetal
movement, which part of the body is opened first to determine the cause of death?
A. Head
B. Chest
C. Abdomen
D. Back
----------------------------------------
3. The body of a 45-year-old male with a known history of HIV infection has been sent for postmortem
investigations. What is the best method of organ removal in the autopsy of this case?
(or)
What is the best method of organ removal in the autopsy of HIV +ve case?
A. Letulle’s method
B. Ghon’s method
C. Rokitansky method
D. Virchow method
----------------------------------------
4. Which of the following is dissected using the instrument shown?
(or)
Which of the following is dissected using the instrument shown?

431
A. Femur
B. Liver
C. Spinal cord
D. Brain
----------------------------------------
5. The forensic team has begun the autopsy of a 55-year-old man who became unconscious following a
road traffic accident. A CT scan conducted six months prior to the injury had confirmed an aneurysm in
the anterior communicating artery. Given this information, the forensic doctor decides to perform a
cranial autopsy. Where should the incision in the scalp be started to proceed with the autopsy?
(or)
Where should the forensic doctor make the scalp incision to perform the cranial autopsy
A. At occiput
B. Behind the ear lobe
C. In front of the ear lobe
D. At vertex
----------------------------------------
6. The body of the 63-year-old man who was a known case of adenocarcinoma of the stomach has
been sent for an autopsy. He had been diagnosed with cancer after an esophagogastroduodenoscopy
was done 3 years ago. The forensic doctor will now examine the stomach after performing which of the
following procedures?
(or)
What procedure will the forensic doctor perform to examine the stomach during the autopsy?
A. Double ligation
B. Single ligation
C. Cut open
D. Triple ligation
----------------------------------------
7. A 55-year-old male, presents to the emergency department with complaints of severe chest pain,
radiating to his left arm and jaw. He has a history of hypertension and hyperlipidemia. An immediate
electrocardiogram (ECG) shows ST-segment elevation in the anterior leads (V1 to V4).The patient

Page 2

432
could not survive and is declared dead.Which of the following method is very useful for demonstrating
the distribution of infarction?
(or)
Which heart dissection method is useful for demonstrating infarction distribution?
A. Inflow outflow method
B. Ventricular slicing method
C. Window method
D. Perfusion method
----------------------------------------
8. The forensic doctor is about to begin the autopsy of the 50-year-old farmer who had allegedly
committed suicide by consuming an organophosphate. The doctor prefers an organ for the detection of
the poison as the sample can be collected with ease, a high concentration of toxins can be detected in
that tissue, and also because of the availability of a large database of drug concentrations. Which organ
is the doctor about to examine?
(or)
Which organ is the preferred choice for detecting organophosphate poisoning in the autopsy of a
50-year-old farmer who allegedly committed suicide?
A. Stomach
B. Spleen
C. Kidney
D. Liver
----------------------------------------
9. The body of the 25-year-old who had allegedly committed suicide by hanging in the hostel premises
has been brought for autopsy. The doctor decides to collect the specimens via the most commonly
choosen method, though the anatomical relations of the organs cannot be maintained by this method.
How will the organs be collected if he has chosen Virchow's method?
(or)
How are organs collected for autopsy using Virchow's method
A. Organs are removed En masse
B. Organs are removed En block
C. Organs are removed one by one
D. In situ dissection combined with en bloc removal
----------------------------------------
10. A 40-year-old male is found deceased at home. The circumstances surrounding his death are
unclear, and there is no known medical history of significant illnesses. Law enforcement has requested
a thorough autopsy to determine the cause of death.All of the following are types of incisions made to
perform an autopsy, except?
(or)
All of the following are types of incisions made to perform an autopsy, except?

Page 3

433
A. ‘I’ shaped
B. ‘Y’ shaped
C. Modified ‘I’ shaped
D. Modified ‘Y’ shaped
----------------------------------------

Correct Answers
Question Correct Answer

Question 1 4
Question 2 3
Question 3 3
Question 4 3
Question 5 2
Question 6 1
Question 7 2
Question 8 4
Question 9 3
Question 10 3

Solution for Question 1:


Correct Option D - 2 is true, 1 is false:
1. Small intestine is opened along the ante- mesenteric border
• The small intestine is opened along the mesenteric border.
2. Large intestines are opened along anterior taenia
• The large intestine is opened along the anterior taenia.

Solution for Question 2:


Correct Option C - Abdomen:
• The abdomen is opened first during the autopsy of a new-born.
Incorrect Options:
Option A - Head:
• The purpose of conducting an autopsy on a newborn is to know whether the baby is live born or not.
• The head should not be opened first, as it may cause death to the live child.
Option B - Heart:

Page 4

434
• The heart should not be opened first, as it may cause death to the live child.
Option D - Back:
• The back should not be opened first, as it may cause spinal cord damage to the live child.

Solution for Question 3:


Correct Option C - Rokinansky method:
• Rokitansky's technique: It involves in situ dissection in part, combined with en block removal.
• It is the preferred choice in patients with highly transmissible diseases, such as HIV, and hepatitis B.
Incorrect Options:
Option A - Letulle’s method:
• Lettulle's technique: Cervical, thoracic, abdominal, and pelvic organs are removed en masse and
dissected as organ blocks.
• It has the advantage of leaving all attachments intact.
Option B - Ghon’s method:
• Ghon's Technique: Cervical, thoracic, abdominal organs and urogenital system are removed as organ
blocks.
• The neuronal system is removed as another block.
Option D - Virchow method:
• Virchow's technique: Individual organs are removed one by one.
• The cranial cavity is exposed first, followed by the thoracic, cervical, and abdominal organs. The
spinal cord is removed from the back.
• In this, the anatomico-pathologic relations are not preserved.

Solution for Question 4:


Correct Option C - Spinal cord:
• The instrument shown in the image is a rachiotomy saw, which is used for dissecting the spinal cord
Incorrect Options:
Option A - Femur:
• Rachiotomy saw is not used in the dissection of the femur.
Option B - Liver:
• Multiple parallel cuts are made along the liver & examine it.
• Liver is preferred for the detection of poisons as the sample can be collected with ease, a high
concentration of toxins can be detected in that tissue, and also because of the availability of a large
database of drug concentrations.

Page 5

435
• Rachiotomy saw is not used in the dissection of the liver.
Option D - Brain:
• In poisoning cases, the cranial cavity is opened 1st, where the smell associated with the poison can
be perceived.
• Ideally, the brain has to be dissected after fixation with formalin (10%) for a week.
• Sometimes, fresh dissection can also be done.
• Among fresh dissection methods, the cranial cutting method [multiple cranial slices are made &
parenchyma of the brain is studied] is better.
• Rachiotomy saw is not used in the dissection of the brain.

Solution for Question 5:


Correct Option B - Behind the ear lobe:
• During an autopsy, a coronal incision is made in the scalp. This incision begins at the mastoid process
just behind one ear, extends over the vertex of the scalp, and ends at the back of the opposite ear
Incorrect Options:
Option A - At occiput: A coronal incision is carried over the vertex of the scalp, not at occiput.
Option C - In front of the ear lobe: A coronal incision is made in the scalp, which starts from the mastoi
d process just behind one ear, not in front of the ear lobe.
Option D - At vertex: A coronal incision is made in the scalp, which starts from the mastoid process just
behind one ear, and is carried over the vertex of the scalp to the back of the opposite ear (intermastoid
al incision).

Solution for Question 6:


Correct Option A - Double ligation:
• Since the stomach is connected both to the esophagus and duodenum so double ligature along the
greater curvature of the stomach is usually done.
Incorrect Options:
Option B - Single ligation: Since the stomach is connected both to the esophagus and duodenum, singl
e ligation is not useful here.
Option C - Cut open: Since the stomach is connected both to the esophagus and duodenum, the cut o
pen technique is not usually done to take the stomach.
Option D - Triple ligation: Since the stomach is connected both to the esophagus and duodenum, triple
ligation is not useful here.

Solution for Question 7:

Page 6

436
Correct Option B - Ventricular slicing method:
• It is useful if an ischemic myocardial disease is suspected as it clearly demonstrates the distribution of
infarction.
• In this method, the diaphragmatic aspect is kept over the paper tower to prevent slippage.
• 1-1.5 cm thick cuts parallel to atrio ventricular groove with a long knife.
Incorrect Options:
Option A - Inflow outflow method:
• The most commonly used & easy method to dissect the heart is Inflow outflow method
• I.e Dissection follows the direction of blood flow in the heart
• RA → RV → LA → LV
• It does not demonstrate the distribution of infarction.
Option C - Window method:
• In this method, perfusion was fixed, and windows of various sizes were removed with a scalpel and
sent to the HP study.
• Useful for cardiac museum specimens.
• It does not demonstrate the distribution of infarction.
Option D - Partition method:
• In this method, coronaries and epicardial fats are stripped off.
• Ventricles separated from IV septum and atria removed.
• All are weighted separately.
• It does not demonstrate the distribution of infarction.

Solution for Question 8:


Correct Option D - Liver:
• The doctor prefers the liver for the detection of the poison as the sample can be collected with ease, a
high concentration of toxins can be detected in that tissue, and also because of the availability of a
large database of drug concentrations.
Incorrect Options:
Option A - Stomach:
• In cases of Suicide due to organophosphate poisoning, high concentrations of toxins can be detected
in the liver than in the stomach.
• Thus liver is the preferred organ to preserve.
Option B - Spleen:
• Spleen is also routinely preserved along with the liver.
• But the liver is the most important organ to preserve because of the large concentration of toxins.
Option C - Kidney:

Page 7

437
• The kidney is also routinely preserved along with the liver.
• But the liver is the most important organ to preserve because of the large concentration of toxins.

Solution for Question 9:


Correct Option C - Organs are removed one by one:
• Virchow's technique: Individual organs are removed one by one.
• The cranial cavity is exposed first, followed by the thoracic, cervical, and abdominal organs. The
spinal cord is removed from the back.
• In this, the anatomico-pathologic relations are not preserved.
Incorrect Options:
Option A - Organs are removed En masse:
• Lettulle's technique: Cervical, thoracic, abdominal, and pelvic organs are removed en masse and
dissected as organ blocks.
• It has the advantage of leaving all attachments intact.
Option B - Organs are removed En block:
• Ghon's Technique: Cervical, thoracic, abdominal organs and urogenital system are removed as organ
blocks. The neuronal system is removed as another block.
Option D - In situ dissection combined with en bloc removal:
• Rokitansky's technique: It involves in situ dissection in part, combined with en block removal. It is the
preferred choice in patients with highly transmissible diseases, such as HIV, and hepatitis B.

Solution for Question 10:


Correct Option C - Modified ‘I’ shaped:
• A modified ‘I’ shaped incision is not a type of incision made to perform an autopsy.
Incorrect Options:
Option A - ‘I’ shaped:
• Incision extending from the chin straight down to the symphysis pubis.
• It is the most common type of incision.
Option B - ‘Y’ shaped:
• 2 incisions each starting from acromion processes, meet in the midline near the xiphisternum and then
extend downwards straight.
Option D - Modified ‘Y’ shaped:
• It is used when a detailed study of the neck is required e.g., Asphyxial death due to neck compression
(hanging or strangulation).

Page 8

438
Page 9

439
Thanatology
1. On examination of the body of the 28-year-old actress, who had allegedly committed suicide by
hanging, the forensic doctor noticed hypostasis (reddish-purple discolouration) in the hands, the feet,
and around the genitalia of the victim. He documented the same as "Suggilation" which can also be
documented as?
(or)
"Suggilation" can also be documented as?
A. Postmortem calorcity
B. Rigor mortis
C. Algor mortis
D. Postmortem staining
----------------------------------------
2. The controversial Guru was invited by the National Institute of Sciences to perform his meditation
while being strictly monitored by a team of scientists. The team concluded that he was indeed able to
remain in a state of suspended animation for 1 hour before regaining consciousness. It is defined as a
state of temporary (short- or long-term) slowing or stopping of biological function so that physiological
capabilities are preserved. It may be either hypometabolic or ametabolic in nature. Such a state of
suspended animation also occurs secondary to which one of the following conditions?
(or)
State of suspended animation also occurs secondary to which one of the following conditions?
A. Lead poisoning
B. Cyanide poisoning
C. Burns
D. Drowning
----------------------------------------
3. The forensic team has concluded the autopsy of the 35-year-old clerk who had suddenly collapsed
during his night shift. The final report states that it was a Negative autopsy. What does the team mean
when using that term?
(or)
What is the meaning of a "Negative autopsy" in the final report?
A. No cause of death is found on gross as well as histopathological examination
B. Cause is apparent on gross examination but not on histopathological examination
C. Gross finding are minimal
D. Cause is apparent on gross examination but not found because of constrains on the part of doctor
----------------------------------------
4. While reporting to the police who are investigating the death of the 30-year-old actress, the forensic
team was enquired about the possible time frame of the murder. The forensic team has an approximate
time of death which they determine from various observations. In the eyes, ‘trucking’ or segmentation of
retinal blood vessels is one of the first observable signs. The skin loses its elasticity and luster within

440
the first few hours after death and appears pale. Histological examination of the skin, shows no
morphological changes within 6 hours post-mortem. All of the following can also be studied by the team
to narrow the time frame of death, except?
(or)
All of the following can also be studied to narrow the time frame of death, except ?
A. Cadaveric spasm
B. Algor mortis
C. Rigor mortis
D. Livor mortis
----------------------------------------
5. Which of the following is proof of eyes being open for a few hours after a death?
(or)
Which of the following is proof of eyes being open for a few hours after a death?
A. Kevorkian sign
B. Tach noir
C. Both of the above
D. None of the above
----------------------------------------
6. A group of National Geographic documentators was scouring the marshes for a good location to
shoot the next episode of "Man in the Wild" when they accidentally discovered the remains of a corpse
among the vegetation. They immediately informed the concerned authorities and the body was
collected by the forensic team for further investigation. There was a thick layer of adipocere formation
which the team knew would have preserved certain features and injuries that would aid in the
determination of identity and in the cause of death of the body. Which among the following statements
is not true of adipocere formation?
(or)
Which among the following statements is not true of adipocere formation?
A. It’s a modification of putrefaction
B. Its developed in the presence of moist air
C. It occurs in dead bodies lying in water
D. Body has rotten & foul smell
----------------------------------------
7. Which of the following is Nysten’s law?
(or)
Which of the following is Nysten’s law?
A. Cadaveric rigidity does not appear in living people
B. Cadaveric rigidity appears faster in older and the young than in the middle age
C. Cadaveric rigidity occurs in men earlier than in the women

Page 2

441
D. Cadaveric rigidity affect successively the masticatory muscles, those of the face and the neck, those
of the trunk and arms and finally those of the lower limbs
----------------------------------------
8. A 22-year-old woman went into the woods early in the morning to walk her dog. The dog, on
identifying a different scent, led the woman to a decomposed body half exposed from the ground
behind an old oak tree. The late-night shower the previous night must have eroded the soil and
exposed the body. Putrefaction of the body is mainly caused by the action of the enzyme Lecithinase,
which is produced by which one of the following organisms?
(or)
Which organism produces the enzyme Lecithinase, responsible for the putrefaction of the body?
A. Streptococcus pyogens
B. Clostridium welchii
C. Pseudomonas aeruginosa
D. None
----------------------------------------
9. Which of the following is not an exclusion of brainstem death?
(or)
Which of the following is not an exclusion of brainstem death?
A. Patient under effect of drug overdose
B. Patient with severe head injury
C. Core body temperature is below 35 degree Celsius
D. Patient is suffering from severe metabolic/endocrine disturbance
----------------------------------------
10. The gas has to be analysed by gas chromatography. What is the expected colour change if the
Pyrogallol test is used to confirm the presence of antemortem air embolism?
(or)
What is the expected colour change in Pyrogallol test used to confirm the presence of antemortem air
embolism?
A. Orange to brown
B. Green to yellow
C. Colourless to brown
D. Colourless to pink
----------------------------------------
11. The body of the 10-year-old boy has been recovered from the lake. He had allegedly strolled away
from his family who had picnicked near the lake for the long weekend. The body displayed cadaveric
spasm . Which statement among the following about cadaveric spasm is not true?
(or)
Which statement among the following about cadaveric spasm is not true?

Page 3

442
A. Same mechanism as rigor mortis
B. Can be seen in cases of drowning
C. Also known as instantaneous rigor
D. Molecular death does not occur
----------------------------------------
12. One of the following terms is the study of how organic remains pass from the biosphere to the
lithosphere, and this includes processes affecting remains from the time of death of an organism (or the
discard of shed parts) through decomposition, burial, and preservation as mineralized fossils or other
stable biomaterials. What is the term for the Study of the postmortem process of a dead body and its
interpretation?
(or)
What is the term for the Study of the postmortem process of a dead body and its interpretation?
A. Thanatology
B. Putrefaction
C. Taphonomy
D. Entomology
----------------------------------------
13. The bodies of the 2 students who had been swept away by a flash flood have been recovered. The
body of the first has been found washed up on the bank near a field. The second had been washed
further down and was seen among the vegetation of the marshes near the mouth of the river. What is
the major factor that influences the onset and duration of algor mortis?
(or)
What is the major factor that influences the onset and duration of algor mortis?
A. Muscle built of the person
B. Environmental temperature
C. The damage sustained by the tissues
D. Age
----------------------------------------
14. Degloving is seen in which of the following?
(or)
Degloving is seen in which of the following?
A. Hanging
B. Rigor mortis
C. Mummification
D. Putrefaction
----------------------------------------
15. When the body of the 25-year-old woman was examined by the forensic doctor, the muscles
appeared to be in a flaccid state. When the doctor struck the lower third of the quadriceps femoris

Page 4

443
muscle, above the patella with a reflex hammer, it caused an upward movement of the patella. The
doctor immediately documented the same as Zasko's phenomenon. Which stage will this phenomenon
be seen in?
(or)
Zasko's phenomenon seen at which stage ?
A. Rigor mortis
B. Decomposition
C. Primary relaxation
D. Secondary relaxation
----------------------------------------
16. When the body of the 26-year-old woman was brought in for an autopsy, it appeared to be in rigor
mortis, helping the doctor understand that the time of death was at least 6 to 8 hours prior. Rigor mortis,
the postmortem rigidity, is the third stage of death. It is one of the recognizable signs of death,
characterized by stiffening of the limbs of the corpse caused by chemical changes in the muscles
postmortem. It follows all of the following rules, except?
A. Shapiro
B. Nysten
C. Morrison
D. Rule of 12
----------------------------------------
17. What is the time since death in the below image of a body found dead in summer in India?
(or)
What is the time since death in the below image of a body found dead in summer in India?

A. 12-18 hours
B. 1-2 days
C. 3-5 days
D. 7-10 days
----------------------------------------
18. Among the following options mark the incorrect statement(s) about Cadaveric spasm

Page 5

444
(or)
Which statement(s) is/are incorrect about Cadaveric spasm?
A. Pugilistic attitude
B. Occurs immediately after death
C. Intensity of muscle contraction is much more as compared to rigor mortis
D. Only selected muscles involved
E. Flexion of elbows, knees and clenching of hand into a fist
----------------------------------------
19. Autolysis is the breakdown of the body by endogenous substances. It proceeds most rapidly in
organs such as the pancreas and stomach. It may predominate in more arid conditions and can
eventually result in mummification. Which component of the cell contributes towards autolysis?
(or)
Which component of the cell contributes towards autolysis?
A. Lecithinase
B. Lipase
C. Lysosomes
D. ATPase
----------------------------------------
20. Which of the following conditions favour mummification?
(or)
Which of the following conditions favours mummification?
A. Warm & Humid climate
B. Absence of moisture
C. Clostridium perfringes
D. Intrinsic Lipases
----------------------------------------
21. Which of the following is used for determining time since death by algor mortis?
(or)
What is used to determine the time since death through algor mortis?
A. Madea’s formula
B. Calorimetry formula
C. Nysten’s rule
D. Henssge nomogram
----------------------------------------
22. All are false regarding muscular changes after death, except?
(or)

Page 6

445
All are false regarding muscular changes after death, except?
A. Rigor mortis immediately after the death
B. Cadaveric spasm occurs 1 hour after death
C. Rigor mortis involves voluntary muscles & involuntary muscles
D. Cadaveric spasm involves voluntary muscles & involuntary muscles
----------------------------------------
23. Which of the following is a true statement about exhumation?
(or)
Which of the following is a true statement about exhumation?
A. Police can order for exhumation
B. Executive magistrate can order for exhumation
C. Postmortem cannot be done on exhumated body
D. CrPC 174 is related to enquiry of exhumated body
----------------------------------------
24. According to the Transplantations of Human Organ and Tissue Act, all of the following specialists
are authorized for pronouncing brain death except?
(or)
According to the Transplantation of Human Organs and Tissues Act, which specialist is not authorized
to pronounce brain death?
A. Neurosurgeon
B. Doctor performing the liver transplant
C. Doctor attending the patient
D. RMP in charge of the hospital
----------------------------------------
25. During the autopsy, core body temperature is best measured in a case of sodomy, from which part?
(or)
What is the best site to measure core body temperature during an autopsy in a case of sodomy?
A. Rectum
B. Inferior surface of liver
C. Intra-aural (thermometer up to tympanic membrane)
D. Intra-nasal (thermometer up to cribriform plate)
----------------------------------------
26. Which of the following distinguishes an antemortem blister from a postmortem blister?
(or)
Which of the following distinguishes an antemortem blister from a postmortem blister?

Page 7

446
A. Presence of Albumin & Chloride in blister fluid
B. Gas in blister
C. Enzymatic reaction are absent
D. Absence of hyperemia around the blister
----------------------------------------
27. Algor mortis, the second stage of death, is the change in body temperature post-mortem until the
ambient temperature is matched. This is generally a steady decline that is proportional to the difference
between the temperatures of the internal and external environment. In all the following conditions, this
decline is not seen for a few hours after death (due to postmortem caloricity) except?
(or)
In all the following conditions, steady decline between the temperatures of the internal and external
environment is not seen for a few hours after death (due to postmortem caloricity) except?
A. Pontine haemorrhage
B. Bacteremia
C. Postmortem glycogenolysis
D. Status epilepticus
----------------------------------------
28. Swiss- cheese liver found in.
(or)
Swiss- cheese liver seen in :
A. Adipocere
B. Alcoholism
C. Putrefaction
D. Drowning
----------------------------------------
29. Which of the following is not associated with putrefaction?
(or)
Which of the following is not associated with putrefaction?

Page 8

447
A. Bacterial activity.
B. Involvement of Clostridium welchii or Clostridium perfringens.
C. Conversion of organic matter into fossils.
D. Production of lytic lecithinase enzyme.
----------------------------------------
30. Which of the following is present in the image below:
(or)
Which of the following is present in the image below:

A. Rigor mortis
B. Algor mortis
C. Suggillation
D. Marbling
----------------------------------------

Correct Answers
Question Correct Answer

Question 1 4
Question 2 4
Question 3 1
Question 4 1
Question 5 2
Question 6 4
Question 7 4
Question 8 2
Question 9 2
Question 10 3
Question 11 1

Page 9

448
Question 12 3
Question 13 2
Question 14 4
Question 15 3
Question 16 3
Question 17 1
Question 18 1,5
Question 19 3
Question 20 2
Question 21 4
Question 22 3
Question 23 2
Question 24 2
Question 25 2
Question 26 1
Question 27 3
Question 28 3
Question 29 3
Question 30 3

Solution for Question 1:


Correct Option D - Postmortem staining:
• Postmortem Staining Synonyms: Livor Mortis, Hypostasis, lucidity, cogitation, vibices, suggilation,
postmortem or cadaveric lividity, darkening of death.
Incorrect Options:
Option A - Postmortem calorcity: Postmortem caloricity is a
phenomenon where the body temperature of a corpse rises or remains unusually high for up to 2
hours after death instead of falling.
Option B - Rigor mortis: Rigor mortis (stiffness of death) is the state of the muscles in a
dead body when they become stiff or rigid with some degree of shortening. It is a
sign of death and indicates molecular death of the muscle involved.
Option C - Algor mortis: Algor mortis is the cooling of the body that normally takes place after death, w
here the body temperature equilibrates with its environmental temperature.

Solution for Question 2:


Correct Option D - Drowning:

Page 10

449
• When a person falls into water, he becomes panic & voluntarily holds the breath.
• Later, due to air hunger, reflex Iinspiratory effort occurs & water enters rapidly into the airway.
• Suspended animation/ apparent death also occurs secondary to drowning.
Incorrect Options:
Option A - Lead poisoning:
• Lead poisoning causes microcytic hypochromic anemia.
• In lead poisoning, 5-pyrimidine nucleotidase is inhibited & ribosome degradation doesn't happen,
resulting in basophilic stippling of RBC.
• Suspended animation/apparent death does not occur in lead poisoning.
Option B - Cyanide poisoning:
• In cyanide poisoning, brick-red hypostasis is seen.
• Suspended animation/ apparent death does not occur in cyanide poisoning.
Option C - Burns:
• If the temperature is more, an even shorter duration of contact can produce deeper injuries. 45°C for 6
hrs exposure causes deep burns. 64°C for 45 sec exposure causes deep burns.
• 45°C for 6 hrs exposure causes deep burns.
• 64°C for 45 sec exposure causes deep burns.
• Suspended animation/ apparent death does not occur in burns.
• 45°C for 6 hrs exposure causes deep burns.
• 64°C for 45 sec exposure causes deep burns.

Solution for Question 3:


Correct Option A - No cause of death is found on gross as well as histopathological examination:
• Negative autopsy: In about 2–5% of all postmortem examination cases, the cause of death remains
unknown, even after all laboratory examinations including biochemical, microbiological, virological,
microscopic and toxicological examination.
Incorrect Options:
Option B - Cause is apparent on gross examination but not on histopathological examination:
• If the cause is apparent on gross examination, it is not a negative autopsy.
Option C - Gross findings are minimal:
• If the gross findings are minimal, it is termed an obscure autopsy.
• Additional tests are done to determine the cause of death i.e, Histopathological examination [HPE].
Microbiological examination. Chemical/ Toxicological examination etc.
• Histopathological examination [HPE].
• Microbiological examination.
• Chemical/ Toxicological examination etc.

Page 11

450
• Histopathological examination [HPE].
• Microbiological examination.
• Chemical/ Toxicological examination etc.
Option D - Cause is apparent on gross examination but not found because of constraints on the part of
the doctor:
• If the cause is apparent on gross examination, it is not a negative autopsy.

Solution for Question 4:


Correct Option A - Cadaveric Spasm:
• Cadaveric spasm is a condition in which the muscles of the body which were in a state of contraction
immediately before death, continue to be so after death without passing through the stage of primary
relaxation.
• Cadaveric spasm, being an antemortem phenomenon, reflects the last act of the subject performed
before and at the time of his death. The cause and the manner of death may be judged.
Incorrect Options:
Option B - Algor Mortis:
• Algor mortis is the cooling of the body that normally takes place after death, where the body
temperature equilibrates with its environmental temperature. It is a sign of death. It helps in the
estimation of the time of death. Rapid cooling of a dead body delays the processes of rigor mortis and
decomposition. If the heat is preserved for a longer period, then both processes start early.
• It is a sign of death.
• It helps in the estimation of the time of death.
• Rapid cooling of a dead body delays the processes of rigor mortis and decomposition. If the heat is
preserved for a longer period, then both processes start early.
• It is a sign of death.
• It helps in the estimation of the time of death.
• Rapid cooling of a dead body delays the processes of rigor mortis and decomposition. If the heat is
preserved for a longer period, then both processes start early.
Option C - Rigor mortis:
• Rigor mortis (stiffness of death) is the state of the muscles in a dead body when they become stiff or
rigid with some degree of shortening.
• It is a sign of death and indicates molecular death of the muscle involved.
Option D - Livor Mortis:
• Postmortem staining or PM staining is bluish or purplish-red discolouration resulting from the
gravitational settling of blood in the toneless capillaries and venules of the dependent parts of the dead
body. It is a sign of death. The time since death can be roughly estimated from the formation, extension
and fixation of the PM staining. It indicates the posture of the body at the time of death.
• It is a sign of death.

Page 12

451
• The time since death can be roughly estimated from the formation, extension and fixation of the PM
staining.
• It indicates the posture of the body at the time of death.
• It is a sign of death.
• The time since death can be roughly estimated from the formation, extension and fixation of the PM
staining.
• It indicates the posture of the body at the time of death.

Solution for Question 5:


Correct Option B - Tach noir:
• Tache noir: Dust deposition in the sclera in the form of two triangles on either side of the iris when
eyes remained open.
• It is brown in colour and later turns black. Appears after 3-4 hours after death.

Incorrect Options:
Option A - Kevorkian sign: Kevorkian sign is a postmortem change that occurs in the retinal vessels. It i
s characterized by fragmentation of blood column in the retinal vessels. It appears within minutes after
death and lasts for about 1 hour. It can be visualized by using an ophthalmoscope.
Option C - Both of the above: Tache noir is used as proof of eyes being open for a
few hours after death.
Option D - All of the above: Tache noir is used as proof of eyes being open for a few hours after death.

Solution for Question 6:


Correct Option D - Body has rotten & foul smell:

Page 13

452
• Adipocere has a rancid butter/ammonical smell.
Incorrect Options:
Option A - It’s a modification of putrefaction: Adipocere is a modification of decomposition/putrefaction.
Option B - Its developed in the presence of moist air: A warm, moist and anaerobic environment favour
s adipocere formation.
Option C - It occurs in dead bodies lying in water: Adipocere occurs more rapidly in bodies submerged
in water than in damp soil.

Solution for Question 7:


Correct Option D - Cadaveric rigidity affects successively the masticatory muscles, those of the face an
d the neck, those of the trunk and arms, and finally those of the lower limbs:
• Nysten’s law: Rigor mortis affects first the muscles of the jaw, followed by those of the face and neck,
then muscles of the trunk and arms, and lastly the legs and feet.
Incorrect Options:
Option A - Cadaveric rigidity does not appear in living people:
• Though Cadaveric rigidity does not appear in living people, it is not Nystein's rule.
Option B - Cadaveric rigidity appears faster in older and the young than in the middle age:
• Cadaveric rigidity absent in fetuses < 2 months of gestation [IUL]
• Cadaveric rigidity is not seen in the fetus as the muscle/ protein development is absent [it occurs after
2 months]
Option C - Cadaveric rigidity occurs in men earlier than in the women:
• Cadaveric rigidity occurs in women earlier than in men because of the muscle bulk.

Solution for Question 8:


Correct Option B - Clostridium welchii:
• Putrefaction is mainly caused by the action of the enzyme lecithinase produced by Clostridium welchi.
Incorrect Options:
Option A - Streptococcus pyogens:
• Group A Streptococcus, often known as group A strep or Streptococcus pyogenes, can result in
non-suppurative sequelae as well as invasive and non-suppurative illness.
• Enzyme lecithinase is not produced by Streptococcus pyogens.
Option C - Pseudomonas aeruginosa:
• Usually after surgery, Pseudomonas aeruginosa can result in infections of the blood, lungs
(pneumonia), or other body parts.
• Enzyme lecithinase is not produced by Pseudomonas aeruginosa.

Page 14

453
Option D - None:
• Putrefaction is mainly caused by the action of the enzyme lecithinase produced by Clostridium welchi.

Solution for Question 9:


Correct Option B: Patient with severe head injury
• A patient with a severe head injury is not an exclusion from brainstem death.
• The essential requirements for the diagnosis of brain death include the establishment of cessation of
all brain functions, i.e. cerebral and mainly brainstem functions using primarily the clinical criteria and
partly by confirmatory paraclinical/ laboratory tests which include electroencephalogram (flat isoelectric
EEG) and somatosensory evoked potentials (SSEP) and tests to measure cerebral blood flow.
Incorrect Options
Option A: Patient under effect of drug overdose
• The most important reversible conditions/confounding factors that must be excluded include drug
intoxication.
• Drug intoxication: Presence of sedation, neuromuscular blockade, or drugs causing CNS depression.
Option C: Core body temperature is below 35 degree Celsius
• The most important reversible conditions/ confounding factors that must be excluded contain
hypothermia.
Option D: Patient is suffering from severe metabolic/endocrine disturbance
• The most important reversible conditions/ confounding factors that must be excluded include severe
electrolyte, acid-base or endocrine abnormalities.

Solution for Question 10:


Correct Option C - Colourless to brown:
• Gas from the heart in addition to pyrogallol solution will change from colourless to brown in presence
of free oxygen.
Incorrect Options:
Option A - Orange to brown: Gas from the heart in addition to pyrogallol solution will change from colou
rless to brown in presence of free oxygen, not orange to brown.
Option B - Green to yellow: Gas from the heart in addition to pyrogallol solution will change from colourl
ess to brown, not green to yellow.
Option D - Colourless to pink: Gas from the heart in addition to pyrogallol solution will change from colo
urless to brown, not pink.

Solution for Question 11:

Page 15

454
Correct Option A - Same mechanism as rigor mortis:
• Mechanism– Not clearly known; predisposing factors– sudden death, excitement, fear, exhaustion,
nervous tension, etc.
Incorrect Options:
Option B - Can be seen in cases of drowning:
• Cadaveric spasm is the most important sign of antemortem drowning.
• Plants and weeds in the hands of victims are seen in drowning.
Option C - Also known as instantaneous rigor:
• Cadaveric spasm is also known as instantaneous spasm/ cataleptic rigidity.
Option D - Molecular death does not occur:
• Molecular death does not occur in cadaveric spasm.

Solution for Question 12:


Correct Option C - Taphonomy:
• Forensic taphonomy is the interdisciplinary study and interpretation of postmortem processes of
human remains.
Incorrect Options:
Option A - Thanatology: The study deals with death in all its aspects.
Option B - Putrefaction: The process of gradual dissolution and liquefaction of the tissues.
Option D - Entomology: The study of the form and behaviour of insects.

Solution for Question 13:


Correct Option B - Environmental temperature:
• Environmental temperature is the major factor that influences the onset and duration of algor mortis.
• The rate of fall of body temperature is directly proportional to the difference between the temperature
of the dead body and the environmental temperature.
Incorrect Options:
Option A - Muscle built of the person:
• Muscle built of the person is not the major factor that influences the onset and duration of algor mortis.
• Obese bodies cool slowly, and lean bodies rapidly, since fat is a bad conductor of heat.
Option C - The damage sustained by the tissues:
• The damage sustained by the tissues is not the major factor that influences the onset and duration of
algor mortis.
Option D - Age:

Page 16

455
• Age: The rate of loss of heat is more in children and the elderly, compared to adults because the
surface area of the body is more in relation to the body volume.
• Thus age is not the major factor that influences the onset and duration of algor mortis.

Solution for Question 14:


Correct Option D - Putrefaction:
• Degloving also called avulsion, is a type of severe injury that happens when the top layers of your skin
and tissue are ripped from the underlying muscle, connective tissue, or bone.
• Degloving is seen in putrefaction.
Incorrect Options:
Option A - Hanging:
• Hanging (self-suspension) is a form of asphyxia which is caused by the suspension of the body by a
ligature which encircles the neck, the constricting force being the weight of the body.
• The ligature mark on the neck is the most important and specific sign of death from hanging.
Option B - Rigor mortis:
• Rigor mortis (Latin, stiffness of death) is the state of the muscles in a dead body when they become
stiff or rigid with some degree of shortening.
• It is a sign of death and indicates molecular death of the muscle involved.
Option C - Mummification:
• It is the rapid dehydration/desiccation and shrivelling of the dead body from the evaporation of water,
with the preservation of natural appearances and features of the body.
• It is a modification of putrefaction (dry decomposition).
• The entire body loses weight and becomes thin, stiff, brittle, and odourless.
• The process of normal decomposition of the dead body is prevented, as the growth of the
microorganisms is retarded.

Solution for Question 15:


Correct Option C - Primary relaxation:
• Tendon reaction/ Zasko's phenomenon: This can be seen up to 1 to 2 hours after death. This is a
stage of primary relaxation.
• This can be seen up to 1 to 2 hours after death.
• This is a stage of primary relaxation.
• Primary relaxation or flaccidity of the muscles: Muscles lose their tonicity and become flaccid, but the
muscular tissues are still alive, their chemical reaction is alkaline and responds to electrical stimuli.
• This can be seen up to 1 to 2 hours after death.

Page 17

456
• This is a stage of primary relaxation.
Incorrect Options:
Option A - Rigor mortis:
• Rigor mortis is the state of the muscles in a dead body when they become stiff or rigid with some
degree of shortening.
• The phase of primary relaxation of the muscles continues for about an hour which is followed by
stiffening or rigidity. It indicates molecular death of the concerned muscles.
Option B - Decomposition:
• Putrefaction is a process by which complex organic body tissue breaks down into simpler inorganic
compounds or elements due to the action of saprophytic microorganisms or due to autolysis.
• Putrefaction usually follows the disappearance of rigor mortis.
Option D - Secondary relaxation:
• Time of occurrence: After rigor mortis passes off.

Solution for Question 16:


Option C: Morrison
• Rule of Morrison: This rule states that the crown heel length of the fetus in centimetres, divided by five
gives the age in months.
• It does not relate to rigor mortis.
Option A: Shapiro
• Rigor mortis is a biochemical process which is occurring simultaneously in all the muscles.
• In small muscles, rigidity appears earlier
• In larger muscles, rigidity appears later
Option B: Nysten
• Nysten's rule: Rigor mortis occurs in a specific sequence
• Eyelid →Neck →Jaw →Face →Thorax →Upper Limb → Abdomen →Lower Limb →Fingers / Toes.
Option D: Rule of 12
• Rule of 12: Rigor mortis starts in 60 minutes and reaches a maximum in 12 hrs. Rigor mortis stays in
the body for 12 hrs and disappears from the body in another 12 hrs.
• Rigor mortis starts in 60 minutes and reaches a maximum in 12 hrs.
• Rigor mortis stays in the body for 12 hrs and disappears from the body in another 12 hrs.
• Rigor mortis starts in 60 minutes and reaches a maximum in 12 hrs.
• Rigor mortis stays in the body for 12 hrs and disappears from the body in another 12 hrs.

Page 18

457
Solution for Question 17:
Correct Option A - 12-18 hours:
• In the image greenish discolouration is only on the right iliac fossa, hence 12-18 hours.
Incorrect Options:
Option B - 1-2 days: This patch of discolouration usually appears between 1–2 days in winter.
Option C - 3-5 days: In the image greenish discolouration is only on the right iliac fossa, hence 12-18 h
ours is the answer.
Option D - 7-10 days: In the image greenish discolouration is only on the right iliac fossa, hence 12-18
hours is the answer.

Solution for Question 18:


Correct Options
Option A - Pugilistic attitude:
• Fencing attitude/Heat Stiffening (Pugilistic or Boxer’s attitude): Occurs in case of deaths due to
burning and falling in hot liquid.
• The body attains a flexing posture which looks like a fencing posture or a defence posture in boxing.
Option E - Flexion of elbows, knees and clenching of hand into a fist:
• Fencing attitude/Heat Stiffening: The legs are flexed at the hips and knees, the arms are flexed at the
elbows and held out in front of the body and the fingers are hooked like claws.
Incorrect Options:
Option B - Occurs immediately after death: The cadaveric spasm occurs immediately after death.
Option C - Intensity of muscle contraction is much more as compared to rigor mortis: In Cadaveric spas
ms, the Intensity of muscle contraction is much more as compared to Rigor Mortis.
Option D - Only selected muscles involved: Only Selected muscles are involved in Cadaveric spasms.

Solution for Question 19:


Correct Option C - Lysosomes:
• Autolysis: Autolysis is the self-digestion of tissues. Soon after the death, cell membranes become
permeable and break down, with the release of cytoplasm-containing enzymes.
• Lysosomes and their digestive enzymes (mainly hydrolases) are released from the cells.
Incorrect Options:
Option A - Lecithinase:
• A specific class of phospholipase called lecithinase works on lecithin.
• It can be made by Listeria monocytogenes, Staphylococcus aureus, Clostridium perfringens, or
Pseudomonas aeruginosa.

Page 19

458
• Myonecrosis and hemolysis are brought on by the lecithinase in C. perfringens alpha-toxin.
• Lecithinase: Major bacterial enzyme responsible for putrefaction.
Option B - Lipase:
• The body employs the enzyme lipase to digest dietary fats so they may be absorbed in the intestines.
• The pancreas, mouth, and stomach all manufacture lipase.
• Intrinsic lipases favours adipocere.
Option D - ATPase:
• After death, there is continuous hydrolysis of the ATP, and as long as glycogen is available in the
muscle, there is re-synthesis of ATP. In this process, once the muscle glycogen is exhausted, no
further re-synthesis of ATP is possible and the muscle loses softness, elasticity and extensibility due to
the formation of viscid actomyosin complex giving rise to rigor mortis in the muscle.
• After the pH of the muscle becomes 5.5, the release of autolytic enzymes stored in lysosomes takes
place. The major proteolytic enzymes are cathepsins and calpains. These enzymes act on the
myofibrillar proteins and hydrolyze them. As a result, the actomyosin complex is broken down and
muscles become soft again. This is known as the resolution of rigor which occurs during the stage of
secondary relaxation, due to decomposition.

Solution for Question 20:


Correct Option B - Absence of moisture:
• Ideal conditions for mummification: High temperature, dry condition, absence of moisture, and free
circulation of air.
Incorrect Options:
Option A - Warm & Humid climate:
• A warm & humid climate favours adipocere, not mummification.
Option C - Clostridium perfringes:
• Clostridium perfringes favour adipocere, not mummification.
Option D - Intrinsic Lipases:
• Intricis lipases favour adipocere, not mummification.
• Ideal conditions for Adipocere formation: Warm temperature, moisture, less air, bacteria, and
fat-splitting enzymes (lipases).

Solution for Question 21:


Correct Option D - Henssge nomogram:
• Time since death from algor mortis is from the nomogram method devised by Henssge.
Incorrect Options:
Option A - Madea’s formula: Formulae used for vitreous potassium: Madea’s formula.

Page 20

459
Option B - Calorimetry formula: A recent method to determine the time since death from PM staining is
known as calorimetry, which shows an increasing pallor of the hypostasis during the first 24 h.
Option C - Nysten’s rule: Nysten’s law: Rigor mortis affects first the muscles of the jaw, followed by tho
se of the face and neck, then muscles of the trunk and arms, and lastly the legs and feet.

Solution for Question 22:


Correct Option C - Rigor mortis involves voluntary muscles & involuntary muscles:
• Rigor mortis involves both voluntary muscles & involuntary muscles.
Incorrect Options:
Option A - Rigor mortis immediately after the death: Onset within 1–2 h after death, takes about 9–12 h
to develop from head to foot, persists for another 12 h, and takes 12 h to pass off (Rule of 12).
Option B - Cadaveric spasm occurs 1
hour after death: Cadaveric spasm or instantaneous rigor mortis is a rare form of muscular stiffening th
at occurs at the moment of death and persists into the period of rigor mortis.
Option D - Cadaveric spasm involves voluntary muscles & involuntary muscles: Usually restricted to a
selected group of voluntary muscles.

Solution for Question 23:


Correct Option B - Executive magistrate can order for exhumation:
• An executive magistrate can order for exhumation (Written order from the first class magistrate/
district magistrate/ executive magistrate).
Incorrect Options:
Option A - Police can order for exhumation: Police cannot order exhumation.
Option C - Postmortem cannot be done on exhumated body: The sole purpose of exhumation is to perf
orm or reperform Postmortem only.
Option D - CrPC 174 is related to enquiry of exhumated body: 176 CrPC is related to the enquiry of ex
humated body.

Solution for Question 24:


Correct Option B - Doctor performing the liver transplant:
• A transplant surgeon can not be a part of the team due to a conflict of interest.
Incorrect Options:
Option A - Neurosurgeon: A Neurologist/Neurosurgeon nominated by a panel may certify brain death.
Option C - Doctor attending the patient: As per the latest amendment, if a neurologist is not available, a
ny surgeon, physician, anaesthetist or intensivist, nominated by the medical in-charge of the hospital m

Page 21

460
ay certify brain death.
Option D - RMP in charge of the hospital: The RMP in charge of the hospital where brain death has oc
curred is authorized for pronouncing brain death.

Solution for Question 25:


Correct Option B - Inferior surface of liver:
• In cases of sodomy, the temperature is measured from the Inferior surface of the liver.
Incorrect Options:
Option A - Rectum: Best site to measure core body temperature after death is Rectum.
Option C - Intra-aural (thermometer up to tympanic membrane): Other suitable sites are the external au
ditory canal, nasal passages or inserting a thermometer into the peritoneal cavity by a small opening.
Option D - Intra-nasal (thermometer up to the cribriform plate): Other suitable sites are the external aud
itory canal, nasal passages or inserting a thermometer into the peritoneal cavity by a small opening.

Solution for Question 26:


Correct Option A - Presence of Albumin & Chloride in blister fluid:
• Antemortem blister differs from postmortem blister by the Presence of Albumin & Chloride in blister
fluid.The first picture shows a postmortem blister, and the second one is an antemortem blister.
Incorrect Options:
Option B - Gas in blister: Gas is present in post-mortem blisters.
Option C - Enzymatic reaction are absent Enzymatic reaction are absent in postmortem blister
Option D - Absence of hyperemia around the blister: In
post-mortem blisters, there is an absence of hyperemia around the blister.

Solution for Question 27:


Correct Option C - Postmortem glycogenolysis:
• Not seen in postmortem glycogenolysis.
• Postmortem glycogenolysis: Compulsory phenomenon which occurs in all dead bodies, and which
starts soon after death (produces up to 140 calories).
Incorrect Options:
Option A - Pontine hemorrhage:
• Disturbed regulation of heat production before death– Heat stroke, Pontine hemorrhage.
Option B - Bacteremia:

Page 22

461
• In Postmortem Caloricity, instead of cooling, the temperature of the dead body remains high for the
initial 2 hour or so.
• Heat production due to excessive bacterial activity, as in septicaemic conditions, cholera, and other
fevers.
Option D - Status epilepticus:
• In Postmortem Caloricity, instead of cooling, the temperature of the dead body remains high for the
initial 2 hour or so and is seen in Status epilepticus.

Solution for Question 28:


Correct Option C - Putrefaction:
• Swiss- cheese liver is found in putrefaction.
• The liver assumes a ‘honey comb’ (‘foamy’ or ‘Swiss cheese’) appearance due to the formation of air
bubbles. It becomes greenish in color, and later changes to coal-black.
Incorrect Options:
Option A - Adipocere:
• Adipocere is the formation of an offensive, sweet rancid smelling, soft, whitish, or grayish-white,
crumbly, waxy, and greasy material (similar to soap) occurring in fatty tissues of a dead body. It is a
modification of decomposition.
• Distribution: It forms at any site where fatty tissue is present.
• The liver is prominent and retains its shape.
Option B - Alcoholism:
• Swiss-cheese liver is found in putrefaction, not in alcoholism.
Option D - Drowning:
• Liver, spleen, and kidneys are congested in Drowning.

Solution for Question 29:


Correct Option C - Conversion of organic matter into fossils:
• Fossilization is a process that involves the preservation of organic material in rock or mineralized form
over an extended period. It is not related to putrefaction, which is the decomposition and breakdown of
organic matter.
Incorrect Options:
Option A - Bacterial activity: Putrefaction is stimulated by bacterial activity. Bacteria, particularly specie
s like Clostridium welchii or Clostridium perfringens, play a
significant role in the decomposition of organic matter during putrefaction.
Option B - Involvement of Clostridium welchii or Clostridium perfringens: Clostridium welchii or Clostridi
um perfringens are commonly associated with putrefaction. These bacteria are known to thrive in anae
robic conditions and contribute to the breakdown of tissues during the putrefaction process.

Page 23

462
Option D - Production of lytic lecithinase enzyme: Lytic lecithinase is an enzyme that is commonly invo
lved in putrefaction. It is produced by bacteria, including Clostridium species, and aids in the decompos
ition of organic material.

Solution for Question 30:


Correct Option C - Suggillation:
• The given image is showing postmortem staining which is also known as suggillation.
Incorrect Options:
Option A - Rigor mortis:
• Rigor mortis (stiffness of death) is the state of the muscles in a dead body when they become stiff or
rigid with some degree of shortening. It is a sign of death and indicates molecular death of the muscle
involved.
Option B - Algor mortis:
• Algor mortis is the cooling of the body that normally takes place after death, where the body
temperature equilibrates with its environmental temperature.
Option D - Marbling:
• The blood vessels provide an important route through which the bacteria can spread with ease
throughout the body.
• Their passage is marked by the decomposition of hemoglobin to sulphmethemoglobin in the blood
vessels, which causes greenish or reddish-brown staining of the inner walls of the superficial vessels.
• This is seen as linear branching patterns, which gives a ‘marbled’ (‘road map’) appearance of the skin.
• Marbling commences after 24 h. Putrefactive odor is noticed at about the same time.
• By 36-48 h, marbling is prominent.

Page 24

463
Human Identification
1. What is True regarding the sequence of permanent teeth eruption?
(or)
What is the sequence of permanent teeth eruption in humans?
A. Central Incisor, Lateral Incisor, Canine, 1st Premolar, 2nd Premolar, 1st Molar, 2nd Molar & 3rd
Molar
B. 1st Premolar, Central Incisor, Lateral Incisor, Canine, 2nd Premolar, 1st Molar, 2nd Molar & 3rd
Molar
C. 1st Molar, Central Incisor, Lateral Incisor, Canine, 1st Premolar, 2nd Premolar, 2nd Molar & 3rd
Molar
D. 1st Molar, Central Incisor, Lateral Incisor, 1st Premolar, 2nd Premolar, Canine, 2nd Molar & 3rd
Molar
----------------------------------------
2. Cheiloscopic techniques have been used in 85 cases, including 65 burglary cases, 15 cases of
homicide, and five cases of assault. In 34 cases the identification was positive, which means that
Cheiloscopic techniques were equal in value to other types of forensic evidence. It has also been
included in evidence for presentation in court. Cheiloscopy is the study of ?
(or)
Cheiloscopy is the study of ?
A. Foot
B. Fingers
C. Palate
D. Lips
----------------------------------------
3. After the search and rescue operations following a massive earthquake, forensic experts are
investigating and identifying those deceased. Patient’s relative gives a history of tattoo, however it was
not found during autopsy. What should be dissected to find it?
(or)
What specific anatomical structures should be dissected to locate a missing tattoo reported by the
patient's relative during autopsy?
A. Lymph node
B. Skin
C. Spleen
D. Kidney
----------------------------------------
4. An autopsy was performed on the 15-year-old boy who had succumbed to injuries sustained in a
road traffic accident. The doctor also examined the skull and the brain for signs of damage. He noticed
that the base of his sphenoid was not yet fused with the occiput. By which age is the fusion of the
bones expected to happen?

464
(or)
At what age is the fusion of the sphenoid and occiput bones typically expected to occur
A. 20 years
B. 30 years
C. 40 years
D. 50 years
----------------------------------------
5. According to Locard’s Principle of Exchange, when two objects come into contact with each other,
there is always a transfer of some material between them. This is helpful in crime investigations,
especially in sexual offenses. Which one of the following methods of personal identification was
described by Locard?
(or)
Which one of the following methods of personal identification was described by Locard?
A. Podography
B. Dactylography
C. Poroscopy
D. Cheiloscopy
----------------------------------------
6. A 9-year-old boy's body was received by the forensic pathologist for an autopsy. He had succumbed
to injuries he had sustained while falling off a tree he was attempting to climb. Examination of the boy's
dentition has revealed delayed tooth eruption, enamel hypoplasia, micrognathia, and anterior open bite.
In all of the following conditions, dentition is delayed except?
(or)
In all of the following conditions, dentition is delayed,except?
A. Rickets
B. Hypothyroidism
C. Malnutrition
D. Congenital syphilis
----------------------------------------
7. Palato-print is commonly taken from which part of palate:
(or)
Palato-print taken from which part of the palate:
A. Anterior part
B. Lateral wall
C. Medial wall
D. Posterior part
----------------------------------------

Page 2

465
8. Corneal tattooing may be done with:
(or)
Corneal tattooing may be done with:
A. Gold chloride
B. Calcium chloride
C. Copper sulfate
D. Potassium permanganate
----------------------------------------
9. Find the correct match regarding appearance of epiphyses:
(or)
Find the correct match regarding appearance of epiphyses:
A. Talus–7th month
B. Calcaneum–3rd month
C. Scaphoid–3 years
D. Hamate–5 years
----------------------------------------
10. In measuring lengths of long bones using osteometric board for assessing stature, which of the
following is correct
(or)
What is the correct method for measuring the lengths of long bones using an osteometric board to
assess stature?
A. Femur - Head to lateral condyle
B. Tibia - medial condyle to tip of medial malleolus
C. Fibula - tip of the head to tip of lateral malleolus
D. Humerus - capitulum to head
----------------------------------------
11. The newly appointed Sub-Inspector of the Idukki district was determined to find evidence of
Georgekutty's involvement in the murder of an 18-year-old boy. There has been no sign of the missing
boy or his remains for almost 2 years. A determined investigation by the SP's team led the police to
excavate their own police station. The remains of a body have been discovered from the excavated site
and sent for dating. Which statement among the following, about the dating of bones, is not true?
(or)
Which among the following about the dating of bones, is not true?
A. In 50-100 years, the nitrogen content is more than 3.5 g%
B. Fresh bone shows 15 amino acids
C. Blood pigments persist up to 100 years
D. Precipitin tests are negative after 5 years

Page 3

466
----------------------------------------
12. Permanent impairment of fingerprints occurs in all ,except.
(or)
Permanent impairment of fingerprints occurs in all ,except:
A. Leprosy
B. Acanthosis nigricans
C. Electrical injuries
D. Radiation injuries
----------------------------------------
13. Which among the following is relatively not a method of identification in living individuals?
(or)
Which is not a method of identification in living individuals?
A. Spectrogram
B. Calligraphy
C. Gait analysis
D. Superimposition
----------------------------------------
14. Read the following statements regarding Eunuchs, which is true. Penis and scrotum are removed
before puberty in Hijrahs Genetalia of Zenana are intact
A. Both are true
B. Both are false
C. 1 is true, 2 is false
D. 2 is true, 1 is false
----------------------------------------
15. Which of the following is not a valid method for determining the sex of a bone?
(or)
A certain syndicate has acquired the rights to excavate a part of a forest area for the construction of a
factory. During the excavation, the remains of a couple of bodies have been recovered. It has been
sent to the forensic lab for the determination of its identity. The forensic team would use all of the
following formulae to determine the sex of the bone. Which one among them is not the right method?
A. Sciatic notch index - Width of notch/ Depth of notch x 100
B. Washburn ischiopubic index - Pubic length/ Ischial length x 100
C. Corporobasal index - Length of 1st sacral vertebra/ Length of base of sacrum x 100
D. Sternal index - Manubrial length/ Body length x 100
----------------------------------------
16. Forensic dentistry or forensic odontology is the handling, examination, and evaluation of dental
evidence in criminal justice cases. Forensic dentists are involved in assisting investigative agencies to

Page 4

467
identify recovered human remains in addition to the identification of whole or fragmented bodies. Which
of the following is/are superadded teeth in permanent dentition?
(or)
Which of the following is/are superadded teeth in permanent dentition?
A. Maxillary 3rd molar
B. Mandibular 1st molar
C. Maxillary 2nd molar
D. Premolars
E. Permanent Incisors
----------------------------------------
17. Finger prints are obtained upto a depth of
(or)
Finger prints are obtained upto a depth of
A. 2 mm
B. 0.2 mm
C. 6 mm
D. 0.6 mm
----------------------------------------
18. Determine the sequence of long bones used in stature estimation?
(or)
The sequence of the long bones in stature estimation is
A. Femur > tibia > humerus > radius
B. Tibia > femur > humerus > radius
C. Femur > Humerus > tibia > radius
D. Tibia > humerus > femur > radius
----------------------------------------
19. This refers to a forensic identification science that is associated with all of the ridges on the volar
areas and not just on the fingertips or fingerprint identification implies. It is not only more encompassing
but has methodologies and philosophies consistent with other forensic disciplines. Which discipline
among the following, is being defined?
(or)
Which forensic identification science covers all the ridges on the volar areas, rather than just focusing
on fingerprints identification?
A. Dactylography
B. Poroscopy
C. Ridgeology
D. Rugoscopy

Page 5

468
----------------------------------------
20. The drumstick shape in the image help to determine: (refer to the image below)
(or)
The drumstick shape in the image help to determine:

A. Age
B. Sex
C. Stature
D. Race
----------------------------------------
21. Consider the following four events of development of fetus: Centers of ossification for calcaneum
Centers of ossification for lower end of femur Centers of ossification for talus Centers of ossification for
cuboid What is the order in which they appear from lower to higher gestation?
A. 1, 3, 2, 4
B. 1, 3, 4, 2
C. 1, 4, 2, 3
D. 1, 2, 4, 3
----------------------------------------
22. The forensic dentist was invited to observe the dentition of a body that was excavated from the
garden of a businessman. He has remained uncooperative during questioning. The forensic team has
thus, decided to proceed with the investigations to gather evidence to file charges. The dentist has
observed that all permanent teeth have erupted. The dentist would agree that all of the following
statements are true of permanent dentition, except?
(or)
All of the following statements are true of permanent dentition, except?
A. Anterior teeth are inclined forward
B. Roots of molar are larger
C. Eruption starts with central incisor
D. They are ivory white in color
----------------------------------------

Page 6

469
23. The forensic doctor allotted to the case has attended a course that has certified him to identify a
person based on characteristic arrangements of lines of a particular body part. He has made a
thorough search and found some prints on a bottle and a whisky glass found at the crime scene. He
documented the same as "Figure linearum labiorum rubroum". Which one of the following is the doctor
examining?
(or)
"Figure linearum labiorum rubroum" Signify ?
A. Nose print
B. Lip print
C. Ear print
D. Palate rugae
----------------------------------------
24. As part of the autopsy of the body of a young man, the doctor had an X-ray done to determine the
ossification status of the bone to narrow down on the age. The following image was the x-ray of the hip
joint done postmortem. He notices that the center for the head, greater and lesser trochanter of the
femur appeared but has not fused. The triradiate cartilage has not obliterated. Which age range will the
doctor interpret the victim to be of, on viewing below X-Ray?
(or)
Which age range will be interpreted of the victim on viewing the below X-ray?

A. More than 18 years


B. Less than 15 years
C. More than 15 years
D. Less than 11 years
----------------------------------------
25. In the skeletal remains in a building suspected to be of a male, the length of humerus is 24.5 cm,
the stature of the person will be:
(or)
What is the estimated stature of a male based on a humerus length of 24.5 cm in skeletal remains?
A. 130.095 cm
B. 93.59 cm

Page 7

470
C. 143.00 cm
D. 110.00 cm
----------------------------------------
26. Which of the following is not correct regarding teeth features and ethnicity:
(or)
Which statement about teeth features and ethnicity is incorrect?
A. Upper third molar is most commonly absent in Mongolians
B. In negroes the cusps of molars are wide and deep and shovel shaped cusps in incisors
C. Caucasians have carabelli cusps
D. More cusps in negroids
----------------------------------------
27. "Locard Exchange" principle was given by?
(or)
"Locard Exchange" principle was given by?
A. Dr. Edmond Locard
B. Sir Francis Galton
C. Prof. Alec Jeffreys
D. Mathieu Orfila
----------------------------------------
28. The fingerprint pattern showing two or more delta is:
(or)
The fingerprint pattern showing two or more delta is:
A. Loop
B. Plain arch
C. Tented arch
D. Whorls
----------------------------------------
29. The minimum age of this person is:
(or)
The minimum age of this person is:

Page 8

471
A. More than 12 years
B. 5 years
C. 6 years
D. 18 years
----------------------------------------
30. The body of a young man has washed ashore on a beach in Chennai. The police have circulated
the pictures of the victim to cross-check with missing reports across the state. The forensic team was
handed the body to determine the cause of death. The forensic team studied the palatal rugae as its
design and structure are unchanged on exposure to chemicals, heat, disease, or trauma. All of the
following statements regarding rugoscopy are true, except?
(or)
All of the following statements regarding rugoscopy are true, except?
A. Palatal rugae are used as a method of identification
B. Palatoprints do not change during growth
C. Primary rugae are less than 3 mm
D. Secondary rugae are 3-5 mm
----------------------------------------
31. Which of the following differentiate female skull from male skull?
(or)
Which of the following differentiate female skull from male skull?
A. Mastoid process bigger
B. Prominent frontal eminence
C. Orbit square with round edges
D. Prominent Zygomatic arch
----------------------------------------
32. A boy alleging age 9 years is brought for medical examination. The total number of teeth at this age
would be:
(or)
How many teeth would a 9-year-old boy typically have during a medical examination?

Page 9

472
A. 20
B. 24
C. 28
D. 32
----------------------------------------
33. Which method of identification is shown in the picture?
(or)
Which method of identification is shown in the picture?

A. Dactylography
B. Cheiloscopy
C. Ridgeology
D. Poroscopy
----------------------------------------
34. A 28-year-old woman has been rescued from the bungalow of a businessman. She is a native of
Thailand and had been allured with the promise of a job by a few men, who the police are on the
lookout for. The forensic team has begun the physical examination of the victim. They found concentric
teeth marks on the right forearm. What is the first procedure to be done with the finding?
(or)
What is the initial step in investigating the concentric teeth marks on the victim's right forearm during a
forensic examination?
A. 2 swab technique for saliva collection
B. Keep scale for measuring below the mark and take photographs
C. Prepare cast using vinyl polysiloxane
D. Skin is removed and preserved
----------------------------------------
35. The charred body has been recovered from the dense vegetation close to the highway. The police
were led to the spot by a sniffer dog that was tracking a 25-year-old IT employee who had been missing
since the previous evening. The team has to collect a sample to identify the victim. A sample of which
tissue from a burnt body, is best for identification?
(or)

Page 10

473
Which tissue sample from a burnt body is most suitable for identification?
A. Hair
B. Bone
C. Teeth
D. Blood
----------------------------------------

Correct Answers
Question Correct Answer

Question 1 4
Question 2 4
Question 3 1
Question 4 1
Question 5 3
Question 6 4
Question 7 1
Question 8 1
Question 9 1
Question 10 3
Question 11 4
Question 12 2
Question 13 4
Question 14 1
Question 15 3
Question 16 1,2,3
Question 17 4
Question 18 1
Question 19 3
Question 20 2
Question 21 1
Question 22 3
Question 23 2
Question 24 2
Question 25 1
Question 26 2

Page 11

474
Question 27 1
Question 28 4
Question 29 3
Question 30 3
Question 31 2
Question 32 2
Question 33 4
Question 34 1
Question 35 3

Solution for Question 1:


Correct Option D
- 1st Molar, Central Incisor, Lateral Incisor, 1st Premolar, 2nd Premolar, Canine, 2nd Molar &
3rd Molar:
• 1st Molar (6 years), Central Incisor (7 years), Lateral Incisor (8 years), 1st Premolar (9 years), 2nd
Premolar (10 years), Canine (11 years), 2nd Molar (12-14 years) & 3rd Molar (17-25 years).

Solution for Question 2:


Correct Option D - Lips:
• The study of lip prints is called cheiloscopy.
Incorrect Options:
Option A - Foot:
• Podogram: Study of footprints.
Option B - Fingers:
• Poroscopy: Is the term applied to a specialized study of pore structure found on the papillary ridges of
the fingers as a means of identification.
• Ridgeology: Refers to friction ridge identification that is associated with all the ridges on the volar
areas and not just on the fingertips.
Option C - Palate:
• Palatoscopy/Palato-print/Rugoscopy: It is the study of palatal rugae in order to establish identity.

Solution for Question 3:


Correct Option A - Lymph node:
• Faint or disappeared tattoo marks can be made out on the histological study of the regional lymph
nodes.

Page 12

475
Solution for Question 4:
Correct Option A - 20 years:
• Basiocciput and basisphenoid: 18-20 years.

Solution for Question 5:


Correct Option C - Poroscopy:
• Microscopic study of pores formed by mouths of ducts of subepidermal sweat glands.
Incorrect Options:
Option A - Podography:
• Podography: Study of foot prints.
Option B - Dactylography:
• Dactylography: Study of finger prints.
Option D - Cheiloscopy:
• Cheiloscopy: Study of lip prints.

Solution for Question 6:


Correct Option D - Congenital Syphilis:
• In syphilis, teeth may be premature or even present at birth.
Incorrect Option A/ B/ C: Rickets/ Hypothyroidism/ Malnutrition
• In ill-nourished children, and especially in rickets, and in hypothyroidism dentition may be delayed.

Solution for Question 7:


Correct Option A - Anterior part:
• Palato-print is commonly taken from anterior part of palate.

Solution for Question 8:


Correct Option A - Gold chloride:
• Corneal tattooing may be done with Gold or Platinum chloride.

Page 13

476
Incorrect Option B/ C/ D - Calcium chloride/ Copper sulfate/ Potassium permanganate:
• Options B, C, D are wrong.
• India ink is the most commonly used dye for corneal tattooing.
• Two other methods exist, chemical dyeing with gold or platinum chloride and carbon impregnation.

Solution for Question 9:


Correct Option A - Talus–7th month:
• Here, Talus: 7th month is the correct match regarding the appearance of epiphyses.
Incorrect Options:
Option B - Calcaneum–3rd month:
• Calcaneum: 5th month of intra-uterine age.
Option C - Scaphoid–3 years:
• Scaphoid and trapezoid: 5 years.
Option D - Hamate–5 years:
• Hamate: 2 months

Solution for Question 10:


Correct Option C - Fibula - tip of the head to tip of lateral malleolus:
• Here, it is the correct option in measuring lengths of long bones using osteometric board for assessing
stature.
Incorrect Options:
Option A - Femur - Head to lateral condyle:
• Femur: Head to the medial condyle, not lateral condyle.
Option B - Tibia - medial condyle to tip of medial malleolus:
• Tibia: Lateral condyle to the tip of medial malleolus.
Option D - Humerus - capitulum to head:
• Humerus: Trochlea to head.

Solution for Question 11:


Correct Option D - Precipitin tests are negative after 5 years:
• Precipitin tests can detect the presence of specific proteins, including blood proteins, on bones for a
longer period than just 5 years. The persistence of proteins on bones can vary depending on

Page 14

477
environmental conditions and preservation factors, but it is generally longer than 5 years.
Incorrect Options:
Option A - In 50-100 years, the nitrogen content is more than 3.5 g%:
• The nitrogen content of bones decreases over time, and after 50-100 years, it tends to drop below 3.5
g% due to the degradation of organic components.
Option B - Fresh bone shows 15 amino acids:
• Fresh bone contains a variety of amino acids, and the analysis of these amino acids can provide
valuable information for dating bones and determining their age.
Option C - Blood pigments persist up to 100 years:
• Blood pigments, such as heme, can persist in bones for extended periods, even up to 100 years or
more, under certain conditions. They can be used as indicators of bloodstains on bones.

Solution for Question 12:


Correct Option B - Acanthosis nigricans:
• Permanent impairment of fingerprints occurs in all except acanthosis nigricans.
• Ridge alteration occurs in acanthosis nigricans.
Incorrect Options:
Option A/ C/ D- - Leprosy/ Electrical injuries/ Radiation injuries:
• Permanent impairment of fingerprints occurs in leprosy, electrical injuries, and radiation injuries.

Solution for Question 13:


Correct Option D - Superimposition:
• Superimposition is not a method of Identification in living individuals.
• Superimposition is the technique applied to determine whether the skull is that of the person in the
photograph. Done in dead patients.
Incorrect Options:
Option A/ B/ C - Spectrogram/ Calligraphy/ Gait analysis:
• Spectrogram, Calligraphy, and gait analysis are done on living subjects.

Solution for Question 14:


Correct Option A - Both are true:
1. Penis and scrotum are removed before puberty in Hijrahs
• In hijrahs, penis, and scrotum are removed by cutting before puberty.

Page 15

478
2. Genetalia of Zenana are intact
• The genitalia of the zenana are intact

Solution for Question 15:


Correct Option C - Corporobasal index - Length of 1st sacral vertebra/ Length of base of sacrum x 100:
• Corporobasal index - Length of 1st sacral vertebra/ Length of base of sacrum x 100 is WRONG as In
the FORMULA BREADTH is used.
Incorrect options.
Option A - Sciatic notch index - Width of notch/ Depth of notch x 100: Sciatic notch index -
Width of notch/ Depth of notch x 100 is the correct formula.
Option B - Washburn ischiopubic index - Pubic length/ Ischial length x
100: Washburn ischiopubic index - Pubic length/ Ischial length x 100 is the correct formula.
Option D - Sternal index - Manubrial length/ Body length x 100: Sternal index -
Manubrial length/ Body length x 100 is the correct formula.

Solution for Question 16:


Correct Options:
Option A/ B/ C - Maxillary 3rd molar/ Mandibular 1st molar/ Maxillary 2nd molar:
• Superadded permanent teeth are those which do not have deciduous predecessors. They erupt
behind the temporary teeth.
• All the permanent molars are superadded permanent teeth. The first permanent molar tooth of each
side erupts, while all the other deciduous teeth are present in the jaw.
Incorrect Options:
Option D - Permanent Incisors:
• Successional permanent teeth are those which erupt in place of deciduous teeth. eg: Permanent
incisors, canines & premolars.

Solution for Question 17:


Correct Option D - 0.6 mm:
• Prints can be obtained from the dermis if the epidermis is lost, histological section up to a depth of 0.6
mm from the finger pad surface can give satisfactory results.
Incorrect Options:Option A/B/C

Page 16

479
Solution for Question 18:
Correct Option A - Femur > tibia > humerus > radius:
• Stature can be determined from long bones, such as the femur, tibia, humerus, or radius.
• A useful rule of thumb is that the humerus is 20%, the tibia 22%, the femur 27%, and the spine 35%.

Solution for Question 19:


Correct Option C - Ridgeology:
• Ridgeology refers to friction ridge identification that is associated with all the ridges on the volar areas
and not just on the fingertips.
Incorrect Options:
Option A - Dactylography:
• Dactylography (dermatoglyphics, galton system) is the study of fingerprints as a method of
identification.
Option B - Poroscopy:
• Poroscopy is the term applied to a specialized study of pore structure found on the papillary ridges of
the fingers as a means of identification.
Option D - Rugoscopy:
• Rugoscopy is the study of palatal rugae in order to establish identity.

Solution for Question 20:


Correct Option B - Sex:
• The image in question shows Davidson body. It can be used to identify sex.
• Neutrophilic leucocytes: Sex chromatin often presents as a nuclear attachment to one of the nuclear
lobes, resembling a drumstick k/as Davidson body.
• Found in polymorphonuclear leukocytes in females.
Other options are incorrect

Solution for Question 21:


Correct Option A - 1, 3, 2, 4:

Page 17

480
The order in which they appear from lower to higher gestation:
1. Centers of ossification for calcaneum
• 20 weeks: Center of ossification for calcaneum appears.
3. Centers of ossification for talus
• 28 weeks: Center of ossification for talus appears.
2. Centers of ossification for lower end of femur
• Ossification centers for lower end of femur (36–37 weeks).
4. Centers of ossification for cuboid
• Cuboid and Capitate appear at birth.

Solution for Question 22:


Correct Option C - Eruption starts with central incisor:
• The eruption of permanent teeth usually begins with the first molars, specifically the lower first molars.
This is followed by the central incisors.
Incorrect Options:
Option A - Anterior teeth are inclined forward: Anterior (front) teeth, including incisors, are typically incli
ned forward, allowing for effective biting and cutting of food.
Option B - Roots of molars are larger: The molars, especially the permanent molars, tend to have large
r and more complex root structures compared to other teeth.
Option D - They are ivory white in color: Permanent teeth are usually ivory white in color, although ther
e can be variations in shade.

Solution for Question 23:


Correct Option B - Lip print:
• "Figure linearum labiorum rubroum," is a Latin term referring to lip prints. Lip prints, also known as
cheiloscopy, involve the study of characteristic arrangements of lines, ridges, and grooves found on the
lips. Just like fingerprints and palm prints, lip prints can be unique to each individual and are used for
forensic identification.
Incorrect Options:
Option A - Nose print:
• Nose prints are not typically used for forensic identification, as they do not have the same distinct
ridge patterns and characteristics as fingerprints, palm prints, or lip prints.
Option C - Ear print:
• Ear prints refer to the patterns and characteristics found on the surface of the ear. Ears have four
basic shapes: oval, round, rectangular, and triangular. The shapes of ear lobes and the tips of ears are
of various types. Most of the ear prints are found on doors or windows.

Page 18

481
Option D - Palate rugae:
• In the anterior part of the palate, the structural details like the rugae are individual specific and
permanent. The Palato prints can be used in the same way as fingerprints.

Solution for Question 24:


Correct Option B - Less than 15 years:
• Here in question, center for the head, greater and lesser trochanter of femur appeared but not fused;
triradiate cartilage not obliterated. So the age is more than 11 years and less than 15 years.

Solution for Question 25:


Correct Option A - 130.095 cm:
• The multiplying factor for humerus in calculating stature in males is 5.31.
• Stature = length of humerus x 5.31.
• 24.5 x 5.31 = 130.095 cm.
• Stature: 130.095 cm.

Solution for Question 26:


Correct Option B
- In negroes the cusps of molars are wide and deep and shovel shaped cusps in incisors:
• Large teeth with more cusps in their molars with two lingual cusps on mandibular first premolars are
common.
• Shovel-shaped upper central incisors seen in Mongolians.
Incorrect Options:
Option A - Upper third molar is most commonly absent in Mongolians:
• In Mongolians, congenital lack of upper 3rd molars can be seen.
Option C - Caucasians have carabelli cusps:
• Caucasians have carabelli cusps.
Option D - More cusps in negroids:
• Large teeth with more cusps in their molars with two lingual cusps on mandibular first premolars are
common.

Page 19

482
Solution for Question 27:
Correct Option A - Dr. Edmond Locard:
• "Locard Exchange" principle was given by Dr. Edmond Locard.
• The key principle underlying crime scene investigation is a concept that has become known as
Locard's Exchange Principle. It states that whenever someone enters or exits an environment,
something physical is added to and removed from the scene.
Incorrect Options:
Option B - Sir Francis Galton:
• Systemized fingerprinting study for usage in personal identification.
Option C - Prof. Alec Jeffreys:
• DNA fingerprinting and DNA profiling.
Option D - Mathieu Orfila:
• Father of Forensic Toxicology, "Lafarge" arsenic poisoning case in France.

Solution for Question 28:


Correct Option D - Whorls:
• The fingerprint pattern showing two or more delta is Whorls.
• Whorl: 25-35% of fingerprint patterns are whorls. Whorls have at least two deltas and a core. Any
fingerprint pattern that contains equal or more than 2 deltas will be a whorl pattern.
Incorrect Options:
Option A - Loop:
• Loop: 60-70% of fingerprint patterns are loops. These are the most common fingerprint pattern.
• Loops pattern must have one delta and one core.
Option B - Plain arch:
• A fingerprint pattern in which the ridges enter on one side of the impression and flow or tend to flow
out the other side with arise or wave in the center.
Option C - Tented arch:
• The tented arch and the arch comprise about 5% of fingerprint patterns found on the human fingertips
and thumbs. The tented arch has a ‘tent pole’ thrusting up from the mound at the base in the middle of
the print.

Solution for Question 29:


Correct Option C - 6 years:
• The image shows an X-ray of the Wrist joint which helps estimate age between 6 to 12 years.

Page 20

483
• Pisiform has not appeared. (It appears at 10- 12 years of age) so, age is less than 12 years.
• Age is more than 6 years and less than 12 years.
Other options are incorrect

Solution for Question 30:


Correct Option C - Primary rugae are less than 3 mm:
• Primary rugae are 5-10 mm or more.
Incorrect Options:
Option A - Palatal rugae are used as a method of identification:
• Harrison Allen, in 1889, suggested the use of palatal rugae for identity verification.
Option B - Palatoprints do not change during growth:
• Palatine rugae are unique and can be used for identification in circumstances when it is difficult to
identify a dead person through dental records or fingerprints.
Option D - Secondary rugae are 3-5mm:
• It is true regarding Rugoscopy: Secondary rugae are 3-5 mm.

Solution for Question 31:


Correct Option B - Prominent frontal eminence:
• Frontal eminences & parietal eminences are more prominent in Females.
Incorrect Options:
Option A - Mastoid process bigger:
• Mastoid process is small and pointed in females.
Option C - Orbit square with round edges:
• Orbit square with rounded and sharp margins in Females.
Option D - Prominent zygomatic arch:
• Zygomatic arch: Not prominent.

Solution for Question 32:


Correct Option B - 24:
• The total number of teeth at this age would be 24.
• At 6-11 years: Period of Mixed dentition.

Page 21

484
Solution for Question 33:
Correct Option D - Poroscopy:
• Poroscopy: Circular openings shown in the picture are openings of sweat pores in between the ridges.
• These openings are unique so it helps in identification of an individual. This is called poroscopy.
Incorrect Options:
Option A - Dactylography:
• Dactylography is the study of fingerprints.
Option B - Cheiloscopy:
• Cheiloscopy is the study of lip printing.
Option C - Ridgeology:
• Rideology is the study of flow of ridges.

Solution for Question 34:


Correct Option A - 2 swab technique for saliva collection:
• Swabbing of saliva is the first step to be done to identify or exclude assailant .
Incorrect Options:
Option B - Keep scale for measuring below the mark and take photographs:
• Photograph: Bite mark is photographed from different angles during the Bite mark investigation.
• Bite marks can be matched for identification purposes with the dental profile of an accused.
Option C - Prepare cast using vinyl polysiloxane:
• Impression of bite mark: Plastic substance (rubber or silicone based) or plaster of Paris is laid over the
bite mark that hardens and produces permanent negative cast of the lesion.
Option D - Skin is removed and preserved:
• Skin carrying the bite is removed and preserved in formalin during autopsy.

Solution for Question 35:


Correct Option C - Teeth:
• Teeth: DNA can be extracted from the pulp of the teeth even in a burnt body.
Incorrect Options:
Option A/ B/ D - Hair/ Bone/ Blood:
• Hair: Singed.

Page 22

485
• Blood: Burned
• Bone: Difficult to extract DNA from charred bones.

Page 23

486
Previous Year Questions
1. Which of the following changes does not occur in chronic starvation?
A. Hyperthermia
B. Exhaustion
C. Heart atrophy
D. Hypotension
----------------------------------------
2. What does the observation of contracted hands with no primary relaxation during a post-mortem
examination indicate?
A. Rigor mortis
B. Livor mortis
C. Cadaveric spasm
D. Algor mortis
----------------------------------------
3. What is the term used to describe the comprehensive examination of death in all its aspects?
A. Thanatology
B. Forensic anthropology
C. Toxicology
D. Molecular death
----------------------------------------
4. After the police discovered a skull bone that had been separated from its body, the family members
of a person who had gone missing aim to verify if the bone belongs to their loved one. Which of the
below methods can be employed to determine the identity of an individual by comparing a beheaded
skull bone with a photograph of the person?
A. Superimposition
B. Anthropometry
C. DNA fingerprinting
D. Cheiloscopy
----------------------------------------
5. What is the method of conducting a mass autopsy, involving the removal of organs from the tongue
to the prostate?
A. Virchow technique
B. Rokitansky technique
C. Ghon technique
D. Letulle technique
----------------------------------------

487
6. What is the appropriate autopsy procedure to be performed on the body of a 42-year-old patient who
was HIV-positive and has passed away?
A. Ghon technique
B. Letulle technique
C. Virchow technique
D. Rokitansky technique
----------------------------------------
7. What is the likely cause of a deceased person having a body temperature of 39 degrees Celsius?
A. Cyanide poisoning
B. Septicemia
C. Corrosive poisoning
D. Intra-abdominal hemorrhage
----------------------------------------
8. Post-mortem examination of a patient reveals the following. What is this finding known as?

A. Hesitation cuts
B. Grazed abrasion
C. Laceration
D. Patterned bruise
----------------------------------------
9. Identify the postmortem change given in the image.

Page 2

488
A. Marbling
B. Tattooing
C. Lividity
D. Filigree burns
----------------------------------------
10. On a hot summer afternoon, a man who had been working in a field suddenly collapsed. Despite
showing no physical signs of dehydration and having normal serum electrolyte levels, the doctor
discovered that his body temperature was 106 degrees Fahrenheit. Among the given options, which
symptom is least expected to be observed in this patient?
A. Hot skin
B. Hypotension
C. Sweating
D. Disorientation
----------------------------------------
11. The below phenomenon is due to

A. Case of torture phalanges.


B. Case of suspension or hanging more than 72 hours
C. Case of immersion in water for 36 hours
D. Case of colliquative liquefaction due to immersion in hot water
----------------------------------------
12. Which of the following is true regarding the post-mortem change shown in the image given below?

Page 3

489
A. Seen within first 12 hours of death
B. It occurs due to the accumulation of deoxygenated blood
C. It results due to bacterial activity
D. It is seen in case of lightning
----------------------------------------
13. What would be the most appropriate description for the situation in which a woman delivers twins,
and the father, suspecting that he is not the biological father of both children, requests DNA testing,
which confirms that he is indeed not the father of one of the twins?
A. Superfetation
B. Superfecundation
C. Suppositious child
D. Atavism
----------------------------------------
14. A child, who has been subjected to harm under the Protection of Children from Sexual Offences
(POCSO) Act, is presented to the Department of forensic medicine to determine their age. The
provided image displays an X-ray of the hand. What is the probable age range of the child?

A. 4 years
B. 7 years
C. 10 years
D. 13 years

Page 4

490
----------------------------------------
15. What phenomenon can be identified in a deceased individual when there is swelling of the face,
protrusion of the tongue, and other observable findings as shown in the image?

A. Electrocution
B. Crocodile burns
C. Scalds
D. Putrefaction
----------------------------------------
16. What is the term used to describe the following observation made during an autopsy in cases of
drowning?

A. Cadaveric spasm
B. Cutis anserina
C. Rigor mortis
D. Washerwoman’s hands
----------------------------------------
17. Please determine the smell resulting from the alteration depicted in the accompanying picture.

Page 5

491
A. Odorless
B. Pungent
C. Putrid
D. Offensive
----------------------------------------
18. Match the following with the respective ages of fusion : Medial end of clavicle Sacrum as a single
bone Crista scapulae Lambdoid suture 22-25 years 50 years 45 years 21-25 years
Medial end of clavicleSacrum as a single boneCrista scapulaeLambdoid suture 22-25 years50 years45 years21-25 years

A. A-1, B-2, C-3, D-4


B. A-4, B-1, C-2, D-3
C. A-4, B-3, C-1, D-2
D. A-1, B-4, C-3, D-2
----------------------------------------
19. A pelvis is brought for forensic examination. Which of the following characteristics will identify the
given pelvis as a male? Triangular obturator foramen U-shaped pubic angle Evered ischial tuberosity
Large greater sciatic notch Large acetabulum Deep preauricular sulcus
A. 2,3,4,5
B. 4,5,6
C. Only 5
D. 1 and 3
----------------------------------------
20. Please match the following findings observed during postmortem examination with their
corresponding time of appearance: A. Adipocere formation B. Relaxation of urethral sphincter C.
Marbling of veins D. Liquefaction of tissues 24 hrs 2-3 days 4-5 days 8-10 days
A. Adipocere formationB. Relaxation of urethral sphincterC. Marbling of veinsD. Liquefaction of tissues 24 hrs2-3 days4-5 days8-10 days

A. A-4, B-1, C-2, D-3


B. A-3, B-1, C-2, D-4
C. A-4, B-2, C-1, D-3
D. A-1, B-2, C-3, D-4

Page 6

492
----------------------------------------
21. A patient came to the casualty after a road traffic accident. A lacerated wound of 4x1 cm, which is
bone – deep, is noted over the scalp of the forehead. Which of the following is the correct mechanism
of this lacerated wound?
A. Shearing force
B. Impact of a semi sharp-edged object
C. Overstretching of skin and subcutaneous tissue
D. Tissue crushed between weapon and underlying bone
----------------------------------------
22. A skull was recovered from a forest. According to inquest papers, a girl had gone missing 15 days
back. The skull was sent to the forensic research lab. Which of the following would identify it as a
female skull? Large frontal and parietal eminence Heavy cheek bones Less pronounced glabella
Square orbits Small mastoid
A. 1 and 4 only
B. 2,3 and 4
C. 1,3 and 5
D. 3,4 and 5
----------------------------------------
23. What observations would you expect to find during a postmortem examination of a 20-year-old
man's body recovered from the sea, indicating death due to drowning in seawater? Hypernatremia
Hyponatremia Hyperkalemia Myocardial anoxia Hemodilution
A. 1 and 3
B. 1 and 4
C. 2,3 and 5
D. 2,4 and 5
----------------------------------------
24. Which of the following statements accurately describes the cause of the observed finding?

A. Lightning strike
B. Electrocution
C. Marbling

Page 7

493
D. Trickling down of acid
----------------------------------------
25. Please determine the specific fingerprint pattern displayed in the provided image.

A. Whorl
B. Composite
C. Loop
D. Arch
----------------------------------------
26. Rigor mortis occurs first in:
A. Eyelids
B. Intestines
C. Myocardium
D. Neck
----------------------------------------
27. Which of the following joint is the best predictor of age of 16 to 17 years by X-ray?
A. Knee
B. Elbow
C. Hip
D. Ankle
----------------------------------------
28. What is the odor of the mummified body?
A. Odorless
B. Pungent
C. Putrid
D. Offensive
----------------------------------------
29. What is the estimated age range of an individual with fusion of the medial end of the clavicle and
complete fusion of the sternum?

Page 8

494
A. <18 years
B. 18-22 years
C. 22-25 years
D. >25 years
----------------------------------------
30. What can be observed in cases of freshwater drowning? A. Haemodilution B. Hypokalemia C.
Hyponatremia D. Arrhythmia
A. A,B,C,D
B. A,B,C
C. B,C,D
D. A,C,D
----------------------------------------
31. Identify the correct pair i. Mummification ii. Adipocere iii. Marbling iv. Lividity of death

A. i -4, ii-3, iii-2, iv-1


B. i -3, ii-4, iii-2, iv-1
C. i -3, ii-1, iii-4, iv-2
D. i -3, ii-4, iii-1, iv-2
----------------------------------------
32. Please determine the following alteration observed during the postmortem examination.

A. Tattooing

Page 9

495
B. Suggillation
C. Putrefaction
D. Decomposition
----------------------------------------
33. As an intern assigned to the casualty department, you are presented with an MLC (Medico-Legal
Case) brought in by the police. The patient's physical condition indicates that she was pronounced
dead upon arrival. Before referring the patient for postmortem examination, what would you state as the
probable reason for her demise?

A. Drowning
B. Hanging
C. Strangulation
D. Traumatic asphyxia
----------------------------------------
34. A patient who was involved in a road traffic accident 20 minutes ago has been brought in for
autopsy. The post-mortem examination indicates rigidity in the neck, upper limbs, and lower limbs, with
no signs of initial relaxation. What does this indicate?
A. Rigor mortis
B. Heat stiffness
C. Cadaveric spasm
D. Cold stiffness
----------------------------------------
35. Identify the condition shown in the image.

Page 10

496
A. Postmortem lividity
B. Marbling
C. Fillgree burns
D. Subcutaneous emphysema
----------------------------------------
36. What is the initial postmortem change observed in a deceased body?
A. Maggot formation
B. Putrefaction
C. Greenish discoloration of right iliac fossa
D. Mummification
----------------------------------------
37. Paradoxical undressing is seen in :
A. Heart stress
B. Voyeurism
C. Hypothermia
D. LSD overdose
----------------------------------------
38. What is the commonly used method for preserving viscera in toxicology autopsy?
A. Glycerin
B. Rectified spirit
C. Formalin
D. Saturated salt solution
----------------------------------------
39. In the case of corrosive acid ingestion, the stomach is surgically opened.
A. Lesser curvature
B. Greater curvature
C. Vertical

Page 11

497
D. Pylorus
----------------------------------------
40. Identify the post-mortem change described below.

A. Tattooing
B. Suggilation
C. Putrefaction
D. Decomposition
----------------------------------------
41. The postmortem change seen in the image below corresponds to which of the following:

A. Marbling
B. Algor mortis
C. Rigor mortis
D. Postmortem lividity
----------------------------------------
42. What is the correct option concerning the techniques used for organ removal? A. Rockitansky – In
situ B. Virchow – en masse C. Ghon’s – en block D. Letulle – One by one
A. A and C are correct
B. A, B and C are correct
C. B and D ,A are correct
D. A,B,C and D are correct

Page 12

498
----------------------------------------
43. At what age does the fusion of the basiocciput and basisphenoid occur?
A. 18-22 years
B. 22-25 years
C. 14-16 years
D. 12-14 years
----------------------------------------
44. Which of the following trait is feature of the female skull ?
A. Larger teeth
B. Mastoid medium-large
C. Mandible Rounded
D. Zygomatic is more pronounced
----------------------------------------
45. A girl was allegedly kidnapped by a man. In court, she says that she is a major and is married to the
man, X ray images of her elbow, wrist and pelvis are given. What is the approximate age of the girl?

A. 14 to 15 years
B. 16 to 17 years
C. 18 to 19 years
D. 21 to 22 years
----------------------------------------

Page 13

499
46. Determining the age at which a person died and identifying their sex are crucial aspects of forensic
odontology in identifying individuals. Teeth are highly reliable in the process of age estimation and are
exceptionally resilient compared to other body parts. This is especially valuable when identifying bodies
in mass disasters and natural calamities. Gustafson (1950) introduced the first scientific technique for
estimating age in adults, which involved analyzing longitudinal sections of teeth in the central area. This
technique involved assigning scores ranging from 0-3 based on the presence and extent of age-related
changes such as attrition, periodontal ligament retractions, secondary dentin formations, root
translucency, and root resorption. Among Gustafson's criteria, the most dependable criterion is?
A. Root resorption
B. Paradentosis
C. Translucency of root
D. Attrition
----------------------------------------
47. What is the estimated gestational age of the fetus discovered in the water canal, considering its
crown heel length measured at 25 cm?
A. 6 months
B. 5 months
C. 4 months
D. 3 months
----------------------------------------
48. What bone is the most reliable for determining the sex of an individual?
A. Skull
B. Pelvis
C. Long bones
D. Sternum
----------------------------------------

Correct Answers
Question Correct Answer

Question 1 1
Question 2 3
Question 3 1
Question 4 1
Question 5 4
Question 6 4
Question 7 2
Question 8 1
Question 9 1

Page 14

500
Question 10 3
Question 11 3
Question 12 3
Question 13 2
Question 14 4
Question 15 4
Question 16 1
Question 17 1
Question 18 2
Question 19 3
Question 20 1
Question 21 4
Question 22 3
Question 23 2
Question 24 1
Question 25 4
Question 26 3
Question 27 4
Question 28 1
Question 29 4
Question 30 4
Question 31 4
Question 32 2
Question 33 1
Question 34 3
Question 35 2
Question 36 3
Question 37 3
Question 38 4
Question 39 2
Question 40 2
Question 41 1
Question 42 1
Question 43 1
Question 44 3

Page 15

501
Question 45 4
Question 46 3
Question 47 2
Question 48 2

Solution for Question 1:


• Hyperthermia refers to elevated body temperature. In chronic starvation, the body's metabolic rate
decreases, and thermoregulation mechanisms work to conserve heat rather than generate excess
heat. Therefore, hyperthermia is not a characteristic change seen in chronic starvation.
Incorrect Choices:
• Option b. Chronic starvation leads to the depletion of energy reserves in the body, resulting in fatigue,
weakness, and exhaustion.
• Option c. In chronic starvation, the body conserves energy by reducing its metabolic rate. This can
result in muscle wasting, including the heart muscle, leading to heart atrophy.
• Option d. Hypotension refers to low blood pressure. In chronic starvation, the body tries to conserve
fluids and maintain blood volume, which can result in decreased blood pressure.

Solution for Question 2:


• Cadaveric spasm, or cataleptic rigidity, occurs when the muscles that are contracted during life,
become stiff and rigid immediately, after death, without passing into the stage of primary relaxation. The
spasm originates by normal nervous stimulation of the muscles and hence, is usually limited to a single
group of involuntary muscles and frequently involves the hands.
Incorrect Choices:
• Option a. Rigor Mortis, also called death stiffening or cadaveric rigidity, is a state of stiffening of
muscles, sometimes with slight shortening of the fibres. There is individual cell death that occurs when
rigor mortis sets in. It affects all muscles in the body including voluntary and non-voluntary, contracted
or uncontracted, unlike cadaveric spasms.
• Option b. Livor Mortis, or post-mortem hypostasis, is the purplish discoloration of skin in the
dependent parts of the body caused by the collection of blood in those parts due to gravitational pull.
The purplish colour is due to the deoxyhemoglobin.
• Option d. Algor Mortis is the cooling of the body post death. The body does not follow Newton's law
when losing temperature. The fall in temperature is because of the cessation of heat production by the
somatic cells of the body.

Solution for Question 3:


• Thanatology deals with death in all aspects including somatic, systemic and clinical, and molecular or
cellular death.
Incorrect Choices:

Page 16

502
• Option b. Forensic anthropology involves the study of human remains. It involves applying the skeletal
analysis, descriptive data, body marks, and photographs of the individual together to get to forensic
inferences. It only deals with one aspect of death and not all of them.
• Option c. Toxicology is the study of analysing suspect samples for the presence of toxins. It only deals
with toxins that can be a cause of death- one aspect of death and not all of them.
• Option d. Molecular death is the death of cells and tissues individually. It occurs one to two hours after
the stoppage of the vital functions. It only deals with one aspect of death and not all of them.

Solution for Question 4:


• The superimposition technique is a method used in forensic anthropology to assist in the analysis of
an unknown skull. The technique involves using the ante mortem image of the victim superimposed by
the image of the recovered skull. The principle behind the study is that the shape of the face is related
to the bony contours of the facial part of the skin.
Incorrect Choices:
• Option b. Anthropometry or Bertillon system involves analysis of the descriptive date of individuals,
body marks on individuals, the individual's body measurements and the photographs of particular
aspects of the individuals. It does not require an ante mortem photograph of the face of the victim.
• Option c. DNA Fingerprinting is a method of forensic analysis that uses the study of analysing DNA
found on articles of the crime scene and victim. The DNA has 95% of silent genes which are very
specific to different individuals. The number of repeats in a repetitive DNA can be used to match with
suspect DNAs to come to a conclusion. This technique does not require an ante mortem photograph of
the face of the victim.
• Option d. Cheiloscopy is a technique of forensic analysis that involves human identification based on
lips. It uses a method similar to that of fingerprint analysis. It does not require an ante mortem
photograph of the face of the victim and is mainly a study to detect the accused from a set of suspects.

Solution for Question 5:


• Letulle technique is named after Maurice Letulle which involves the removal of organs together in one
mass from tongue to prostate. This method of autopsy is thus also called “en masse”. The anatomy of
the organs and organ systems can be preserved in the greatest possible due to this. Forensic
pathologists can get a better overall picture due to the preservation of anatomical linkages.
Incorrect Choices:
Option a. Virchow technique: This approach is named after Rudolf Virchow. Instead of removing all org
ans at once, this method involves the sequential removal and examination of individual organs. It's a m
ore time-consuming process but provides the opportunity to inspect each organ in detail, in its anatomi
c position. The Virchow technique does not fit the description in the question because it does not involv
e the "en masse" removal of organs.
Option b. Rokitansky technique: All of the bodily organs are removed at once using the Rokitansky tec
hnique. In other words, the organs such as the heart, lungs, liver, kidneys, spleen, and others are remo
ved in one piece and dissected on the autopsy table.

Page 17

503
Option c. Ghon technique: Anatomical relationships are maintained by using the Ghon approach, often
known as "en bloc". Instead of one enormous block, the thoracic, abdominal, and pelvic organs are re
moved using this procedure in three separate blocks. The dissection is made much more manageable
by dividing the huge block into these three portions while still keeping the anatomy between the organs
.

Solution for Question 6:


Option D - Rokitansky procedure
• The Rokitansky procedure is a local (or in situ) dissection that begins at the neck and proceeds
downward while also removing the organ in one piece.
• The larynx is punctured first to separate the esophagus and pharynx, followed by the larynx and
trachea, and finally, the chest organs are severed to expose the abdominal organs.
• As the patient was HIV positive, the use of this technique prevents the spread of the disease.
Incorrect Choices:
Option a. Ghon technique: The thoracic, cervical, and abdominal organs are removed using the "en blo
c" procedure in the Ghon technique, which is comparable to the Rokitansky but does not use an in situ
dissection hence is not a useful form of autopsy for an HIV-positive individual.
Option b. Letulle technique: The cervical, thoracic, abdominal, and pelvic organs are removed as a sin
gle mass using the Letulle technique, sometimes known as the en masse technique, and are then subd
ivided into organ blocks. The linkages between the organs and the circulatory supply can both be pres
erved using this procedure but it cannot prevent the spread of infections such as HIV.
Option c. Virchow technique: Instead of removing all organs at once, this method involves the sequenti
al removal and examination of individual organs. It's a more time-consuming process but provides the
opportunity to inspect each organ in detail, in its anatomic position. This method is not useful for the au
topsy of an HIV-positive individual as it can cause the spread of disease.

Solution for Question 7:


• Septicemia, or sepsis, refers to a severe infection that spreads through the bloodstream. This
condition causes systemic inflammation and could lead to multiple organ dysfunction or failure. A
significant symptom of sepsis is high body temperature or fever, which aligns with the scenario
described in the question. A body temperature of 39 degrees Celsius (102.2 degrees Fahrenheit)
suggests an active systemic infection, making septicemia the most probable cause.
Incorrect Choices:
Option a. Cyanide poisoning: Cyanide is a potent and rapidly acting poison. Symptoms of cyanide pois
oning include headache, confusion, nausea, and can result in cardiac arrest. Cyanide poisoning may le
ad to death swiftly but it typically does not result in a
sustained elevated body temperature, especially not after death.
Option c. Corrosive poisoning: Corrosive substances can cause severe damage to the tissues they co
me into contact with. However, these substances do not typically cause fever. In cases of corrosive poi
soning, the primary symptoms are related to the areas of contact, which could include the mouth, throa
t, and stomach.

Page 18

504
Option d. Intra-abdominal hemorrhage: This condition refers to bleeding within the abdomen, which co
uld be due to several causes such as trauma or a
ruptured blood vessel. While intra-abdominal hemorrhage is a
serious condition, it does not generally cause a high body temperature.

Solution for Question 8:


Correct Option A - Hesitation cuts:
• Hesitation cuts refer to superficial wounds, often found on the wrists or forearms, which are thought to
be self-inflicted as part of a suicide attempt. These wounds are usually not immediately life-threatening
being relatively shallow in nature. This contrasts with more decisive, deeper cuts that may pose a
greater immediate danger due to more severe blood loss.
Incorrect Options:
Option B - Grazed abrasion: Grazed abrasions are brought on by the body being dragged across a rou
gh surface, which causes the epidermis to scrape. The depth of grazed injuries varies according to the
body's speed and the surface's roughness. The given image does not indicate a grazed abrasion.
Option C - Laceration: A laceration is a wound that results from soft bodily tissue being torn. This kind
of wound frequently has uneven and jagged edges. The image is not indicative of laceration.
Option D - Patterned bruise: A patterned injury is one that has a clear pattern that may mimic the featur
es of the object that caused it. The pattern may be brought on by the body coming into contact with a
patterned surface or being struck by a weapon or other instrument. The image doesn’t show a
patterned bruise.

Solution for Question 9:


Correct Option A - Marbling:
• Marbling is a postmortem change that occurs in the body after death. It refers to the development of a
network of dark, irregularly shaped lines or patches on the skin. These lines or patches seen in the
image resemble the patterns found in marble, hence the term marbling.
Incorrect Options:
Option B - Tattooing: The given image is not of Tattooting.
Option C - Lividity: The given image is not of Lividity
Option D - Filigree burns: The given image is not of Filigree burns

Solution for Question 10:


Correct Option C:
• In the given scenario, with a body temperature of 106 degrees Fahrenheit, the symptom that is least
likely to be seen in this patient is sweating.

Page 19

505
• Heatstroke, also known as sunstroke, is a severe form of heat-related illness characterized by
elevated body temperature. In heatstroke, the body's thermoregulatory mechanisms fail, leading to a
rapid rise in body temperature. Sweating is the body's natural mechanism to cool down, but in cases of
severe heatstroke, the body's ability to sweat may be impaired
Incorrect Options:
Option A. Hot skin: This option is commonly seen in cases of heatstroke. When the body's thermoregul
atory mechanisms fail, the body temperature rises, and the skin can become hot to the touch. This sym
ptom is often associated with heatstroke and is not least likely to be seen in the given scenario.
Option B. Hypotension: Heatstroke can lead to dehydration and a
decrease in blood volume, which can result in hypotension (low blood pressure). Hypotension is a
common finding in severe heatstroke and is not least likely to be seen in the given scenario.
Option D. Disorientation: Heatstroke can cause neurological symptoms, including disorientation and alt
ered mental status. The rise in body temperature can affect brain function and lead to confusion, disori
entation, and even loss of consciousness. Disorientation is commonly observed in severe heatstroke a
nd is not least likely to be seen in the given scenario

Solution for Question 11:


Correct Option C- Case of immersion in water for 36 hours:
• Immersion in water for an extended period can lead to specific postmortem changes. These changes
include maceration of the skin, separation of the epidermis, and swelling due to water absorption.
Additionally, there may be a softening and detachment of the skin, known as a "washerwoman's hands"
or "glove-like" appearance. Other features that can be observed include skin slippage, wrinkling, and a
pale appearance.
Incorrect Options:
Option A- Case of torture phalanges: Torture can cause various injuries and damage to the phalanges,
such as fractures, dislocations, or soft tissue injuries. Torture-related injuries would have different char
acteristics and patterns than those resulting from water immersion.
Option B- Case of suspension or hanging more than 72 hours: Prolonged suspension or hanging can l
ead to characteristic postmortem changes, such as livor mortis (postmortem lividity), rigor mortis (stiffe
ning of the body), and changes in the neck due to ligature marks or fractures.
Option D- Case of colliquative liquefaction due to immersion in hot water: Colliquative liquefaction refer
s to the process of tissue breakdown and liquefaction that occurs in certain conditions, such as prolong
ed immersion in warm or hot water.

Solution for Question 12:


Correct Option C: It results from bacterial activity:
• The image shows post-mortem marbling.
• Post-mortem marbling appears as discoloration of prominent superficial veins to a green-brown color.
• Clostridium welchii bacteria form Hydrogen Sulphide gas in the superficial veins, which combine with
the Hb to form Sulfhemoglobin (SulfHb), which leads to these changes.

Page 20

506
Incorrect Options:
Option A. Seen within first 12 hours of death: Post-mortem marbling can begin to appear after
Option B. It occurs due to the accumulation of deoxygenated blood: Post-mortem marbling appears du
e to the formation of sulfhemoglobin.
Option D. It is seen in the case of lightning: Lightning injuries do not correspond to the vascular channe
ls or vessels and is associated with pink color changes.

Solution for Question 13:


Correct Option B:
• Superfecundation refers to the fertilization of two or more ova from different acts of sexual intercourse,
resulting in the conception of multiple embryos with different genetic fathers. In the given scenario, the
woman gave birth to twins, but DNA testing revealed that one child did not belong to the father.
• This indicates that two separate acts of sexual intercourse with different partners occurred, leading to
the fertilization of two different ova by two different spermatozoa. As a result, the twins have different
genetic fathers, confirming the occurrence of superfecundation.
Incorrect Options:
Option A. Superfetation: Superfetation refers to the occurrence of a
second pregnancy during an ongoing pregnancy, where a woman conceives another fetus while alread
y pregnant. This is not applicable in the given scenario as there is no indication of a
second pregnancy occurring during an ongoing pregnancy.
Option C. Suppositious child: A suppositious child refers to a child who is falsely or fraudulently claime
d or substituted as one's own. In the given scenario, there is no indication of false claims or substitution
of the child, as the DNA testing revealed that one child did not belong to the father. Therefore, supposi
tious child is not the appropriate term for this case.
Option D. Atavism: Atavism refers to the reappearance of a characteristic or trait in an organism that is
not present in its immediate ancestors but was present in previous generations. It is not applicable in th
e given scenario, as the issue at hand is related to the genetic paternity of the twins, rather than the re
appearance of ancestral traits.

Solution for Question 14:


Correct Option D:
• The X-ray image of the hand shows that all carpal bones are visible and the lower epiphysis of the first
metacarpal has not yet united. These findings indicate that the child's skeletal development is
consistent with an age of approximately 13 years.
Incorrect Option:
Option A: 4 years: By the age of 4 years, the carpal bones in the hand would not be fully developed, a
nd the lower epiphysis of the first metacarpal would likely be fused.
Option B: 7 years: By the age of 7 years, most of the carpal bones would be visible on X-ray, but the lo
wer epiphysis of the first metacarpal would usually be fused.

Page 21

507
Option C: 10 years: By the age of 10 years, the carpal bones would be well-developed and most of the
m would be fused. The lower epiphysis of the first metacarpal would also be fused by this age.

Solution for Question 15:


Correct option: D
• Putrefaction refers to the decomposition or decay of a body after death. It is a natural process that
occurs as a result of the breakdown of tissues by bacteria and enzymes in the body. Various signs and
changes are observed during putrefaction, including bloating of the face and other body parts, as well
as tongue protrusion.
Incorrect options:
Option A and B. Electrocution: Electrocution refers to the injury or death caused by an electric shock.
While electrocution can cause various physical signs and symptoms, such as burns and internal organ
damage, it does not specifically result in bloating of the face, tongue protrusion, or putrefaction. Electro
cution-related findings would typically involve specific burn patterns or injury marks associated with the
electrical source.crocodile burns are seen in hight voltage electric burns.
Option C. Scalds: Scalds are burns caused by contact with hot liquids or steam. While scalds can resul
t in various skin-related changes, such as blisters and skin peeling, they do not lead to the specific findi
ngs of bloating of the face, tongue protrusion, or putrefaction. Scalds are acute injuries and do not invol
ve postmortem processes.

Solution for Question 16:


Correct Option: A
• A condition known as cadaveric spasm occurs when a group of muscles that were actively utilised
soon before death suddenly become stiff and inflexible. The majority of cases of this "instantaneous
rigour" affect the hands, while extremely occasionally cadaveric spasms might affect the entire body.
Incorrect Options:
Option B. Cutis anserina: Commonly referred to as goosebumps, this short local change in the skin occ
urs when tiny muscles contract in response to cold, fright, or excitement. This skin alteration is the resu
lt of a series of events that begin with a stimulus like cold or fright.
Option C. Rigor mortis: Rigour mortis is the stiffening or rigidity of the body after death. It happens whe
n adenosine triphosphate (ATP) levels fall below a certain point. Due to the muscle cells' lack of integrit
y, calcium ions flood muscle fibres after death. These ions cause the actin and myosin filaments to bin
d, which results in contraction. The ATP-driven pumping of calcium ions back into the muscle cells' sar
coplasmic reticulum causes the muscles to relax. The absence of ATP prevents the muscles from relax
ing and keeps the actinomyosin complex formed during the contraction, which makes the muscles rigid
and hard.
Option D. Washerwoman’s hands: Skin maceration, which starts happening in warm water within minut
es and takes longer to become noticeable in cold water immersion, is the main indicator of immersion.
The skin starts to seem saggy, pallid, and wrinkled, similar to a "washerwoman's skin."

Page 22

508
Solution for Question 17:
Correct Option: A
• The change seen in the image is the mummified body. The bodies, if protected, can be preserved for
years, and they are practically odorless.
• Marked dehydration before death favours mummification. It takes 3 months to 1 year for the entire
body to be mummified.
• The absence of moisture and the continuous action of dry warm air are necessary for this process.
The change seen in the image is the mummified body. The bodies, if protected, can be preserved for y
ears, and they are practically odorless.
Marked dehydration before death favours mummification. It takes 3 months to 1
year for the entire body to be mummified.
The absence of moisture and the continuous action of dry warm air are necessary for this process.
Incorrect Options:
Option B. Pungent: Decomposition is the main reason for the pungent strong odour that surrounds a
dead body. The tissues of a decomposing body are broken down by bacteria and enzymes, generating
numerous gases and chemicals that add to the odour.
Option C. Putrid: The advanced phases of decomposition are what cause the putrid odour that is assoc
iated with a dead body. The body's tissues disintegrate and produce a
variety of gases and chemicals throughout this process, creating a vile and repulsive odour.
Option D. Offensive: The decomposition process is what gives a dead corpse its offensive odour. As th
e corpse decomposes, bacteria and enzymes disintegrate the tissues, generating gases and substanc
es that give off a very foul odour.

Solution for Question 18:


A. Medial end of clavicle:
The correct match is option 4 - 21-25 years.
Explanation: The medial end of the clavicle refers to the innermost portion of the collarbone. The fusion
of the medial end of the clavicle with the shaft of the clavicle typically occurs around the ages of 21-25
years. This fusion marks the completion of growth and development of the clavicle.
B. Sacrum as a single bone:
The correct match is option 1 - 22-25 years.
Explanation: The sacrum is a large triangular bone at the base of the spine, composed of fused vertebr
ae. The fusion of the sacrum, where individual segments of the sacral vertebrae become a single bone,
generally occurs between the ages of 22-25 years. This fusion contributes to the stability of the pelvic r
egion.
C. Crista scapulae:
The correct match is option 2 - 50 years.

Page 23

509
Explanation: The crista scapulae refers to a bony ridge present on the scapula, also known as the shou
lder blade. The fusion of the crista scapulae typically occurs around the age of 50 years. This fusion is
a normal age-related change and does not have any significant functional implications.
D. Lambdoid suture:
The correct match is option 3 - 45 years.
Explanation: The lambdoid suture is a fibrous joint that connects the parietal bones and the occipital bo
ne in the skull. The fusion of the lambdoid suture generally occurs around the age of 45 years. The clo
sure of this suture is a part of the natural aging process.

Solution for Question 19:


Correct option - Option -C - Only 5
Large acetabulum:
• The acetabulum is the socket in the pelvis that forms the hip joint. In males, the acetabulum tends to
be larger and deeper compared to females.
• A large acetabulum is an indicator of a male pelvis.
Incorrect Choices:
Options A, B, and D:
1. Triangular obturator foramen:
• In males, the obturator foramen is typically more oval or rounded in shape, while in females, it tends
to be more triangular.
2. U-shaped pubic angle:
• In males, the pubic angle tends to be more narrow and V-shaped, while in females, it is wider and
U-shaped. However, a U-shaped pubic angle is not exclusive to males and can also be found in some
females.
3. Everted ischial tuberosity:
• In males, the ischial tuberosity tends to be more projected or pronounced compared to females.
However, the presence of an everted (pronounced) ischial tuberosity is not a definitive indicator of a
male pelvis.
4. Large greater sciatic notch:
• In males, the greater sciatic notch is generally narrower compared to females, where it tends to be
wider. A large greater sciatic notch is more commonly seen in female pelvises, so it does not indicate a
male pelvis.
6. Deep preauricular sulcus:
• This characteristic is not directly related to determining the sex of the pelvis and is not exclusive to
either males or females.

Solution for Question 20:

Page 24

510
Correct Choice: A
A. Adipocere formation:
• The correct match is option 4 - 8-10 days.
• Adipocere formation, also known as grave wax, is a postmortem change that occurs when body fat
undergoes hydrolysis and saponification. This process is influenced by environmental factors, such as
moisture and temperature. Adipocere formation typically takes place in the later stages of
decomposition, usually around 8-10 days after death.
B. Relaxation of urethral sphincter:
• The correct match is option 1 - 24 hours.
• After death, the muscles of the body start to relax due to the absence of vital functions. The relaxation
of the urethral sphincter, which controls the release of urine, is one of the early postmortem changes. It
occurs relatively soon after death, typically within 24 hours.
C. Marbling of veins:
• The correct match is option 2 - 2-3 days.
• The marbling of veins, also known as hypostasis or livor mortis, is the pooling and settling of blood in
the dependent parts of the body after death. This phenomenon occurs due to gravity and the cessation
of blood circulation. The marbling of veins usually becomes visible around 2-3 days after death.
D. Liquefaction of tissues:
• The correct match is option 3 - 4-5 days.
• Liquefaction of tissues, also referred to as autolysis, is a postmortem change that involves the
breakdown of tissues by the action of enzymes released from cells. This process leads to the softening
and liquefaction of body tissues. The liquefaction of tissues generally occurs around 4-5 days after
death.

Solution for Question 21:


• Lacerated wounds are caused by blunt force trauma that results in a tear or cut in the skin and
underlying tissues. In this particular case, the wound is described as being "bone-deep," which
indicates that the injury extends to the underlying bone.
• Tissue crushed between weapon and underlying bone: This option correctly describes the mechanism
of the lacerated wound. When tissue is crushed between a weapon or object and the underlying bone,
it can result in a deep laceration that involves both the skin and the underlying tissues, reaching the
bone
Incorrect Choices:
Option A: Shearing force: Shearing force refers to the application of a force that causes one layer of tis
sue to move parallel to another. This type of force can result in various types of injuries, such as abrasi
ons and avulsions, but it is not typically associated with lacerated wounds. In a shearing force injury, th
e skin and tissues are more likely to be separated or torn apart rather than causing a
deep laceration that extends to the bone.
Option B: Impact of a semi sharp-edged object: While an impact from a
semi sharp-edged object could cause a laceration, it would not necessarily result in a wound that is bo

Page 25

511
ne-deep. Lacerations caused by sharp objects are typically superficial or involve only the soft tissues ra
ther than reaching the underlying bone.
Option C: Overstretching of skin and subcutaneous tissue: Overstretching of the skin and subcutaneou
s tissue would typically result in a different type of injury, such as an abrasion or a
stretch mark. It would not cause a deep laceration that extends to the bone.

Solution for Question 22:


Correct Choice: C

1. Large frontal and parietal eminence:


• In general, females tend to have less prominent frontal and parietal eminences compared to males.
Eminences refer to the prominent areas or bulges on the skull. Therefore, a large frontal and parietal
eminence can be an indication of a male skull rather than a female skull.
3. Less pronounced glabella:
• The glabella refers to the smooth area between the eyebrows and above the nasal bridge. In general,
females tend to have a smoother and less pronounced glabella compared to males. A smoother
glabella can be an indication of a female skull.
5. Small mastoid:
• The mastoid process is a bony prominence located behind the ear. In general, males tend to have a
more robust and prominent mastoid process compared to females. A narrower mastoid can be
indicative of a female skull.
Incorrect Choices:
Options A, B, and D: 2. Heavy cheekbones:
• This characteristic is not included in the correct answer.
• Heavy cheekbones, also known as prominent zygomatic arches, can occur in both males and
females. It is not a reliable indicator of gender as the size and prominence of the cheekbones can vary
among individuals regardless of their sex.
4. Square orbits:
• This characteristic is not included in the correct answer.
• The shape of the eye sockets or orbits does not reliably indicate the gender of the skull. The shape of
the orbits can vary among individuals and is not specifically associated with either males or females.

Solution for Question 23:


Correct Choice: B
1. Hypernatremia: This characteristic is included in the correct answer (1 and 4).
• Seawater contains a higher concentration of salt (sodium chloride) compared to the body's internal
fluids. When a person drowns in seawater, they inhale and ingest seawater, leading to an increase in

Page 26

512
sodium levels in the body. Hypernatremia, which refers to an elevated sodium concentration, can be
observed in cases of seawater drowning.
4. Myocardial anoxia: This characteristic is included in the correct answer (1 and 4).
• Myocardial anoxia, which refers to a lack of oxygen supply to the heart muscle, can be observed in
cases of seawater drowning. When a person drowns in seawater, the inhaled water can enter the lungs
and impair the exchange of oxygen and carbon dioxide. This can lead to hypoxia (lack of oxygen) and
subsequent myocardial anoxia.
Incorrect Choices:
Options A, C, and D: 2. Hyponatremia:This characteristic is not included in the correct answer.
• Hyponatremia is a condition characterized by low levels of sodium in the blood. In cases of seawater
drowning, hyponatremia is unlikely to occur because seawater has a higher salt concentration than the
body's fluids. Hyponatremia is more commonly associated with freshwater drowning, where water with
a lower salt concentration is ingested or aspirated.
3. Hyperkalemia: This characteristic is not included in the correct answer.
• Hyperkalemia refers to an elevated potassium level in the blood. While hyperkalemia can occur in
certain drowning scenarios, such as freshwater drowning, it is not typically associated with seawater
drowning. Seawater contains relatively low levels of potassium, so it is less likely to cause
hyperkalemia.

5. Hemodilution: This characteristic is not included in the correct answer.


Hemodilution refers to a decrease in the concentration of red blood cells or other blood components du
e to an increase in the fluid volume. While seawater drowning can cause fluid shifts and changes in blo
od composition, hemodilution is not a specific finding associated with seawater drowning.

Solution for Question 24:


Correct Choice: A
• The given finding is known as Lichtenberg's figures or filigree burns which are pathognomonic for
lightning injury. It is a superficial, thin, tortuous, fern-like pattern of erythema in the skin. It indicates the
path taken by the electrical discharge and tends to follow skin creases and long axis of the body.
• They are also known as arborescent burns or feathering as they have a resemblance to the branches
of a tree. Lichtenberg's figures occur due to the rupture of small vessels. They are usually seen over
the shoulder or flank within an hour of injury and disappear in 1-2 days.
Incorrect Choices:
Option B. Electrocution: Electrocution or electrical burns can cause the following lesions:
• Joule burns: These are elevated blisters that form when the epidermis or the epidermal-dermal
junction of the skin has been split by firm contact with an electrical wire.
• Spark lesion: The current jumps the gap as a spark when the contact is less firm, creating an air gap
between the skin and conductor. As a result, a little portion of the keratin in the outer skin melts. The
so-called spark lesion forms when the keratin cools down into a hard, brownish lump that is typically
raised above the surrounding surface.

Page 27

513
• Burns from high-tension currents are the cause of the crocodile skin effect. Sparking can happen over
a significant distance of centimeters in high-voltage burns, such as those caused by high-tension grid
transmission wires where the voltage is in the multi-kilovolt range. This may result in many spark
lesions and a crocodile-skin effect.
Option C. Marbling: Bacteria like clostridium multiply along the blood vessels &
produces H2S gas. H2S combines with Hb & forms sulf-hemoglobin. This sulf-Hb stains vessel walls gi
ving greenish, linear, branching pattern on the skin [Corresponding to vascular channels]. This is k/a M
arbling. The appearance of marbling = 36-72 hrs after death. It helps to determine T.S.
Option D. Tricking down of acid: Trickling down of acid can cause redness, burns, blistering, and pain.

Solution for Question 25:


Correct Option: Option D: Arch: An arch fingerprint pattern is the simplest and least common type. It ap
pears as a continuous wave-like pattern without any deltas or cores. In an arch pattern, the ridges ente
r from one side of the fingerprint and exit from the opposite side, forming a
slight upward or downward curve.
Other options
Option A: Whorl: A whorl fingerprint pattern consists of one or more concentric circles or spiral-like patt
erns. It appears as a circular or oval shape with ridges that form a spiral or circular pattern. Whorls can
be subdivided into several categories, such as plain whorl, central pocket loop whorl, double loop whorl
, and accidental whorl.
Option B: Composite: A composite fingerprint pattern is a
combination of two or more different fingerprint patterns. It typically consists of a
combination of loops, whorls, or arches in the same print. For example, a
composite pattern may have a loop on one side and a whorl on the other.
Option C: Loop: A
loop fingerprint pattern is characterized by ridges that enter from one side of the fingerprint, form a curv
e or loop, and exit from the same side. Loops can be further categorized as radial loops or ulnar loops
based on the direction the loop opens towards (towards the thumb or little finger, respectively). The ma
jority of fingerprint patterns fall into the loop category.

Solution for Question 26:


Correct Option C: Myocardium.
• Rigor mortis refers to the stiffening of the body's muscles after death due to biochemical changes in
muscle tissue. It typically begins within a few hours after death and progresses gradually over the next
6 to 12 hours.
• Rigor mortis first appears in the heart muscle within an hour after death.
Incorrect Options- Option A/B/D
• Among the voluntary muscles, rigor mortis usually develops sequentially and follows a descending
pattern, the so-called Nysten’s law: It first appears in the muscles of the eye lids (orbicularis oculi) (3–5

Page 28

514
h), then in jaw, facial muscles (4–5 h), neck, thorax (5–7 h) upper limb (from shoulder to the hand) (7–9
h), abdomen, lower limb (from the hip to the foot) (9–11 h), and lastly in the small muscles of fingers
and toes (11–12 h).
• It first appears in the muscles of the eye lids (orbicularis oculi) (3–5 h), then in jaw, facial muscles (4–5
h), neck, thorax (5–7 h) upper limb (from shoulder to the hand) (7–9 h), abdomen, lower limb (from the
hip to the foot) (9–11 h), and lastly in the small muscles of fingers and toes (11–12 h).
• It first appears in the muscles of the eye lids (orbicularis oculi) (3–5 h), then in jaw, facial muscles (4–5
h), neck, thorax (5–7 h) upper limb (from shoulder to the hand) (7–9 h), abdomen, lower limb (from the
hip to the foot) (9–11 h), and lastly in the small muscles of fingers and toes (11–12 h).

Solution for Question 27:


Correct Option D : Ankle - 16 to 17 years - Ankle joint
Incorrect Options
Option A: Knee - At 18 to 19 years - knee, shoulder and wrist joint
Option B: Elbow- At age of 16 years elbow joint
Option C: Hip - At age of 17 to 18 years

Solution for Question 28:


Correct Option A: Odorless
• Mummification is a process characterized by the rapid dehydration and desiccation of a dead body,
resulting in the preservation of its natural appearances and features through evaporation of water. Here
are the key points about mummification: Definition: Rapid dehydration and desiccation of the body,
preserving its natural appearance. Modification of putrefaction: Mummification is a form of dry
decomposition, distinct from the wet decomposition seen in putrefaction. Physical changes: The entire
body loses weight, becomes thin, stiff, and brittle. It also becomes odorless. Prevention of
decomposition: The process of normal decomposition, which involves the growth of microorganisms, is
prevented or significantly retarded in mummification. This is due to the rapid dehydration and lack of
moisture, which inhibits microbial growth.
• Definition: Rapid dehydration and desiccation of the body, preserving its natural appearance.
• Modification of putrefaction: Mummification is a form of dry decomposition, distinct from the wet
decomposition seen in putrefaction.
• Physical changes: The entire body loses weight, becomes thin, stiff, and brittle. It also becomes
odorless.
• Prevention of decomposition: The process of normal decomposition, which involves the growth of
microorganisms, is prevented or significantly retarded in mummification. This is due to the rapid
dehydration and lack of moisture, which inhibits microbial growth.

Page 29

515
Mummification is a process characterized by the rapid dehydration and desiccation of a dead body, res
ulting in the preservation of its natural appearances and features through evaporation of water. Here ar
e the key points about mummification:
• Definition: Rapid dehydration and desiccation of the body, preserving its natural appearance.
• Modification of putrefaction: Mummification is a form of dry decomposition, distinct from the wet
decomposition seen in putrefaction.
• Physical changes: The entire body loses weight, becomes thin, stiff, and brittle. It also becomes
odorless.
• Prevention of decomposition: The process of normal decomposition, which involves the growth of
microorganisms, is prevented or significantly retarded in mummification. This is due to the rapid
dehydration and lack of moisture, which inhibits microbial growth.
Incorrect Options:
Option B - Pungent: This option is not correct. A pungent odor refers to a
strong, sharp, and often unpleasant smell. It is not typically associated with a mummified body, as the
process of mummification involves the preservation of the body and the prevention of decomposition.
Option C - Putrid: This option is not correct. The term "putrid" refers to a
foul or rotten smell associated with decomposition. A mummified body does not have a
putrid odor because the mummification process hinders decomposition.
Option D - Offensive: This option is not correct. An offensive odor refers to a
strong, unpleasant, and offensive smell. Similar to the previous options, a mummified body does not ha
ve an offensive odor due to the preservation process that prevents decomposition.

Solution for Question 29:


The approximate age of a person in whom the medial end of the clavicle is fused and the sternum is co
mpletely fused is typically greater than 25 years. The fusion of the clavicle and sternum occurs as part
of the natural aging process and is considered a sign of skeletal maturity. Therefore, individuals who ex
hibit complete fusion of the clavicle and sternum are typically adults over the age of 25.

Solution for Question 30:


Correct Option D: A, C, D
In freshwater drowning, the following are seen:
A: Haemodilution: Freshwater ingestion during drowning can cause hemodilution, leading to a
decrease in the concentration of blood components.
C: Hyponatremia: Freshwater ingestion can result in dilution of sodium levels in the blood, leading to hy
ponatremia.
D: Arrhythmia: Imbalances in electrolytes, such as hyponatremia and other factors related to drowning,
can contribute to cardiac arrhythmias.
B: Hypokalemia is not typically associated with freshwater drowning. Hypokalemia is more commonly s
een in saltwater drowning due to the loss of potassium-rich fluids from the body.

Page 30

516
Solution for Question 31:
Correct option : D
• Mummification: Mummification is the preservation of a dead body, typically achieved through the
removal of moisture and the use of chemicals, natural processes, or environmental conditions. It
involves drying out the body to prevent decomposition. Therefore, it is correctly paired with option 3.
• Adipocere: Adipocere, also known as "grave wax," is a waxy substance that forms on a corpse under
specific conditions, such as when the body is buried in a moist environment or submerged in water. It is
a result of the hydrolysis of fats in the body. Thus, it is correctly paired with option 4.
• Marbling: Marbling, in the context of death, refers to the discoloration or mottling of the skin after
death. It occurs due to the settling of blood and the pooling of blood in the lower parts of the body.
Therefore, it is correctly paired with option 1.
• Lividity of death: Lividity of death, also known as livor mortis, is the process by which blood settles in
the lower parts of the body after death, resulting in a purple or reddish discoloration of the skin. It
occurs due to the gravitational pull on the blood. Thus, it is correctly paired with option 2.
Therefore, the correct pair is: i - Mummification (3) ii - Adipocere (4) iii - Marbling (1) iv -
Lividity of death (2)
Incorrect options:
The other choices are incorrect as they do not match the correct pairings

Solution for Question 32:


Correct Option : B
• Suggillation, also known as livor mortis or postmortem lividity, refers to the discoloration of the skin
that occurs after death. It happens as a result of the settling of blood in the dependent parts of the body,
due to the gravitational pull. This pooling of blood gives a reddish or purplish discoloration to the skin,
which can be observed in the areas closest to the ground or pressure points as seen in the given
image.
Incorrect options:
Option A. Tattooing: Tattooing is not a postmortem change but rather a
deliberate process of injecting ink into the skin during a
person's lifetime to create permanent designs or patterns.
Option C. Putrefaction: Putrefaction refers to the process of decomposition of body tissues after death.
It involves the breakdown of proteins by bacteria and enzymes, leading to the release of foul-smelling g
ases, bloating of the body, and eventual disintegration. While putrefaction is a
postmortem change, it is not the correct answer in this context.
Option D. Decomposition: Decomposition refers to the process by which organic matter breaks down in
to simpler substances after death. It involves various physical, chemical, and biological changes, includ
ing putrefaction, autolysis (self-digestion of cells), and skeletonization. While decomposition is a gener
al term for the overall process of breakdown, it is not the specific answer related to the given context.

Page 31

517
Solution for Question 33:
Correct Option A: Drowning:
Drowning occurs when a person's airway is submerged in water or another liquid, leading to asphyxia.
The presence of frothing from the nostrils can be indicative of water entering the airway during the dro
wning process. This frothy material consists of a mixture of water, air, and mucus.
Incorrect Option:
Option B. Hanging: Hanging is a form of asphyxia caused by constriction of the neck due to a
ligature or suspension. Frothing from the nostrils is not typically associated with hanging.
Option C. Strangulation: Strangulation involves the external compression of the neck, leading to asphy
xia. Frothing from the nostrils is not commonly observed in cases of strangulation.
Option D. Traumatic asphyxia: Traumatic asphyxia is a condition that occurs when there is a
sudden and severe compression of the chest, leading to a
lack of oxygen supply to the body. Frothing from the nostrils is not a
characteristic feature of traumatic asphyxia.

Solution for Question 34:


Correct Option C: Cadaveric spasm
• Cadaveric spasm occurs due to ATP depletion, and is usually seen in accidental cases, in which the
person performs sudden physical movements in distress just before death.
• It affects voluntary group of muscles(legs, neck, eyes) and short group of muscles(hands).
• Aka instantaneous rigor i.e Occurs immediately after death.
• It is spasm of group of voluntary muscles which are used at the time of death.
• It is purely Antemortem Origin
• It cannot be produced artificially
• An important component of cadaveric spasm is the absence of primary relaxation.
Aka instantaneous rigor i.e Occurs immediately after death.
It is spasm of group of voluntary muscles which are used at the time of death.
It is purely Antemortem Origin
It cannot be produced artificially
Incorrect Options:
Option A: Rigor mortis - This is not correct. Rigor mortis is a postmortem change resulting in the stiffeni
ng of the body muscles due to chemical changes in their myofibrils. Rigor mortis helps in estimating the
time since death as well to ascertain if the body had been moved after death.
Option B: Heat stiffness - The muscle proteins undergo coagulation and as a
result the body posture stiffen up. It usually occurs in temperatures above 65 °C

Page 32

518
Option D: Cold stiffness -
This occurs due to freezing of the body fluids, due to sub zero temperatures of around <-5■

Solution for Question 35:


Correct Option B:
• The image shows post-mortem marbling, which is Due to sulfhemoglobin accumulation and formation.
• Post-mortem marbling, also known as livor mortis or hypostasis, refers to the settling of blood in the
dependent parts of the body after death.
• It appears as a purple or bluish discoloration of the skin due to the accumulation of deoxygenated
blood in the blood vessels.
• This phenomenon occurs within a few hours after death and is caused by the pooling of blood under
the influence of gravity.
Incorrect Options:
Option A. Postmortem lividity: Refers to the settling of blood in the dependent parts of the body after de
ath, resulting in a bluish-purple discoloration. It is different from marbling.
Option C. Filigree burns: Refers to a
specific pattern of burns resembling delicate lacework. It is unrelated to the image shown.
Option D. Subcutaneous emphysema: Refers to the presence of air or gas in the subcutaneous tissues
, causing a characteristic crackling sensation on palpation. It is unrelated to the image shown.

Solution for Question 36:


Correct Options C-Greenish discoloration of right iliac fossa:
• Greenish discoloration of the right iliac fossa, known as the Greenish Line of Hochenegg, is one of the
early signs of decomposition and is observed before other changes such as maggot formation,
putrefaction, or mummification.
Incorrect Options:
Option A- Maggot formation: Maggot formation occurs during the later stages of decomposition, after th
e initial discoloration of the right iliac fossa.
Option B- Putrefaction: Putrefaction, the breakdown of body tissues, occurs after the initial discoloratio
n.
Option D- Mummification: Mummification is a
specific form of postmortem preservation and does not occur as the first change in a dead body.

Solution for Question 37:

Page 33

519
Correct Ans: C
• Paradoxical undressing is a phenomenon that can occur in cases of severe hypothermia. When a
person is experiencing extreme cold and their core body temperature drops significantly, paradoxical
undressing refers to the behaviour of the individual inexplicably removing their clothing, often resulting
in being found partially or completely naked.
Incorrect option:
Option A. Heart stress: Heart stress, also known as cardiac stress or cardiac strain, refers to excessive
strain or demand placed on the heart. It is not associated with paradoxical undressing.
Option B. Voyeurism: Voyeurism is a paraphilic disorder characterized by deriving sexual pleasure fro
m observing others without their consent. It is unrelated to paradoxical undressing.
Option D. LSD overdose: LSD (Lysergic acid diethylamide) is a hallucinogenic drug, and an overdose o
f LSD can lead to various physiological and psychological effects. However, paradoxical undressing is
not a recognized symptom or consequence of LSD overdose.

Solution for Question 38:


Correct option D: Saturated salt solution -
Saturated salt solution is commonly used for the preservation of viscera in toxicology autopsy. It is a hy
pertonic solution that helps to prevent decomposition and maintain the integrity of the tissues for further
toxicological analysis. The high salt concentration creates an osmotic environment that inhibits bacteri
al growth and enzymatic degradation.
Incorrect Options
Option A: Glycerin is not commonly used for the preservation of viscera in toxicology autopsy. While it
has some preservative properties, it is not as effective as saturated salt solution for maintaining tissue i
ntegrity and preventing decomposition.
Option B: Rectified spirit, which is a type of alcohol, is not typically used for preservation in toxicology a
utopsy. It may have some disinfectant properties but is not suitable for long-term tissue preservation.
Option C: Formalin is commonly used for general tissue preservation and fixation in anatomical patholo
gy, but it is not specifically preferred for toxicology autopsy. It may affect the results of toxicological ana
lyses and can interfere with certain tests.

Solution for Question 39:


In cases of corrosive acid ingestion, if surgical intervention is necessary to address severe damage, th
e stomach is typically surgically opened along the greater curvature.
Here's an explanation of the correct option and the other options:
Lesser Curvature: The maximum damage is on the lesser curvature
Greater Curvature: Because, in cases like acid or corrosive poisoning, the maximum damage is on the
lesser curvature (Magenstrasse). · To examine the lesser curvature, it opened from greater curvature b
y the double ligature method.

Page 34

520
Vertical: The term "vertical" is not typically used to describe the surgical opening of the stomach. Instea
d, surgical approaches are often described in relation to specific anatomical features like the curvature
s.
Pylorus: The pylorus is the lower part of the stomach that connects to the small intestine. In cases of c
orrosive acid ingestion, the surgical focus is often on the affected areas of the stomach, and the pyloru
s may or may not be involved depending on the extent of the damage.

Solution for Question 40:


Correct Ans: B
• Suggillation, also known as livor mortis or lividity, refers to the discoloration of the skin that occurs
after death due to the settling of blood in the dependent areas of the body. This happens because, after
death, blood circulation ceases, and gravity causes blood to pool in the lowest parts of the body. This
results in a purplish-red or bluish discoloration of the skin in those areas.
Other options
Option A. Tattooing: Tattooing refers to the permanent insertion of ink or pigment into the dermis layer
of the skin. It is not a post-mortem change but rather a
deliberate form of body modification performed during a person's lifetime.
Option C. Putrefaction: Putrefaction is the process of decomposition of organic matter after death. It inv
olves the breakdown of tissues by bacteria and other microorganisms, leading to the release of foul-sm
elling gases and the decay of body tissues. While putrefaction is a
post-mortem change, it is not the correct option in this case.
Option D. Decomposition: Decomposition refers to the process by which organic substances break do
wn into simpler forms. It includes various chemical and physical changes that occur after death, such a
s the breakdown of proteins, fats, and carbohydrates. Putrefaction is one of the stages of decompositio
n. While decomposition is a general term for post-mortem changes, it is not the specific term used to d
escribe the discoloration of the skin seen in suggillation.

Solution for Question 41:


Correct Ans: A
• Marbling: Marbling refers to the mottled appearance of blood vessels seen on the surface of the skin
or internal organs due to the formation of blood clots after death. This change occurs as a result of the
breakdown of blood cells and the settling of blood components. Marbling is commonly observed in the
early stages of decomposition.
Incorrect option:
Option B. Algor mortis: Algor mortis refers to the cooling of the body after death. It is the gradual loss of
body heat until the body reaches the ambient temperature. It is not directly associated with any visual
changes on the body.
Option C. Rigor mortis: Rigor mortis is the stiffening of the muscles after death due to the coagulation o
f muscle proteins. It is a temporary condition that usually starts within a

Page 35

521
few hours after death and resolves after 24 to 48 hours.
Option D. Post mortem lividity: Post mortem lividity, also known as livor mortis, is the pooling or settling
of blood in the dependent parts of the body after death. It results in a
purplish or bluish discoloration of the skin in the lower areas due to gravity.

Solution for Question 42:


Correct Ans: A and C are correct statment
• Rockitansky – In situ: This technique refers to the removal of organs one by one while keeping them
in their original position within the body. It allows for a more detailed examination and dissection of
each organ individually.
• Ghon's – en block: Ghon's technique involves the removal of multiple organs as a single unit or block.
This technique is commonly used in organ transplantation procedures to preserve the integrity of the
organs and minimize ischemic time.
Incorrect Option:
Option B. A, B, and C are correct:
• Rockitansky – In situ: Explained above.
• B is incorrect- Virchows – One by one:technique, also known as the "one by one" technique, is a
method used in autopsies. hence does not match as written above.
• Ghon's – en block: Explained above.
Option C. B and D are correct:.
• B and D both are incorrect - Virchows – One by one:technique, also known as the "one by one"
technique, is a method used in autopsies. It involves the sequential removal of organs for examination
and analysis, allowing for a systematic and thorough assessment of each organ individually.
• Letulle method, also known as “en masse”. In this method, the entire organ block is removed as one.
hence does not match as written above.
Option D. A, B, C, and D are correct:
• B and D both are incorrect

Solution for Question 43:


Correct Option A: 18-22 years
• The fusion of the basiocciput (the base of the occipital bone) with the basisphenoid (part of the
sphenoid bone) occurs ib 18-22 years
Incorrect Options:
Option B - 22-25 years: This option is not correct. By the age of 22-25 years, the fusion of the basiocci
put with the basisphenoid has usually already taken place.
Option C - 14-16 years: This option is not correct. The fusion of the basiocciput with the basisphenoid o
ccurs later than 14-16 years of age. At this stage, the cranial bones are still undergoing development a

Page 36

522
nd growth.
Option D - 12-14 years: This option is not correct. The fusion of the basiocciput with the basisphenoid o
ccurs later than 12-14 years of age. During early adolescence, the cranial bones are still in the process
of development and have not yet fused completely.

Solution for Question 44:


Correct Option: C Mandible Rounded
• Mandible Rounded in female and Mandible is squared in male
Incorrect options:
Option A. Larger teeth: This choice is incorrect because the size of teeth is not a distinguishing feature
between male and female skulls. Tooth size can vary among individuals, but it is not directly related to
sexual dimorphism.
Option B. Mastoid medium-large : This choice is also incorrect. Mastoid medium-large in male ,
in females it is small.
Option D. Zygomatic is more pronounced: This choice is incorrect. Zygomatic is more pronounced in m
ale and it is less pronounced in femlae.

Solution for Question 45:


Correct Option: D
• To determine the approximate age of an individual based on X-ray images of their elbow, wrist, and
pelvis, a method called skeletal age assessment can be used. This method relies on the assessment of
ossification centers and fusion of growth plates in these areas, which occur at predictable ages during
development.The X-RAY findings are consistent with the girl’s age as 21-22 years.
• Findings : Elbow: In a 21-22 year old female, the epiphyseal plates in the elbow region should be
closed, indicating the completion of growth and fusion. The bones of the elbow joint, including the
humerus, radius, and ulna, should appear fully developed and fused. The joint space should be
minimal, and the bony contours should be well-defined. Wrist: At this age, the growth plates in the wrist
region should also be closed, indicating the completion of growth. The carpal bones of the wrist,
including the scaphoid, lunate, triquetrum, pisiform, trapezium, trapezoid, capitate, and hamate, should
be well-formed and have fused together. The bony architecture of the wrist should be well-defined.
Pelvis: In a 21-22 year old female, the pelvis should have completed its growth and reached its adult
form. The ilium, ischium, and pubis bones of the pelvis should be fully developed and fused. The
sacroiliac joints should be stable, and the symphysis pubis should be fused with minimal or no joint
space.
• Elbow: In a 21-22 year old female, the epiphyseal plates in the elbow region should be closed,
indicating the completion of growth and fusion. The bones of the elbow joint, including the humerus,
radius, and ulna, should appear fully developed and fused. The joint space should be minimal, and the
bony contours should be well-defined.
• Wrist: At this age, the growth plates in the wrist region should also be closed, indicating the
completion of growth. The carpal bones of the wrist, including the scaphoid, lunate, triquetrum, pisiform,
trapezium, trapezoid, capitate, and hamate, should be well-formed and have fused together. The bony

Page 37

523
architecture of the wrist should be well-defined.
• Pelvis: In a 21-22 year old female, the pelvis should have completed its growth and reached its adult
form. The ilium, ischium, and pubis bones of the pelvis should be fully developed and fused. The
sacroiliac joints should be stable, and the symphysis pubis should be fused with minimal or no joint
space.
• Elbow: In a 21-22 year old female, the epiphyseal plates in the elbow region should be closed,
indicating the completion of growth and fusion. The bones of the elbow joint, including the humerus,
radius, and ulna, should appear fully developed and fused. The joint space should be minimal, and the
bony contours should be well-defined.
• Wrist: At this age, the growth plates in the wrist region should also be closed, indicating the
completion of growth. The carpal bones of the wrist, including the scaphoid, lunate, triquetrum, pisiform,
trapezium, trapezoid, capitate, and hamate, should be well-formed and have fused together. The bony
architecture of the wrist should be well-defined.
• Pelvis: In a 21-22 year old female, the pelvis should have completed its growth and reached its adult
form. The ilium, ischium, and pubis bones of the pelvis should be fully developed and fused. The
sacroiliac joints should be stable, and the symphysis pubis should be fused with minimal or no joint
space.
Incorrect Option:
The other options are incorrect as they are inconsistent with the above X-ray findings

Solution for Question 46:


Correct option C
• Teeth play a crucial role in the identification of individuals, especially in forensic odontology.
Gustafson's technique, developed by Gustafson in 1950, is a widely used method for age estimation in
adults based on dental characteristics. Among Gustafson's criteria, the most reliable criterion for age
estimation is the translucency of the root (Option C). The technique involves attributing scores for
various age-related changes observed in teeth, such as attrition, periodontal ligament retractions,
secondary dentin formations, root translucency, and root resorption. The translucency of the root is
considered one of the most reliable indicators of age, as it gradually increases with age due to the
deposition of secondary dentin and changes in the root structure.
Incorrect options
Option A: Root resorption, Option B: Paradentosis & Option D: Attrition - While these criteria are also c
onsidered in Gustafson's technique, the translucency of the root is considered the most reliable indicat
or of age estimation among Gustafson's criteria.

Solution for Question 47:


Correct Option B: 5 months
The calculation of age of fetus can be done by two methods
RULE OF HASSE (for upto 5 months of age) - calculates Age(months) = crown heel length(cm)

Page 38

524
MORRISON RULE (for more than 5 months of age)- Age(months) = Crown heel length (cm) / 5
• Since 3 options are of 5 months or less, Rule of hasse can be used first to see if we come to an
answer.
• Square root of 25 = 5 cm which is the answer
Incorrect Options:
Option A: 6 months would’ve been the answer if crown heel length was 30 cm.
Option C: 4 cm would’ve been the answer if crown heel length was 16 cm
Option D: 3 months would’ve been the answer if crown heel length was 9 cm.

Solution for Question 48:


Correct Option B: Pelvis
• Single best bone for sex identification is the pelvis. It provides 95% accurate sex differentiation.
Incorrect Options:
Option A: Skull -
Skull bone provides only 90-92% accuracy, whereas pelvis provides 95% and thus is not a
correct option
Option C: Long bones provide only 80-85 %
accuracy and are best used in combination with other bones.
Option D: Sternum
• Ashley’s rule is used to differentiate sex through the estimation of length of sternal bone. However,
individually it does not provide accurate assessment of sex differentiation.
• Ashley’s rule- Sternal bone in males is usually >149mm, whereas in females it is <149.

Page 39

525
Forensics and Psychiatry
1. Sleep drunkenness is also called as?
A. Somnolentia
B. Somnambulism
C. Automatism
D. Hypnotism
----------------------------------------
2. A 32-year-old female librarian, has been displaying unusual behavior and thought patterns that have
raised concerns among her friends and colleagues. Over the past few months, she has become
increasingly preoccupied with the idea that a famous author is madly in love with her. This is known as?
(or)
Syndrome characterized by delusion that the patient is passionately loved by another person is also
known as:
A. Ekbom’s syndrome
B. De Clerambault’s syndrome
C. Querulous paranoia
D. Othello syndrome
----------------------------------------
3. In which of the following situations, the accused is punishable under Law:
A. If under the influence of an insane delusion, a person thinks another man is attempting to kill him and
he kills that man in self-defense
B. If under the influence of an insane delusion, a person thinks another man to be a wild animal and
kills him
C. If under an insane delusion, a person thinks that another person has caused serious injury to his
character, family or property and kills him.
D. If under the intoxication produced without his will, a person kills another person and has no
recollection of it.
----------------------------------------
4. Rules for criminal responsibility of the insane are all, except:
A. Morrison’s rule
B. American Law institute’s rule
C. New Hampshire doctrine
D. Curren’s rule
----------------------------------------
5. A person is responsible for his criminal act, if he was suffering from which of the following condition:
A. Somnambulism
B. Somnolentia

526
C. Post traumatic automatism
D. Alcohol intoxication with his will
----------------------------------------
6. True about delusions is all ,except:
A. They are false belief, but firm
B. It is primarily a disorder of perception
C. Not associated with intellectual background
D. It remains despite of contrary evidence
----------------------------------------
7. The will is invalid if:
A. Made by deaf, dumb or blind persons
B. Made during lucid intervals of mental illness
C. Suicide by testator immediately after making the will, in the absence of any mental illness
D. Made by drunk person
----------------------------------------
8. A person falsely perceives that his close friend has been replaced by an exact duplicate. This
phenomenon is referred to as:
A. Cotard syndrome
B. Fregoli syndrome
C. Capgras syndrome
D. Delusional perception
----------------------------------------
9. A 10-year-old girl has been exhibiting signs of fear and anxiety every night. As the sun sets and
darkness begins to envelop her room, she becomes increasingly anxious. She insists on keeping all the
lights on, and even a dimly lit hallway can cause distress. The condition she is suffering from is?
(or)
Fear of darkness is called:
A. Nyctophobia
B. Mysophohbia
C. Claustrophobia
D. Agarophobia
----------------------------------------
10. A Sudden and irresistible force compelling a person to commit an offense consciously is known as:
A. Illusion
B. Obsession
C. Twilight state

Page 2

527
D. Impulsiveness
----------------------------------------
11. Which of the following features is not true about advance directive?
A. The patient can appoint a representative to take on behalf of the patient
B. It is applicable at the time of emergency
C. Psychiatrist (or) caregiver can approach mental health board if not satisfied in the case
D. None of the above
----------------------------------------
12. Which of the following features is true about feigned insanity?
A. Gradual onset
B. Presence of predisposing factor
C. Exaggerated facial appearance
D. Motive is usually absent
----------------------------------------
13. What is the method chosen to perform Hara-kiri(seppuku) ?
A. Stabbing heart
B. Cutting the genitalia
C. Shooting in mouth
D. Stabbing in abdomen
----------------------------------------

Correct Answers
Question Correct Answer

Question 1 1
Question 2 2
Question 3 3
Question 4 1
Question 5 4
Question 6 2
Question 7 4
Question 8 3
Question 9 1
Question 10 4
Question 11 2
Question 12 3

Page 3

528
Question 13 4

Solution for Question 1:


Correct Option A - Somnolentia:
• Somnolentia: It is often called sleep-drunkenness.The person would be awake and drowsy or
confused for a long time
Incorrect Options:
Option B - Somnambulism: Somnambulism: Sleep-walking
Option C - Automatism: Automatism: An act committed during a
state of unconsciousness or grossly impaired consciousness.
Option D - Hypnotism: Hypnotism: Mesmerism

Solution for Question 2:


Correct Option B - De Clerambault’s syndrome:
• Erotomania (de Clérambault’s syndrome): Condition in which a person holds a delusional belief that
another person, usually of a higher social status, is in love with him/her.
Incorrect Options:
Option A - Ekbom’s syndrome: Ekbom syndrome (delusions of parasitosis): A
form of psychosis wherein the patients acquire a
delusional belief that they are infested with parasites, whereas in reality no such parasites are present.
Option C - Querulous paranoia: The terms querulous paranoia (Kraepelin, 1904) and litigious paranoia
have been used to describe a paranoid condition that manifested itself in querulant behavior. The conc
ept had, until 2004, disappeared from the psychiatric literature; largely because it had been misused to
stigmatize the behavior of people seeking the resolution of valid grievances.
Option D - Othello syndrome: Delusion of infidelity/ jealousy (Othello syndrome): A person holds a
delusional belief that his spouse is unfaithful.

Solution for Question 3:


Correct Option C - If under an insane delusion, a person thinks that another person has caused serious
injury to his character, family or property and kills him:
• Here the accused is punishable under Law because, under the law, no one can kill a person in
revenge.
Incorrect Options:
Option A - If under the influence of an insane delusion, a person thinks another man is attempting to kill
him and he kills that man in self-defense:His delusion makes him believe that another individual is atte
mpting He will not be held criminally responsible.

Page 4

529
Option B - If under the influence of an insane delusion, a person thinks another man to be a
wild animal and kills him: To be a wild animal and kills him: He will not be held criminally responsible.
Option D - If under the intoxication produced without his will, a
person kills another person and has no recollection of it: Nothing is an offense which is done by a pers
on who at the time of doing it, by reason of intoxication, is incapable of knowing the nature of the act, o
r what he is doing is either wrong or contrary to law; provided that thing which intoxicated him was adm
inistered to him without his knowledge or against his will.

Solution for Question 4:


Correct Option A - Morrison’s rule:
• Morrison rule (1964): This rule states that the crown heel length of the fetus in centimetres, divided by
five gives the age in a month, when CHL is more than 25cm.
Incorrect Options:
Option B - American Law institute’s rule: American Law Institute test (Brawner decision/rule): It combin
es elements of the M’Naughten rule and the irresistible impulse rule. "A person is not responsible for cri
minal conduct if at the time of such conduct as a result of mental disease or defect he lacks substantial
capacity to appreciate the wrongfulness of his conduct or to conform his conduct to the requirements o
f the law."
Option C
- New Hampshire doctrine: The Irresistible Impulse test or the New Hampshire doctrine: Argues that a
person may have known an act was illegal, but because of mental impairment, he couldn’t control his a
ctions.
Option D - Curren’s rule: Curren’s rule (1964): States that an accused person is not criminally responsi
ble if at the time of committing the act, he did not have the capacity to regulate his conduct to the requir
ements of the law as a result of mental disease or defect.

Solution for Question 5:


Correct Option D - Alcohol intoxication with his will:
• Voluntary intoxication is no excuse for the commission of a crime.
Incorrect Options:
Option A - Somnambulism:
• A person is NOT responsible for his criminal acts done during Somnambulism.
• SOMNAMBULISM: It means, walking during sleep.
Option B - Somnolentia:
• A person is NOT responsible for his criminal acts done during Somnolentia.
• SOMNOLENTIA (semi-somnolence): It is often called sleep-drunkenness and is midway between
sleep and waking.

Page 5

530
Option C - Post traumatic automatism: A
person is NOT responsible for his criminal acts done during: Post-traumatic automatism.

Solution for Question 6:


Correct Option B - It is primarily a disorder of perception:
• Delusion is primarily a disorder of thought.
Incorrect Options:
Option A - They are false belief, but firm: Delusion is a false belief in something which is not a fact.
Option C
- Not associated with the intellectual background: It is not associated with the intellectual background.
Option D
- It remains despite of contrary evidence: True Statement, It remains despite of contrary evidence.

Solution for Question 7:


Correct Option D - Made by drunk person:
• Will is invalid when made by imbecile or drunk persons or under insane delusions because the
testator was incapable of rational views and judgment.
Incorrect Options:
Option A/ B/ C - Made by deaf, dumb or blind persons/ Made during lucid intervals of mental illness/ Su
icide by testator immediately after making the will, in the absence of any mental illness:
• Will is valid under special circumstances: Made by deaf, dumb or blind persons. Made during lucid
intervals of mental illness. Suicide by testator immediately after making the will, in the absence of any
mental illness.
• Made by deaf, dumb or blind persons.
• Made during lucid intervals of mental illness.
• Suicide by testator immediately after making the will, in the absence of any mental illness.
• Made by deaf, dumb or blind persons.
• Made during lucid intervals of mental illness.
• Suicide by testator immediately after making the will, in the absence of any mental illness.

Solution for Question 8:


Correct Option C - Capgras syndrome:
• Capgras syndrome: The false belief that a familiar person or place has been replaced with an exact
duplicate.

Page 6

531
Incorrect Options:
Option A - Cotard syndrome: Cotard syndrome is a
false belief that they are already dead, are putrefying, or have lost their blood or internal organs.
Option B - Fregoli syndrome: Fregoli delusion: Delusion of doubles, is a
false belief that different people are in fact a single person who changes appearance.
Option D - Delusional perception:
• Delusional perception: A Schneiderian first-rank symptom in which a person believes that a normal
percept (product of perception) has a special meaning for him or her.
• Eg: A cloud in the sky may be misinterpreted as meaning that someone has sent that person a
message to save the world. While the symptom is particularly indicative of schizophrenia, it also occurs
in other psychoses, including mania (in which it often has grandiose undertones).

Solution for Question 9:


Correct Option A - Nyctophobia:
• Nyctophobia: Fear of darkness.
Incorrect Options:
Option B - Mysophohbia: Mysophobia: Fear of contamination.
Option C - Claustrophobia: Claustrophobia: Fear of closed spaces.
Option D - Agarophobia: Agoraphobia: Fear of crowded open space

Solution for Question 10:


Correct Option D - Impulsiveness:
• A sudden and irresistible force compelling a person to commit an offense consciously is known as an
impulse.
Incorrect Options:
Option A - Illusion: Illusion is a
false interpretation by the senses of an external object or stimulus which has a
real existence, e.g., when a person sees a dog and mistakes it for lion.
Option B - Obsession: Obsession: In this, a
single idea, thought, or emotion is constantly entertained by a person which he recognizes as irrational,
but persists inspite of all efforts to drive it from his mind. It is a disorder of content of thought.
Option C - Twilight state: Twilight state is seen in visual hallucinations, Epilepsy, Head injury, Hysteria,
and Punch-drunkenness.

Solution for Question 11:

Page 7

532
Correct Option B - It is applicable at the time of emergency
• Advance directive is not applicable at the time of emergency.
Incorrect Options:
Option A - The patient can appoint a representative to take on behalf of the patient: True statement ab
out the advance directive, that the patient can appoint a representative to take on behalf of the patient.
Option C - Psychiatrist (or) caregiver can approach mental health board if not satisfied in the case: Psy
chiatrist (or) caregiver can approach mental health Board of anyone of them are not satisfied in the cas
e.

Solution for Question 12:


Correct Option C - Exaggerated facial appearance:
• In cases of feigned insanity, the person voluntarily exaggerates the facial appearance.
Incorrect Options:
Option A - Gradual onset: In feigned insanity, its Sudden onset.
Option B - Presence of predisposing factor: Predisposing factors are absent in feigned insanity.
Option D - Motive is usually absent: In feigned insanity, Motive is always present.

Solution for Question 13:


Correct Option D - Stabbing in abdomen:
• The Hara-kiri method of suicide involves stabbing into the abdomen.
• Hara-kiri (seppuku): It is an unusual type of suicidal disembowelment connected with Japanese
Samurai warriors.
• The victim with a short sword inflicts a single large abdominal stab wound into the left side, drawing
the blade across to the right side and then turning it upwards producing an L-shaped cut. The sudden
evisceration of the internal organs causes an immediate decrease in intra-abdominal pressure and
cardiac return resulting in collapse and death.

Page 8

533
Option A, B and D does not explain the mode of suicide correctly.

Page 9

534

You might also like